You are on page 1of 1646

OCN

800 Questions and Explanation


2020 edition
. Amanda forte
Question 1
• Question 1: A 72-year-old man with a history of prostate cancer presents with lower back pain, and
worsening fecal incontinence. He states he frequently becomes constipated and has leakage of feces
before he can make it to the bathroom. Recently the incontinence has been more severe, and he finds
he has to use a finger to help relieve himself of the stool. MRI shows metastasis to the S4 and S5
vertebrae.
• What is the mechanism behind his fecal incontinence?
• Choices:
• 1. Loss of voluntary control of the external anal sphincter
• 2. Loss of voluntary control of the internal anal sphincter
• 3. Failure to contract the puborectalis muscle
• 4. Loss of the recto sphincteric reflex
Answer: 1

• Answer: 1 - Loss of voluntary control of the external anal sphincter


• Explanations:
• Voluntary control of defecation includes relaxing the external anal sphincter
and puborectalis muscle.
• The urge to defecate stems from the contraction of the external anal sphincter
mediated by the rectosphincteric reflex.
• The rectosphincteric reflex causes relaxation of the internal anal sphincter.
• The myenteric plexus mediates the passage of stool through the distal large
intestine, rectum, and anal sphincter through peristaltic movements.
Question 2

• Question 2: A 45-year-old male presents to the clinic complaining of progressive gait


instability, paresthesias of his right upper extremity, and left-sided facial weakness for about
4 weeks. He has a past medical history of chronic bronchitis, HIV, congenital hydronephrosis
of his right kidney, hepatitis C and Kaposi sarcoma. Further review of systems is positive for
a productive cough and shortness of breath over the past few days. On examination, he is
mildly tachypneic. A 3/5 weakness of his left facial musculature and right lower extremity is
appreciated. The patient is given antibiotics and a 2-week follow up is scheduled.
• What is the next best step in the management of the patient?
• Choices:
• 1. MRI brain with contrast
• 2. Stereotactic brain biopsy
• 3. Lumbar puncture with cerebrospinal fluid (CSF) analysis
• 4. Fluorodeoxyglucose (FDG) PET imaging
Answer: 1

• Answer: 1 - MRI brain with contrast


• Explanations:
• HIV positive patients should always be tested for CNS lymphomas if they
present with symptoms involving motor coordination.
• MRI brain is often the best first imaging test used in the diagnosis of primary
CNS lymphoma.
• Lesions are usually hypointense on T1 weighted sequences and hyperintense
on T2 and show post-contrast enhancement.
• When CNS lymphoma is suspected it is important to not prescribe steroids, as
they often cause a rapid resolution of symptoms and false negative biopsies
can delay diagnosis.
Question 3
• Question 3: A 65-year-old male patient recently diagnosed with rectal cancer
within 1 cm of the anal canal undergoes CT imaging for the staging. Results of
CT reveal a possible extension beyond the rectal wall. If rectal cancer is
metastatic.
• which of the following group of lymph nodes would most likely contain the
cancerous cells?
• Choices:
• 1. Inguinal nodes
• 2. Para-aortic nodes
• 3. Internal Iliac nodes
• 4. Superior mesenteric nodes
Answer: 3
• Answer: 3 - Internal Iliac nodes
• Explanations:

• Lymphatic drainage of the rectum follows the venous drainage.


• Upper two-thirds of rectum drains into the inferior mesenteric lymph nodes
and paraaortic nodes.
• Lower one-third of rectum drains into the internal iliac nodes.
• The lymphatic drainage below the dentate line in the anal canal is to the
inguinal nodes
Question 4

• Question 4: A 65-year-old male with a history of taking betel quid presents to


the clinic with the complaint of swelling in the mouth near the left lower molar
and 20 pounds weight loss for the past two months. He was prescribed
antibiotics by his PCP with no change in the sign and symptoms. What is the
most likely cause of his symptoms?
• Choices:
1. Oral abscess
2. Oral cancer
3. Fungal infection
4. Parasitic infestation of the oral cavity
Answer: 2
• Answer: 2 - Oral cancer
• Explanations:
• The most likely explanation is oral cancer. Oral cancer can manifest as
swelling/thickening inside the oral cavity.
• There could be dysphagia or dysphonia associated with the cancer.
• General symptoms such as fatigue, weight loss can also be seen with oral
cancer.
• An oral abscess usually responds to antibiotic treatment
Question 5
Question 5: A 47-year-old female presents to a health care provider after
discovering a lump in her right breast. She has not had any other symptoms.
Physical examination reveals a 2.5 cm, firm, fixed, nontender mass in the upper,
outer quadrant of the right breast. Diagnostic mammography shows the presence
of suspicious calcifications and biopsy demonstrates invasive ductal carcinoma.
Molecular testing reveals that the tumor is positive for HER2 and negative for
estrogen receptor and progesterone receptor.
The patient undergoes partial mastectomy and sentinel lymph node biopsy, which
indicates the presence of regional nodal metastasis. A post-operative positron
emission tomography (PET) scan shows no evidence of distant metastases.
In addition to radiation therapy, treatment with which of the following is most
appropriate?
• Choices:
1. Anastrozole and doxorubicin
2. Cyclophosphamide alone
3. Tamoxifen alone
4. Trastuzumab and paclitaxel
Answer: 4
• Answer: 4 - Trastuzumab and paclitaxel
• Explanations:
• Trastuzumab is indicated for the neoadjuvant treatment of HER2-positive breast
cancer, in combination with a taxane like paclitaxel or anthracycline-based
chemotherapy.
• The primary mechanism of action of trastuzumab is the blockade of HER2, a type of
epidermal growth factor receptor.
• Patients receiving trastuzumab should be monitored for the development of
cardiotoxicity.
• Trastuzumab has a similar mechanism of action, though not identical, to pertuzumab,
a newer antineoplastic biologic agent.
Question 6
• Question 6: A student is performing a routine physical exam on a patient with a
known family history of stomach cancer. The supervising clinician advises the
student to pay close attention to which anatomic structure found in the
posterior neck triangle during the physical examination?
• Choices:
1. Left supraclavicular lymph node
2. Right supraclavicular lymph node
3. Right occipital lymph node
4. Left occipital lymph node
Answer: 1
• Answer: 1 - Left supraclavicular lymph node
• Explanations:
• The left supraclavicular lymph node, also known as Virchow node, drains lymphatic
fluid from the abdominal cavity after passing through the thoracic duct.
• Cancer of the stomach can be detected by physical examination through a finding
known as Troisier sign, which includes an enlarged, firm, painless left supraclavicular
node.
• The right supraclavicular lymph node drains lymphatic fluid from the thorax and can be
used to diagnose cancers of the lung or esophagus by similar physical exam findings.
• Routine cervical lymph node palpation should include both supraclavicular lymph
nodes in the posterior neck triangles due to their significance in draining carcinomas
from various regions.
Question 7
• Question 7: A 55-year-old G2P2 presents to the office complaining of vaginal
bleeding. A speculum examination demonstrates a 3x2cm exophytic lesion on
the right vaginal sidewall with no other lesions. A biopsy is carried out which
returns positive for well-differentiated squamous cell carcinoma. In order to
stage cancer, a pelvic exam is carried out whereby a limited spread to the
subvaginal tissue not involving the pelvic sidewall is found. The spread is also
not found to be anywhere in the bladder or rectum. Which of the following is
the stage of cancer in this patient?
• Choices:
1. Stage IVA
2. Stage II
3. Stage I
4. Stage III
Answer: 2

• Answer: 2 - Stage II
• Explanations:
• Staging is one of the most important prognostic factors of vaginal cancer.
• Stage II includes involvement of subvaginal tissue without extension to the pelvic
sidewall.
• Treatment is often dependent on the stage of the disease.
• Stage I is limited to the vaginal wall.
• Stage III refers to an extension beyond true pelvis or involvement of bladder or
rectal mucosa.
• Stage IVA refers to spread to adjacent organs and/or direct extension beyond the
true pelvis
Question 8
• Question 8: A 62-year-old male with no significant past medical history presents
to the clinic for weight loss, fatigue, and vague abdominal pain for the past two
months. He tried using multivitamins but did not feel better.
• He has smoked a pack of cigarettes daily for twenty five years and drinks whiskey
every night. Upon further questioning, he reveals he is tying his belt two notches
tighter than two months ago. Upon physical examination, his conjunctivae
appear pale and yellow. The lower border of the liver is barely palpable in the
right hypochondrium, and a notched smooth mass is felt in the left
hypochondrium.
• Which of the following is the best initial test to reveal the underlying diagnosis?
• Choices:
• 1. CT abdomen
• 2. Liver function tests
• 3. Erythrocyte sedimentation rate
• 4. Ultrasound of the abdomen
Answer: 4
• Answer: 4 - Ultrasound of the abdomen
• Explanations:
• The patient most likely has a pancreatic mass, occluding the biliary duct leading to jaundice and the portal
vein leading to splenomegaly. An abdominal ultrasound is indicated as the initial test, which may be followed
by a CT or an MRI. Abdominal ultrasound is the most often recommended best initial test while investigating
hepatic, pancreatic, and biliary malignancies.
• Doppler ultrasound is superior to normal ultrasound and contrast-enhanced ultrasound is superior to Doppler
ultrasound for the detection of portal vein thrombosis.
• Possible ultrasound findings include increased portal vein diameter, a hyperechoic shadow in the portal vein
and dilated CBD (common bile duct) as seen in this patient due to pancreatic cancer.
• In a patient with no apparent cause for the thrombus, recommended blood tests should encompass extensive
procoagulant factors workup, including antiphospholipid syndrome, protein C, S, antithrombin III levels, factor
V and Leiden mutation.
Question 9

• Question 9: A 65-year-old male patient presents to the hospital for routine follow up of hypertension and to
discuss smoking cessation. He has smoked a pack of cigarettes a day since he was in high school. Today, he is
complaining of swelling and pain in both wrists. No other joints are affected. He denies injury, fever, morning
stiffness, and rashes. He also reports a 20-pound weight loss in the last three months and denies shortness of
breath, chest pain, or hemoptysis. He does not have any gastrointestinal or genitourinary symptoms. His blood
pressure is well controlled. The physical exam is remarkable for palpable effusion in both wrists without
evidence of synovial inflammation, and limited range of motion due to pain. There is no small joints tenderness.
He has gross clubbing of the fingers in both hands. Which of the following is the most appropriate next step in
the evaluation of this patient?
• Choices:
• 1. Rheumatoid factor
• 2. ANA
• 3. Joint aspiration with synovial fluid analysis
• 4. Chest X-ray
Answer: 4
• Answer: 4 - Chest X-ray
• Explanations:
• This patient's presentation is most consistent with hypertrophic osteoarthropathy, most likely secondary to a pulmonary
malignancy. Given the patient's smoking history and recent weight loss, a chest x-ray would be the best test to screen for
pulmonary neoplasm.
• Hypertrophic osteoarthropathy (HOA), consists of the presence of digital clubbing, increased the periosteal activity of the
tubular bones, arthralgias, and joint effusion and is characterized by abnormal proliferation of the skin, soft tissues and
osseous tissues in the distal parts of extremities. There are two forms of the disease - primary and secondary. Primary HOA
(PHO), also called pachydermoperiostosis, comprises of digital clubbing, periostosis, and pachydermia and is a rare genetic
disease, while secondary hypertrophic osteoarthropathy is associated with a variety of pulmonary, cardiac and other
conditions.
• ANA would be the correct answer if SLE were suspected. Given this patient's weight loss, smoking history, and lack of rash
and morning stiffness, the more likely diagnosis is secondary hypertrophic osteoarthropathy. This patient has bilateral
arthropathy, making gout and septic arthritis less likely.
• The rheumatoid factor would be more appropriate as a screen for rheumatoid arthritis (RA). While RA may present with wrist
effusion and pain, it would be extremely unlikely for RA to be the cause of his distal joint swelling and pain. He also denies
Question 10
• Question 10: A 65-year-old female patient presents to the clinic with unbearable pain in her abdomen. She
received a diagnosis of advanced rectal cancer, and chemotherapy appeared to slow down the clinical course
for about a year. At the time of the visit, however, the clinical conditions worsened, and the patient shows a
cachectic appearance and has a likely hydro-electrolyte imbalance. She suffers from intense pain in the lower
abdomen with a burning in the perianal region despite treatment with high-dose strong opioids, including
tapentadol 400 mg/day, acetaminophen, and pregabalin. In addition, she reports several episodes of pain
spikes in a day.
• Which of the following is the most effective treatment for this patient?
• Choices:
1. Rapid onset opioids combined with tapentadol 400 mg/day
2. Transdermal patch fentanyl in equianalgesic dosing
3. Increase the dose of tapentadol 400 mg/day
4. Epidural analgesia
Answer: 4

• Answer: 4 - Epidural analgesia


• Explanations:
• Patients with terminal cancer may have pain that is refractory to systemic analgesics.
• Continuous epidural analgesia is an effective strategy for the treatment of mixed (somatic plus neuropathic)
cancer pain.
• The patient completed the three steps of the World Health Organization (WHO) pain ladder with inadequate
pain control. However, it would be possible to perform minimally invasive or even invasive therapies in the
early stages of the disease, after a careful case-by-case analysis
• Rapid onset opioids cannot be used to control background analgesia. Furthermore, these drugs cannot be used
in more than ¾ administrations a day, and higher doses of tapentadol are not recommended. Although
fentanyl patches are approved by the United States-
• Food and Drug Administration (US-FDA) for patients who are opioid resistant this strategy is often ineffective as
the kinetics of absorption is not predictable; furthermore, the onset of action can take several hours and
depends on the body temperature, previous dosing of opioid and other factors such as site of application,
Question 11
• Question 11: A 62-year-old man comes to the clinic complaining of a sensation of dry
mouth. He states that it started three weeks ago and he has to keep a bottle of water
with him at all times. He was recently diagnosed with squamous cell carcinoma of the
tongue and opted to receive radiation therapy. He was recently started on hydralazine
for high blood pressure. His last dose of radiation was two days ago. His vital signs
show a BP of 130/85 mmHg, HR 72/minute, and RR of 18/minute. On physical exam,
his oral mucosa is mildly atrophic accompanied by cracked lips. Which of the following
is the most appropriate cause of the patient's complaint?
• Choices:
1. Age
2. Hydralazine
3. Radiation therapy
4. Squamous cell carcinoma
Answer: 3
• Answer: 3 - Radiation therapy
• Explanations:
• Radiation therapy of any head and neck cancer is associated with xerostomia.
• Almost every patient receiving radiation for head and neck cancer complains of
dry mouth.
• Xerostomia in these patients is caused by the hypofunction of salivary glands,
either by gland atrophy or loss of nerve impulses to the gland due to radiation.
• Antihypertensive drugs have been associated with xerostomia but an acute
presentation after receiving radiation therapy is more likely in this patient.
Squamous cell carcinoma does not cause symptoms of dry mouth by itself
Question 12
• Question 12: A 60-year old farmer presents with several lesions on his head,
neck, and nose. He says he has had these skin lesions for many years, but they
do not seem to be going away. He has been a farmer all his life and has no other
pertinent medical history. On physical exam, you note two lesions that appear
waxy and have a central depression. The edges have a pearly appearance.
Excisional biopsy of one of the lesions reveals that it is a malignancy. What is the
most common subtype of malignancy in people with such lesions?
• Choices:
1. Nodular
2. Pigmented
3. Cystic
4. Superficial
Answer: 1
• Answer: 1 - Nodular
• Explanations:
• Nodular basal cell cancer is the most common variety of cancer. This type of
cancer usually occurs on the head, neck, and upper back.
• Nodular basal cell cancers may have waxy papules and a pearly appearance that
may be eroding, bleeding or crusting.
• Pigmented lesions are next most common and also occur on sun-exposed
areas.
• Cystic basal cell cancers are translucent and often appear grey and are common
on the nose and forehead.
Question 13
• Question 13: A 72-year-old male, former smoker, with untreated stage 4 small-
cell lung cancer of the lungs, presents to the emergency department with a
continuous uncontrollable, debilitating severe back pain and spasm of 2 weeks
duration, refractory to acetaminophen. Physical exam reveals no signs of cord
compression. The patient is currently on palliative care management. After
hospital admission for evaluation, he was found to have multiple metastatic
lesions in the spine. Which of the following is the best initial therapy for this
patient?
• Choices:
1. Spinal decompression
2. Immediate-release hydromorphone administration
3. Naproxen administration
4. Extended-release hydromorphone administration
Answer: 2
• Answer: 2 - Immediate-release hydromorphone administration
• Explanations:
• The patient is complaining of severe back pain refractory to acetaminophen.
Treatment consists of opioid administration.
• Even though the patient may have a limited lung function capacity, opioid
administration should not be stopped as the patient is at a terminal stage of his
disease.
• Immediate-release hydromorphone should be chosen over extended-release as
it has an onset of action at 15-30 mins. Extended-release hydromorphone has
an onset of action at 6 hours.
• Spinal decompression is not recommended as the cord is not compromised and
the patient has a poor frailty index that does not to favor surgical procedures.
Question 14
• Question 14: A 39-year-old male presents to the clinic with anemia and
lymphocytosis. A majority of the lymphocytes were found to be atypical and
had a deeply clefted nuclei. Flow cytometry showed a kappa restricted CD20+,
CD19+, and CD10+ population.
• Which of the following is considered a negative prognostic factor of the
patient's disease?

• Choices:
1. Age more than 60 years
2. The diameter of the largest involved lymph node is more than 5 cm
3. Low beta2 microglobulin
4. Increased ionized calcium
Answer: 1
• Answer: 1 - Age more than 60 years
• Explanations:
• The prognosis and risk assessment of follicular lymphoma (FL) is determined based
on the FLIP2 study that includes beta2 macroglobulin higher than the upper limit of
normal, bone marrow involvement, hemoglobin of less than 12g/dl, longest
diameter of the largest involved node of more than 6 cm and age greater than 60
years.
• The 5-year progression-free survival based on the risk factors is low risk (0 risk
factors), 80%; intermediate risk (1-2 risk factors), 51%; High risk (3-5 risk factors),
19%.
• Management of FL depends on the stage of the disease.
• FL is the second most common type of non-Hodgkin lymphoma (NHL) and accounts
Question 15
• Question 15: A 28-year-old female follows up at the clinic to discuss her
prognosis from cervical cancer. The care team decides to have a complete
cultural assessment of the patient.
Which of the following is the most appropriate reason for this assessment?
• Choices:
1. To assure the patient is being treated the same as everyone else
2. To assure the quality of care given to this patient is appropriate according to
her family needs
3. To avoid stereotyping the patient and incorrect assumptions while enchaining
patient-centered care
4. To ensure that her spiritual needs are met
Answer: 3

• Answer: 3 - To avoid stereotyping the patient and incorrect assumptions while


enchaining patient-centered care

• Explanations:
• The primary purpose of cultural assessments is to avoid stereotyping the
patient and incorrect assumptions while enchaining client-centered care.
• Family relationships are also very important in a cultural assessment.
• The standard of care for each patient may be different based on cultural needs.
• Although spirituality is important in a cultural assessment, avoiding stereotypes
takes priority.
Question 16
• Question 16: A 67-year-old male presents to the office with the
complaint of feeling tired all the time for the past four months. He
has lost 10 pounds over the past two months. He also feels shortness
of breath on exertion. He has no significant past medical history. He
has been smoking one pack of cigarettes daily for the past 25 years.
His family history is significant for colon cancer. Physical examination
is significant for pallor and koilonychia. Labs show hemoglobin of
10.7 g/dL. His last colonoscopy was at the age of 65 and was
unremarkable.

What is the next best step in the management of this patient?


• Choices:
• 1. Blood transfusion
• 2. Colonoscopy
• 3. Chest x-ray
• 4. CT scan chest
Answer: 4
• Answer: 4 - CT scan chest
• Explanations:
• Malignancy is a common cause of new-onset fatigue. Elderly patients often present with
new onset of fatigue and weight loss, which might be an indication of some underlying
malignant process. If patients with fatigue have risk factors of malignancy, they should be
screened according to age-appropriate cancer screening protocols like colonoscopy if the
patient age is 50 years or above, low dose CT or chest x-ray if the patient is a chronic
smoker and between 55-80 years of age.
• Although both a CT scan of the chest and a colonoscopy would be recommended in this
patient, the next best step would still be the CT scan.
• A history of unremarkable colonoscopy two years back makes a lung cancer more likely in
this patient.
• The patient should never be reassured without doing any diagnostic workup.
Question 17
• Question 17: A patient is started on a chemotherapeutic agent for a
malignancy. The patient develops a hemolytic anemia with a uremic syndrome.
Which of the following is the most likely drug that caused these conditions?

• Choices:
1. Mitomycin
2. Cyclosporine
3. Cyclophosphamide
4. Azathioprine
Answer: 1
• Answer: 1 - Mitomycin
• Explanations:
• Hemolytic uremic syndrome has been associated with a various etiologies, most
commonly E. coli infection.
• Chemotherapeutic medications that can cause this condition include mitomycin,
bleomycin, gemcitabine, and cisplatin.
• Mitomycin causes hemolytic uremic syndrome in less than 10% of patients. This
syndrome is characterized by destruction of red blood cells, damage to blood
vessel walls, and kidney failure. This occurs in less than 2% of patients treated
with mitomycin, it can occur at any time but is usually seen after several cycles of
therapy.
• Mitomycin often causes low blood counts. Less common side effects include
nausea and vomiting, diarrhea, hair loss, bladder inflammation, pneumonitis, and
rarely pulmonary fibrosis.
Question 19
• Question 19: A 65-year-old male is undergoing a novel treatment for pancreatic
adenocarcinoma. His specialist is injecting hyaluronidase directly into the
pancreatic tumor in order to achieve better uptake of the chemotherapy in the
cancer cells. Within minutes of administration of the hyaluronidase, the patient
becomes diaphoretic, begins wheezing, and his blood pressure drops to 86/50
mmHg, while his heart rate is measured at 140/min. What is the next best step
in the management of this patient?
• Choices:
1. Stop the hyaluronidase infusion
2. Administer diphenhydramine
3. IM epinephrine
4. IV methylprednisolone
Answer: 1
• Answer: 1 - Stop the hyaluronidase infusion
• Explanations:
• This patient is having an anaphylactic reaction to the hyaluronidase. It is crucial to
stop the offending agent.
• The patient would require epinephrine to help stabilize his blood pressure and
inhibit bronchospasms. However, this should be done after stopping the
hyaluronidase infusion.
• The short time between drug administration and symptom onset suggests that the
patient's anaphylaxis is caused by a hypersensitivity reaction to the injected
hyaluronidase.
• Animal-derived hyaluronidase is immunogenic and may cause allergic reactions.
Human recombinant hyaluronidase (rHuPH20) is better tolerated and with no
evidence of allergenicity.
Question 20
• Question 20: A 52-year old male presents to the emergency department after a
highway motor vehicle accident. During his laboratory workup, an MRI reveals
a mass in the pancreas. Upon further testing, you suspect that the patient has
a non-functioning pancreatic neuroendocrine tumor. Which of the following is
a clinical manifestation of a nonfunctioning pancreatic neuroendocrine tumor?

• Choices:
• 1. Tremors
• 2. Palpitations
• 3. Necrolytic rash
• 4. Abdominal pain
Answer: 4
• Answer: 4 - Abdominal pain
• Explanations:
• Nonfunctioning tumors are the most common subtype of pancreatic
neuroendocrine tumors.
• These tumors may release certain substances, such as chromogranin A and
pancreatic polypeptide.
• These tumors do not present with a hormonal syndrome and are most
commonly diagnosed incidentally.
• If symptoms are present, they are due to mass effect and include abdominal
pain, weight loss, anorexia, nausea, obstructive jaundice, intraabdominal
hemorrhage, and a palpable mass.
Question 21
• Question 21: A 60-year-old male presents to the primary care clinic for his
yearly checkup. He states that he has not seen his primary care physician for
12 years because he has always been healthy. The patient reveals that he has
had vague epigastric abdominal pain for the past month and decided to get a
checkup at the insistence of his wife. He also reports intermittent episodes of
diarrhea and a 25-pound weight loss over the last three months, which he
attributes to stress. He is sent to the local hospital for abdominal imaging,
which reveals a mass in the head of the pancreas and multiple gallstones.
Further testing reveals an HbA1c level of 8.5%.

Which of the following cells most likely constitute his pancreatic tumor?
• Choices:
• 1. Alpha cells
• 2. Beta cells
• 3. Pancreatic polypeptide cells
• 4. Delta cells
Answer: 4
• Answer: 4 - Delta cells
• Explanations:
• The delta cells of the islets of Langerhans release somatostatin.
• Tumors made up of delta cells are called somatostatinomas and are usually
malignant.
• Around 55% of somatostatinomas occur in the pancreas and the remaining
45% occur in the duodenum and rarely in the jejunum.
• The classic triad of symptoms in patients with somatostatinomas is diabetes
mellitus, cholelithiasis, and diarrhea or steatorrhea.
• These symptoms are more common with pancreatic somatostatinomas.
Question 22
• Question 22: A 65-year-old male, who smokes a pack per day, notices bright red
blood in his stool. The patient does not regularly see a healthcare provider and
has never had a colonoscopy. The patient admits to a diet high in red meat with
minimal fiber intake. He denies abdominal pain or changes in bowel habits. The
patient has his blood drawn for basic labs. What is the most appropriate next
step in the management of this patient?
• Choices:
1. Reassurance and watchful waiting
2. CT scan of the chest, abdomen, and pelvis
3. Colonoscopy
4. Assess Serum CEA
Answer: 3
• Answer: 3 - Colonoscopy
• Explanations:
• The U.S. Preventive Services Task Force (USPSTF) currently recommends screening for
colorectal cancer beginning at age 50. Different options exist for cancer screening;
however, colonoscopy provides the highest sensitivity and specificity with the ability
to remove polyps or
• lesions for a biopsy if warranted.
• Patients presenting with painless bloody stools, over the age of 50 without a previous
colonoscopy, should be suspected with the primary diagnosis of colon cancer.
• Smoking and diets high in red meat have correlations with the increased risk of
colorectal cancer.
• The initial presentation of colorectal cancers include changes in stool caliber, rectal
bleeding, or iron deficiency anemia
Question 23
• Question 23: A 55-year-old male presents to the clinic with complaints of fatigue
and painful fingers and toes. He also states that that feels a burning sensation
after hot showers at times. On review of systems, it is noted that he also has
headaches, night sweats, and a weight loss of 30 pounds. Laboratory
investigations reveal a hemoglobin of 18.1 g/dL and low serum erythropoietin.
Which following best explains why oxygen uptake within tissue can be impaired
in this patient?
• Choices:
• 1. Arteriolar constriction to divert blood flow
• 2. Increased intravascular resistance
• 3. Defective elastic components within vessels
• 4. Hyperactive vascular smooth muscle cells
Answer: 2
• Answer: 2 - Increased intravascular resistance
• Explanations:
• As the most likely diagnosis is polycythemia vera, a diversion of blood flow does
is an associated feature. In hypovolemic shock, blood from the distal
extremities can be diverted to the core needs of the body.
• With polycythemia vera as the most likely diagnosis, hyperviscosity of the blood
due to thrombocytosis leads to an increase in resistance to blood flow.
• Any intravascular resistance to blood flow can impair oxygen delivery to tissue.
• Defective elastic components of blood vessels are associated with connective
tissue disorders, and not with polycythemia vera. Hyperactive vascular smooth
muscle cells is not a phenomenon associated with polycythemia vera.
Question 24
• Question 24: A 28-year-old female presents to the emergency department with right breast
pain. She has a history of drug abuse and has recently completed a drug rehabilitation
program. On physical examination, she is noted to have a right breast lump that is tender.
Laboratory tests and radiologic tests confirm the diagnosis of breast cancer. The treatment
protocol for breast cancer is started including a surgery. Postoperatively, the patient inquires
if there is a non-opioid intervention that can help her with the pain. Which of the following
would best help the patient manage her pain acutely?
• Choices:
• 1. T10 epidural steroid injection
• 2. Lumbar epidural catheter placement
• 3. Local anesthetic infiltration at abscess site
• 4. Right-sided pectoralis II block
Answer: 4

• Answer: 4 - Right-sided pectoralis II block


• Explanations:
• The pectoralis II (PECS II) block can be used to provide analgesia in a variety of anterior thoracic wall surgeries,
most commonly breast surgery.
• The PECS II block targets the interfacial plane between the pectoralis major and pectoralis minor as for PECS I,
but also targets the interfacial plane between the pectoralis minor and the serratus anterior, aiming to block
intercostal nerves 3-6, intercostobrachial and the
• long thoracic nerves necessary for axillary node dissection.
• Patient refusal or infection at the site of injection are absolute contraindications to performing a PECS block.
• An epidural steroid injection may have some beneficial effects on pain in the T10 dermatome, but the T10
dermatome is significantly below the level of the breast (T2-T6). Lumbar epidural catheter placement would
also not help manage pain in the breast. Finally, a local anesthetic (locally infiltrated) has been shown to have
decreased efficacy in infection due to acidotic conditions at the site. Therefore local anesthetic infiltration would
not work effectively.
Question 25
• Question 25: A 65-year-old female, current smoker, with a history of active
breast cancer with recent mastectomy, hypertension, and diabetes begins
complaining of chest pain and shortness of breath three days into her intensive
care unit (ICU) stay. Her vitals shows a blood pressure of 146/88 mmHg, a pulse
of 112/min, and oxygen saturation of 94% on room air.
Which of the following are significant risk factors for venous thrombosis in this
patient?
• Choices:
• 1. Hypertension, smoking, and diabetes
• 2. Breast cancer, recent mastectomy, and current ICU admission
• 3. Hypertension, tachycardia, and smoking
• 4. Breast cancer, smoking, and low oxygen saturation
Answer: 2
• Answer: 2 - Breast cancer, recent mastectomy, and current ICU admission
• Explanations:
• Virchow's classic triad describes the three main risk factors for venous thrombotic
disease and includes (1) venous stasis, (2) hypercoagulable state, and (3) vessel wall
disruption or injury. This patient demonstrates one risk factor in each of these
categories, plus smoking, which is also another separate risk factor.
• This patient is at risk for venous stasis secondary to her non-ambulatory status during
her ICU admission.
• This patient has the potential for a hypercoagulable state secondary to her active
breast cancer.
• This patient has the potential for vessel wall disruption or injury secondary to her
recent surgery (mastectomy).
Question 26

• Question 26: Which of the following is associated with lung cancer screening
using annual low-dose computed tomography?
• Choices:
• 1. Reduction in neither lung cancer incidence nor lung cancer mortality
• 2. Reduction in lung cancer incidence
• 3. Reduction in lung cancer mortality
• 4. Reduction in both lung cancer incidence and lung cancer mortality
Answer: 3
• Answer: 3 - Reduction in lung cancer mortality
• Explanations:
• The landmark National Lung Screening Trial demonstrated that annual low-
dose CT screening among at-risk individuals was associated with a significant
reduction in lung cancer mortality.
• More cancers were detected in the screening arm; therefore, the incidence of
diagnosed cancers was higher, not lower.
• All-cause mortality was also decreased, though this did not reach statistical
significance.
• The United States Preventive Services Task Force recommends annual
screening in adults 55 to 80 years of age who have a 30 pack-year smoking
history and currently smoke or have quit within the past 15 years.
Question 27
• Question 27: A 65-year old male presents to the clinic with a history of
metastatic prostate cancer. He recently received radiation to his spine for severe
pain. He did obtain significant pain relief but now has returned with moderate
to severe pain in the same area. What is the current recommendation for
providing relief to these patients?
• Choices:
• 1. Repeat radiation treatment
• 2. Surgical vertebroplasty
• 3. Start on bisphosphonates
• 4. Start an infusion pump for pain relief
Answer: 1
• Answer: 1 - Repeat radiation treatment
• Explanations:
• Radiation is often given to patients with metastatic cancer for pain relief, as the
condition of the patient refers to recurrence. Depending on the severity, about
40%-70% of patients find relief. If the pain recurs, then retreatment with radiation
is still recommended.
• Retreatment adhering to normal tissue-dosing constraints should be considered
for patients with persistent or recurrent pain from peripheral or spinal bone
metastases for more than 1 month after external-beam radiotherapy.
• The latest study shows that the use of radionuclides, surgery, or bisphosphonates
does not reduce the need for external beam radiation therapy.
Question 28
• Question 28: A 23-year-old woman, with no significant previous medical history,
presents with post-coital bleeding and vaginal discharge for approximately 2
months. She reports being monogamous, and she first experienced sexual
intercourse at the age of 19. She does not have a family history of cancer or
diethylstilbestrol (DES) exposure. A 4-centimeter polypoid mass at her cervix
was detected during a pelvic examination. A cervical biopsy and a conventional
pap smear were obtained. The cytological specimen was negative for
intraepithelial lesions or malignancy (NILM). The histopathological specimen
shows cervical tissue involved with malignant infiltration consisting of large
round cells with clear cytoplasm and glandular formation.
The specimen also shows some hobnail tumor cells. Based on the underlying
diagnosis, which of the following is the most likely predisposing risk factor?
• Choices:
• 1. Early menarche
• 2. DES (Diethylstillbosterol) exposure in utero
• 3. Late menopause
• 4. Smoking
Answer: 2
• Answer: 2 - DES (Diethylstillbosterol) exposure in utero
• Explanations:
• This patient exhibits the typical clinical presentation and histopathological features characteristic of clear cell
adenocarcinoma of the cervix.
• This is a malignant neoplasm composed mainly of clear or hobnail cells arranged in solid, tubulocystic or papillary
patterns. This rare tumor is histologically similar to clear cell adenocarcinoma of the ovary, endometrium, and vagina
where they are more common. There are two groups of patients with clear cell carcinoma of the cervix, those related
to DES exposure in utero and the other group without intrauterine
• DES exposure.
• The diagnosis of clear cell adenocarcinoma of the cervix can only be established based on the histopathological
examination of the biopsy or surgical specimen since clinical and radiological findings are not specific.
• Vaginal bleeding is the most common initial symptom of clear cell adenocarcinoma of the cervix. However, vaginal
bleeding is not specific for clear cell adenocarcinoma. In this case, the patient presented with post-coital bleeding and
vaginal discharge for approximately two months.
• Pap smear is less efficient in diagnosing clear cell adenocarcinoma of the uterine cervix, as it was the case in this
Question 29
• Question 29: A young woman presents with a 3 cm left neck mass. She does not
report any other symptoms. Excisional biopsy of the mass reveals a uniform
population of cells consisting of few large cells, lymphocytes, eosinophils, and
plasma cells. Immunohistochemistry reveals positive CD 15 and CD 30 cells in
the majority of the large cells. What is the most likely diagnosis?
• Choices:
• 1. Sarcoidosis
• 2. Tuberculosis
• 3. Hodgkin lymphoma
• 4. Acute lymphoblastic leukemia
Answer: 3

• Answer: 3 - Hodgkin lymphoma Explanations:


• Hodgkin lymphoma usually presents with large owl-eye nuclei or Reed-
Sternberg cells. Most cells in Hodgkin lymphoma are uniform in size. The
malignancy presents between 15 and 34 years of age or after 55 years of age.
Symptoms of Hodgkin lymphoma may include weight loss, pruritus, low-grade
fever, and loss of appetite. The four classic types of Hodgkin lymphoma are
nodular sclerosis, mixed cellularity, lymphocyte depletion, and lymphocyte-rich.
Nodular lymphocyte predominant is treated differently.
Question 30
• Question 30: A 58-year-old female presents to the clinic for a follow up of
abnormal mammogram findings in the upper outer quadrant of the left breast.
Ultrasound-guided biopsy reveals a 3 cm mass positive for invasive ductal
carcinoma, grade 2, ER-positive, PR positive, Her2 negative.The patient wishes
to discuss breast conservation therapy. Which of the following is the best
treatment choice for this patient?
• Choices:
• 1. Simple mastectomy with whole breast radiation
• 2. Partial mastectomy, sentinel lymph node biopsy, and whole breast radiation
• 3. Simple mastectomy with sentinel lymph node biopsy
• 4. Partial mastectomy with sentinel lymph node biopsy, no radiation
Answer: 2
• Answer: 2 - Partial mastectomy, sentinel lymph node biopsy, and whole breast radiation
• Explanations:
• The patient has a T2 lesion and is, therefore, a candidate for breast conservation therapy.
• All patients undergoing partial mastectomy should have sentinel lymph node mapping
performed for proper axillary staging.
• Breast conservation therapy has shown to have equivalent survival compared to
mastectomy when combined with whole breast radiation. If a patient cannot undergo
postoperative radiation, this is considered a contraindication to breast conservation
therapy.
• Radiation may be omitted from patients over 70 years old who take are candidates and take
adjuvant tamoxifen.
Question 31
• Question 31: A 62-year-old female presents for assessment for hospice
services. The patient has a history of lung cancer with debilitating abdominal
and bone pain due to metastasis severely impacting her quality of life. After
discussing the matter further with the palliative physician ,the patient chooses
to proceed. The appropriate paperwork is filed, and four days later, a rejection
letter is received by the clinic. The submitted forms describe the patient as a
"62-year-old female with lung cancer and bone cancer, with significant pain,
and a prognosis of five months."
• What correction to the paperwork is most likely to resolve the near-
immediate cause of rejection?
Choices:
• 1. List the patient's age with days in addition to specifying years
• 2. Write an addendum which describes how the patient's pain has
prevented her from running a half-marathon, one of her life goals
• 3. More thoroughly document the patient's diagnoses, including
designation as primary and secondary neoplasms
• 4. Nothing can be done at this time as the patient's prognosis is beyond
hospice eligibility
Answer: 3
• Answer: 3 - More thoroughly document the patient's diagnoses, including designation as primary and secondary
neoplasms
• Explanations:
• Hospice eligibility depends on more than only a physician determining a prognosis of six months or less until
death. The careful and thorough documentation of hospice appropriate diagnoses is also vital to obtain approval
for hospice services.
• When describing hospice appropriate diagnoses in Centers for Medicare & Medicaid Services (CMS) paperwork,
it is important to designate primary versus secondary neoplasms as appropriate to avoid delay in acceptance.
• Documentation should aim to be complete, clear, and factual regarding the patient’s circumstances and the
impact their diagnoses have on their prognosis.
• Hospice appropriate diagnoses involve conditions which will lead to the patient's death, as well as impact the
patient's ability for activities of daily living and requirement of symptom management. Whether or not
diagnoses impact the ability of patients to achieve life-long goals(such as running a half-marathon) does not
directly impact hospice service eligibility.
Question 32

• Question 32: A 65-year-old Asian male presents with a longitudinal band of


hyperpigmentation down the center of his right thumb nail. It extends to the
proximal nail fold and has been progressively widening and getting darker in the
last few months. Where do you want to biopsy?

• Choices
• 1. Nail fold
• 2. Nail plate
• 3. Nail bed
• 4. Nail matrix
Answer: 4
• Answer: 4 - Nail matrix
• Explanations:
• created an ABCDEF rule to aid in the clinical diagnosis of subungual melanoma.
Concerning features include:
• A – Age 50 to 70years-old; Asian/African American/Native American
• B – Brown and Black band; Breadth equal to or greater than 3 mm; Borders
irregular
• C– Change in nail band; or no Change with treatment
• D – Digit affected (thumb more than great toe more than index finger)
• E – Extension of pigment to nail fold (Hutchinson’s sign)
• F – Family or personal history of melanoma or dysplastic nevi
:Continue
• Hutchinson sign is an extension of pigment to the nail fold, and it raises suspicion
for melanoma. Biopsy solely of this feature can be deceptively encouraging.
• The nail plate originates from the nail matrix, so pigment in the plate is derived from a
lesion at the nail matrix. For longitudinal melanonychia, the biopsy must include the nail
matrix.
• A longitudinal biopsy would be another option if this lesion were located near a lateral
edge of the nail.
• Collectively, the nail unit has a lower density of melanocytes than normal skin.
Compared to the rest of the nail unit, though, the nail matrix has the highest
concentration of melanocytes.
Question 33
• Question 33: A patient with a mediastinal mass undergoes a lymph node
biopsy. The pathologist reports that certain nuclei have an owleye appearance.
What is the most likely diagnosis?
• Choices:
• 1. Malaria
• 2. Tuberculosis
• 3. Cytomegalovirus
• 4. Hodgkin's lymphoma
Answer: 4

• Answer: 4 - Hodgkin's lymphoma


• Explanations:
• Owl eye appearance is often referred to as the appearance of a bilobed nucleus
with prominent eosinophilic inclusion-like nucleoli.
• These cells are often known as Reed Sternberg cells and seen in patients with
Hodgkin lymphoma.
• Reed Sternberg cells are large, multinucleated, and have a bilobed nucleus
resembling an owl eye.
• Reed Sternberg cells are often CD 30 and CD 15 positive.
Question 34
• Question 34: A 50-year-old male has a squamous cell carcinoma of the
right side of the laryngeal surface of the epiglottis. A 3 cm node is palpable
on the left side of the neck.
• What is the neck staging of this tumor?
• Choices:
• 1. N1
• 2. N2
• 3. N2a
• 4. N2c
Answer: 4

• Answer: 4 - N2c
• Explanations:
• According to the tumor, node, metastases classification by the American Joint
Committee on Cancer, N2c represents "bilateral orcontralateral positive nodes,
none more than 6 cm in diameter."
• Therefore, the neck staging is N2c because the node is on the contralateral side
of the tumor.
• N2a is metastasis in a single ipsilateral lymph node which is more than 3 cm but
not more than 6 cm at its greatest dimension.
• N2b is metastasis in multiple ipsilateral lymph nodes, but none are more than 6
cm at there greatest dimension.
Question 35

• Question 35: A 65-year-old female patient was diagnosed with paranasal sinus
cancer (adenoid cystic cancer) and treated with surgicaexcision and
postoperative radiotherapy (70Gy). One month after the completion of
radiotherapy, she developed an acute or cutaneous fistula. What is the most
appropriate management strategy in this case?
• Choices:
• 1. Incision and drainage of the fistula
• 2. Repair with prosthetic closure
• 3. Repair with surgical intervention
• 4. Antibiotic courses
Answer: 2
• Answer: 2 - Repair with prosthetic closure
• Explanations:
• Cutaneous fistula after radiotherapy is a therapeutic challenge. High doses (60
Gy or higher) are usually prescribed in patients with paranasal sinus cancer, and
therefore, irreversible tissue damage is associated.
• Prosthetic closure is a convenient alternative to surgical repair in this patient.
The aim of the treatment is to minimize infection risk and cosmetic burden
associated with the fistula.
• Prostheses could be used intraorally, extra orally, or as a combination.
• Such or cutaneous fistulas are frequently repaired with surgical intervention. But
surgical repair is difficult in this case because the fistula appeared in the
immediate post-therapy period.
Question 36

• Question 36: A 7-year-old girl presents to the clinic for the evaluation of
intermittent vaginal bleeding. Laboratory tests revealed an undetectable
estradiol level and low gonadotropin levels. On physical examination, pubic or
axillary hair is absent. A vaginoscopy is carried out to evaluate the cause of
vaginal bleeding. At the time of vaginoscopy, a polypoid pedunculated mass is
identified arising from the anterior lip of the cervix. The lesion is very friable with
a narrow stalk, and it is excised at its base. Clinically, botryoid
rhabdomyosarcoma was the suspected diagnosis. However, intraoperative frozen
section evaluation revealed a turboelectric and a papillary configuration with
focal areas of necrosis. The papillae were mostly lined by a single layer of tumor
cells having abundant clear cytoplasm with a centrally located round to oval
nuclei. Which of the following is the most likely diagnosis in this patient?
• Choices:
• 1. Endometrioid adenocarcinoma
• 2. Clear cell adenocarcinoma
• 3. Serous adenocarcinoma
• 4. In situ adenocarcinoma
Answer: 2

• Answer: 2 - Clear cell adenocarcinoma


• Explanations:
• Clear cell carcinoma of the cervix typically causes prolonged vaginal bleeding. When vaginal bleeding
occurs in a young woman or a child, itis often mistaken for anovulatory bleeding or precocious puberty.
• The etiology and pathogenesis of clear cell carcinoma of the cervix are unclear. However, many reports
have associated this cancer subtype with prior intrauterine exposure to diethylstilbestrol (DES), a synthetic
nonsteroidal estrogen hormone and teratogen with the ability to cross the placenta.
• The diagnosis of clear cell adenocarcinoma of the cervix can only be established based on
histopathological examination of the biopsy or surgical specimen since clinical and radiological findings are
not specific.
• Histologically, there are three growth patterns in clear cell adenocarcinoma; turboelectric, papillary, and
solid. The cells can range from flat to hobnail with mild to severe atypia. The cytoplasm can be optically
clear or eosinophilic. Mitotic figures are infrequent.
Q u estio n 3 7

• Question 37: A 65-year-old male presented to a clinic with a chief complaint of


bilateral heel ulcers and pain. The patient was diagnosed with polycythemia
Vera two years ago and has been on hydroxyurea treatment with good
tolerance. On physical exam, his blood pressure is120/70 mmHg, heart rate is
70 bpm, respiratory rate is 12 breaths/min, and SpO2 is 98% at room air. On the
right peri malleolar area, there is a 1cm stage 1 ulcer. Laboratory investigations
revealed a hemoglobin of 14.5 gm/dl, hematocrit of 42 %, normal platelet
count, and normal white blood cell count. His hemoglobin A1c is 5.4. The
bilateral ankle-brachial index is 1.1. Histological examination showed
perivascular leukocyte infiltration without vasculitis.

• What is the most likely etiology of the skin ulcer?


• Choices:
• 1. Sequelae of diabetes
• 2. Cardiovascular
• 3. Drug toxicity
• 4. Hematological
Answer: 3
• Answer: 3 - Drug toxicity
• Explanations:
• Long term therapy with hydroxyurea in myeloproliferative disorders results in
cutaneous side effects.
• It is known to cause ulcerative, non-ulcerative skin lesions, and gangrene of
toes.
• It is also known to cause cutaneous carcinomas. Patients under hydroxyurea
therapy require long-term monitoring.
• In patients with cutaneous side effects, discontinuation of the drug is
recommended.
Question 38
• Question 38: A 64-year-old male is evaluated in the clinic for a rash of one-day
duration. The patient has a history of coronary artery disease and non-small
cell lung cancer. He takes ASA 81 mg daily, atorvastatin 80 mg daily, metoprolol
succinate 50 mg daily, and afatinib 40 mg daily. Vital signs are significant for
body temperature of 99.1 F, blood pressure 115/75 mm Hg, respiratory rate
18/min, heart rate 69 bpm. The physical exam is significant for several
maculopapular rashes on chest and abdomen, multiple small blisters on upper
extremities otherwise unremarkable. Patient denies any fever, chills, night
sweats.

• Which one is the most appropriate initial step in the management of this
patient?
• Choices:
• 1. Discontinue ASA and atorvastatin
• 2. Continue the same medication regimen and check back in two weeks
• 3. Discontinue afatinib
• 4. Send the patient home with an antihistamine prescription without any
change in medications
Answer: 3
• Answer: 3 - Discontinue afatinib
• Explanations:
• Dermatologic reactions are reported to be the most common side effects of tyrosine
kinase inhibitors.
• Afatinib is recommended to be discontinued in case of any serious and life-threatening
bullous, blistering, or exfoliative skin lesions.
• This patient has developed several maculopapular rashes on chest and abdomen, multiple
small blisters on upper extremities, likely secondary to afatinib, thus afatinib should be
discontinued immediately with immediate oncology follow up.
• Impaired left ventricular function, bullous eruption, hand-foot syndrome, severe diarrhea,
hepatotoxicity, and interstitial lung disease are some of the adverse effects of afatinib.
However, afatinib is reported to have a predictable and manageable side effect profile
based on a
• Keating et al. study.
Question 39
• Question 39: A 65-year-old female presents with complaints of pain in the lower
abdomen. She denies any vaginal discharge but says she has constant pain in the
lower abdomen. This has been ongoing for a few months. She has lost weight
and has no appetite. Ultrasound reveals that she has a large mass in the ovary.
Biopsy reveals that she has a Krukenberg tumor.
• Her primary tumor resides in which organ?
• Choices:
• 1. Kidney
• 2. Lung
• 3. Skin
• 4. Stomach
Answer: 4
• Answer: 4 - Stomach
• Explanations:
• The Krukenberg tumor is adenocarcinoma metastasized to the ovary.
• Most commonly the cancer is from the stomach, but it can originate from anywhere in the
gastrointestinal tract, breast, or bladder.
• The stomach is the primary site for this tumor 70% of the time.
• The stomach and colorectal site together account for 90% of this tumor.
Question 40

• Question 40: A 67-year-old female presents with a 3-cm mass in her left breast
that has been confirmed to be malignant. The surgeon recommends
lumpectomy, axillary node dissection, and postoperative radiation treatment.
What is the current status of axillary node dissection in patients with breast
cancer undergoing a lumpectomy and radiation?
• Choices:
• 1. The procedure improves outcome
• 2. The procedure improves survival
• 3. The procedure has not been shown to improve outcome or survival
• 4. Axillary node dissection is superior to sentinel lymph node biopsy
Answer: 3
• Answer: 3 - The procedure has not been shown to improve outcome or survival
• Explanations:
• Studies have shown that in women undergoing breast conservation therapy
with lumpectomy and postoperative radiation, axillary lymph node dissection
did not significantly affect overall or disease-free survival.
• The morbidity from axillary lymph node dissection has led researchers to study
alternative axillary treatment strategies, such as observation, radiation, and
systemic chemotherapy.
• These methods have demonstrated low axillary failure rates and no significant
difference in overall survival.
• Sentinel lymph node biopsy is associated with less morbidity than axillary lymph
node dissection and can offer excellent regional control in select patients with
early-stage breast cancer.
Question 41
• Question 41: A 15-year-old boy presents to the clinic for follow up. The night before, he
was seen in the emergency department for nasal congestion, cough, and fever and was
diagnosed with a viral upper respiratory infection. The patient's clinic chart shows a 4 kg
weight loss over the past year. Earlier during the winter, the patient reported that he felt
his lungs had decreased capacity during band practice. He had these symptoms for one
month, but then they resolved. Physical examination shows a thin, pleasant boy. His
weight is 61 kg, the temperature is 37.6 C, and blood pressure is 110/68 mmHg. Heart
rate is 125 beats/min, but the cardiac examination is otherwise normal. A chest x-ray
reveals a large anterior mediastinal mass. Biopsy confirms the diagnosis and the patient
is started on chemotherapy.

• Which of the following cells are the most likely expected on the patient's biopsy?
• Choices:
• 1. Hurthle cells
• 2. Reed-Sternberg cells
• 3. Kupffer cells
• 4. Langerhans cells
Answer: 2

• Answer: 2 - Reed-Sternberg cells


• Explanations:
• Hodgkin and nonHodgkin lymphoma account for 10-15% of childhood cancers.
They are the most common type of cancer in children, after
• leukemia and brain tumors. The nonspecific symptoms seen during the
presentation of Hodgkin lymphoma include weight loss, fatigue, fevers, anorexia
and night sweats.
• A chest radiograph should always be considered if Hodgkin disease is part of a
clinician's differential diagnosis. If a mass is seen, a biopsy should be performed.
• In Hodgkin's lymphoma, the hallmark tumor cells are Reed-Sternberg cells.
• The cure rates for pediatric Hodgkin's lymphoma using multiagent
chemotherapy and radiation therapy are amongst the highest in pediatric
oncology.
Question 42
• Question 42: A 68-year-old Asian male with a past medical history of alcohol use disorder,
active tobacco smoker, and obesity presents to the clinic for a regular follow up visit. He was
recently admitted for alcohol-induced hepatitis. He was discharged 2 days back with oral
steroids. He is concerned about his risk of liver cancer development in the future.

Which of the following is the correct information to educate the patient?


• Choices:
• 1. Diet modifications such as vegetables, white meat, and fish can reduce the risk of liver cancer
• 2. Male are less prone than female to develop liver cancer
• 3. Young adults are more prone to develop liver cancer than their older counterparts
• 4. There is no racial predisposition to liver cancer development
Answer: 1
• Answer: 1 - Diet modifications such as vegetables, white meat, and fish can reduce the risk of liver cancer
• Explanations:
• A population-based study and meta-analysis suggested that consumption of white meat is associated with
reduced risk. In addition, the consumption of fish, omega-3 fatty acids, and vegetables have been associated
with reduced risk. Higher dietary intake of vitamin E has also been associated with a decreased risk of
hepatocellular carcinoma (HCC) among patients both with and without a self-reported history of
• liver disease, or a family history of HCC. Some studies have suggested that consumption of red meat and
saturated fat are associated with an increased risk of HCC.
• Men who are Hepatitis B surface antigen (HBsAg) positive appear to be at increased risk for HCC compared with
women. In a prospective cohort study that followed 23,820 residents of Taiwan who were between the ages of
30 and 65, the cumulative lifetime incidence of HCC for patients who were positive for HBsAg was higher in men
than in women (27 versus 8 percent).
• The majority of hepatocellular carcinoma occurs in patients with chronic liver disease or cirrhosis. Thus, older
patients with long-standing
• liver disease are more likely to develop HCC.
• Population-based studies in the United States have identified racial and ethnic variations in the incidence of
HCC, and they conclude that
• Asians/Pacific Islanders (APIs) have higher rates of HCC compared with other groups.
Question 43

• Question 43: A 56-year-old woman undergoing chemotherapy for diffuse large B-cell lymphoma
presents at the hospital for several days of fever of unknown origin. She states that her temperature
at home was 101.5F and is 101.7F in the hospital today. She denies any other associated symptoms
other than just "feeling bad," which she relates to being caused by the chemotherapy. Labs are
drawn, and her CBC is notable for an absolute neutrophil count (ANC) of 396/microliter.

• What is most likely responsible for the findings in this patient?


• Choices:
• 1. Brain
• 2. Heart
• 3. Kidneys
• 4. Bone marrow
Answer: 4
• Answer: 4 - Bone marrow
• Explanations:
• This patient is presenting with a case of neutropenic fever secondary to her treatment
with cytotoxic chemotherapeutic agents.
• Neutropenic fever is a condition in which a patient has a temperature of greater than
101F or greater than 100.4F for one hour in the setting of a low neutrophil count.
Neutropenia is defined as an absolute neutrophil count below 500/microliter or below
1000/microliter with an anticipation of less than 500/microliter within 48 hours.
• Neutropenia in the setting of chemotherapy is often the result of bone marrow
suppression, as these agents target rapidly dividing tissues such as bone marrow.
• Bone marrow is responsible for producing erythrocytes, leukocytes, such as neutrophils,
and thrombocytes which create platelets.
Question 44
• Question 44: A 65-year-old female presents to the clinic with chronic right upper
quadrant pain. The pain has been present for the past 2 weeks, and it has been
associated with occasional nausea and vomiting. Vital signs are within normal limits.
Her past medical history is unremarkable. An MRI of the abdomen demonstrates a
non-enhancing cystic lesion in segment IV of the liver.
• What is the most dangerous complication associated with the sampling in this patient
with the presumed diagnosis?
• Choices:
• 1. Cyst rupture
• 2. Peritoneal carcinomatosis
• 3. Intracystic hemorrhage
• 4. Infection
Answer: 2
• Answer: 2 - Peritoneal carcinomatosis
• Explanations:
• Fine-needle aspiration of cyst fluid is no longer recommended due to the risk of
dissemination of cells and the development of peritoneal carcinomatosis if the
lesion is, in fact, carcinoma.
• The definitive diagnosis of biliary cystadenoma (BCA) and cystadenocarcinoma
is challenging preoperatively, even with image-guided aspiration or sampling.
• The role of cyst fluid analysis for carcinoembryonic antigen (CEA) and CA 19-9
remains controversial as they are neither sensitive nonspecific.
• Cyst rupture, intracystic hemorrhage, and infection are all complications that
can occur in BCAs; however, they are not the most dangerous.
Question 45

• Question 45: A 38-year-old female presents to the clinic for a routine visit.
During the physical exam, a hard lump is discovered on her left neck. The mass
is solid and does not move during swallowing. She denies smoking but claims
she received a small dose of radiation for acne many years ago.
• What is the most likely cause of her mass?
• Choices:
• 1. Papillary carcinoma
• 2. Goiter
• 3. Medullary thyroid cancer
• 4. Thyroglossal duct cyst
Answer: 1
• Answer: 1 - Papillary carcinoma
• Explanations:
• Papillary carcinoma is the most common thyroid cancer and accounts for nearly
60% to 75% of all thyroid cancers.
• Females are affected two to three times more often than males. It is more
common and often remains localized in the young, but is more malignant and
invasive in the elderly.
• Papillary cancer of the thyroid is more common in patients with a history of
radiation exposure. It spreads via the lymphatic system.
• The tumor usually shows as a "cold nodule" on thyroid scintigraphy, and a
needle aspiration may provide a clue to the disorder. Treatment of this condition
is surgical resection, preferably total thyroidectomy, followed by radio-iodine
ablation and thyroid-stimulating hormone suppressive therapy.
Question 46
• Question 46: A 65-year-old female presents to the clinic with complaints of vaginal
bleeding, vague abdominal pain, and nausea. She says she has not been feeling well for
the past few months. She denies any weight loss and has not seen a healthcare provider
for at least 5 years. She has a 30 pack-year history of smoking and drinks three to four
alcoholic beverages daily. She is on no medications and denies any allergies. A physical
exam is unremarkable except for the presence of blood clots at the cervical os.
• Which of the following risk factors would raise the suspicion of uterine cancer?
• Choices:
• 1. Obesity
• 2. Human papillomavirus infection
• 3. Multiple sex partners
• 4. Endometriosis
Answer: 1
• Answer: 1 - Obesity
• Explanations:
• Uterine cancer is not rare and is associated with excess estrogen.
• Exogenous sources of estrogen include tamoxifen use or unopposed estrogen
replacement therapy.
• Tamoxifen increases the risk of endometrial cancer due to its ability to
stimulate the estrogen receptors found on the endometrium.
• Endogenous sources of estrogen include polycystic ovary syndrome, obesity, or
estrogen-secreting tumors such as granulosa cell tumors.
Question 47
• Question 47: A 49-year-old female presents to the clinic with complaints of a right breast lump
with unilateral bloody nipple discharge for seven months. On examination, a well-delineated
mass is present in the upper and outer quadrant of the right breast measuring 6x5 cm. An
anterior axillary lymph node is palpable measuring 2x2 cm. The patient undergoes radical
mastectomy with axillary lymphadenectomy. Histopathological analysis of the mass disclosed
a well-circumscribed lesion showing pleomorphic cells arranged in sheets and cords. The
tumor cells were round or oval with hyperchromatic and prominent nucleoli. The sheets of
cells exhibited pushing borders without evidence of any infiltration. There was a prominent
and diffuse lymphoplasmacytic infiltrate in the stroma with no evidence of vascular invasion.
All the margins and base of the specimen were free of malignant cell infiltration. On
immunohistochemistry, the tumor was triple negative.

• Which of the following is the most likely diagnosis?


• Choices:
• 1. Invasive carcinoma of no special type
• 2. Medullary carcinoma
• 3. Large B-cell lymphoma
• 4. Lymphoepithelioma-like carcinoma
Answer: 2
• Answer: 2 - Medullary carcinoma
• Explanations:
• Classically, the following criteria are used to define medullary carcinoma:1. Syncytial architecture
in greater than 75% of the tumor mass. 2.
• Histological circumscription or pushing margin. 3. Lack of tubular differentiation. 4. Diffuse and
prominent lymphoplasmacytic stroma infiltrate. 5. Round cells with abundant cytoplasm and
pleomorphic high-grade vesicular nuclei containing one or several nucleoli.
• Medullary carcinomas are most often negative for estrogen receptors, progesterone receptors,
and HER2 (human epidermal growth factor-2) and, variably express keratins 5/6 and 14, smooth
muscle actin, EGFR (epidermal growth factor receptor), P-cadherin, p53, and caveolin-
• Most of the patients with medullary carcinoma present with a palpable mass, usually in the upper
outer quadrant. The tumor is often welldefined clinically and on imaging studies. Axillary
lymphadenopathy is seen at the time of presentation in some patients, suggesting the presence
of metastatic disease.
• The treatment for medullary carcinoma, either typical or atypical is similar to invasive ductal
carcinoma. It includes modified or radical mastectomy along with radiation or chemotherapy
depending on the stage.
Question 48
• Question 48: A 42-year-old woman presents to the clinic to assess options for
her cancer-related pain. The patient currently works as a museum curator and
has a history of Von-Willebrand disease, hypertension, diabetes, liver failure,
and breast cancer that has spread. Her vital signs are all within normal
reference ranges, and her pain is scaled to level 3 on the World Health
Organization (WHO) cancer ladder. The patient states that she has thus far
failed the trials of pain pumps. Her pelvic pain is moderate, but the pain in her
right leg is making it difficult for her to work. She is interested in definitive,
permanent options to palliate her pain.

• Which of the following is the most appropriate approach to this patient?


• Choices:
• 1. Open bilateral cordotomy; if hypertension can be controlled, proceed
with the procedure as the patient wants definitive palliation of her pain
• 2. Right unilateral percutaneous cervical cordotomy; if blood glucose can
be adequately controlled, proceed with the procedure to target the
• patient's leg pain
• 3. Punctate midline myelotomy; if the patient's pain remains at level 3 for
2-weeks, proceed with the procedure to address the patient's pelvic pain
• 4. The patient is not a surgical candidate for the intervention which
would meet her needs
Answer: 4

• Answer: 4 - The patient is not a surgical candidate for the intervention which would
meet her needs
• Explanations:
• The cordotomy leads to the nociceptive pathways in the lateral spinothalamic tract
anterolateral column being destroyed in the cervical spinal cord at the level of C1-C2.
• Percutaneous cordotomy is contraindicated in patients with a coagulation disorder
(such as patients with liver failure and Von Willebrand disease).
• Patients must be at level 3 of pain on the World Health Organization (WHO) cancer
ladder with a prognosis of under six months to be a candidate for cordotomy.
• In otherwise healthy patients without contraindications, risk of severe complications
with unilateral cordotomy is low.
Question 49

• Question 49: An 82-year-old male is brought to the emergency department with


complaints of fatigue, constipation, anorexia, and confusion. He has a 35-pack-year
smoking history and says he was told six months ago that he has some sort of cancer, but
did not think much of it and has not received any treatment. On review of his medical
records, a biopsy report confirming squamous cell carcinoma of the lung is found.

• Which of the following is most often associated with this patient's diagnosis?
• Choices:
• 1. SIADH
• 2. Lambert Eaton syndrome
• 3. Cushing Syndrome
• 4. Hypercalcemia
Answer: 4
• Answer: 4 - Hypercalcemia
• Explanations:
• Lung cancer is associated with a variety of paraneoplastic syndromes.
• Most commonly squamous cell lung cancer is associated with hypercalcemia.
• Hypercalcemia usually results from the secretion of PTHrP or parathyroid
hormone-related protein.
• SIADH is more commonly associated with small cell lung cancer as is Lambert-
Eaton syndrome.
Question 50

• Question 50: A 65-year-old male presents with worsening headaches, double


vision, along with left facial numbness and tingling for the past 3 weeks. Further
review of systems is positive for a right-sided hemiparesis and testicular
swelling. He has a past medical history of non- Hodgkin’s lymphoma for which
he is currently in remission after completing treatment 8 months ago. He was
recently diagnosed with bronchogenic adenocarcinoma for which he is
undergoing treatment. An unenhanced head CT shows a high-density mass.
Which of the following is needed to obtain a definitive diagnosis?
• Choices:
• 1. MRI brain with gadolinium contrast
• 2. Stereotactic brain biopsy
• 3. Methionine PET scan
• 4. Breast biopsy
Answer: 2
• Answer: 2 - Stereotactic brain biopsy
• Explanations:
• Approximately 10% to 15% of patients with systemic non-Hodgkin's lymphoma experience CNS
involvement usually within the first year after diagnosis of systemic disease.
• High-risk features of systemic non-Hodgkin’s lymphoma that increases the likelihood of CNS
dissemination include the diffuse large B cell histological subtype, presence of extranodal disease as
well as testicular involvement.
• The leptomeninges is often the primary site of involvement in two-third of cases, while one-third of
lesions present intraparenchymal.
• Patients can present with cranial or spinal neuropathy and headaches. Intraparenchymal lesions are
often periventricular or superficial.
• MRI is often the first imaging modality to detect CNS involvement by systemic disease but cannot
differentiate lymphoma from other metastases. A stereotactic biopsy is usually needed for definitive
diagnosis. The diffuse large B cell variant is often the most common histological subtype seen.
Question 51
• Question 51: A patient with bone cancer is being treated with a long-acting
morphine formula. He presents with vague complaints of abdominal
distension, nausea, hard stools, and straining. The working diagnosis is opioid-
induced constipation. The Bristol stool chart is used to characterize his
constipation.
• The chart uses which criteria to assess constipation?
• Choices:
• 1. Stool size
• 2. Stool color
• 3. Stool form
• 4. Stool weight
Answer: 3
• Answer: 3 - Stool form
• Explanations:
• The Bristol stool form scale, or Bristol stool chart, can be used to classify the
form of feces into seven categories.
• Type 1: sperate hard lumps, like nuts (hard to pass). Type 2: sausage-shaped but
lumpy. Type 1 indicates severe constipation. Type 2 indicates mild constipation.
• Type 3: like a sausage but with cracks on its surface. chart 4: like a sausage or
snake, smooth and soft. Types 3 and 4 indicate normal stools.
• Type 5: soft blobs with clear-cut edges (passed easily). Type 6: fluffy pieces with
ragged edges, a mushy stool. Type 7: water, no solid pieces, entirely liquid. Type
5 indicates a lack of fiber in the diet. Type 6 indicates mild diarrhea. Type 7
indicates severe diarrhea.
Question 52
• Question 52: A patient has a 3 cm, mobile, group II lymph node. Fine-needle
aspiration reveals squamous cell carcinoma. The physical examination and
radiographic evaluation reveal no evidence of a primary site of lesion and no
distant metastases.
• Which of the following is most likely the primary site?
• Choices:
• 1. Nasopharynx
• 2. Floor of mouth
• 3. Base of tongue
• 4. Infrahyoid epiglottis
Answer: 3
• Answer: 3 - Base of tongue
• Explanations:
• The primary site is detected in approximately 40% of patients presenting with a
metastatic upper cervical lymph node from an unknown primary site. Specifically, in the
head and neck of patients who have undergone a modern evaluation with flexible fiber-
optic endoscopy, CT or MRI of the head and neck, and directed biopsies.
• Approximately 40% of detected primary sites are found in the base of the tongue and
40% in the tonsillar fossa.
• With tonsillar cancer, metastasis to regional lymphatics is a frequent occurrence. Neck
metastases are seen in 50% to 65% of patients. Even in patients with a clinically negative
physical exam, 30% to 40% will have neck disease.
• Tonsillar squamous cell cancer also may spread to the retropharyngeal lymph nodes.
Question 53
• Question 53: Which of the following is a risk factor for chest wall recurrence?
• Choices:
• 1. Tumor size of 4 cm or involved axillary nodes
• 2. Unifocal disease
• 3. Nipple involvement
• 4. ER-positive status
Answer: 1
• Answer: 1 - Tumor size of 4 cm or involved axillary nodes
• Explanations:
• Katz et al. found that patients with tumors greater than or equal to 4 cm or at
least four involved nodes experience local recurrence rates in excess of 20%.
• Gross multifocal/multicentric disease is more likely to lead to recurrent
disease.
• ER-positive hormone status permits the use of selective estrogen receptor
modulators, drugs whose benefits in the treatment of breast cancer outweigh
their relatively low toxicity.
• Age younger than 40 and genetic predispositions to breast cancer are
associated with risk of recurrent breast cancer.
Question 54
• Question 54: A 20-year-old female presents to the emergency department with
complaints of a sore throat for a week accompanied by intermittent fevers. On
physical exam, there are swollen lymph nodes in the neck, and the patient has
an enlarged spleen. The lymph nodes are greater than 1 cm in size. The patient
tells you that her primary care provider did something called a mono spot test
and it was negative. A lymph node biopsy reveals the presence of large cells
with bilobed nuclei and eosinophilic inclusion body like nucleoli.

• Which of the following viruses is most likely responsible for the patient's
condition?
• Choices:
• 1. Human papillomavirus
• 2. Rabies
• 3. Measles
• 4. Epstein Barr virus
Answer: 4

• Answer: 4 - Epstein Barr virus


• Explanations:
• 40% of Reed-Sternberg cells are infected with Epstein Barr virus.
• Epstein Barr virus infects naive B cells that undergo germinal center reactions
so the virus ultimately resides in memory B cells. Lymphomas associated with
EBV are of germinal center B cell origin.
• EBV positive patients have an increased risk of Hodgkin lymphoma when
compared to EBV negative patients.
• EBV infected B cells acquire the morphologic and immunophenotypic
characteristics of Reed-Sternberg cells although the mechanism of how this
occurs is not well understood.
Question 55
• Question 55: A 67-year-old patient presents to the clinic two years after a right
modified radical mastectomy for T3N0 breast cancer. She has occasional pain in
her right hand, and in the office, the diameter of her right arm is 2 cm larger
than her left.
• What is the best initial management of this patient's problem?
• Choices:
• 1. Do nothing as this complication will resolve spontaneously
• 2. Perform a lymph node transfer
• 3. Perform a lymphaticovenular anastomosis
• 4. Refer the patient to a lymphedema specialist for evaluation, compression and
sleeve fitting
Answer: 4
• Answer: 4 - Refer the patient to a lymphedema specialist for evaluation,
compression and sleeve fitting
• Explanations:
• This patient has developed lymphedema which is a common complication
after axillary lymph node dissection. It will probably not resolve on its own.
• This patient should be referred to a lymphedema specialist for education,
exercise, and compression.
• Compression is most commonly performed with fitted compressive garments.
• Lymph node transfer is performed as a last resort. This patient should first be
referred to a specialist for conservative management. A
• lymphaticovenular anastomosis is performed as a last resort. This patient
should first be referred to a specialist for conservative management.
Question 56
• Question 56: While receiving chemotherapy for metastatic ovarian carcinoma, a
patient begins to experience discomfort at the site of infusion shortly after the
infusion is started. She describes the pain as a burning sensation. On
examination, there is localized swelling and erythema at the port site.
What is the next best step in the management of this patient?
• Choices:
• 1. Reposition needle/catheter and continue infusion
• 2. Administer IV hydration and continue infusion
• 3. Stop the infusion and apply a warm compress to the site
• 4. Immediately stop the infusion and aspirate the fluid
Answer: 4
• Answer: 4 - Immediately stop the infusion and aspirate the fluid
• Explanations:
• Cisplatin has many possible adverse effects including extravasation during
administration.
• Extravasation is characterized by localized swelling, redness, burning, and
induration at the site of extravasation.
• The infusion should be stopped immediately if extravasation is suspected.
• The extravasated fluid should be aspirated to prevent damage to surrounding
tissues
Question 57
• Question 57: A 53-year-old female patient affected by breast cancer underwent
non-surgical management through chemotherapy. A few weeks after the start of
chemotherapy, she reports progressive difficulties in remembering more recent
episodes, things, or images, and trouble in learning new things. Moreover, she
describes her status as a ‘feeling of mental fogginess.’ These clinical manifestations
affect her everyday life. Which of the following is the most likely diagnosis?
• Choices:
• 1. Brain metastasis
• 2. Chemotherapy-related cognitive impairment
• 3. Postoperative cognitive decline
• 4. Delirium
Answer: 2
• Answer: 2 - Chemotherapy-related cognitive impairment
• Explanations:
• Cognitive impairment is a decline in a variety of neuropsychological tasks after chemotherapy,
in patients with non-CNS cancer. It represents the most frequent complication reported by
patients with breast cancer. Memory decline is the main CNS complication, and its impairment
may vary from short-term memory impairments to severe and permanent damages involving
recalling of visual and verbal long term memories.
• Memory decline is a source of significant concern among patients with cancer and can
negatively impact the patients and cancer survivors’ quality of life.
• Apart from memory, other cognitive functions can be involved. Patients featuring
chemotherapy-related cognitive impairment can express attention difficulties and confusion.
• The clinical picture allows excluding delirium, which is usually characterized by rapid onset
(within hours or a few days) confused thinking and reduced awareness of the environment.
Furthermore, the patient was not scheduled for surgery.
Question 58
• Question 58: A 12-year-old male presents to his provider with a 2-month history of pain in his
lower right thigh. He describes the pain as "achy" in character, moderate in severity, and
persistent in duration. On physical examination, there is a palpable mass in the distal right thigh.
He adds that it is tender. He also exhibits a limited range of motion at the knee. Radiographs of
the lower extremity reveal an osseous lesion with a radio dense matrix and a "sunburst"
appearance. Local soft tissue extension is apparent.
Which organ is the most common site of spread of the most likely lesion in this patient with
inadequate treatment?

• Choices:
• 1. Liver
• 2. Brain
• 3. Lungs
• 4. Adrenal glands
Answer: 3
• Answer: 3 - Lungs
• Explanations:
• Osteosarcoma is a lethal pediatric malignancy which most commonly occurs in the long
bones of the lower extremity. Physical exam findings may include warmth and tenderness in
the affected area, a palpable mass, and a limited range of motion when a joint is involved.
• The lungs are the most common site of extraosseous metastasis. CT imaging of the chest
may reveal pulmonary nodules with density (measured in Hounsfield units) similar to that of
bone due to the production of osteoid by the metastatic osteosarcoma cancer cells.
• The characteristic "sunburst" appearance of osteosarcoma seen on radiography is the result
of aggressive periostitis.
• Extrapulmonary metastasis related to osteosarcoma is exceedingly rare.
Question 59
• Question 59: A 60-year-old female underwent a modified radical mastectomy. On serial
sectioning, 5.5 cm × 5 cm × 2.5 cm wellcircumscribed tumor in the upper outer quadrant is
found. Microscopic examination establishes the diagnosis of medullary breast carcinoma. Four
lymph nodes dissected out of the specimen were metastatic. Immunohistochemically, the tumor
is estrogen receptor/progesterone receptor (ER/PR) and CerbB2 negative. The work-up for the
spread of breast carcinoma did not show metastases.
• Which of the following is the most appropriate pTNM classification of this tumor?

• Choices:
• 1. pT2N1M0
• 2. pT2N2M0
• 3. pT3N2M0
• 4. pT3N3M0
Answer: 3
• Answer: 3 - pT3N2M0
• Explanations:
• The most widely used clinical staging system for medullary breast carcinoma is the one
adopted by both the International Union for Cancer
• Control (UICC) and the American Joint Commission on Cancer (AJCC). It is based on the TNM
system (T, tumor; N, nodes; M, metastases).
• pT3 is defined by a tumor greater than 50 mm in the greatest dimension. The tumor, in this
case, measures 5.5 cm × 5 cm × 2.5 cm.
• pN2 is defined by metastases in 4 – 9 axillary lymph nodes. In this case, four lymph nodes
dissected out of the specimen were metastatic.
• pM0 has no clinical or radiographic evidence of distant metastases. In this case, the work-up
for the spread of breast carcinoma did not show metastases.
Question 60
• Question 60: A 16-year-old male patient presents to the hospital because of a
soft tissue protrusion from his right shoulder. He first noticed it four months ago
and is unsure if it has increased in size since. He thought it might be related to
playing football, but is concerned because it is not going away. On exam, there is
no pain on palpation, but a bruit is heard when the mass is auscultated. Imaging
demonstrates a lytic lesion with low attenuating fluid levels.
Pathology reveals empty cystic spaces resembling aneurysmal bone cysts and
septa populated by pleomorphic cells. The diagnosis is made, and the treatment is
started.
Which of the following could be the most likely outcome?
• Choices:
• 1. A moderate response to chemotherapy
• 2. No significant response to chemotherapy
• 3. Good response to chemotherapy
• 4. Good response to surgery, only utilize chemotherapy for recurrence
Answer: 3
• Answer: 3 - Good response to chemotherapy
• Explanations:
• Telangiectatic osteosarcoma usually shows a better response to chemotherapy than
conventional osteosarcoma.
• Survival rates with current chemotherapy protocols are within the range of 65% after 5 years.
• Chemotherapeutic protocols for telangiectatic osteosarcoma are similar to those of
conventional osteosarcoma. Patients will have two to six cycles of chemotherapy administered
before surgery.
• The common chemotherapeutic agents used include cisplatin, carboplatin, methotrexate,
doxorubicin, and ifosfamide. Among these, at least two agents need inclusion in a dual-drug
therapy regimen.
Question 61
• Question 61: A 65-year-old male presents to the clinic, with the chief
complaints of weakness and fatigue. He has a history of smoking for the past
25 years. On examination, muscle weakness was present in the limbs. The
repetitive nerve stimulation test at high rates showed considerable increment
in the response.
• Which disease is associated with his muscle weakness?
• Choices:
• 1. Benign prostate hypertrophy
• 2. Polyneuropathy
• 3. Lung cancer
• 4. Myasthenia gravis
Answer: 3
• Answer: 3 - Lung cancer
• Explanations:
• Lambert-Eaton myasthenic syndrome (LEMS) causes weakness of skeletal muscles and fatigue
that usually decreases after physical activity.
• LEMS is an autoimmune disease when the body produces autoantibodies against the calcium
(Ca2+) channels present on the nerve terminal membrane of the neuromuscular junction.
• Antibodies against Ca2+ channels in the presynaptic membrane causes a decrease in the
quantal release of acetylcholine; this disrupts the neuromuscular transmission.
• The exact cause of LEMS is unknown, but around 3% of small cell lung carcinoma patients
have LEMS. LEMS is also associated with nonsmall cell lung cancer, lymphosarcoma,
malignant thymoma, carcinoma of breast, prostate, colon, rectum, bladder, kidney; basal cell
carcinoma; leukemia; lymphoproliferative disorders and Hodgkin lymphoma.
Question 62

• Question 62: A 23-year-old female with no significant past medical history


presents to the emergency department with weakness and tremors. She states
that she has had intermittent periods with similar symptoms over the past two
months. Laboratory workup reveals elevated levels of insulin, proinsulin, and c-
peptide and a glucose level of 40 mg/dl. Her family history is significant for
parathyroid tumors and insulinomas. Which of the following is the most likely
cause of her symptoms?
• Choices:
• 1. Factitious insulin administration
• 2. Multiple endocrine neoplasia syndrome type 1
• 3. Multiple endocrine neoplasia syndrome type 2b
• 4. Diabetes mellitus
Answer: 2
• Answer: 2 - Multiple endocrine neoplasia syndrome type 1
• Explanations:
• Multiple endocrine neoplasia syndrome type 1 is an endocrine tumor syndrome
that causes tumors of the pancreas, such as insulinomas and glucagonomas, in
addition to pituitary and parathyroid tumors.
• Insulinomas are tumors of the pancreatic beta cells and represent 10 to 30% of
pancreatic neuroendocrine tumors in patients with MEN1 syndrome.
• Patients with insulinomas have periods of hypoglycemia that often occur during
fasting or after exercise and cause symptoms such as weakness, confusion,
palpitations, and tremors.
• The diagnosis of insulinoma can be confirmed with increased levels of insulin,
proinsulin, and c-peptide and a glucose level below 45 mg/dl.
Question 63
• Question 63: A 50-year-old male presents to the clinic with a palpable, central
neck mass that has been present for several months. He admits to a history of
joint pain and arthritis that has been progressively worsening. He had a kidney
stone 2 weeks ago that brought him to the emergency department (ED) because
he was in excruciating pain. In the ED the patient was found to have
hypercalcemia and a parathyroid hormone level of 10 times the upper limit of
normal. What is the likely pathology?
• Choices:
• 1. Benign primary hyperparathyroidism
• 2. Parathyroid carcinoma
• 3. Papillary thyroid carcinoma
• 4. Medullary thyroid carcinoma
Answer: 2
• Answer: 2 - Parathyroid carcinoma
• Explanations:
• Laboratory evaluation is similar to that of primary hyperparathyroidism, which includes
parathyroid hormone (PTH) and serum calcium
• levels. PTH and calcium levels tend to be higher in parathyroid carcinoma compared with
primary hyperparathyroidism.
• A PTH level three times the upper limit of normal is suspicious, but a PTH level of 10 times
the upper limit of normal has a positive predictive value of 84% for parathyroid carcinoma.
• A palpable neck mass in combination with hyperparathyroidism or hypercalcemia is
suggestive of parathyroid carcinoma.
• Fifty percent of patients with parathyroid carcinoma present with a palpable central neck
mass, kidney stones, or osteoporosis
Question 64
• Question 64: A 65-year-old female patient with no past family history of breast
carcinoma, presented with a lump in the left breast for three months. Physical
examination showed a well-circumscribed, ulcerative mass, in the right upper outer
quadrant of the left breast measuring 5×5 cm and was associated with foul smelling
bloody discharge. A left axillary lymph node was palpable measuring 3×2 cm. Imaging
studies did not disclose distant metastases.
• What is the most accurate TNM stage of this patient?
• Choices:
• 1. Stage IIB
• 2. Stage IIIA
• 3. Stage IIIB
• 4. Stage IIIC
Answer: 3
• Answer: 3 - Stage IIIB
• Explanations:
• T4b: in case of cutaneous ulceration, we classify the breast carcinoma as T4b.
• N1: Since an ipsilateral axillary lymph node was palpable measuring 3x2 cm.
• M0: Imaging studies did not disclose distant metastases.
• Overall, we classify this tumor as T4bN1M0, which corresponds to stage IIIB.
Stage IIIB breast carcinomas correspond to T4, N0 or N1 or
• N2 and M0.
Question 65
• Question 65: A 55-year-old female with a past medical history of
hypertension, hyperlipidemia, and type 2 diabetes mellitus presents to her
provider to discuss her risk of endometrial cancer. She reports that her best
friend recently was evaluated for abnormal uterine bleeding and was
diagnosed with endometrial cancer by an endometrial biopsy. The patient still
reports regular monthly menstrual periods. She has no previous history of
cancer and denies any family history of cancer. The patient wants to know
when she should begin screening for endometrial cancer.
• As her provider, what should the patient be told?
• Choices:
• 1. Perform yearly screening transvaginal ultrasound to assess endometrial
thickness
• 2. Schedule the patient for a one-time screening endometrial biopsy
• 3. There is no indication for screening in this patient
• 4. Use routine PAP smears to screen for endometrial cancer
Answer: 3
• Answer: 3 - There is no indication for screening in this patient
• Explanations:
• There is no published current evidence to support the screening of asymptomatic women for
endometrial cancer. The American Cancer
• Society recommends that patients who have an increased risk of hereditary non-polyposis
colon cancer (HNPCC, or Lynch syndrome) should be screened annually with an endometrial
biopsy starting at age 35. They report a 22% to 50% lifetime risk of endometrial cancer
within this population.
• While no screening is indicated, proper management of the patient’s chronic conditions,
diabetes, and hypertension could help prevent endometrial cancer, as these are known risk
factors.
CONT
• Any postmenopausal bleeding or abnormal uterine bleeding should
be investigated, and this can be done with an endometrial biopsy.
• However, this specific patient was asymptomatic and did not warrant
investigation.
• Atypical endometrial cells may be seen on Pap smears and would
indicate the need for additional endometrial sampling. However, Pap
smears are not indicated or recommended for screening of
endometrial cancer
Question 66
• Question 66: Which organism has been linked to the development of Burkitt
lymphoma?
• Choices:
• 1. Helicobacter pylori
• 2. Epstein-Barr virus
• 3. Human papillomavirus
• 4. Herpes simplex virus
Answer: 2
• Answer: 2 - Epstein-Barr virus
• Explanations:
• Helicobacter pylori is responsible for most gastric ulcers and strongly associated with
gastric marginal zone mucosa-associated lymphoid tissue (MALT) lymphoma.
• The Epstein-Barr virus is mainly responsible for the development of Burkitt lymphoma
in its endemic form.
• Human papillomavirus is responsible for genital warts and is strongly associated with
cancer of the cervix.
• Herpes simplex virus can reactivate in patients with malignancies, especially those
receiving chemotherapy or immunosuppressive treatments, but is not causally
associated with Burkitt lymphoma.
Question 67
• Question 67: A 61-year-old female with a known history of non-small cell lung
cancer with metastasis to bone and EGFR mutation positive presents to the
oncology clinic to discuss treatment options. A drug is mentioned as the current
first-line treatment option for her form of cancer.
The first-line treatment option for this patient is from which of the following
drug classes?
• Choices:
• 1. PCSK9 inhibitors
• 2. Tyrosine kinase inhibitors
• 3. Anti-platelet IIb/IIIa receptors
• 4. TNF alpha inhibitors
Answer: 2
• Answer: 2 - Tyrosine kinase inhibitors
• Explanations:
• Tyrosine kinase inhibitors are the first-line FDA-approved treatment in patients
with metastatic non-small cell lung cancer with nonresistant
• EGFR positive mutation.
• There are three known tyrosine kinase inhibitors (EGFR TKIs): gefitinib, erlotinib,
and afatinib.
• All three tyrosine kinase inhibitors are widely used as a treatment for advanced
non-small cell lung cancer (NSCLC) with proven efficacy.
• Afatinib covalently binds EGFR receptor and is an irreversible inhibitor. It is
currently accumulating trial data for use in other forms of cancer such as breast
cancer
Question 68
• Question 68: A parotid gland tumor would most likely occlude which artery?
• Choices:
• 1. Internal carotid
• 2. Middle meningeal
• 3. Facial
• 4. Superior thyroid
Answer: 2
• Answer: 2 - Middle meningeal
• Explanations:
• The external carotid artery terminates at the maxillary artery.
• The middle meningeal artery is a branch of the maxillary artery within the
parotid gland.
• The internal carotid is not near to the parotid gland.
• The facial artery and the superior thyroid artery branch off from the external
carotid before the parotid gland.
Question 69
• Question 69: A 45-year-old male presents with a small, elevated lesion on his
scalp. The lesion appears red with a smooth shiny surface and a central crater
that appears like a horn. He reports that the lesion developed rather quickly
over the past few weeks and is painless.
• What is the most likely diagnosis?
• Choices:
• 1. Squamous cell cancer
• 2. Keratoacanthoma
• 3. Melanoma
• 4. Actinic keratosis
Answer: 2
• Answer: 2 - Keratoacanthoma
• Explanations:
• Keratoacanthomas are low-grade, malignant lesions that often occur in sun-
exposed areas.
• The lesions may resemble a basal cell cancer but have a very rapid growth rate.
Spontaneous resolution is common with these lesions.
• Because it is hard to differentiate the tumor from a cancer, it is essential to
excise it.
• The pathogenesis of keratoacanthoma is similar to squamous cell cancers.
Question 70
• Question 70: What is the 5-year survival rate for pancreatic cancer?

• Choices:
• 1. 90%
• 2. 70%
• 3. 40%
• 4. < 25%
Answer: 4
• Answer: 4 - < 25%

• Explanations:
• The majority of patients with pancreatic cancer are dead in 4-6 months. The
few who get surgery do live longer. Five-year survival for patients is abysmal.
Rarely do more than 20% make it to 5 years. Nearly 3/ 4 of all pancreatic
cancers occur in the head and neck area. Significant weight loss is the most
classic feature of this cancer. The median time survival for most patients is 4-
6 months.
Question 71
• Question 71: Which of the following organs is the most common site of distant
metastasis in colon cancer?
• Choices:
• 1. Liver
• 2. Skin
• 3. Kidney
• 4. Small intestine
Answer: 1
• Answer: 1 – Liver
• Explanations:
• Stage 1 colon cancers are confined to the wall of the colon. Stage 1 colon
cancer can be cured by surgery alone. When colon cancer is untreated it may
spread to regional lymph nodes. Sixty percent of patients may be cured with
surgery and chemotherapy. Metastatic colon cancer is most commonly found
in the liver. Metastatic colon cancer is, in fact, the most common malignancy
overall of the liver. Although chemotherapy can extend survival, a cure is
seldom possible for patients with stage 4 colon cancer.
Question 72
• Question 72: What is the most common posterior mediastinal tumor?

• Choices:
• 1. Lymphoma
• 2. Thymoma
• 3. Seminoma
• 4. Neurogenic
Answer: 4
• Answer: 4 – Neurogenic
• Explanations:
• Neurogenic tumors are the most common tumors of the posterior
mediastinum. Tumors in the posterior mediastinum may also originate from
vascular, lymphatic, and mesenchymal tissues. Neurogenic tumors account for
about 20% of all adult mediastinal masses and nearly 40% of mediastinal
masses in children. The majority of neurogenic tumors in adults originate from
the nerve sheath and usually are neurofibromas and neurilemomas.
Question 73
• Question 73: What is the most common cause of brain metastasis?

• Choices:
• 1. Lung cancer
• 2. Breast cancer
• 3. Skin cancer
• 4. Unknown primary site
Answer: 1

• Answer: 1 - Lung cancer


• Explanations:
• Brain metastases are seen in about 20% of cancers. The most common
primary tumor associated with brain metastases is lung cancer, which
accounts for about half of brain metastases. Breast cancer is the second
most frequent cause of brain metastases and accounts for about 20% of
all brain metastases. Skin and gastrointestinal cancers each account for
about 10% of brain metastases.
Question 74
• Question 74: A 65-year-old male patient with a past medical history of
hypertension and hyperlipidemia presents to the hospital complaining of
fatigue and abdominal fullness. He states that he had a recent dental
procedure with persistent gingival bleeding, but he did not think much of it at
the time. On physical examination, he is found to have splenomegaly. His
examination reveals a temperature of 98.8 F (37.1 C), a blood pressure of 100/
62 mmHg, a heart rate of 92 beats/ min, and a respiratory rate of 18/ min.
Laboratory workup shows an elevated acid phosphatase that, when oxidized,
turned into purple color. Which of the following cells would characteristically
be seen in his blood smear?
• Choices:
• 1. Hairy cells
• 2. Smudge cells
• 3. Myeloblasts with Auer rods
• 4. Reed-Sternberg cells
Answer: 1
• Answer: 1 - Hairy cells
• Explanations:
• Tartrate-resistant acid phosphatase is characteristically elevated in hairy cell
leukemia. Acid phosphatases with a metallic center turn into purple color when
oxidized. They are also resistant to tartrate, thus called the tartrate-resistant
acid phosphatases. Hairy cell leukemia is named so because of the
characteristic "hairy" appearance of the abnormal lymphocytes. Hairy cell
leukemia is a chronic lymphoproliferative disease in which neoplastic B cells
infiltrate the bone marrow, spleen, and blood, leading to splenomegaly,
anemia, and recurring infections.
Question 75
• Question 75: What mass is most commonly seen in the anterior mediastinum
in adults?
• Choices:
• 1. Thymoma
• 2. Goiter
• 3. Teratoma
• 4. Lymphoma
Answer: 1
• Answer: 1 - Thymoma
• Explanations:
• Thymomas are quite common in the anterior mediastinum. Twenty percent are
associated with myasthenia gravis. Substernal goiter can occur in elderly females
but is not very common. The majority can be removed through a cervical
incision. Teratomas of the superior mediastinum are rare and seen in the second
and third decade of life. Hodgkin lymphoma is the second most common mass in
the anterior mediastinum. Biopsy usually reveals Reed-Sternberg cells.
Question 76
• Question 76: Lentigo maligna usually develops into which of the following?
• Choices:
• 1. Basal cell cancer
• 2. Squamous cell carcinoma
• 3. Melanoma
• 4. Cafe au lait spots
Answer: 3
• Answer: 3 – Melanoma
• Explanations:
• Lentigo maligna melanoma (LMM) is a type of melanoma found in older
individuals that develops within lentigo maligna. Lentigo maligna is a form of in
situ melanoma associated with sun damage. It usually is located on the skin of
the head and neck. The tumor is slow-growing and was previously called a
Hutchinson freckle. Lentigo maligna may appear as a slowly enlarging, irregular
stain.
Question 77
• Question 77: What skin cancer most commonly develops with actinic
keratosis?
• Choices:
• 1. Basal cell carcinoma
• 2. Adenocarcinoma
• 3. Squamous cell carcinoma
• 4. Melanoma
Answer: 3
• Answer: 3 - Squamous cell carcinoma
• Explanations:
• Actinic keratosis or solar keratosis is a premalignant disorder of the skin.
The disorder is more common in fair-skinned individuals and is
frequently associated with sun exposure. Actinic keratosis can progress
to squamous cell carcinoma and should be treated. Actinic keratosis can
range from skin colored to hyperpigmented plaques.
Question 78
• Question 78: Which is true regarding lentigo maligna?
• Choices:
• 1. Is a precursor for cervical cancer
• 2. Is a precursor lesion for melanoma
• 3. Is associated with human papillomavirus
• 4. Usually gives rise to basal cell cancers
Answer: 2
• Answer: 2 - Is a precursor lesion for melanoma
• Explanations:
• Lentigo maligna is a type of melanoma in situ that occurs on sun-exposed skin.
Invasive melanoma may develop within it. This is called lentigo maligna
melanoma. Lentigo maligna was previously known as Hutchinson melanotic
freckle. Lentigo maligna is usually found in elderly individuals. The lesion
appears as a flat patch with variable color and irregular shape. The affected
skin is often thin. On histology, epidermal atrophy is common and there is an
increased number of atypical melanocytes.
Question 79

• Question 79: Where is an Ommaya reservoir placed?


• Choices:
• 1. The subcutaneous tissue of the abdomen
• 2. The epidural space
• 3. The deltoid muscle
• 4. The lateral ventricles
Answer: 4
• Answer: 4 - The lateral ventricles
• Explanations:
• The Ommaya reservoir is an intraventricular medication administration system
for certain chemotherapeutics and pain medications. The ventricular reservoir
allows access to CSF for sampling. The reservoir eliminates the need for
repeated lumbar punctures. Strict aseptic technique is required when accessing
the reservoir; infection is the most common complication. It can also become
clogged and malfunction.
Question 80
• Question 80: A 65-year-old female has had a 3 centimeter, tan, freckle-like
lesion for several years. The lesion has grown slowly and multiple areas of
increased and decreased pigmentation have appeared.
• What is the most likely diagnosis?
• Choices:
• 1. Senile lentigo
• 2. Lentigo maligna melanoma
• 3. Superficial spreading melanoma
• 4. Nodular melanoma
Answer: 2
• Answer: 2 - Lentigo maligna melanoma
• Explanations:
• Early recognition of the clinical forms of melanoma greatly enhances survival.
Lentigo maligna melanoma (LMM) begins as a large, freckle-like lesion that
enlarges over a period of years and develops multiple areas of increased and
decreased pigmentation. LMM is mostly found in the head and neck area and
slightly more common in females. Excisional biopsy is used to make the
diagnosis.
Question 81
• Question 81: Which drug inhibits topoisomerase II and is cell-cycle specific?
• Choices:
• 1. Etoposide
• 2. Bleomycin
• 3. Doxorubicin
• 4. Tamoxifen
Answer: 1
• Answer: 1 - Etoposide
• Explanations:
• Etoposide forms a complex with DNA and the topoisomerase II enzyme.
This enzyme aids in DNA unwinding. Etoposide is slowly administered
intravenously as it can lower blood pressure. It also can be given orally. The
peripheral site of injection can become painful if the drug extravasates.
Etoposide is used to treat Ewing sarcoma, lymphoma, lung cancer, and
testicular cancer.
Question 82
• Question 82: Carcinoembryonic antigen (CEA) is a marker for which of the
following carcinomas?
• Choices:
• 1. Colorectal
• 2. Breast
• 3. Gastric
• 4. Esophageal
Answer: 1
• Answer: 1 - Colorectal
• Explanations:
• CEA is a glycoprotein. In the healthy population, the upper limit of
CEA is about 3 ng/ L for nonsmokers and about 5 ng/ L for smokers.
This is a nonspecific marker for certain types of carcinoma. In breast
cancer, an elevated CEA usually indicates metastatic disease.
Question 83
• Question 83: What should a patient be instructed to do after injection of a
sclerosing agent in the pleura?
• Choices:
• 1. Change position frequently
• 2. Lie still to avoid loosening the chest tube
• 3. Avoid coughing
• 4. Avoid deep breathing
Answer: 1

• Answer: 1 - Change position frequently


• Explanations:
• Sclerosing agents are used to treat recurring pleural effusions.
• If the patient is unable to cough, suctioning may be needed.
• Breathing exercises would help to keep the lungs expanded. Changing
positions helps to distribute the sclerosing agent.
Question 84
• Question 84: A 62-year-old patient with relapsed, diffuse large-cell lymphoma
is admitted for a cycle of DHAP chemotherapy (dexamethasone, high-dose
cytarabine, cisplatin) and develops severe bilateral eye pain and redness. What
is the most probable diagnosis?
• Choices:
• 1. Acute angle glaucoma
• 2. Allergic conjunctivitis
• 3. Cytarabine keratitis
• 4. Retinal vein thrombosis
Answer: 3
• Answer: 3 - Cytarabine keratitis
• Explanations:
• Keratoconjunctivitis is a common acute toxicity of high-dose cytarabine
infusion. Prophylaxis is usually recommended for patients receiving
cytarabine doses greater than 1 g per m2. Corticosteroid eye drops every 4 to
6 hours until 48 hours after the end of cytarabine infusion are the preferred
prophylactic strategy. Topical NSAIDs such as diclofenac can also be used for
relief in patients who develop this complication.
Question 85
• Question 85: Which of the following adverse effects is known to occur with
the use of vincristine?
• Choices:
• 1. Gout
• 2. Priapism
• 3. Neurotoxicity
• 4. Hyperglycemia
Answer: 3

• Answer: 3 – Neurotoxicity
• Explanations:
• Vincristine is used to treat various types of cancers, including
leukemias, Hodgkin disease, and small cell lung cancer. It interferes
with mitosis in cancer cells, leading to cellular apoptosis.
• Vincristine is associated with the development of neuropathy as an
adverse effect. This can manifest as numbness and tingling in the
extremities, hypersensitivity to cold, or foot drop.
Question 86

• Question 86: Which adverse effect should result in a decreased dose


of paclitaxel?
• Choices:
• 1. Nausea
• 2. Myalgia
• 3. Edema
• 4. Peripheral neuropathy
Answer: 4
• Answer: 4 - Peripheral neuropathy
• Explanations:
• Peripheral neuropathy is a common adverse effect of paclitaxel. Patients with
preexisting neuropathies are at a higher risk for developing peripheral
neuropathy than other patients. Certain prior chemotherapy regimens can
increase a patient's risk along with comorbid conditions, such as diabetes
mellitus. For patients who develop severe, peripheral neuropathy, the dose
should be reduced by 20%.
Question 87
• Question 87: A 65-year-old man with non-Hodgkins lymphoma presents to the
emergency department with confusion, nausea, vomiting, and left flank pain.
Serum chemistry shows a calcium level of 14mg/ dL. In order to control his
hypercalcemia, a drug is administered that is endogenously produced by the
parafollicular cells of the thyroid. What is the mechanism of action of this
drug?
• Choices:
• 1. Prevention of bone formation
• 2. Promotion of bone formation
• 3. Prevention of bone resorption
• 4. Promotion of bone resorption
Answer: 3

• Answer: 3 - Prevention of bone resorption


• Explanations: Calcitonin inhibits osteoclasts thereby preventing bone
resorption. It also increases renal excretion of calcium. Both act to
lower serum calcium. Calcitonin is a naturally occurring 32-amino acid
linear polypeptide hormone. It is also used therapeutically, usually in
the form of a nasal spray. It is produced by parafollicular cells (C-cells)
of the thyroid gland in humans and the ultimobranchial glands in
salmon.
Question 88
• Question 88: What percent of myasthenia gravis patients also have a
thymic tumor?
• Choices:
• 1. 5%
• 2. 10%
• 3. 15%
• 4. 20%
A n sw er:2

• Answer: 2 - 10%
• Explanations:
• A thymic tumor is found in about 10% of patients with myasthenia
gravis. Myasthenia gravis occurs most commonly in women under 40
years of age. Men and women are affected equally after 50 years of
age. Chest x-ray, chest CT, or chest MRI may be used to rule out
thymoma. Thymectomy typically is performed when patients
concurrently have a thymoma.
Question 89

• Question 89: Which of the following thyroid disorders cannot be


diagnosed with simple fine-needle biopsy?
• Choices:
• 1. De Quervain thyroiditis
• 2. Thyrotoxic goiter
• 3. Follicular thyroid cancer
Answer: 3
• Answer: 3 - Follicular thyroid cancer
• Explanations: Fine needle biopsies of the thyroid remove vast amounts of
follicular cells from the area of interest. Malignant cells need further
assessment findings than available with needle biopsy. Follicular thyroid
cancer can be difficult to identify by fine needle biopsy and may require
surgical biopsy. Follicular cells can be confused with a lymphoma and of
course, the treatment is different. If you suspect follicular cell cancer, you need
an open biopsy.
Question 90
• Question 90: A patient has a breast cancer with axillary nodes. What
is the assigned stage of this patient's cancer?
• Choices:
• 1. 1
• 2. 2
• 3. 3
• 4. 4
Answer: 3
• Answer: 3 – 3
• Explanations: One has stage 3 cancer when there are positive lymph
nodes. In stage 4, the cancer is most advanced and has spread to other
areas of the body. In stage 1 breast cancer, the tumor is less than 2 cm
and there are no nodes in the axilla. In stage 2 cancers, the tumor is
between two and five centimeters. The cancer may not have spread to
the nodes, or no more than three nodes are positive.
Question 91
• Question 91: A mother brings in her 4-year-old child for evaluation of low
mood and energy over the past 8 weeks. He's stopped playing at school
because the kids have been making fun of the way he walks. His teachers
report that he has fallen out of his chair several times. Although his sleep has
been undisturbed, he awakens earlier than usual, holding his head. He eats a
regular balanced diet. He continues to draw his favorite superheroes but
doesn't like watching TV anymore as he "cannot make sense of what he is
seeing." What is the most likely diagnosis?
• Choices:
• 1. Major depression syndrome
• 2. Adjustment disorder
• 3. Optic glioma
• 4. Medulloblastoma
Answer: 4
• Answer: 4 – Medulloblastoma
• Explanations: Medulloblastoma is a highly malignant tumor with a propensity for local invasion and
distant metastatic spread through the subarachnoid system, i.e., within the brain and along the spinal
cord, also known as "drop mets." Thought to originate from the granule cell precursors in the external
germinal layer (EGL) of the developing cerebellum, tumor growth starts along the floor of the fourth
ventricle and occupies the entirety of the ventricle, subsequently invading both the cerebellar vermis and
the brainstem. The symptomatology, thus, reflects this pattern of growth.
• Children often present with difficulties in coordination and gait (cerebellar signs) and with early morning
headaches, nausea, vomiting, papilledema, and double vision secondary to hydrocephalus from
obstruction of the fourth ventricle.
• The average time from onset of symptoms to diagnosis is 2 to 3 months. Although the patient's mood is
low from being teased at school, he does not have any other symptoms of major depression. He eats a
balanced, regular diet. He does not have anhedonia. His sleep is mostly undisturbed. No changes in his
life have been reported recently, making adjustment disorder also unlikely.
Question 92
• Question 92: A 65-year-male female patient presents with complaints of blurry
vision, weight gain, muscle weakness, easy bruising, and frequent urination.
Laboratory findings indicate an elevated random cortisol level and a fasting
blood sugar 180 mg/ dL, with no previous diagnosis of diabetes. Her blood
pressure has been running high over the last few months. The provider orders a
magnetic resonance imaging (MRI ) of the brain, and an abnormal finding was
noticed. Based on the clinical and radiological findings, a diagnosis of the
pituitary adenoma was made.
• What additional tests would the provider check to diagnose this patient?
• Choices:
• 1. Inferior petrosal sinus sampling
• 2. prolactin
• 3. Thyroid-stimulating hormone (TSH)
• 4. 24 hour urine cortisol
Answer: 4
• Answer: 4 - 24 hour urine cortisol
• Explanations: Pituitary adenomas occur in the pituitary gland. They can be a
functional and nonfunctional adenoma. Recognize the symptoms of
hypercortisolemia that are weight gain, new-onset or worsening diabetes,
worsening hypertension, muscle weakness, easy bruising. Recognize the
proper tests for diagnosis of hypercortisolemia that are 1- Bedtime salivary
cortisol 2- 24-hour urine cortisol 3- low dose dexamethasone suppression test
Recognize to confirm the biochemical diagnosis in hypercortisolemia prior to
invasive testing like MRI head or Inferior Petrosal Sinus Sampling( IPSS). It is
important to check the rest of the pituitary hormone but prolactin and TSH do
not explain her symptoms.
Question 93

• Question 93: What is the first step in managing a febrile patient who is
neutropenic?
• Choices:
• 1. Chest radiograph
• 2. Blood cultures followed immediately by antibiotics
• 3. Blood cultures and wait for the results before beginning antibiotics
• 4. Repeat temperature measurement in 30 minutes
Answer: 2
• Answer: 2 - Blood cultures followed immediately by antibiotics
Explanations: Febrile neutropenia includes a temperature greater
than 101F and an absolute neutrophil count less than 500 cells/
microliter. Empiric antibiotics should be started after taking blood and
urine cultures. Other investigations may follow after initiating
antibiotics and monitoring of the immune system status must be
carried out. If a fungal infection is suspected, appropriate samples for
culture and antifungal therapy should be initiated.
Question 94
• Question 94: Which of these conditions does not predispose to
esophageal cancer?
• Choices:
• 1. Achalasia
• 2. Scleroderma
• 3. Plummer Vinson syndrome
• 4. Zenker diverticulum
Answer: 4

• Answer: 4 - Zenker diverticulum


• Explanations: Zenker diverticulum is not associated with esophageal cancer.
Scleroderma patients have severe reflux esophagitis but most do not live long
enough to develop malignancy. Plummer Vinson is seen in edentulous old
females with anemia, nail changes, and esophageal webs. They can develop a
cervical esophageal cancer. Cancers in achalasia develop in the mid esophagus,
are usually large, and related to stasis. Cancer occurs in about 10 percent over
a 15-year period and the diagnosis is usually delayed.
Question 95
• Question 95: What is the correct term for difficulty swallowing? Choices:
• 1. Odynophagia
• 2. Dysphagia
• 3. Dysarthria
• 4. Dysphasia
Answer: 2
• Answer: 2 - Dysphagia
• Explanations: Dysphagia is the term for problems with swallowing.
Odynophagia is the term for pain with swallowing.
• Dysarthria is difficulty speaking. Dysphasia is difficulty with all forms of
communication.
Question 96
• Question 96: A 60-year-old female was diagnosed with colorectal cancer
following biopsies obtained on routine colonoscopy. She is planned to
undergo adjuvant chemo and radiotherapy.
• What measures should be taken for symptoms of acute radiation enteritis?
• Choices:
• 1. Antibiotics
• 2. Enema
• 3. None
• 4. Surgical resection
Answer: 3
• Answer: 3 - None
• Explanations: Symptoms of acute radiation enteritis often self-resolve within
weeks of supportive treatment with antimotility agents and adequate water
intake; surgery is rarely needed. Mild bleeding and minimal symptoms do not
require further treatment. Oral antibiotics are given to patients suspected of
having bacterial overgrowth along with vitamins and electrolytes replacement if
needed. As a consequence of chronic radiation enteritis leading to the
formation of intestinal strictures, fistulas, and perforations, patients require
surgical procedures.
Question 97
• Question 97: A patient starts a chemotherapeutic agent. He later develops a
"pins and needles" feeling in his legs.
• Which medication is most likely to have caused this effect?
• Choices:
• 1. Chlorhexidine
• 2. Cyclophosphamide
• 3. Prednisone
• 4. Vincristine
Answer: 4
• Answer: 4 - Vincristine

• Explanations: Vincristine is a microtubule inhibitor and often used to treat solid cancers. At least
10% of people develop peripheral neuropathy. Vincristine is delivered via intravenous infusion. Its
main uses are in non-Hodgkin lymphoma (CHOP chemotherapy regimen), Hodgkin lymphoma (MOPP,
COPP, BEACOPP regimen), or the Stanford V chemotherapy regimen in acute lymphoblastic leukemia
(ALL). It also is used to treat nephroblastoma. It may be used to induce remission in ALL with
dexamethasone and L-Asparaginase and is used in combination with prednisone to treat childhood
leukemia. Vincristine occasionally is used as an immunosuppressant for the treatment of thrombotic
thrombocytopenic purpura or chronic immune thrombocytopenic purpura.
• Chemotherapy-induced peripheral neuropathy may be severe, and in some patients, may be a reason
to reduce or avoid using vincristine. The progressive and enduring symptoms include tingling,
numbness, pain, and hypersensitivity to cold, which begins in the hands and feet and progresses to
the arms and legs. Foot drop is one of the first signs. Individuals with a family history of foot drop or
Charcot-Marie-Tooth disease should avoid vincristine.
Question 98
• Question 98: A 17-year old with acute leukemia develops tumor lysis
syndrome with severe hyperuricemia. What drug can be given to increase
the breakdown of uric acid?
• Choices:
• 1. Probenecid
• 2. Furosemide
• 3. Rasburicase
• 4. Colchicine
Answer: 3
• Answer: 3 - Rasburicase
• Explanations: Rasburicase is a new medication for the prevention and
treatment of tumor lysis syndrome in patients receiving chemotherapy for
hematologic malignancies like lymphomas and leukemias. The drug is
presently being assessed for use in other disorders associated with
hyperuricemia. Rasburicase catalyzes the conversion of uric acid to allantoin.
Allantoin is an inactive metabolite of purine metabolism and is ten times more
soluble than uric acid. Therefore, it is more rapidly excreted by the kidneys.
Rasburicase is unique in that it catalyzes the metabolism of existing uric acid
and thus can be used as a treatment and a prophylactic agent for
hyperuricemic symptoms.
Question 99
• Question 99: What is the most common presentation of a prolactinoma in
men?
• Choices:
• 1. Galactorrhea
• 2. Headache
• 3. Azoospermia
Answer: 2
• Answer: 2 - Headache
• Explanations: Unlike women, men tend to present with either a headache or
visual changes in the presence of a prolactinoma. Other symptoms may include
sexual dysfunction, decreased libido, and male infertility. Hyperprolactinemia is
responsible for about 11% of all cases of oligospermia. Blood work usually
reveals hyperprolactinemia, decreased serum testosterone, and
hypogonadism. Prolactin levels should be checked in all men with confirmed
hypogonadism and as part of the routine evaluation of male infertility.
Question 100
• Question 100: Chemotherapy with etoposide commonly is associated with
bone marrow suppression. This can lead to greater risk of infection due to
reduced leukocyte count. When is the risk of infection greatest after initiating
this drug?
• Choices:
• 1. In the first week after initiating treatment
• 2. One to 2 weeks following administration of the drug
• 3. One month after starting therapy
• 4. Six to 8 weeks following administration
Answer: 2
• Answer: 2 - One to 2 weeks following administration of the drug

• Explanations: The greatest risk for infection is about 7 to 14 days following the
initial administration of etoposide. Myelosuppression places these patients at
increased risk for infections. Hand hygiene is of the utmost importance in this
patient dynamic to prevent the spread of contagions. Due to the risk of further
exacerbating myelosuppression, patient education should include the need to
avoid individuals who have recently received vaccinations. Educate patients to
avoid crowded areas, especially during influenza season. Report any changes in
condition such as a sore throat, fever or a cough.
Question 101
• Question 101: A 24-year-old woman with a history of stage III breast cancer is
scheduled for the induction phase of chemotherapy with docetaxel. She is
taking an oral contraceptive for pregnancy prevention. She will be
administered fosaprepitant IV, dexamethasone IV, and ondansetron IV before
infusion for chemotherapy-induced nausea and vomiting prophylaxis (CINV).
What is the most appropriate post-infusion advice for this patient?
• Choices:
• 1. Avoid food for the next 12 hours
• 2. Get renal function tests done 24 hours later
• 3. Employ a backup means of contraception if the patient engages in sexual
activity
• 4. Not operate a motorized vehicle for 24 hours
Answer: 3
• Answer: 3 - Employ a backup means of contraception if the patient engages in
sexual activity

• Explanations: Fosaprepitant can decrease the effectiveness of oral


contraceptive medications. The patient should be made aware to use a backup
means of contraception. Fosaprepitant exerts its antiemetic effects for up to
48 hours. In contrast to promethazine or droperidol the combination of
dexamethasone, ondansetron and fosaprepitant lead to little or no effect on
alertness. The combination of dexamethasone, ondansetron and fosaprepitant
requires no lab monitoring.
Question 102
• Question 102: Which of the following about mucoepidermoid tumors is
correct?
• Choices:
• 1. The majority are located in the periphery
• 2. They are very responsive to radiation
• 3. Surgery is always undertaken after chemotherapy
• 4. The prime factor that determines their behavior is the histology grade
Answer: 4
• Answer: 4 - The prime factor that determines their behavior is the histology
grade

• Explanations: Mucoepidermoid cancers occur in the fifth or sixth decade with


equal occurrence in males and females. Mucoepidermoid is a rare tumor that
arises in the central tracheo-bronchial tree. There is a low grade and high
grade tumor based on histology. The low-grade variety may rarely infiltrate
local organs but is not metastatic. The high grade is an aggressive tumor and
highly metastatic. They may present with bronchial irritation, wheezing cough,
and hemoptysis. Biopsy is safe under bronchoscopy. Surgery is complete
resection as these cancers are resistant to both chemotherapy and radiation.
Question 103

• Question 103: A patient with 4-cm non-small cell cancer within the right middle
lobe has a positive right lower paratracheal lymph node. There are no other
sites of metastatic disease. What is the correct TNM classification for this
patient?
• Choices:
• 1. T2 N1 M0
• 2. T2 N2 M0
• 3. T1 N2 M0
• 4. T1 N1 M1
Answer: 2

• Answer: 2 - T2 N2 M0

• Explanations:
• The 4 cm lesion is a T2 lesion, and unilateral paratracheal lymph node
involvement corresponds to N2 nodal station.
• T2 plus N2 makes stage III A.
• T2 lesions are defined as between 3 to 7 cm in size (3 to 5 cm is T2A and 5 to 7 cm
is T2b). Other reasons to list a lesion as T2 include involvement of the main
bronchus or within 2 cm of the carina and invasion of visceral but not parietal
pleura. N2 lesions are lymph node spread to ipsilateral mediastinal and/ or
subcarinal lymph nodes.
• T2N2M0 lesion is classified as stage IIIA.
:Question 104
• Question 104: When assessing pain intensity, which of the following
individuals has the most validity?
• Choices:
• 1. Patient
• 2. Physician
• 3. Nurse
• 4. Pain specialist
Answer: 1
• Answer: 1 - Patient
• Explanations:
• Self-reporting is the most valid assessment of pain level.
• Pain is whatever the patient says it is at that time.
• Pain is subjective. Nonverbal patients may be able to use a visual scale.
Question 105
• Question 105: What is Multiple myeloma associated with?
• Choices:
• 1. RBC casts
• 2. WBC casts
• 3. Bence Jones protein
• 4. Waxy casts
Answer: 3
• Answer: 3 - Bence Jones protein
• Explanations:
• Glomerulonephritis is associated with RBC casts.
• Pyelonephritis is associated with WBC casts.
• Nephrosis is associated with waxy casts.
• Multiple myeloma is associated with Bence Jones protein.
Question 106
• Question 106: A 65-year-old female patient with a recent history of HER2
positive, ER, and PR negative breast cancer, is started on adjuvant
chemotherapy. Her baseline echocardiogram done before the initiation of
chemotherapy showed an ejection fraction (EF) of 55%. After four cycles of her
chemotherapy, her oncologist notices lower extremity edema and orders repeat
echocardiogram, which now reveals an EF of 35%. Which of the following drugs
was used as a part of her regimen to cause this side effect?
• Choices:
• 1. Docetaxel
• 2. Paclitaxel
• 3. Anastrozole
• 4. Trastuzumab
Answer: 4
• Answer: 4 - Trastuzumab

• Explanations:
• Trastuzumab is a monoclonal antibody directed against human epidermal growth
factor receptor 2 (HER2) and is used in the treatment of breast cancer.
• It is associated with a modest risk of cardiotoxicity, usually presenting as
decreased ejection fraction.
• Trastuzumab cardiotoxicity is usually asymptomatic, and its risk increases in
patients beyond the age of 50.
• The risk of trastuzumab cardiotoxicity is also higher in patients with concurrent
use of anthracyclines.
• Paclitaxel and docetaxel are not associated with cardiotoxicity but can cause bone
marrow suppression.
Question 107
• Question 107: A 45-year-old female with no significant past medical history
presents to the emergency department due to profound weakness and
confusion. Laboratory workup reveals a functioning pancreatic
neuroendocrine tumor.
• Which of the following are the most likely characteristics of her particular
tumor?
• Choices:
• 1. Benign, greater than 2 cm
• 2. Benign, less than 2 cm
• 3. Malignant, greater than 2 cm
• 4. Malignant, less than 2 cm
Answer: 2
• Answer: 2 - Benign, less than 2 cm
• Explanations: The most common functioning pancreatic neuroendocrine
tumors are insulinomas, which are made up of pancreatic beta cells.
Insulinomas most commonly occur in adults between their third and sixth
decades of life. The most common clinical manifestations of insulinomas are
weakness, loss of consciousness, confusion, palpitations, tremors, hunger, and
sweating. The majority of insulinomas are benign and less than 2 cm in size.
Question 108

• Question 108: A 65-year-old man presents to the clinic for follow up. He has a
history of Non-Hodgkins Lymphoma. He has been advised to start combination
chemotherapy with an agent that inhibits microtubules and thus inhibits cell
proliferation. Which of the following is the most appropriate example of such
an agent?
• Choices:
• 1. Vinblastine
• 2. Adriamycin
• 3. Dexamethasone
• 4. Imatinib
Answer: 1
• Answer: 1 - Vinblastine
• Explanations: Vinblastine inhibits microtubules and thus inhibits cell
proliferation. Vinblastine is a vinca alkaloid. Vinblastine is an integral
component of regimens like ABVD use for Non-Hodgkins Lymphoma.
Adriamycin inhibits topoisomerase II. Dexamethasone is a steroid. Imatinib
inhibits BCR-ABL tyrosine kinase.
Question 109
• Question 109: In which of the following will PET scintigraphy most likely be
positive?
• Choices:
• 1. Granular cell myoblastoma
• 2. Carcinoid 3.
• Small cell carcinoma
• 4. Adenocystic
Answer: 3
• Answer: 3 - Small cell carcinoma
• Explanations: PET scintigraphy involves use of positron labeled isotopes. Most
commonly F-18 is used to label a glucose analog, fluorodeoxyglucose (FDG).
With FDG, malignancies show increased uptake due to their high metabolic
activity. There are several well known potential false negatives, usually due to
malignancies that are relatively slow growing. These include carcinoid tumors,
adenocystic carcinomas, and granular cell myoblastomas, as well as low grade
lymphomas and (among lung cancers) bronchoalveolar cell carcinoma, a variant
of adenocarcinoma. SCLC, also known as oat cell cancer, is among the most
rapidly growing and metabolically active lung cancer. F-18 FDG PET is usually
strongly positive in such cases, showing both the primary cancer and metastatic
deposits.
Question 110
• Question 110: What is the second most common site for metastasis from
sigmoid colon cancer?
• Choices:
• 1. Liver
• 2. Lymph nodes
• 3. Lungs
• 4. Brain
Answer: 1
• Answer: 1 - Liver
• Explanations: The liver is the second most common site of metastases from
colon cancer. Most people tend to have multiple liver lesions. Less than 10
percent have a single lesion that may be resected if the primary is controlled.
Colorectal cancer first spreads to lymph nodes.
Question 111
• Question 111: In the United States, what is the most common risk factor for
developing bladder cancer?
• Choices:
• 1. Alcohol consumption
• 2. Smoking
• 3. Exposure to polycyclic aromatic hydrocarbons
• 4. Exposure to solvents
Answer: 2
Answer: 2 - Smoking
Explanations: The majority of bladder cancers are due to environmental causes.
In the United States, smoking is the most common risk factor for developing
bladder cancer. Other environmental risk factors include exposure to
nitrosamines and polycyclic aromatic hydrocarbons. Bladder cancer also is
associated with industrial exposure to paints, solvents, rubbers, and various dyes.
Question 112
• Question 112: A 50-year-old had a normal initial screening colonoscopy. A
history shows the father was diagnosed with colorectal cancer at age 65.
When would you recommend repeating the colonoscopy?
• Choices:
• 1. 3 years
• 2. 5 years
• 3. 10 years
• 4. 20 years
Answer: 3
• Answer: 3 - 10 years
• Explanations: A colonoscopy is the preferred screening method for colorectal
cancer in patients with a family history according to the American College of
Gastroenterologists. The United States Preventive Services Task Force
recommends fecal occult blood testing, sigmoidoscopy, or colonoscopy. A
patient with a normal colonoscopy and family history after age 60 should be
screened every ten years unless a polyp is found. One or two tubular polyps
should prompt repeat at five years. If three to ten tubular polyps are found,
the colonoscopy should be repeated in three years. With a family history
before age 60, colonoscopy should be repeated in five years.
Question 113
• Question 113: What is the most common primary malignant tumor of the
heart?
• Choices:
• 1. Melanoma
• 2. Squamous cell carcinoma
• 3. Angiosarcoma
• 4. Myxoma
Answer: 3
• Answer: 3 - Angiosarcoma
• Explanations: Cardiac sarcomas are the most common primary malignant
tumors of the heart. It should be noted that metastatic tumors are 30
times more common than primary heart tumors. Angiosarcomas can
occur in any age group and often occur in the right ventricle. The
symptoms may vary from chest pain, dyspnea, tachycardia, and
hypertension. The prognosis is very poor.
Question 114
• Question 114: A 61-year-old man born in the US, diagnosed with advanced
testicular cancer presents postoperatively, to the clinician for medical
management. He has not previously received radiation or chemotherapy for his
condition. He has no history of travel and was fully vaccinated as a child and is
currently up to date on vaccinations. His review of systems is positive for
fatigue. The left testicle has been surgically removed. The physical exam is
otherwise unremarkable. Preoperative chest X-ray was unremarkable with no
infiltrations noted. The patient is to be started on a chemotherapy regimen,
including cisplatin.
• Which of the following must be included in the initial workup before
administering cisplatin?
• Choices:
• 1. Complete blood count
• 2. Complete blood count (CBC), basic metabolic panel, and kidney
function test
• 3. Complete blood count, basic metabolic panel, kidney function test,
and a viral panel
• 4. Complete blood count, basic metabolic panel, viral panel, and
tuberculosis screening (PPD)
Answer: 2
• Answer: 2 - Complete blood count (CBC), basic metabolic panel, and kidney
function test
Explanations: Before initiating cisplatin therapy, a complete blood count (CBC)
should be ordered to assess white blood cell baseline levels due to cisplatin-
induced myelosuppression. Serial CBC can help to monitor the side effects of
cisplatin administration and should be assessed before each treatment course.
Nephrotoxicity is a common complication from cisplatin use. Renal toxicity risk is
minimized by ensuring adequate hydration before and after cisplatin
administration. Close monitoring of renal function throughout treatment is
important. Assessing electrolytes, blood-urea nitrogen (BUN), and creatinine
before first administration is crucial to identifying the patient's baseline and
determining if modified renal dosing is required.
• Renal impairment may require a change in dose or aggressive pretreatment to
reduce renal toxicity. Creatinine and electrolytes should be assessed before
administering Cisplatin to monitor changes after treatment. Further
monitoring may be necessary should a patient experience clinical symptoms
such as neuropathy or vision loss. The viral panel is not necessary before
treatment in an asymptomatic patient. This patients fatigue is likely secondary
to advanced testicular cancer. TB test should be assessed before starting
patients on TNFa inhibitors. This is not required before treating with Cisplatin.
Question 115
• Question 115: Which of the following is a paraneoplastic syndrome?
• Choices:
• 1. Hypermagnesemia
• 2. Hypercalcemia
• 3. Hypomagnesemia
• 4. Hypocalcemia
Answer: 2

• Answer: 2 - Hypercalcemia
• Explanations: Hypercalcemia of malignancy is a common paraneoplastic
syndrome. The most common cancers associated with hypercalcemia are
breast, lung and multiple myeloma. The mechanisms of hypercalcemia are by
the production of parathyroid hormone-related protein (PTHrP), osteolytic
metastases and increased production of 1,25-dihydroxyvitamin D by the tumor
cells. Patients with hypercalcemia may have symptoms of polyuria, polydipsia,
anorexia, nausea, and constipation. Chronic hypercalcemia associated with
hypercalciuria can lead to nephrolithiasis and renal dysfunction. Cardiac
arrhythmias can also be caused, and it causes shortening of the QT interval.
Other common paraneoplastic syndromes
• include SIADH and Cushing syndrome. Paraneoplastic syndromes should
not be confused as metastatic manifestations of the malignancy. They
are called by multiple mechanisms which lead to dysregulation of the
immune system. Early and correct identification of paraneoplastic
syndromes is necessary as sometimes these can be the initial
presentation of the malignancy. The treatment aims at improving the
symptoms and treating the underlying malignancy.
Question 116
• Question 116: What is the most common source of metastases to the adrenal
gland?
• Choices:
• 1. Brain
• 2. Colon
• 3. Lung
• 4. Breast
Answer: 3
• Answer: 3 – Lung
• Explanations: Adrenal metastasis can occur from primaries in the lung (39%),
breast (35%), melanoma, gastrointestinal tract, pancreas, kidney among others.
Adrenal glands are the fourth most common site of metastases in malignant
disease, which is significant considering the relatively small size of adrenals. The
metastases to the adrenal gland can be bilateral.
This metastatic spread to the adrenals is mainly hematogenous
Question 117
• Question 117: A 28-year-old lady is being evaluated for a three months history of
episodic palpitations, which spontaneously resolve after 10-12 minutes. She also
reports associated chest pain and diaphoresis and the feeling of impending doom.
Medical history is negative for any cardiac disease, but she was diagnosed with
neurofibromatosis type 1 (NF-1). There is no family history of cardiac disease, but her
younger brother was also diagnosed with NF-1. She is not on any medication. On
physical examination, her temperature is 37.1 C, blood pressure is 156/ 96 mmHg,
pulse rate is 110/ min, and respiratory rate is 14/ min. Heart exam reveals tachycardia,
but no murmurs and skin exam shows profuse sweating, and cold and clammy skin.
The rest of her examination is within normal limits. Laboratory studies reveal increased
urine catecholamines and metanephrines. CT chest, abdomen and pelvis shows a
single tumor along the sympathetic chain on the right above the diaphragm. Which of
the following findings would be most predictive of poor prognosis in this patient?
• Choices:
• 1. Size of the tumor on the imaging study
• 2. Presence of a single tumor in the CT scan
• 3. Nuclear atypia, pleomorphism, necrosis, mitotic figures, and local/
vascular invasion in biopsied lesion
• 4. Distant metastases of the tumors
Answer: 4
• Answer: 4 - Distant metastases of the tumors
• Explanations: The patient's presentation is most consistent with a diagnosis of
paraganglioma. It is critical to detect all lesions by CT and MRI scans of the entire
retroperitoneum rather than the thorax or pelvis to locate the tumor. If the CT
shows 10 Hounsfield units or less, it suggests lipid-rich mass and hence rules out
pheochromocytoma and paraganglioma. If the tumors are still not localized, the
next step should be functional imaging. Metastasis of tumor to the distant
region is associated with poor prognosis and hence, it is the only determinant of
worse outcome. Size of the tumor does not directly determine the prognosis,
but the presence of metastases is the only way to determine the aggressiveness
of the tumor and is associated with bad prognosis. Paragangliomas are
• commonly a single unilateral tumor and multiple tumors are commonly seen
more in familial types (20%). Sympathetic cells may have hyaline globules in H&
E stain, and giant mitochondria with paracrystalline inclusions may be
noticeable on electron microscopy. Nuclear atypia, pleomorphism, necrosis,
mitotic figures, and local/ vascular invasion is common but not indicative of
malignancy. Distant metastasis is the only indicator of malignancy.
Question 118
• Question 118: A 65-year-old female with a history of asthma, hypertension,
hyperlipidemia, and congestive heart failure with preserved ejection fraction is
recently diagnosed with breast cancer requiring chemotherapy. She presents to
the hospital for follow-up before the initiation of a new chemotherapy agent.
She states that aside from fatigue and occasional shortness of breath on
exertion, she has no new complaints. Her social history is significant for social
alcohol use on the weekends with her family and friends but is otherwise
unremarkable. The physical exam is only remarkable for bilateral expiratory
wheezing in the lung fields. Her doctor recommends starting an agent that has
a mechanism of action of inhibiting topoisomerase II and nucleotide replication.
• Which of the following conditions requires modification of the dose?
• Choices:
• 1. Asthma
• 2. Alcohol use
• 3. Breast cancer
• 4. Elevated serum transaminases
Answer: 4
• Answer: 4 - Elevated serum transaminases
• Explanations: Adriamycin, also known as doxorubicin, is classified as
anthracycline and has been known to cause cardiomyopathy and
hepatotoxicity. Patients with cardiomyopathies should be medically optimized,
diastolic dysfunction is not a contraindication to adriamycin, and should be
adjusted for patients with decreased ejection fractions, abnormal hepatic
function, and elevated hepatic enzymes. The mechanism of action of
adriamycin is to inhibit topoisomerase II and nucleotide replication by
intercalation into DNA. Dexrazoxane is a cardioprotective agent often given
with adriamycin.
Question 119

• Question 119: After radiation therapy to the chest, a patient develops a


necrotic skin ulcer, which has failed to heal with conservative medical
management for the past 3 months. What is the best treatment option?
• Choices:
• 1. Local excision with skin graft
• 2. Hyperbaric oxygen treatment
• 3. Application of growth factor to wound
• 4. Myocutaneous flap
Answer: 4
• Answer: 4 - Myocutaneous flap
• Explanations: Skin injuries ranging from erythema to full chronic wounds occur
in about 95% of patients who undergo radiation therapy. Radiation induced
non-healing skin ulcers can be difficult to heal. The areas of the body that are
more susceptible to radiation injury are the skin of the face, neck, trunk, and
extremities. The center of the ulcer is usually avascular and does not heal with
routine medical care. Pandya et al. examined specimens from 27 patients with
OSCC (oral squamous cell carcinoma) who received radiation therapy involving
the jaw and neck. He found significant tissue atrophy and severity of dysplasia,
an increased presence of fibrinous exudative necrosis, vessel wall thickening,
with oropharyngeal ectasia of the salivary
• gland. In addition, dense fibrosis with thick fibers was a common finding of
post-radiation tissue in the same patients. The best method to close them is
the use of muscle flaps. Hyperbaric oxygen treatment has one of the most
frequent applications and delays radiation injuries, with up to one-third of
patients treated with HBO2 for radiation injury in the United States. Even the
mechanism of injury of ionizing radiation to be mainly hypoxia and increased
district fibrosis, better oxygen treatment has been shown to increase
angiogenesis and therefore improving oxygenation as well as reduce fibrosis,
although this effect has not been fully investigated and explain.
Question 120

• Question 120: Besides bleomycin, which of the following drugs can also cause
pulmonary fibrosis?
• Choices:
• 1. Taxol
• 2. Cyclophosphamide
• 3. Busulfan
• 4. Vincristine
Answer: 3
• Answer: 3 - Busulfan
• Explanations: Busulfan is a potent chemotherapeutic agent associated with
lung fibrosis. Busulfan is not widely used because of the availability of better
and safer drugs. Adverse drug reactions of the lung can affect the pleura,
pulmonary parenchyma, pulmonary vascular system, airways, neuromuscular
system, and mediastinum. The physical findings in patients with drug-induced
lung disease are non-specific. Patients may have crackles in the presence of
noncardiac pulmonary edema, decreased breath sounds in the presence of
pleural effusions, and wheezing in the presence of bronchospasm.
Question 121
• Question 121: Which of the following is the best biochemical marker to monitor
possible recurrence after total thyroidectomy for papillary carcinoma?
• Choices:
• 1. Melatonin
• 2. Calcium
• 3. Serum thyroid-stimulating hormone
• 4. Thyroglobulin
Answer: 4
• Answer: 4 - Thyroglobulin
• Explanations: Thyroglobulin is the biomarker marker to follow after total
thyroidectomy. It is a glycoprotein that is produced only by normal or
neoplastic thyroid follicular cells. Thyroglobulin levels should fall to zero after
total thyroidectomy. Any detectable levels after thyroidectomy should be a clue
to neoplastic tissues, thereby predicting recurrence. Also, the presence of anti-
thyroglobulin antibodies leads to low levels of thyroglobulin. These antibodies
should be measured in all patients with differentiated thyroid carcinoma in
order not to miss a possible recurrence. If positive antibodies are present, then
other modalities for screening are required such as imaging by ultrasound, CT
scan, or a whole body iodine-123 scan.
• The other choices are not associated with predictable evidence for recurrence
of papillary carcinoma.
Question 122
• Question 122: A 40-year-old African male with a history of smoking presented
with a neck mass. Subsequent fine-needle aspiration was positive for papillary
thyroid carcinoma. What is the leading risk factor for the development of this
tumor?
• Choices:
• 1. Male sex
• 2. Radiation exposure
• 3. Smoking
• 4. Iodine deficiency
Answer: 2
• Answer: 2 - Radiation exposure
• Explanations: Radiation exposure is the leading risk factor for developing thyroid
cancer. Exposure to moderate levels of radiation to the head and neck can
increase the risk of thyroid cancer. Sources of exposure include radiation therapy
for head and neck lymphomas and radiation from medical imaging, radioactive
iodine exposure, and ionizing radiation exposure. Examples of ionizing radiation
exposure are nuclear weapons testing during the 1950s and 1960s and the 1986
Chernobyl nuclear power plant accident. Other risk factors for developing
thyroid cancer include female sex, genetics, and a diet low in iodine.
Question 123
• Question 123: A 56-year-old female patient with metastatic colorectal cancer
with KRAS wild-type, in whom other treatments have failed, treated with drug
X shows improvement in overall survival and progression-free survival and
preserves quality-of-life in this patient. What is drug X?
• Choices:
• 1. Cisplatin
• 2. Cyclophosphamide
• 3. Cetuximab
• 4. Dexamethasone
Answer: 3
• Answer: 3 - Cetuximab
• Explanations: Cetuximab is an epidermal growth factor receptor (EGFR) inhibitor.
Cetuximab is used to treat colorectal cancer, metastatic, KRAS wild-type (without
mutation) improves overall survival and progression-free survival and preserves
quality-of-life measures in patients with colorectal cancer in whom other
treatments have failed. Non-FDA approved usage include (1) advanced colorectal
cancer, biweekly administration; (2) advanced non-small cell lung cancer (NSCLC),
EGFR-expressing; and (3) unresectable squamous cell skin cancer. A limitation of
use is that patients with a colorectal tumor-bearing mutated K-ras did not benefit
from cetuximab, whereas patients with a tumor-bearing wild-type K-ras did
benefit from cetuximab.
Question 124
• Question 124: A patient with rheumatoid arthritis developed severe
ulcerations of the buccal mucosa after being given a certain chemotherapeutic
agent. What medication was most likely given?
• Choices:
• 1. Dacarbazine
• 2. Procarbazine
• 3. Methotrexate
• 4. L-asparaginase
Answer: 3
• Answer: 3 - Methotrexate
• Explanations: Mucositis is a common complication of high dose methotrexate
administration. This occurs due to inhibition of cell proliferation in epithelial
cells. The mucositis can involve the esophagus and cause severe dysphagia.
Mucositis is a potential complication of most chemotherapeutic agents and
radiation therapy. The mucositis can occur anywhere along the GI tract, but the
oral cavity is commonly affected and can be very debilitating.
Question 125
• Question 125: Which is true about gastric lymphomas?
• Choices:
• 1. It is most common extra nodal site
• 2. It is never associated with H. pylori
• 3. Radical surgery is preferred treatment
• 4. Diagnosis can be made by CT scan
Answer: 1
• Answer: 1 - It is most common extra nodal site
• Explanations: Gastric lymphomas are very rare and account for only 2% of
stomach cancers. The stomach is the most frequent area for extra nodal
Hodgkin lymphoma. Diagnosis can be difficult because of vague symptoms and
usually requires endoscopic biopsy. Traditionally surgery was the preferred
method of treating lymphomas of the stomach but today combined
radiotherapy and chemotherapy are used. Prognosis for stomach lymphomas
depends on surgical resectability, female sex, early stage, and size less than 5
cm.
Question 126
• Question 126: A 35-year-old female enters the radiology department with a
history of a palpable, painful mass in her right breast with increased sensitivity
during the premenstrual phase. She has a weak second-degree family history of
paternal aunt diagnosed at 85 years for breast cancer.
• What would be the best first line of diagnostic imaging for this scenario?
• Choices:
• 1. In lieu of strong family history, a clinical breast examination and
comprehensive breast ultrasound is indicated
• 2. A biopsy of the mass is indicated
• 3. She should be referred for a mammogram at age 40 years
• 4. No imaging, the patient should return if symptoms worsen
Answer: 1

• Answer: 1 - In lieu of strong family history, a clinical breast examination and


comprehensive breast ultrasound is indicated
• Explanations: If no strong first-degree family history is present, and the patient
presents with a cyclical breast symptom, an ultrasound is the best first-line
imaging option at her age. A biopsy is only indicated in a radiology department,
following initial imaging and a raised suspicion of Breast Imaging Reporting and
Data System (BIRADS) 4-5 is reported. In the interim, an annual breast
ultrasound is indicated. Baseline ultrasound imaging is always indicated.
Question 127

• Question 127: What is the most important prognostic indicator for lung
cancer?
• Choices:
• 1. Size of cancer
• 2. Presence of positive lymph nodes
• 3. Involvement of chest wall
• 4. Central location
Answer: 2
• Answer: 2 - Presence of positive lymph nodes
• Explanations: The presence of positive lymph nodes is an important
prognosticator. Once mediastinal nodes are present, a cure is rarely obtained.
Patients with positive lymph nodes have to undergo chemotherapy and/ or
radiation. Despite many advances in treatment, survival rates have not
drastically improved. Prior to surgery, the patient's lymph node status must be
investigated to prevent him or her from undergoing unnecessary surgery.
Question 128
• Question 128: Which of the following factors is most important when treating
hyponatremia?
• Choices:
• 1. Degree of hyponatremia
• 2. Duration of hyponatremia
• 3. Volume status
• 4. Need to replace other electrolytes
Answer: 2
• Answer: 2 - Duration of hyponatremia

• Explanations: Treatment of hyponatremia should be guided by volume status,


as well as the duration and degree of the disorder. Chronic hyponatremia has
already allowed for coping mechanisms and should be gradually corrected.
One should start with isotonic saline in patients who are hypovolemic.
Patients with hypovolemia secondary to diuretics may also require potassium.
In chronic cases, the rate of replacement should be slow and not exceed 12
mEq/ day.
Question 129
• Question 129: What are the indications for radiation therapy after definitive
resection for Stage I or II lung cancer?
• Choices:
• 1. Positive margins and tumor size greater than 4 cm
• 2. Tumor size greater than 4 cm and positive hilar node
• 3. Positive mediastinal node and positive margins
• 4. Positive hilar
Answer: 3
• Answer: 3 - Positive mediastinal node and positive margins
• Explanations: Indications for adjuvant chemotherapy for completely resected
lung cancer are hilar or mediastinal node involvement (Stage II-III) and/ or
tumors greater than 4 cm (T2). Postoperative radiation dose range from 50-54
Gy but can be as high as 66 Gy for positive margins. Stereotactic body radiation
therapy (SBRT) or stereotactic ablative body radiation (SABR) is used for tumors
up to 5 cm in size. Surgical options to resect T1-T2 tumors include
pneumonectomy, lobectomy, and wedge or segmental resection.
Question 130
• Question 130: A 36-year old woman presents to her primary care physician complaining
of extreme fatigue and pain in her left breast. Her physician feels several hard nodules
upon palpation of her left breast and orders a diagnostic mammogram. The results return
positive for breast cancer, and her physician prescribes a combination of chemotherapy
drugs, including cyclophosphamide, prednisone, and oncovin. After several cycles of
treatment, the patient's breast cancer is in remission. Five years later, she returns with
similar episodes of fatigue and is diagnosed with bladder cancer. Which
chemotherapeutic drug could be the cause?
• Choices:
• 1. Cyclophosphamide
• 2. Prednisone
• 3. Oncovin
• 4. Tamoxifen
Answer: 1

• Answer: 1 - Cyclophosphamide
• Explanations: Cyclophosphamide has been associated with bladder
cancer. Other associations with bladder cancer are smoking, chemical
exposure at work, radiation, and bladder infection. Cyclophosphamide
is the only drug listed that irritates the bladder mucosa and is linked
to the development of cancer in these tissues. The development of
secondary malignancies are not as common as some other side
effects like hemorrhagic cystitis, but are still possible and should be
discussed with the patient.
Question 131
• Question 131: A Pancoast tumor is found in which of the following
structures?
• Choices:
• 1. Myocardium
• 2. Bone
• 3. Lung apex
• 4. Colon
Answer: 3
• Answer: 3 - Lung apex
• Explanations: A Pancoast tumor is a lung apical tumor. It is also known
as a superior sulcus tumor. It is associated with Horner syndrome:
miosis (constriction of the pupils), anhidrosis (lack of sweating), ptosis
(drooping of the eyelid), and enophthalmos (sunken eyeball). A
Pancoast tumor can cause Horner syndrome via compression of the
sympathetic ganglia in the neck. The presence of Horner syndrome
symptoms implies extra-thoracic invasion. Most Pancoast tumors are
NSCLC, but the diagnosis of this lesion is based on location, not
histology.
Question 132
• Question 132: What is the next step in a patient with stage II
thymoma where residual tumor is left behind?
• Choices:
• 1. Chemotherapy
• 2. Radiation
• 3. Bone marrow transplantation
• 4. Immunotherapy
Answer: 2
• Answer: 2 - Radiation
• Explanations: In patients who have stage II tumor with macroscopic invasion
into mediastinal fat or pleura, radiation therapy can be administered
postoperatively. Several studies have shown that people who undergo
incomplete thymoma resection and do not receive postoperative radiation have
a high incidence of recurrence. A large study from Memorial Sloan-Kettering
Cancer Center showed that adjuvant radiation therapy did not reduce
recurrence or improve survival in stage II and III disease. However, the current
practice is to administer radiation if there is any evidence of residual tumor
after resection. Studies show that people who benefit most from postoperative
radiation following surgery are (1) female patients, (2) patients with early-stage
thymoma, and (3) those who do undergo surgical removal of the thymoma.
Question 133
• Question 133: A 56-year-old man with a recent diagnosis of metastatic pancreatic
adenocarcinoma presents to the clinic for follow-up on his pain control regimen. He
reports that the oxycodone prescription he has been on is minimally effective in
treating his severe, persistent abdominal pain. Further, he is concerned about its
addictive potential and wishes to discuss other options. He wishes to discuss
smoking a cannabinoid preparation. Which of the following best describes the
legality and use of this drug?
• Choices:
• 1. It is a Schedule 1 drug, which means it is illegal.
• 2. It can be prescribed based on his specific diagnosis.
• 3. The drug has no medical application.
• 4. It is governed by state-specific legislation.
Answer: 4
• Answer: 4 - It is governed by state-specific legislation.
• Explanations: Although marijuana is currently classified as a schedule 1
drug, emerging clinical evidence has motivated some states to enact
legislation deeming medical marijuana legal. Since the Controlled
Substances Act was created in 1970, the Schedule 1 designation has
historically been defined as having no accepted medical use. Despite this
definition, there is growing evidence in favor of the use of medical
marijuana. The schedule list is updated annually in the Title 21 Code of
Federal Regulations. Medical indications include palliative (end-of-life)
conditions, chronic pain associated with a multitude of neurologic or
hematologic/ oncologic (i.e. cancer) diseases, psychiatric conditions, among
others. To date, there is no federal approval for medical marijuana.
Question 134
• Question 134: Osteogenic sarcoma most often affects the which
bone?
• Choices:
• 1. Wrist
• 2. Elbow
• 3. Ribs
• 4. Femur
Answer: 4
• Answer: 4 - Femur
• Explanations: Osteogenic sarcoma is a very aggressive cancer and
seen mostly in young adults or teenagers, though with a second
modal peak in the 60 year range (related to dedifferentiation of
Pagetoid bone.). The tumor tends to occur at the end of long bones
and is most common in the proximal tibial metaphysis or distal
femoral metaphysis. The tumor tends to occur around the knee and is
solid and irregular. Radiographs usually reveal a moth-eaten or a
sunburst appearance. MRI is the imaging modality of choice for
assessment of the primary lesion.
Question 135
• Question 135: Which malignancy can be treated with Bacillus
Calmette-Guerin (BCG) vaccine?
• Choices:
• 1. Basal cell cancer
• 2. Barrett esophagus
• 3. Bladder cancer
• 4. Lung cancer
Answer: 3
• Answer: 3 - Bladder cancer
• Explanations: Bacillus Calmette-Guerin (BCG) immunotherapy is used
to treat superficial urothelial cancers of the bladder. The vaccine has
been shown to decrease the rate of recurrence and progression.
However, there are many studies refuting the claim that BCG is
effective. BCG is administered weekly for 6 weeks. This is followed by
cystoscopy to determine if the vaccine has worked.
Question 136
• Question 136: Why is filgrastim administered during chemotherapy?
Choices:
• 1. Preventing epithelial cell damage
• 2. Preventing renal toxicity
• 3. Increasing bone marrow cells
• 4. Preventing cardiotoxicity
Answer: 3
• Answer: 3 - Increasing bone marrow cells
• Explanations: A major side effect of many chemotherapy drugs is
bone marrow suppression. Filgrastim is a granulocyte colony-
stimulating factor used to stimulate the bone marrow and increase
neutrophil production, maturation, and activation. The pegylated
form of filgrastim, pegfilgrastim, is often used in clinical practice to
increase its half-life. Filgrastim is available as a subcutaneous and
intravenous medication, while pegfilgrastim is only available
subcutaneously. Do not administer filgrastim between 24 hours
before and 24 hours after cytotoxic chemotherapy.
Question 137
• Question 137: A 5-year-old boy is brought to the clinic by his mother for a right
cheek swelling. It has been present for about two months and seems to be
getting bigger. The mother also mentions low-grade fevers and night sweats but
denies any pain. The swelling appears to be firm and matted, extending to a
diameter of 6 cm around the angle of the jaw. No tenderness or sinuses are
appreciated. What is the best initial step in the management of this patient?
• Choices:
• 1. Reassurance
• 2. Sputum for acid-fast bacilli
• 3. Bone marrow biopsy
• 4. Lymph node cytology
Answer: 1
• Answer: 1 - Reassurance
• Explanations: The clinical vignette is most indicative of a diagnosis of
Burkitt lymphoma. Prompt diagnosis is of utmost importance as cure
rates can be as high as 90% when diagnosed early. Lymph node biopsy
is usually the best test to evaluate for Burkitt lymphoma. The typical
microscopy of Burkitt lymphoma shows the "Starry sky" pattern on
H& E staining. Tuberculous lymphadenitis is unlikely in this patient as
there are no risk factors mentioned. Moreover, even if tuberculous
lymphadenitis were suspected, sputum testing would not be the best
test. Bone marrow biopsy is rarely employed since Burkitt lymphoma
is a lymphoid malignancy.
Question 138
• Question 138: The majority of post renal transplant cancers have
been linked to which virus?
• Choices:
• 1. Cytomegalovirus
• 2. Epstein Barr virus
• 3. Herpes virus
• 4. HIV
Answer: 2
• Answer: 2 - Epstein Barr virus
• Explanations: Epstein-Barr virus (EBV) is associated with lymphomas
especially in patients who have had renal transplants. Hepatitis B and
C are linked to hepatocellular carcinoma. Human herpes virus 8 is
linked OT Kaposi sarcoma and lymphomas. Human papilloma viruses
are linked to cervical, penile, and head an neck cancers. Antiviral
medications are often given to renal transplant patients to reduce
infections.
Question 139
• Question 139: A 65-year-old man presents to the clinic to discuss the
results from a recent colonoscopy. The biopsy report shows
adenocarcinoma. Which of the following factors, if present, is most
likely to have contributed to his risk of colon cancer?
• Choices:
• 1. Colon cancer in a cousin at age 58 years
• 2. A glass of alcohol every week
• 3. 20-pack-year smoking history, quit one year ago
• 4. Hypertension
Answer: 3
• Answer: 3 - 20-pack-year smoking history, quit one year ago
• Explanations:
• Colorectal Cancer is the fourth most common cancer and the second leading cause of cancer death in
the United States (US), thus representing a major public health problem.
• Community-based estimates reveal an increased prevalence with age and male gender while several
studies have also reported an increased risk of colon cancer in patients with a first-degree relative
family history, smoking history, excess alcohol intake, diabetes mellitus and higher body-mass index
which can be a reflection of high-fat, low-fiber diet and low physical activity.
• Smoking history is a strong risk factor for colon cancer.
• Although first-degree relative family history is a risk factor for developing colon cancer, having a
cousin diagnosed with cancer is not a risk factor for the disease.
• Although excess alcohol intake is a risk factor for developing colon cancer, one glass of beer is not
considered a risk factor.
• Hypertension is not a risk factor for colon cancer.
Question 140
• Question 140: On chest x-ray, what is the earliest presentation of non-
small cell lung cancer?
• Choices:
• 1. Cavity
• 2. Solitary nodule
• 3. Hilar adenopathy
• 4. Consolidation
Answer: 2
• Answer: 2 - Solitary nodule
• Explanations: Ninety-five percent of patients with non-small cell lung
cancer (NSCLC) are symptomatic at presentation and most have some
evidence of lung pathology on chest x-ray. A solitary pulmonary
nodule is seen in the majority with non-small cell cancer. Of course, a
majority of solitary pulmonary nodules (SPNs) are not malignant, but
depending on clinical and imaging context, many require workup. The
workup of an SPN may vary from institution to institution and can
involve careful history, comparison to old studies, CT (including
dynamic contrast enhancement characteristics), and PET/ CT.
Question 141
• Question 141: A 49-year-old presents with loss of weight, anemia, and
fatigue. Blood work reveals a WBC of 88 and a Hb of 9, He has a mildly
enlarged spleen and liver. Numerous petechiae are visible on the skin.
Genetic analysis reveals a Bcr/ abl translocation. He most likely has:
• Choices:
• 1. Acute myeloid leukemia
• 2. Acute lymphoblastic leukemia
• 3. Chronic myelogenous leukemia
• 4. Chronic lymphocytic leukemia
Answer: 3
• Answer: 3 - Chronic myelogenous leukemia
• Explanations: The Philadelphia (Ph) chromosome is translocation
between chromosomes 9 and 12 t( 9;22)( q34; q11). The BCR ("
breakpoint cluster region") gene from chromosome 22q11 fuses with
the ABL at chromosome 9q34. ABL stands for the name of the
leukemia virus, Abelson. It is present in 95% of patients with CML.
Patients with CML and without the Ph chromosome have worse
prognosis. The Ph chromosome is see in 25 to 30% of adults with ALL
and 2 to 10% of children.
Question 142
• Question 142: Turcot syndrome type 1, also known as brain tumor-
polyposis syndrome type 1, most commonly is associated with which
type of brain tumor?
• Choices:
• 1. Medulloblastoma
• 2. Pilocytic astrocytoma
• 3. Ependymoma
• 4. Glioblastoma multiforme
Answer: 4
• Answer: 4 - Glioblastoma multiforme
• Explanations: Medulloblastomas are the most common brain tumor
seen in patients with Turcot syndrome type 2. Turcot syndrome type 2
is due to a homozygous mutation of the APC gene. Although they are
uncommon, pilocytic astrocytomas can arise in patients with Turcot
syndrome type 1. Although they are uncommon, ependymomas can
arise in patients with Turcot syndrome type 1. Glioblastoma
multiforme is the most commonly associated brain tumor with Turcot
syndrome type 1. Turcot syndrome type 1 is due to MMR gene
mutations. This tumor is high-grade.
Question 143
• Question 143: What is the primary toxicity of bleomycin?
• Choices:
• 1. Nephritis
• 2. Hemorrhagic cystitis
• 3. Lung fibrosis
• 4. Neuropathy
Answer: 3
• Answer: 3 - Lung fibrosis
• Explanations: Bleomycin is a glycopeptides and often used to treat
lymphomas. Bleomycin is felt to work by preventing incorporation of
thymidine into DNA strands. Bleomycin chelates metal iron and
produces free radicals which damage the DNA. The most serious side
effects of bleomycin is lung fibrosis and impaired lung function. The
lung toxicity is related to oxygen free radicals. Other side effects of
bleomycin include alopecia, dermatographism and raynaud
phenomenon.
Question 144
• Question 144: A 30-year-old female presents with an easily palpable and
firm nodule on the left side of her thyroid. High-resolution ultrasound of
the anterior region of the neck reveals a 1.3 cm nodule in the left lower
pole of the thyroid gland. FIne-needle aspiration (FNA) revealed neoplastic
papillae with a central fibrovascular core outline by several layers of cells
with crowded nuclei. What is the prognosis associated with the most likely
malignancy?
• Choices: 1. 5-year survival rate of > 60%
• 2. 5-year survival rate > 90%
• 3. 5-year survival rate < 20%
• 4. 1-year survival rate < 20%
Answer: 2
• Answer: 2 - 5-year survival rate > 90%
• Explanations: These histological findings describe papillary thyroid
cancer, which has a very good survival rate with treatment. The first step
in evaluating a thyroid nodule is to asses thyroid function by checking the
thyroid-stimulating hormone (TSH). If TSH is low, then the radioactive
iodine uptake scan is done. Fine-needle aspiration biopsy (FNAB) is the
best measure for determining the histopathologic diagnosis. Ultrasound
findings associated with malignancy include microcalcifications, irregular
margins, hypoechogenicity, taller-than-wide shape, and increased
vascularity. Contrary to papillary thyroid cancer, anaplastic thyroid cancer
has a very poor prognosis with a 1-year survival rate of < 20%.
Question 145
• Question 145: Which of the following is the most common
malignancy of the female pelvic region?
• Choices:
• 1. Endometrial
• 2. Vaginal
• 3. Cervical
• 4. Fallopian tube
Answer: 1
• Answer: 1 - Endometrial
• Explanations: Uterine cancer is the most common gynecological
malignancy in women. Ovarian cancer is the second most common.
Cervical cancer is the third most common. The incidence of
endometrial cancer is high in white females compared to African
Americans, however, the mortality rates are higher in the latter.
Question 146
• Question 146: Which of the following is most likely to cause very high
levels of alkaline phosphatase?
• Choices:
• 1. Multiple myeloma
• 2. Paget disease
• 3. Prostate cancer with bone metastases
• 4. Bone fracture
Answer: 3
• Answer: 3 - Prostate cancer with bone metastases
• Explanations: Elevated levels of alkaline phosphatase are seen in
many disorders. Among bone disorders, the highest levels are seen in
patients with prostate cancer with metastatic bone disease. Other
disorders associated with elevation of alkaline phosphatase include
osteosarcoma, Paget disease, renal osteodystrophy, and fractured
bone. If it is unclear why alkaline phosphatase is elevated, isoenzyme
studies using electrophoresis can confirm the source of the ALP.
Question 147
• Question 147: Which of the following adverse events has been
commonly associated with the use of vincristine?
• Choices:
• 1. Gastric ulcers
• 2. Diarrhea
• 3. Constipation
• 4. Pulmonary rales
Answer: 3
• Answer: 3 - Constipation
• Explanations: Constipation might result from high dose vincristine
therapy. Vincristine is known to cause neuropathy manifested by
tingling and numbness. Constipation may be managed by daily
enema. Pulmonary rales are caused by drugs such as bleomycin that
have pulmonary toxicity.
Question 148
• Question 148: A 63-year-old female undergoes a lumpectomy and an
axillary node dissection. The biopsy reveals an intraductal cancer, and
four of the lymph nodes are positive. What is the appropriate next
step in her management?
• Choices:
• 1. Radiation
• 2. Chemotherapy
• 3. Tamoxifen
• 4. Radical mastectomy
Answer: 2
• Answer: 2 - Chemotherapy
• Explanations: Adjuvant therapy is of benefit to women who have four
or more positive lymph nodes. Patients who have four or more lymph
nodes usually have a much poorer prognosis than patients who have
no nodes. Prophylactic oophorectomy has not been shown to
improve survival. Multiple drug therapy is more effective than single-
dose chemotherapy.
Question 149
• Question 149: Phlebitis occurs when parenteral nutrition contains
which of the following?
• Choices:
• 1. Low glucose
• 2. High glucose
• 3. Low sodium
• 4. High sodium
Answer: 2
• Answer: 2 - High glucose Explanations: Phlebitis is associated with a
high glucose content. High potassium rates can lead to phlebitis
Central administration of total parenteral nutrition avoids these
complications. 3% saline can cause phlebitis.
Question 150
• Question 150: What is the most frequent mode of transmission in
nosocomial infections?
• Choices:
• 1. Airborne transmission
• 2. Contact transmission
• 3. Common vehicle transmission
• 4. Droplet transmission
Answer: 2
• Answer: 2 - Contact transmission
• Explanations: Contact transmission is the most common and frequent
cause of nosocomial infection. Hand washing is the single most
important measure of infection prevention. Isolation precautions also
prevent transmission by common routes. The other choices are
involved with nosocomial infections, but are not as frequently the
cause.
Question 151
• Question 151: A 29-year-old female comes into the primary care office. The patient
has complaints of headaches and an inability to become pregnant. The patient has
been trying to get pregnant for one year now. The patient's husband has one child
of his own. Her past medical history is unremarkable. On physical examination, she
is found to have bitemporal hemianopia. The clinician order an MRI scan for the
patient. What part of the brain would most likely show the lesion on the MRI scan
resulting in her visual field defects?
• Choices:
• 1. Wing of sphenoid
• 2. Petrosal sinus
• 3. Foramen ovale
• 4. Sella turcica
Answer: 4
• Answer: 4 - Sella turcica
• Explanations: The compression of the optic chiasm will lead to
bitemporal hemianopia. The sella turcica is located above the optic
chiasm. Enlargement of the pituitary gland can cause compression of
the optic chiasm and infertility symptoms. The most common cause of
compression of the optic chiasm is from a prolactinoma. A
prolactinoma compressing the optic chiasm will lead to bitemporal
hemianopia. The overproduction of the prolactin hormone results in
the inhibition of follicle-stimulating hormone (FSH) and luteinizing
hormone (LH). The suppression of FSH and LH causes infertility.
Question 152
• Question 152: The ultimate prognosis of a patient with carcinoma of
the esophagus depends upon which of the following?
• Choices:
• 1. Technique of resection
• 2. Pre-operative chemotherapy combined with radiation
• 3. Postoperative chemotherapy
• 4. Pathologic staging
Answer: 4
• Answer: 4 - Pathologic staging
• Explanations: Staging is essential for prognosis. How the esophagus is
resected and other adjuvant therapies do not affect survival. Staging
is accomplished with ultrasound. Stage I is disease limited to the
submucosa. Stage II is disease limited to, and not extending past, the
muscularis. Stage III is penetrating, full-thickness involvement to peri-
esophageal tissue. Stage IV is involvement of adjacent organs.
Question 153
• Question 153: What is the five-year survival rate for bone metastasis
from breast cancer?
• Choices:
• 1. Less than 1%
• 2. 2% to 5%
• 3. 15%
• 4. More than 40%
Answer: 3
• Answer: 3 - 15%
• Explanations: Bone metastasis from breast cancer has a five-year
survival rate of 15%. For the majority of patients with bone
metastases, the prognosis is poor. In women with breast cancer, the
bone is the most frequent site of metastasis. At least two-thirds of
patients with breast cancer experience distant bone relapse. Breast
cancers with estrogen and progesterone receptor positivity, a low
mitotic rate, and intermediate histologic grade, have the highest risk
of skeletal spread.
Question 154
• Question 154: What is the primary malignancy that most frequently
metastasizes to the adrenal gland?
• Choices:
• 1. Melanoma
• 2. Hepatocellular cancer
• 3. Contralateral adrenocortical carcinoma
• 4. Non-squamous cell lung cancer
Answer: 1
• Answer: 1 - Melanoma
• Explanations: Adrenal metastases can be unilateral or bilateral and can vary in
size. They are usually clinically silent and do not disrupt adrenal hormonal
function. The adrenal gland is the fourth most common site for metastatic
disease. Melanoma metastasizes to the adrenal gland about 50% of the time and
to the breast and lung about 30% to 40% of the time. Although melanoma more
frequently metastasizes to the adrenal gland, it has a much lower prevalence
than lung or breast cancer. Thus, a given adrenal gland metastasis is more likely
from the lung or breast, not from melanoma. Even in patients with known
primary malignancies, incidental adrenal adenomas are common. MRI can
usually distinguish between these processes in a patient with an adrenal mass,
especially for melanoma metastases. These classically show an increased signal
on T1 due to the paramagnetic properties of melanin.
Question 155
• Question 155: A 76 year-old-female with stage 4 pancreatic cancer has been
maintained on morphine along with the hydrocodone for breakthrough pain. Over the
last few weeks, she reports that the hydrocodone "is not reducing my breakthrough
pain like it used to." The pain physician decides to do an opioid rotation to oxycodone
from hydrocodone. Which of the following is true concerning opioids and cancer
pain?
• Choices:
• 1. Opioids that can suppress and cause respiration depression should be avoided.
• 2. While converting from one opioid to another, start with 50% to 75% of the
equianalgesic dose.
• 3. Naloxone should always be used if the respiratory rate is less than 10 per minute.
• 4. The dose should always be rapidly increased if the pain is intense.
Answer: 2
• Answer: 2 - While converting from one opioid to another, start with 50% to 75% of
the equianalgesic dose.
• Explanations: Tolerance between different opioids varies, so when changing from
one opioid to another, it is most appropriate to start with 50% to 75% of the
equianalgesic dose. Adjunctive medications, most with sedating effects, as well as
co-analgesics (NSAIDs or acetaminophen), are recommended to decrease opioid
requirements and/ or improve analgesia. Respiratory depression is uncommon in
patients treated with opioids at stable doses for cancer pain. Most of these patients
develop tolerance to opioids; however, the patient needs to be monitored during
dose escalations or conversion to other opioids. The dose of an opioid often has to
be increased due to the development of tolerance, but in pain due to cancer, dose
adjustments are often necessary due to disease progression, e.g., metastasis to
bones or other organs or eroding into or compressing nerves.
Question 156
• Question 156: A patient is receiving vincristine sulfate. When
assessing the patient, it is necessary to do which of the following?
Choices:
• 1. Note any changes in urinary patterns
• 2. Check skin surfaces for color changes
• 3. Ask the client for presence of metallic taste in mouth
• 4. Check hand grasps and deep tendon reflexes
Answer: 4
• Answer: 4 - Check hand grasps and deep tendon reflexes
• Explanations: Assess the patient's hand grasps and deep tendon
reflexes. Side effects of vincristine sulfate include peripheral
neuropathies. Assessing hand grasps and deep tendon reflexes can
help to determine if the patient's sensorimotor functioning is intact.
Urinary pattern disturbances occur with cytoxan and adriamycin.
Fluorouracil may cause skin surface changes. A metallic taste in the
mouth occurs with fluorouracil and cisplatin.
Question 157
• Question 157: A 55-year-old man with a past medical history of testicular
cancer diagnosed at the age of 35, presents to the hospital with shortness
of breath. On exam, his vital signs are stable except for a respiratory rate of
22/ min and an oxygen saturation of 90% on room air. His lung exam reveals
dry crackles. A CT chest is completed, which shows pulmonary fibrosis.
Which of the following medications, if taken, is most likely to have caused
the patient's condition? Choices:
• 1. Dexamethasone
• 2. Bleomycin
• 3. Doxorubicin
• 4. Vincristine
Answer: 2
• Answer: 2 - Bleomycin
• Explanations: Bleomycin is associated with pulmonary fibrosis and
impaired lung function. Bleomycin induces sensitivity to oxygen
toxicity and promoted proinflammatory cytokines IL-18 and IL-1beta
to cause lung injury. Bleomycin is used in the treatment of testicular
cancers. Doxorubicin causes cardiotoxicity.
Question 158
• Question 158: A 43-year-old woman presents to her provider. She complaints of irregular menstrual
cycles for the past two years; the intervals between her periods have extended as long as 3 or 4
months. She denies painful cramps but notes that the periods are heavier than before the irregularity
began. She denies pain during intercourse. She also notes that she has been gaining weight for the
past year, estimating that she is nearly 10.3 kg (22lb) heavier than she was at her last annual checkup.
Upon examination, she is 160 cm (5'3'') tall and weighs 77 kg (170 lb). Her face is round and flushed,
and her weight is concentrated around her abdomen; her arms and legs are comparatively thinner.
Her breast and pelvic examinations are unremarkable, except for thick, reddish-purple streaks around
her breasts. All her vitals are unremarkable except for a blood pressure of 160/ 100 mmHg. Laboratory
findings show an abnormal overnight dexamethasone suppression test and high 24-hour urine free
cortisol. Her adrenocorticotropic hormone (ACTH) level is 3 pg/ ml (Reference value: > 10 pg/ ml).
Which of the following is the most likely diagnosis?
• Choices:
• 1. Adrenocortical Carcinoma
• 2. Lung Cancer
• 3. Pituitary adenoma
• 4. Pheochromocytoma
Answer: 1
• Answer: 1 - Adrenocortical Carcinoma
• Explanations: Adrenocortical carcinoma (ACC) can present classically in three
different forms. About one-third of them present with symptoms of
hormonal excess, another third present with non-specific symptoms, and the
last third are diagnosed with imaging studies when they present for other
medical conditions. Hormonal excess could be due to hypercortisolism,
hyperandrogenism, or both. Hypercortisolism (Cushing’s syndrome) can
present as plethora (rounded face), diabetes mellitus, osteoporosis, purple
striae on the abdominal wall, muscle weakness, truncal obesity, and muscle
atrophy. Glucocorticoid-mediated mineralocorticoid receptor activation due
to saturation of 11-ß-hydroxysteroid dehydrogenase 2 (HSD11B2) system can
present with hypokalemia and
• hypertension. Malignant adrenal tumors usually present with a clinical presentation of
hypercortisolism, rapidly progressing muscle weakness, and pronounced muscle
weakness. Hyperandrogenism can present with male pattern baldness, virilization,
hirsutism, and menstrual abnormalities. The initial evaluation is in part guided by clinical
symptoms (e.g., cushingoid features, hirsutism, and new hypertension with or without
hypokalemia) — blood and urine test help to identify functional versus non-functional
tumors. Most patients with cortisol-secreting tumors will have suppressed ACTH (< 10
pg/ mL) and increased cortisol on a spontaneous 8: 00 AM blood draw. The diagnosis of
hypercortisolism is usually established by a 1 mg dexamethasone suppression test (DST),
midnight salivary cortisol, or elevated 24-hour urine free cortisol. This patient has a
Cushing ´ s syndrome with a low level of ACTH, that means that the next study is to
control de adrenal glands with a CT image and biopsies. Low level of ACTH rules out a
diagnosis of paraneoplastic syndrome of lung cancer and pituitary syndrome.
Pheochromocytoma does not have a clinical picture similar to Cushing's syndrome.
Question 159
• Question 159: Which of the listed antineoplastic agents carries a
requirement for a special restricted distribution program as a result of
its teratogenic profile?
• Choices:
• 1. Vincristine
• 2. Thalidomide
• 3. Carboplatin
• 4. Cyclophosphamide
Answer: 2
• Answer: 2 - Thalidomide
• Explanations: Thalidomide has a well-recognized teratogenic profile.
As a result, in the United States, the drug may not be given except
under a special program known by the acronym S.T.E.P.S. (System for
Thalidomide Education and Prescribing Safety). Prescribing and
dispensing thalidomide is limited only to physicians and pharmacists
registered in this program. Thalidomide currently is used primarily in
the treatment of multiple myeloma.
Question 160
• Question 160: What is the most common type of leukemia in adults?
Choices:
• 1. Acute myelogenous leukemia
• 2. Chronic myelogenous leukemia
• 3. Acute lymphocytic leukemia
• 4. Chronic lymphocytic leukemia
Answer: 4
• Answer: 4 - Chronic lymphocytic leukemia
• Explanations: Chronic lymphocytic leukemia (CLL) is the most
common leukemia in adults in the Western world. CLL is more
frequently diagnosed in the elderly. The median age at diagnosis is 72
years. Most cases of CLL are B-cell neoplasms, although T-cell CLL is
also recognized. B-cell CLL can be diagnosed by immunophenotyping
of peripheral blood. The leukemic cells typically are positive for
normal B-cell surface antigens (CD19, CD20, CD21, and CD23) and
CD5, which is more typical of T cells.
Question 161
• Question 161: Which histologic type of lung cancer is most frequently
associated with hypercalcemia?
• Choices:
• 1. Small cell
• 2. Squamous cell
• 3. Adenocarcinoma
• 4. Carcinoid
Answer: 2
• Answer: 2 - Squamous cell
• Explanations: Squamous cell carcinoma of the lung is the most common cause of
humoral hypercalcemia of malignancy, cancer-associated hypercalcemia occurring
in the absence of bony metastases. The mechanism of hypercalcemia in squamous
cell carcinoma is the production of parathyroid hormone-related protein (PTHrP) by
the tumor. PTHrP shares some homology with parathyroid hormone (PTH), binds to
the PTH receptor, and exerts similar effects on calcium metabolism as does PTH.
When a patient with lung cancer presents with hypercalcemia, PTHrP should be
measured. In patients with PTHrP-associated hypercalcemia, serum PTH is low as its
production is suppressed. Though hypercalcemia from the described mechanism
can occur with any stage of lung cancer, it typically is associated with advanced
disease and poor survival. In those cases in which the tumor is fully resectable, both
the elevated PTHrP and the associated hypercalcemia are reversible.
Question 162
• Question 162: What is the average clinical latency period between
asbestos exposure and development of mesothelioma?
• Choices:
• 1. 2 years
• 2. 5 years
• 3. 15 years
• 4. 35 years
Answer: 4

• Answer: 4 - 35 years
• Explanations: There is a long period between development of
mesothelioma and prior asbestos exposure. Most cases of
mesothelioma occur after 30-40 years. Even though asbestosis
exposure is less of an issue in the United States, mesotheliomas still
continue to be seen with an increased frequency. The majority of
patients present with symptoms in the 6th or 7th decades of life.
Mesothelioma also occurs in children but this is not thought to be
related to asbestos exposure,
Question 163
• Question 163: A 52-year-old female presents with a newly diagnosed
breast tumor. She is unsure of her exact staging but states that she
remembers her tumor is T1. Under the TMN classification, a T1 tumor
is most consistent with which of the following?
• Choices:
• 1. A large tumor with extensive invasion
• 2. Involvement of several regional lymph nodes
• 3. Carcinoma in situ
• 4. A small, minimally invasive tumor
Answer: 4
• Answer: 4 - A small, minimally invasive tumor
• Explanations: T is used for the classification of tumor size and
invasion. T1 through T3 are indicative of progressively larger or
invasive tumors. A large tumor with extensive invasion (answer choice
1) is more consistent with a T3 tumor than T1. Carcinoma in situ is
indicated by Tis. The involvement of regional nodes is classified by N,
not T. The TNM classification has a prognostic and diagnostic impact.
Question 164
• Question 164: Which of the following biomarkers is considered to be
useful in diagnosing prostate cancer?
• Choices:
• 1. Carcinoembryonic antigen (CEA)
• 2. Cancer antigen 125 (CA-125)
• 3. Alpha-fetoprotein (AFP)
• 4. Prostate-specific antigen (PSA)
Answer: 4
• Answer: 4 - Prostate-specific antigen (PSA)
• Explanations: Prostate-specific antigen (PSA) is high in males with
significant prostate cancer. There is debate as to whether PSA has
utility as a screening test. CEA and AFP may be used as biomarkers for
colorectal cancer. CA-125 is a biomarker for ovarian cancer.
Question 165
• Question 165: A 41-year-old woman with stage IIB breast cancer presents to the
clinic to receive her first chemotherapy regimen of docetaxel and
cyclophosphamide. Docetaxel has a boxed warning for bone marrow suppression,
and cyclophosphamide also causes bone marrow suppression. The patient's
absolute neutrophil count is currently 5767/ microL. She is prescribed pegfilgrastim.
Which of the following best describes the mechanism of action of this medication?
• Choices:
• 1. Preventing epithelial cell damage
• 2. Increasing lymphocyte production
• 3. Increasing neutrophil production
• 4. Preventing the breakdown of bone tissue
Answer: 3
• Answer: 3 - Increasing neutrophil production
• Explanations: A major adverse effect of many chemotherapy drugs is
bone marrow suppression. Filgrastim and pegfilgrastim are used to
stimulate the bone marrow by increasing neutrophil production. The
pegylated form of filgrastim, known as pegfilgrastim, is often used in
clinical practice due to its longer half-life. Pegfilgrastim can cause
leukocytosis; therefore, it is important to monitor the patient's white
blood cell counts when receiving this medication.
Question 166
• Question 166: Which parotid gland tumor has a tendency to invade
along nerves?
• Choices:
• 1. Pleomorphic adenoma
• 2. Warthin’s
• 3. Lymphomas
• 4. Adenoid cystic
Answer: 4
• Answer: 4 - Adenoid cystic
• Explanations: Adenoid cystic cancers are rare and usually occur in
salivary glands. Patients survive for decades with this cancer because
it is very slow growing. Adenoid cystic cancer have a propensity to
invade nerves and cause pain or nerve paralysis. Even though growth
is slow, distant spread can occur to the lung followed by the liver.
Question 167
• Question 167: A patient receiving chemotherapy has developed
myelosuppression. What is the therapeutic priority in such an
instance?
• Choices:
• 1. Infections
• 2. Reduced capacity for physical activity
• 3. Anemia
• 4. Renal damage
Answer: 1
• Answer: 1 - Infections
• Explanations: Myelosuppression results in the bone marrow
producing insufficient erythrocytes or leukocytes. Leukopenia leaves
the patient susceptible to infection. Anemia is also a consideration
but can be treated with packed red blood cells. Infection is the
overriding concern.
Question 168
• Question 168: Which is true of nasopharyngeal cancer?
• Choices:
• 1. It is a cause of hemoptysis
• 2. It is a cause of dysphagia
• 3. It is a cause of otalgia
• 4. It is a cause of hematemesis
Answer: 3
• Answer: 3 - It is a cause of otalgia
• Explanations: The most common presentation of nasopharyngeal
cancer is cervical adenopathy. Diagnosis is made by biopsy of the
nodes. Other symptoms may include trismus, pain, otalgia, otitis
media, and nasal regurgitation. Cranial nerve palsies and hearing loss
are late features of the cancer. Large neglected masses can cause
bleeding and nasal obstruction. Metastatic spread usually migrates to
the bones and causes bone pain.
Question 169
• Question 169: Which chromosome has the gene locus responsible for
NF-2?
• Choices:
• 1. 17g11.2
• 2. 22q12.2
• 3. Xq27.3
• 4. 15g11
Answer: 2
• Answer: 2 - 22q12.2
• Explanations: NF-2 is known as Merlin and acts as a tumor
suppressor. Decreased production or function results in increased
likelihood of tumors of the central and peripheral nervous systems.
Half of patients with NF-2 have a de nova mutation.
Question 170
• Question 170: A previously healthy 16-year-old male with no recent
illnesses presents to his family physician complaining of increasingly
worsening knee pain over several weeks. His doctor orders a
radiograph, and the radiologist reports a finding of the Codman triangle
in the distal femur. What is the most likely treatment course?
• Choices:
• 1. Open reduction internal fixation
• 2. External casting
• 3. Surgical resection and chemotherapy
• 4. Antibiotics
Answer: 3
• Answer: 3 - Surgical resection and chemotherapy
• Explanations: Codman triangle is most often a sign of aggressive bone
lesions, the most common being osteosarcoma. Osteosarcoma is the
most likely diagnosis in a young patient with Codman triangle who
presents with gradually worsening bone pain. Osteosarcoma is an
aggressive neoplasm that requires aggressive treatment with surgical
resection followed by chemotherapy. Osteosarcoma is most
commonly found in the long bones of young patients, the most
common being the femur. Antibiotics would be the treatment choice
in the case of osteomyelitis. The patient's clinical presentation
supports a diagnosis of osteosarcoma over osteomyelitis.
Question 171
• Question 171: A 72-year-old man presents to the clinic with a recent
onset of urinary hesitation. Upon physical exam, he is found to have a
nodular and hard prostate. Which of the following factors is
associated with the worst prognosis in this patient?
• Choices:
• 1. PSA of 5 ng/ mL (reference range: < 4 ng/ mL)
• 2. Hard prostate
• 3. Gleason score of 11
• 4. Urinary symptoms
Answer: 3
• Answer: 3 - Gleason score of 11
• Explanations: The Gleason score is a histologic analysis that is used to
assess prostate cancer. The lower the Gleason score, the more the
cells appear to be normal. The higher the Gleason score, the worse
the prognosis. A PSA of 5 ng/ mL is elevated; however, the PSA is
usually significantly more elevated in advanced disease (> 10 ng/ mL).
Question 172
• Question 172: Which diagnosis should be considered in a 65-year-old
male who presents with weight loss, anorexia, back pain, and
migratory thrombophlebitis?
• Choices:
• 1. Ankylosing spondylitis
• 2. Pott disease
• 3. Pancreatic adenocarcinoma
• 4. Cholangitis
Answer: 3
• Answer: 3 - Pancreatic adenocarcinoma Explanations: Pancreatic
cancer often does not present with symptoms, and the diagnosis is
usually delayed. Loss of weight, painless jaundice, and dark urine are
often late signs of this cancer. Trousseau sign is the formation of
migratory thrombophlebitis in any vein of the body. Diabetes mellitus
often precedes the cancer and may be a clue for the cancer.
Question 173
• Question 173: Which is true about metastases?
• Choices:
• 1. Sarcomas usually spread to the liver
• 2. Some are dormant for many years
• 3. Are common with basal cell cancer
• 4. Can often occur during cancer surgery
Answer: 2
• Answer: 2 - Some are dormant for many years Explanations:
Metastases indicate aggressive tumors and often have a poor
prognosis. Spread can be by hematogenous or lymphatic means.
Sarcomas are locally aggressive but may spread to lymph nodes or the
lungs. Basal cell carcinoma rarely metastasizes. Very rarely, cancer can
be spread by surgery. This is a commonly believed myth. Some
metastases can be dormant for years. Malignant melanoma can recur
years later.
Question 174
• Question 174: Which of the following is correct regarding the staging
of non-small cell lung cancer using the TMN staging system?
• Choices:
• 1. T2 indicates tumor size is between 3 cm and 7 cm
• 2. N0 indicates spread to regional lymph nodes
• 3. M1a indicates cancer has spread to distant lymph nodes or other
organs (i.e. liver bones or brain)
• 4. T1 indicates there is no evidence of primary tumor
Answer: 1
• Answer: 1 - T2 indicates tumor size is between 3 cm and 7 cm
Explanations: TNM staging describes tumor size (T), lymph node
involvement (N) and metastasis (M). T2 indicates the tumor has one
or more of the following features: growth into the visceral pleura,
involvement of the main bronchus but greater than 2 cm away from
the carina, is between 3cm and 7 cm in length, or partial occlusion of
airways. N0 indicates no spread to regional lymph nodes. M1a
indicates malignant pleural effusion or separate tumor nodule in a
contralateral lobe tumor with pleural nodules.
Question 175
• Question 175: A 16-year-old patient with a diagnosis of acute
lymphocytic leukemia receiving chemotherapy presents to the
emergency room with complaints of lethargy, nausea, vomiting,
arthralgias, and flank pain. What is the mechanism of action of the
medication used to prevent these symptoms?
• Choices:
• 1. Cyclo-oxygenase (COX) inhibitor
• 2. Xanthine oxidase inhibitor
• 3. Angiotensin-converting enzyme (ACE) inhibitor
• 4. Beta-lactam inhibitors
Answer: 4
• Answer: 4 - Beta-lactam inhibitors Explanations: Tumor lysis syndrome is a clinical
condition that can occur spontaneously or after initiation of chemotherapy
associated with the following metabolic disorders: hyperkalemia,
hyperphosphatemia, hypocalcemia, and hyperuricemia leading to end-organ
damage. The symptoms experienced by this patient such as arthralgias and flank
pain are probably due to hyperuricemia. Xanthine oxidase (XO) is an enzyme that
catalyzes the oxidation of hypoxanthine to xanthine, and of xanthine to uric acid.
Inhibiting this enzyme will help lower uric acid levels. Allopurinol inhibits
xanthine oxidase, preventing the conversion of urate to uric acid. Tumor lysis
syndrome is associated with acute leukemias and high-grade non-Hodgkin
lymphomas. The rapid cell turnover can overwhelm the renal system leading to
an increase of circulating metabolites. Excessive uric acid builds up in the kidney
precipitating into uric acid crystals and eventually leading to nephrolithiasis.
Question 176
• Question 176: Which is a cell cycle specific agent that is well known to
cause peripheral neuropathy?
• Choices:
• 1. Cyclophosphamide
• 2. Busulfan
• 3. Vincristine
• 4. Taxol
Answer: 3
• Answer: 3 - Vincristine Explanations: Vincristine is an antineoplastic
plant alkaloid. Vincristine is a cell cycle (M phase) specific agent and
inhibits mitotic spindle formation. Toxicity of vincristine includes
peripheral neuropathy but does not cause myelosuppression. The first
feature of peripheral neuropathy is a foot drop. In many cases, even
after the drug is discontinued the neuropathy still persists. Individuals
with diabetes or other types of neuropathy should not be treated
with vincristine.
Question 177
• Question 177: What is the most efficient method to prevent the
spread of infection in hospitalized patients?
• Choices:
• 1. Wearing gloves
• 2. Have the patient use antibiotic soap
• 3. Hand washing
• 4. Wear gowns
Answer: 3
• Answer: 3 - Hand washing Explanations: Hand washing is the most
efficient way to prevent infection spread. Infectious spread in
hospitals is not patient-patient, but patient-hospital worker-patient,
so hospital worker needs to wash hands frequently and before each
patient encounter. Gloving and gowning is required in certain cases
per hospital infectious disease protocols, but is not needed
universally, unlike hand washing. The responsibility for minimizing the
spread of infection in hospitals is the hospital employees', not the
patients.
Question 178
• Question 178: Which of the following antiemetic medications has
actions on the 5-HT3 receptor?
• Choices:
• 1. Ondansetron
• 2. Metoclopramide
• 3. Scopolamine
• 4. Chlorpromazine
Answer: 1
• Answer: 1 - Ondansetron Explanations: Ondansetron is a serotonin 5-
HT3 receptor antagonist with excellent antiemetic effects. It is
commonly used to treat chemotherapy induced emesis. It can be
administered orally or IV and works well. It is a selective 5-HT3
receptor with no activity against dopamine or muscarinic receptors.
The one downside is the cost and hence it is not always the drug of
first choice.
Question 179
• Question 179: Which of the following increases the incidence of
tumor lysis syndrome?
• Choices:
• 1. Tumor with low sensitivity to chemotherapy
• 2. Tumor with a low proliferation rate
• 3. Preexisting renal impairment
• 4. Race
Answer: 3
• Answer: 3 - Preexisting renal impairment Explanations: The precise
incidence of tumor lysis syndrome (TLS) is not known. There are
inherent risk factors that can increase the incidence of TLS. Risk
factors include tumor burden, tumor with a high rate of proliferation,
high sensitivity of the tumor to chemotherapy, and preexisting renal
disease or impairment. The predilection for TLS is not related to race
or sex. In a study that queried the database of the National Inpatient
Sample (NIS), the most common malignancies associated with TLS are
non-Hodgkin lymphoma 30%, solid tumors 20%, acute myeloid
leukemia (AML) 19%, and acute lymphocytic leukemia 13%.
Question 180
• Question 180: Which medication has been found to lower the risk of
colorectal cancers?
• Choices:
• 1. Omeprazole
• 2. Aspirin
• 3. Azathioprine
• 4. Misoprostol
Answer: 2
• Answer: 2 - Aspirin Explanations: There is now considerable evidence
that daily aspirin may help prevent colorectal cancers. Several studies
have now shown that aspirin can help prevent not only colorectal
cancers but also lung and brain cancers. The question that remains is
when to start aspirin and what other lifestyle changes are needed.
More recent data indicate that the effectiveness of aspirin varies
depending on the race of the patient. Factors like physical activity,
body mass, and plasma C peptide levels do not affect the activity of
aspirin. Go to the next page if you knew the correct
Question 181
• Question 181: Which of the following malignancies with bone
metastases has the best 5-year survival rate?
• Choices:
• 1. Lung
• 2. Breast
• 3. Thyroid
• 4. Prostate
Answer: 3
• Answer: 3 - Thyroid
• Explanations: Thyroid bone metastasis has the best 5-year survival
rate. The survival rate is 40%. The 5-year survival rate for metastatic
prostate cancer is about 29%. The 5-year survival rate for non-
metastatic prostate cancer is about 100%.
Question 182
• Question 182: Which one of the following is an acute complication of
radiation therapy?
• Choices:
• 1. Constrictive pericarditis
• 2. Mucositis
• 3. Alopecia
• 4. Lung fibrosis
Answer: 2
• Answer: 2 - Mucositis
• Explanations: Acute complications of radiation therapy include
mucositis, skin erythema, and bone marrow toxicity. Constrictive
pericarditis and lung fibrosis are chronic complications. Mediastinal
irradiation leads to increased risk of myocardial infarction.
Question 184
• Question 184: What is the most common form of malignant
melanoma?
• Choices:
• 1. Lentigo maligna melanoma
• 2. Superficial spreading melanoma
• 3. Nodular melanoma
• 4. Acral lentiginous melanoma
Answer: 2
• Answer: 2 - Superficial spreading melanoma
• Explanations: Malignant melanoma is not as common as squamous cell carcinoma and
basal cell carcinoma but is more serious and life threatening because of the potential
for distant metastasis. It is the leading cause of death as a result of a skin disease, and
the incidence is increasing. Studies have shown that the prevalence of melanoma
increases with proximity to the equator. Persons with skin types that are sensitive to the
effects of ultraviolet radiation are at higher risk. Also at higher risk are persons with red
or blond hair, freckles, and fair skin who burn easily and tan with difficulty. Superficial
spreading melanoma is the most common type. These lesions are usually 2 to 3 cm in
diameter and tend to affect patients in their fifties and sixties. Nodular melanomas are
usually smaller in diameter than the other two types and are slightly raised and often
fairly uniform in color. As these lesions tend to spread deeply into the underlying tissue,
they have a worse prognosis than superficial spreading melanoma.
Question 185
• Question 185: A 65-year-old man presents to the hospital with complaints of fatigue and
weakness. He reports that he has had increasing trouble going on a walk with friends
because of the increasing fatigue. Physical examination is largely unremarkable. Results from
routine lab testing are notable for hemoglobin of 7.7g/ dL (reference range > 13.5g/ dL) and
leukocyte count of 45/ microliter (reference range: 4.5-11.0/ microliter). Differential count
on peripheral blood smear is: neutrophils 43%, lymphocytes 11%, eosinophils 4%, basophils
5%, monocytes 5%, promyelocytes 4%, myelocytes 10%, metamyelocyte 8%, bands 10%.
Which of the following is an appropriate therapy to start the patient on?
• Choices:
• 1. Cetuximab
• 2. Trastuzumab
• 3. Imatinib
• 4. Vemurafenib
Answer: 3
• Answer: 3 - Imatinib
• Explanations: This patient with anemia, leukocytosis and myeloid
precursors likely has chronic myeloid leukemia. Imatinib is a drug that can
be used for chronic myeloid leukemia. Imatinib mesylate is a 2-
phenylaminopyrimidine derivative protein tyrosine kinase inhibitor initially
targeted to the platelet-derived growth factor receptor.[ 9] Subsequently, it
has additionally been found to inhibit other protein tyrosine kinases such as
c-kit( gastrointestinal stromal tumors) and BCR-ABL fusion protein
(Philadelphia chromosome chronic myelogenous leukemia). These protein
tyrosine kinases as a whole phosphorylate specific amino acids on substrate
proteins which induce signal transduction resulting in changes to cell
• biology including cell growth, differentiation, and death; constitutive
activation of which, through mutation or other means, can lead to
malignancy. Blocking of this constitutive action has been shown to
induce downstream apoptosis without further differentiation.
Imatinib, as a therapeutic, does just that - it blocks constitutive action
of protein tyrosine kinase by working as a competitive inhibitor of the
ATP binding cleft of ABL inducing apoptosis of leukemic cells.
Cetuximab is typically used for colorectal cancer. Trastuzumab is
typically used for breast cancer. Vemurafenib is typically used for
melanoma.
Question 186
• Question 186: What is the most important prognostic feature of rectal
carcinoma?
• Choices:
• 1. Nodal involvement
• 2. Size
• 3. Histology
• 4. Stage
Answer: 4
• Answer: 4 - Stage
• Explanations: Histopathologic features such as perineural invasion,
poor differentiation, or lymphovascular involvement and T4 tumor
stage are also poor prognostic features besides stage. Clinical features
that portend a poor prognosis include obstruction or perforation.
Postsurgical elevations of CEA indicate recurrence and worse survival
rates. For all cancers, staging is the most important for prognosis.
Question 187
• Question 187: A 33-year-old female who is diagnosed with medullary
carcinoma of the thyroid has also been found to be borderline
hypertensive. What is the next step in her investigation?
• Choices:
• 1. Renal artery imaging
• 2. Urinary metanephrines
• 3. CT scan of the abdomen
• 4. Start beta blockers
Answer: 2
• Answer: 2 - Urinary metanephrines
• Explanations: MEN 2 can present with tumors of the endocrine
system, including the adrenals, thyroid, and parathyroids. In a patient
with a medullary cancer of the thyroid, a few patients also may have a
pheochromocytoma. Prior to a thyroidectomy, a pheochromocytoma
must be ruled out by measuring urinary metanephrines.
Measurement of calcitonin is the key to the diagnosis of medullary
thyroid cancer. The levels can be used for staging, treatment and
prognosis.
Question 188
• Question 188: 64-year-old female patient receiving FOLFOX for stage
IV colon cancer develops painful erythema of the palms and soles
associated with blisters. What is the most likely diagnosis?
• Choices:
• 1. Graft-versus-host disease (GVHD)
• 2. Erythema multiforme (EM)
• 3. Toxic epidermal necrolysis (TEN)
• 4. Acral erythema
Answer: 4
• Answer: 4 - Acral erythema
• Explanations: Acral erythema is also known as palmar-plantar
erythrodysesthesia, palmoplantar erythrodysesthesia, hand-foot syndrome,
and Burgdorf reaction. This is an adverse event caused by many classic
chemotherapeutic agents and the newer molecular targeted therapies
FOLFOX is the combination of 5 fluorouracil (5FU) and oxaliplatin, given for
stage IV colorectal cancer. 5FU is one of the chemotherapeutic drug known
for causing acral erythema. Acral erythema is one of the dose-limiting
toxicity with prolonged infusion of 5-fluorouracil (5-FU). It is important to
differentiate acral erythema from the more dangerous graft-versus-host
disease. The distinction of acral erythema from GVHD is an important one
since the treatments are distinct.
Question 189
• Question 189: Which of the following medications has no estrogen-
receptor agonist activity?
• Choices:
• 1. Tamoxifen
• 2. Raloxifene
• 3. Toremifene
• 4. Fulvestrant
Answer: 4
• Answer: 4 - Fulvestrant
• Explanations: Fulvestrant is the only one from the above choices with
no estrogen-receptor agonist activity. It competitively binds to
estrogen receptors on tumors and other tissue targets. Raloxifene and
toremifene have estrogen agonist activity at bone and uterus, but
antagonize estrogen receptors in breast tissue.
Question 190
• Question 190: A patient complaining of severe right hand and finger
pain presents to you with an apical lung mass. Diagnosis of a superior
sulcus tumor is made. Which nerve is most commonly associated with
this syndrome?
• Choices:
• 1. Median nerve
• 2. Upper brachial plexus
• 3. Radial nerve
• 4. Ulnar nerve
Answer: 4
• Answer: 4 - Ulnar nerve
• Explanations: The tumor involves the lower brachial plexus and the
ulnar nerve is mainly affected. Patients have hand pain and hand
muscle wasting. Horner syndrome occurs with invasion of the
sympathetic chain. Involvement of the lower brachial plexus causes
pain and paresthesias along the ulnar nerve distribution in the hand.
If the stellate ganglion is involved, the patient may present with
Horner syndrome.
Question 191
• Question 191: Which of the following is a malignant tumor?
• Choices:
• 1. Keratoacanthoma
• 2. Mycosis fungoides
• 3. Rhabdomyoma
• 4. Atrial myxoma
Answer: 2
• Answer: 2 - Mycosis fungoides
• Explanations: Mycosis fungoides is a cutaneous T cell lymphoma that
is slow growing. Early findings may be of dark, dry patches of the skin
that may be pruritic. It is often misdiagnosed as eczema or psoriasis
and then recognized after biopsy. Often, mycosis fungoides is
misdiagnosed as psoriasis or eczema at first and only recognized after
several biopsies.
Question 192
• Question 192: Which of the following drugs can cause severe
constipation?
• Choices:
• 1. Methotrexate
• 2. Cyclophosphamide
• 3. Vincristine
• 4. Adriamycin
Answer: 3
• Answer: 3 - Vincristine
• Explanations: Vinca alkaloids work by inhibiting assembly of
microtubule structures. Peripheral neuropathy, constipation, and hair
loss are common side effects. Drug induced constipation can be
managed with enemas. Cyclophosphamide can cause hemorrhagic
cystitis.
Question 193
• Question 193: Which of the following drugs has been indicated for the
treatment of metastatic colon cancer?
• Choices:
• 1. Imatinib
• 2. Abciximab
• 3. Bevacizumab
• 4. Moronumab
Answer: 3
• Answer: 3 - Bevacizumab
• Explanations: Bevacizumab is a drug approved for use in the
treatment of metastatic colon cancer when used with 5-FU. Imatinib
is used for chronic myelogenous leukemia. Abciximab is used as an
antiplatelet agent. Moronumab does not exist.
Question 194
• Question 194: What is meant by stage T2B prostate cancer?
• Choices:
• 1. Metastasis has occurred
• 2. The tumor has extended through the capsule
• 3. The tumor is confined to the prostate
• 4. The tumor is invading adjacent structures
Answer: 3
• Answer: 3 - The tumor is confined to the prostate
• Explanations: Stage T2B refers to prostate cancer that is confined to one
lobe or less. All T2 stages refer to cancer confined within the prostate with
T2A referring to cancer of less than half a lobe and T2C referring to cancer
involving both lobes. Patients with this stage have an excellent prognosis.
An important part of evaluating prostate cancer is determining the stage.
The most common system is the four-stage TNM (Tumor/ Nodes/
Metastases) system. Its components include the size of the tumor, the
number of involved lymph nodes, and the presence of metastases. The key
distinction is whether or not the cancer is confined to the prostate. T1 and
T2 cancers are found only in the prostate. T3 and T4 cancers have spread
elsewhere.
Question 195
• Question 195: Which of the following statements about
craniopharyngioma is TRUE?
• Choices:
• 1. The tumors are usually malignant
• 2. These tumors are uniformly solid
• 3. Patients often develop signs and symptoms of excess growth
hormone
• 4. The tumors may compress the optic chiasm and cause visual
symptoms
Answer: 4
• Answer: 4 - The tumors may compress the optic chiasm and cause
visual symptoms
• Explanations: Craniopharyngiomas generally have malignant behavior
and benign histology. They originate from Rathke pouch remnants and
are commonly cystic. They are found in the suprasellar and sellar
region and compress the optic chiasm, hypothalamus, pituitary, and
third ventricle. They are most common between the ages of 14 and
19 and again over 65. Treatment is surgical and may be followed by
radiation.
Question 196
• Question 196: What is the specificity for detecting precancerous or
cancerous lesions with the Papanicolaou test?
• Choices:
• 1. 40% to 50%
• 2. 55% to 65%
• 3. 70% to 80%
• 4. 85% to 100%
Answer: 4
• Answer: 4 - 85% to 100%
• Explanations: The specificity for detecting precancerous or cancerous
lesions with the Papanicolaou test ranges from 86% to 100%.
Sensitivity ranges from 30% to 90%. The sensitivity and specificity of
the conventional test are similar to the liquid-based test. Both the
sensitivity and specificity are impacted by high intraobserver
variability.
Question 197
• Question 197: A 65-year-old man comes to the emergency department complaining of
intermittent diarrhea, rectal pain, and mild gastrointestinal bleeding. Although vitally stable, he
seems to be in significant distress due to bleeding and rectal pain. Upon questioning, the
patients state that he was diagnosed with Stage 3 rectal cancer for which he received
neoadjuvant chemoradiation. He also says that during this treatment, the patient frequently
complained of bloody diarrhea and rectal pain. He was admitted to the hospital last month due
to the same problem, and a follow-up colonoscopy was significant for radiation-induced
enteritis. Based on the patient's history, which of the following surgical approach can lead to
fewer post-operative complications in this patient?
• Choices:
• 1. Resection of whole irradiated bowel
• 2. Adhesiolysis
• 3. Bypass surgery
• 4. Grafting
Answer: 1
• Answer: 1 - Resection of whole irradiated bowel
• Explanations: Simple procedures like adhesiolysis and bypass have
been shown to less beneficial as compared to removal of the entire
diseased bowel. It is important to resect the whole irradiated bowel in
the first operation to reduce the need for more surgical procedures.
High postoperative morbidity is associated with surgery with a study
showing it to be 74.8% after the first surgical procedure and many
required parenteral nutrition after it. It has been observed that
repeated surgical procedures may be needed and they do not seem to
increase the morbidity.
Question 198
• Question 198: What are the most common locations for melanoma in
the head and neck?
• Choices:
• 1. Occipital scalp and cheek
• 2. Occipital scalp and nose
• 3. Anterior scalp and cheek
• 4. Anterior scalp and nose
Answer: 1
• Answer: 1 - Occipital scalp and cheek
• Explanations: Melanoma of the head and neck account for 25% of all
melanomas. The most common locations of melanoma in the head
and neck are the occipital scalp and cheek. Head and neck melanoma
is managed differently from other parts of the body due to its
aggressive nature. The two most recognized prognostic factors in
melanoma are Breslow depth or thickness greater than1 mm and
ulceration. Location of lesions in head and neck melanoma is
important as lymphatic drainage occurs to different basins. Sentinel
lymph node biopsy has largely replaced elective neck dissection in its
management.
Question 199
• Question 199: A 34-year-old male presents to the clinic reporting his sister
was recently diagnosed with breast cancer at 36 years of age. He is curious if
he has an increased risk of breast cancer given his sister's recent diagnosis.
How would you address his concern?
• Choices:
• 1. Because she is a female, you do not have an increased risk. No further
action is warranted
• 2. You should have prophylactic mastectomies given your young age and high
heritable nature of breast cancer
• 3. You may have an increased risk and further action may be warranted
• 4. We should start you on prophylactic tamoxifen to decrease the risk
Answer: 3
• Answer: 3 - You may have an increased risk and further action may be warranted
• Explanations: Similarly to females, a male has a higher risk of breast cancer if they
have a first or second-degree relative with breast cancer. Interestingly, studies have
shown that an affected sibling or parent of either gender may increase the risk of
breast cancer in either the males or females in the family. Bevier et al. (2012) noted
in their study that the relative risk (RR) of breast cancer was similar in either
offspring when the father or mother was affected (RR = 1.73 and 1.74, respectively),
but the risk was slightly higher in the females when the brother is affected as
opposed to when the sister is affected (RR = 2.48 and 1.39, respectively). The data is
not clear regarding guidelines for the risks of male breast cancer if a sibling is
affected, but the patient may warrant further investigation and monitoring such as
BRCA testing, mammography, and routine clinical examinations. No screening
guidelines exist for male breast cancer in the general population.
Question 200
• Question 200: A 65-year-old female patient came to the hospital to establish care
after she moved town. On physical exam, it is noticed that she has an enlarged liver
and spleen. Her hemoglobin is 9.3 g/ dL, white blood cell count of 5500, and
platelet count of112000. The rest of the laboratory markers are normal except for a
total protein of 10.2 g/ dl with albumin of 3.0 g/ dL. Serum protein electrophoresis
shows a protein M spike, and immunofixation confirms an IgM-kappa monoclonal
protein. Which of the following is the best option to confirm the diagnosis?
• Choices:
• 1. Liver biopsy
• 2. Urine protein electrophoresis
• 3. Bone marrow biopsy and aspirate
• 4. Serum viscosity
Answer: 3
• Answer: 3 - Bone marrow biopsy and aspirate
• Explanations: Waldenstrom macroglobulinemia (WM) is diagnosed with
monoclonal IgM paraproteinemia and with the presence of a
lymphoplasmacytic picture in the bone marrow. A bone marrow aspiration
and biopsy with immunophenotyping will help differentiate WM from IgM
monoclonal gammopathy of undetermined significance, IgM multiple
myeloma (MM), and IgM-secreting lymphomas such as marginal zone
lymphoma. The bone marrow aspirate should be evaluated for the MYD88
L265P mutation, which is present in 90% to 95% of patients with WM and
can help to differentiate WM from other conditions. Serum viscosity is useful
when patients present with symptoms of hyperviscosity. Liver biopsy is not
used to confirm the diagnosis of WM.
Question 201
• Question 201: Which of the following statements about risk factors
for colorectal cancer is true?
• Choices:
• 1. High-fat intake is protective
• 2. Inflammatory bowel disease decreases the risk
• 3. Females are more likely than males to develop colorectal cancer
• 4. Gardner syndrome is associated with the development of colorectal
cancer
Answer: 4
• Answer: 4 - Gardner syndrome is associated with the development of
colorectal cancer
• Explanations: Gardner syndrome is characterized by hereditary
polyposis and is associated with the development of colorectal cancer.
It is an autosomal dominant condition. Other factors that increase the
risk of colorectal cancer are male gender, increasing age,
inflammatory bowel disease, and high-fat intake. Besides Gardner
syndrome, other familial causes of colon cancer include familial
polyposis coli and lynch syndrome. Factors associated with low risk
include calcium intake, estrogen replacement therapy and folate
consumption.
Question 202
• Question 202: An 8-year-old African American male is brought to the clinician by his
mother for a 6-week history of persistent left knee pain and limp. The clinician palpates
a tender mass in the distal femur. Work up is initiated and reveals a primary bone tumor.
Following biopsy to confirm the diagnosis, the patient undergoes preoperative
chemotherapy and limb-sparing resection with the placement of a prosthesis. The
patient does very well, and he plans to follow up with medical oncology for long term
surveillance. An elevation in which biochemical marker may indicate the recurrence of
his primary neoplasm?
• Choices:
• 1. Alkaline phosphatase (ALP)
• 2. CA19-9
• 3. Alpha-fetoprotein (AFP)
• 4. Carcinoembryonic antigen (CEA)
Answer: 1
• Answer: 1 - Alkaline phosphatase (ALP)
• Explanations: The biochemical marker most closely associated with
osteosarcoma is serum alkaline phosphatase (ALP). ALP levels are useful for
diagnosis, prognosis, treatment response, and recurrence detection. ALP
levels are high due to the increased osteoblastic activity associated with
osteosarcoma. Extremely high levels have been linked to heavy tumor
burden and are generally considered a poor prognostic indicator. Response
to therapy may be monitored with the levels of these enzymes because they
may rise with residual disease or recurrence. Serum alkaline phosphatase
levels should be interpreted in the context of age as innately higher values
are physiologic in the younger population. Furthermore, the ALP level may
be normal at the presentation in up to half of patients.
Question 203
• Question 203: A 35-year-old asymptomatic woman presents to her
primary care provider for routine annual care. She has a family history
of an older brother recently being diagnosed with hereditary colorectal
cancer, and her father died of colon cancer. Which of the following is
the gold standard test to diagnose colon cancer?
• Choices:
• 1. Consider offering genetic tests, after genetic counseling
• 2. Total colonoscopy
• 3. Computed tomography colonography
• 4. Endoscopy with a colonic capsule
Answer: 2
• Answer: 2 - Total colonoscopy
• Explanations: It is suggested that in individuals with a family history in
the first degree of classical familial adenomatous polyposis with positive
genetic test or in those families with clinical criteria in which the causal
mutation has not been identified, the screening strategy should be annual
colonoscopy starting at 12 at 15 years, up to 30-35 years of age and then
it is suggested to continue screening every five years. In the population
with suspicion or family history of hereditary nonpolyposis colorectal
cancer, it is suggested that the screening strategy be colonoscopy every
two years, from 20 to 25 years, or five to ten years before the age of the
youngest case diagnosed in the family. Before performing a colonoscopic
• surveillance examination, consider the characteristics of the
adenoma, the patient's age and wishes, and the presence of
significant comorbidity. The patient's condition should be established
before attending each test; this will avoid complications. It is
suggested that the age of onset of screening, fecal occult blood
immunochemistry every two years or colonoscopy every ten years, for
non-hereditary colorectal cancer at average risk is 50 years of age and
the completion of screening for colorectal cancer at average risk is at
75 years of age.
Question 204
• Question 204: A 65-year-old male with chronic tobacco use presents
with a new central lung mass and weight loss of 20 pounds in the last
2 months. What is his likely diagnosis?
• Choices:
• 1. Adenocarcinoma of the lung
• 2. Mesothelioma
• 3. Small cell carcinoma
• 4. Hamartoma
Answer: 3
• Answer: 3 - Small cell carcinoma
• Explanations: Small cell carcinomas of the lung are centrally located
and associated with tobacco use. They may cause ectopic hormone
production such as antidiuretic hormone, parathyroid hormone, or
adrenocorticotropic hormone. Hamartomas (peripheral) and
bronchial adenomas (central) are benign lung lesions.
• Mesothelioma is associated with asbestos exposure.
• Small cell carcinomas of the lung most commonly occur in the sixth to
eighth decade of life.
Question 205
• Question 205: Which surface marker is found on T cells?
• Choices:
• 1. CD3
• 2. CD56
• 3. CD16
• 4. CD21
Answer: 1
• Answer: 1 - CD3 Explanations: CD3 is the pan-T Cell marker found on
many types of T cells.
• NK cells express CD16 and 56. CD21 is a B cell marker.
• T cells have CD specific markers, unique among immune cells.
Question 206
• Question 206: An 18-year-old male with a recent diagnosis of stage III
Hodgkin disease will receive intravenous chemotherapy with the
medication mechlorethamine hydrochloride. To help control the most
common side effect of this medication, what should be done?
Choices:
• 1. Administer an antiemetic before beginning each treatment
• 2. Fluids daily through a peripheral IV
• 3. Make sure the room is cool and pleasant as to minimize anxiety
• 4. Provide the client with medium portioned, frequent meals to
maintain nutrition status
Answer: 1
• Answer: 1 - Administer an antiemetic before beginning each treatment
Explanations: Administer an antiemetic before each treatment. The most
common side effect of mechlorethamine includes nausea and vomiting,
which usually last from 12 to 24 hours. Without antiemetics, the patient
may have difficulty keeping oral fluids down. Maintaining a cool
environment is helpful when patients are subject to nausea and vomiting.
Antiemetics can help control nausea and vomiting. It is advisable to serve
small, frequent meals when patients are subject to nausea and vomiting.
Because breakthrough emesis is difficult to reverse, it is vital to prevent the
symptoms from occurring. The choice of antiemetic depends on the risk of
emesis but one may consider using a neurokinin receptor antagonist,
selective 5HT antagonist, and dexamethasone.
Question 207
• Question 207: During exploration for resection of a proven 3
centimeter, non-small-cell cancer, another 1-centimeter nodule is
palpated in the same lobe. Which of the following is the correct stage
of this cancer?
• Choices:
• 1. Stage I
• 2. Stage II
• 3. Stage III
• 4. Stage IV
Answer: 3
• Answer: 3 - Stage III
• Explanations: This is considered stage III. Locally advanced disease is
found in nearly 40% of patients with lung cancer. In many cases, the
other nodule in the same lobe is discovered at the time of
histopathology. Currently, there is no consensus on how to manage
patients with stage 3 lung cancer. Concurrent chemotherapy and
radiation mildly improve survival compared to sequential
chemoradiation.
Question 208
• Question 208: A 65-year-old woman presents with fatigue and bone pains at the
clinic. On examination, she has a blood pressure of 140/ 85 mmHg, a pulse of 85/
min, and a respiratory rate of 25/ min. Otherwise, her examination is
unremarkable. The patient’s investigations suggest calcium of 3.1 mmol/ L,
creatinine of 181 umol/ L, and hemoglobin of 10g/ dL. CT scan notes osteolytic
lesions on the right and left femur. Which of the following is the mechanism of
action of the best treatment for this patient?
• Choices:
• 1. Dopamine receptor agonist
• 2. Mu receptor agonist
• 3. Cyclooxygenase inhibitor
• 4. Proteosome inhibitor
Answer: 4
• Answer: 4 - Proteosome inhibitor
• Explanations: Bortezomib is used for the treatment of multiple
myeloma and for the treatment of relapsed or refractory mantle cell
lymphoma. Bortezomib works by inhibiting proteasomes.
Proteasomes regulate protein homeostasis in the cell. Bortezomib is
an irreversible inhibitor of chymotrypsin-like activity at the 26S
proteasome. This results in signaling cascade activation, the arrest of
the cell cycle, and apoptosis.
Question 209
• Question 209: A 66-year-old female patient presents with breast discharge. She has had
this symptom for 3 weeks on and off accompanied with weight loss of 10 lb over 6
months. She was previously diagnosed with uterine fibroids at age 25 for which she
underwent myomectomy. She has also been diagnosed with benign ovarian cysts for
which she undergoes regular follow up. Her vitals include pulse rate: 85/ minute, blood
pressure: 105/ 80 mm Hg, and temperature: 98.6 degrees Fahrenheit. Which of the
following characteristics of nipple discharge would suggest basement membrane invasion
of breast ductal/ lobular cells?
• Choices:
• 1. Unilateral bloody discharge
• 2. Bilateral serous discharge
• 3. Discharge with menses
• 4. The discharge associated with serum prolactin level elevation
Answer: 1
• Answer: 1 - Unilateral bloody discharge
• Explanations: Unilateral spontaneous discharge is the most suspicious
for breast cancer, which involves the basement membrane invasion of
cancerous cells. Bloody breast discharge in women more than 30
years of age warrant further evaluation. The investigation for blood
breast discharge should include mammography and possibly
galactography, ductoscopy, or MRI. The discharge associated with
serum prolactin level elevation is a result of an endocrinopathy and
has a low suspicion for breast cancer. Bilateral serous breast discharge
indicates a benign pathology.
Question 210
• Question 210: What is the most common carcinoma of the oral
cavity?
• Choices:
• 1. Squamous cell
• 2. Basal cell
• 3. Transitional cell
• 4. Carcinoid
Answer: 1
• Answer: 1 - Squamous cell
• Explanations: The most common cancer of the head and neck is
squamous cell carcinoma. Squamous cell carcinoma comprises 53% of
oral cancers. Others are adenoid cystic 15%, mucoepidermoid 10%,
adenocarcinoma 4%, and anaplastic 4%. Smoking and alcohol are the
strongest risk factors for these cancers. Squamous cell carcinoma
usually starts as premalignant lesions, such as leukoplakia. Often the
cancer will present as an ulcerated lesion.
Question 211
• Question 211: At what age should screening for prostate cancer be
started?
• Choices:
• 1. 30
• 2. 40
• 3. 50
• 4. All ages
Answer: 3
• Answer: 3 - 50
• Explanations: There is conflicting evidence about the benefit of prostate cancer
screening. So far, there has been a reduction in the diagnosis of localized prostate
cancer by about 30% but no change in the mortality or incidence of advanced or
metastatic disease. The United States Preventive Task Force concludes that the
current evidence is insufficient to justify prostate cancer screening in men
younger than age 75 years. Medicolegally, it is best to discuss screening with each
patient individually. Risk factors and patient concerns should be taken into
account. There is a general consensus that men older than 75 years of age will
gain little from screening. There is some evidence that a baseline PSA test in the
early 40' s will give good predictive value for future prostate cancer risk over the
next 20-25 years. If a PSA is done at age 40 and is < 1, prostate cancer risk is
judged to be very low.
Question 212
• Question 212: Penile cancer usually spreads to which group of lymph
nodes?
• Choices:
• 1. Para-aortic
• 2. Pelvic
• 3. Inguinal
• 4. Axillary
Answer: 3
• Answer: 3 - Inguinal Explanations: The most common site of
metastasis is the inguinal lymph nodes (ILNs) Metastatic spread
occurs in a stepwise fashion from the penis to the sentinel lymph
node, to other superficial inguinal nodes, to deep inguinal nodes, to
the node of Cloquet, to the pelvic lymph nodes, and then distant
metastasis. The sentinel lymph node is the first lymph node where
penile cancer metastasizes. It is usually located in the superior-medial
zone of the superficial inguinal node group. Penile cancer can spread
bilaterally to the inguinal lymph nodes because the penile lymphatics
drain to both sides. The most common sites for distant metastases
include the lung, liver, bone, and brain.
Question 213
• Question 213: Rhabdomyosarcomas are tumors of:
• Choices:
• 1. Epithelial
• 2. Neural tissue
• 3. Smooth muscle
• 4. Skeletal muscle
Answer: 4
• Answer: 4 - Skeletal muscle
• Explanations: Rhabdomyosarcomas originate from skeletal muscle.
The most common sites of origin are the legs and arms but can occur
at any skeletal muscle. Rhabdomyosarcoma is diagnosed more
commonly in children than in adults. These tumors are very resistant
to radiation and chemotherapy. Surgery is the treatment of choice but
recurrences are common.
Question 214
• Question 214: A 79-year old patient recently underwent surgery for
prostate cancer. During the procedure, local metastases were noted.
Which of the following drugs would be an appropriate choice for post-
operative metastatic treatment?
• Choices:
• 1. Oxandrolone
• 2. Leuprolide
• 3. Finasteride
• 4. Tamoxifen
Answer: 2
• Answer: 2 - Leuprolide
• Explanations: Leuprolide is an agonist of the GnRH receptors, which downregulate
follicle stimulating hormone (FSH) and luteinizing hormone (LH). This activity
leads to hypogonadism (lowered testosterone levels) and this helps to suppress
hormone-responsive cancer. Leuprorelin or leuprolide is a hormone used to treat
prostate cancer. In women, it is used for breast cancer, endometriosis, uterine
fibroids, and early puberty. It is given by intramuscular or subcutaneous injection.
Side effects include hot flashes, unstable mood, insomnia, headaches, and
injection site pain. Other side effects include high blood sugar, allergic reactions,
and problems with the pituitary gland. Use during pregnancy may cause harm to
the fetus. Leuprorelin is in the gonadotropin-releasing hormone analog family of
medications. It works by decreasing gonadotropin, testosterone, and estradiol.
Question 215
• Question 215: Which breast cancer has the highest tendency for
invasion of dermal lymphatics?
• Choices:
• 1. Paget
• 2. Infiltrating ductal carcinoma
• 3. Inflammatory breast
• 4. Cystosarcoma phyllodes
Answer: 3
• Answer: 3 - Inflammatory breast
• Explanations: Inflammatory breast cancer is a very aggressive cancer
that usually occurs in females. The cancer has a high degree of
aneuploidy with overexpression of p53 and high levels of E-cadherin.
Histology usually reveals the presence of cancer cells in the
subdermal lymphatics on skin biopsy. Inflammatory breast cancer is
usually negative for ER and PR receptors and hence a very poor
survival.
Question 216
• Question 216: A 55-year-old man is brought to the clinic with a mass on his neck.
He says that the mass has been there for the past 3 months and that he has not
come in earlier because it was painless, and he thought it would resolve. He has
not felt anxious, depressed, lethargic, or slow. Laboratory analysis demonstrated a
normal TSH, and a fine needle aspirate reveals papillary thyroid cancer. Upon
further discussion, the patient is deemed a poor surgical candidate. Which of the
following radionuclides is most useful in the treatment of his pathology?
• Choices:
• 1. Technetium-99m
• 2. Iodine-131
• 3. Fluorine-18
• 4. Nitrogen-13
Answer: 2
• Answer: 2 - Iodine-131
• Explanations: The first step in the evaluation of a thyroid mass is to assess
the TSH. A low TSH would indicate thyroid hyperactivity, which would
essentially exclude malignancy; however, a normal TSH given the context of
a painless thyroid mass raises the possibility of malignancy. If malignancy is
suspected, a fine needle aspirate should be performed to assess the thyroid
histologically. Surgery is one of the first-line treatments for thyroid
malignancies. Radioactive ablation of the thyroid gland is a potential
treatment modality for thyroid malignancy. This involves the utilization of
radioactive Iodine (the thyroid gland is the only structure in the body that is
involved in Iodine uptake). This allows for the localization of the iodine and
the destruction of the thyroid gland.
Question 217
• Question 217: What is the most common site of metastasis from
bladder carcinoma?
• Choices:
• 1. Bone
• 2. Liver
• 3. Lymph nodes
• 4. Lung
Answer: 3
• Answer: 3 - Lymph nodes
• Explanations: Only 5 percent of patients have metastatic disease at
presentation. The most common site of metastasis from bladder
carcinoma is the lymph nodes. It can also spread to the liver, bone,
lung, and central nervous system. In addition to metastatic spread,
bladder cancer can spread by direct invasion of adjacent structures
such as the rectum, uterus, or prostate.
Question 218
• Question 218: A 68-year-old female presents with a history of bleeding via the
rectum every time she has a bowel movement. She has also noticed a change
in her bowel habits and feels tired all the time. A digital rectal exam reveals a
5 cm mass that is fixed to the mucosa. Biopsy reveals a lesion with dysplasia
and abnormal epithelial cells. What is the most important prognostic feature
of this patient's pathology?
• Choices:
• 1. Nodal involvement
• 2. Size
• 3. Histology
• 4. Presence of symptoms
Answer: 1
• Answer: 1 - Nodal involvement
• Explanations: When rectal cancer has spread to lymph nodes far from
the rectum, cancer cannot be cured. Survival time is usually less than
seven months. The rectum is about 12 to 15 centimeters in length and
the prognosis of rectal cancers is dependent on nodal involvement or
staging. Local excision of rectal cancer is not recommended, as there
is a high risk of recurrence. Abdominal perineal resections are done
for lower third rectal cancers and this means a permanent colostomy.
Question 219
• Question 219: A 49-year-old woman is brought to the emergency department by her
neighbor after a seizure. The neighbor states that the patient has been complaining of
headaches and weakness for several weeks now but has always hated seeing a doctor,
so she never followed up on the complaints. A physical examination is
noncontributory. Further laboratory evaluation reveals serum calcium of 15.4 mg/ dl
and a serum phosphate of 2 mg/ dl. Chest radiography reveals multiple lung lesions.
Which of the following conditions is most likely the cause of the finding in this patient?
• Choices:
• 1. Acute renal failure
• 2. Metastatic breast cancer
• 3. Vitamin D toxicity
• 4. Hyperthyroidism
Answer: 2
• Answer: 2 - Metastatic breast cancer
• Explanations: Hypercalcemia is defined as serum calcium concentration,
two standard deviations above the mean values. The normal serum calcium
ranges from 8.8 mg/ dL-10.8 mg/ dL. The patient in the given scenario is
experiencing symptoms of hypercalcemia, which can be summarized as
"groans, bones, stones, moans, thrones and psychic overtones." This
includes abdominal pain, nausea, vomiting, bone pain, fatigue, malaise,
polyuria, polydipsia, constipation, lethargy, headache, confusion,
depression, and memory loss. Calcium levels over 14 mg/ dL are associated
with malignancy. In patients with hypercalcemia associated with
malignancy, the PTH-related peptide levels are elevated. The PTH related
peptide increase levels of
• calcium by causing activation of the parathyroid hormone receptor,
excessive bone resorption, and increased reabsorption of calcium
from the distal convoluted tubules of the kidneys. The patient most
likely has a primary lung malignancy that has metastasized to the
breast and is causing hypercalcemia of malignancy.
Question 220
• Question 220: What is the most common type of soft tissue tumor?
Choices:
• 1. Neuroma
• 2. VIPoma
• 3. Lipoma
• 4. Watkins tumor
Answer: 3
• Answer: 3 - Lipoma
• Explanations: Superficial subcutaneous lipomas are the most common
type of soft tissue tumor. Lipomas are soft, asymptomatic, and usually
do not enlarge quickly. Recurrence of an excised lipoma is not
common. Malignant transformation of a lipoma into a liposarcoma is
extremely rare.
Question 221
• Question 221: A 65-year-old man presented to the emergency department with right-sided
abdominal pain, fever, jaundice, nausea, and vomiting. His symptoms started two days ago and
have been progressively getting worse. His oral temperature is 39.2 C (102.5 F), his blood
pressure is 105/ 65 mm Hg, and his pulse rate is 95/ min. On physical exam, the patient has
jaundice. The abdominal examination reveals right upper abdominal tenderness and guarding.
The remainder of the physical exam is normal. The liver function test is remarkable for alkaline
phosphatase 160 u/ L, ALT: 130 u/ L, AST: 152 U/ L, and total bilirubin 4 g/ dl. Abdominal CT
with contrast revealed a sharply demarcated mass with rim enhancement. What is the most
likely diagnosis in this patient?
• Choices:
• 1. Acute cholangitis
• 2. Hepatitis
• 3. Hepatic abscess
• 4. Hepatocellular carcinoma
Answer: 4
• Answer: 4 - Hepatocellular carcinoma
• Explanations: The hepatic abscess is one of the cystic liver lesions, usually
from an infectious process in the gastrointestinal system that carried the
infection to the liver via the portal venous system, and Escherichia Coli is
the most common microorganism. The imaging characteristic of a hepatic
abscess is a double target sign, which is a central area of low attenuation,
surrounded by two layers of rings, in which the inner rim is high attenuated
and outer ring has low attenuation. Immunodeficiency is one of the risk
factors for developing bacterial abscess. Patients can present with right
upper quadrant abdominal pain, fever, jaundice, malaise, and anorexia.
Patient presentations with fever, jaundice, and right upper quadrant pain
can be seen in
• acute cholangitis, too, but there is no rim enhancement in CT of patients
with acute cholangitis. Heterogenous liver enhancement can be seen in
contrast-enhanced CT of patients with acute cholangitis. Imaging
manifestation of viral hepatitis can range from normal to nonspecific
findings of hepatomegaly, gallbladder wall thickening, or accumulation of
fluid around the portal vein. The diagnosis is mainly made based on
clinical features and serology testing. Biphasic hepatic CT with arterial
and portal venous phase can appreciate tumors that are hypervascular
with more sensitivity than the portal phase alone. Usually, it can be
appreciated as a hyperattenuating lesion in the late arterial phase (~ 35
seconds) and then rapidly wash out. So, there is no enhancement in the
portal venous phase.
Question 222
• Question 222: What is the most common histology of bladder cancer
in a patient with a Schistosoma haematobium infection?
• Choices:
• 1. Adenocarcinoma
• 2. Columnar cell
• 3. Squamous cell
• 4. Transitional cell
Answer: 3
• Answer: 3 - Squamous cell
• Explanations: Bladder cancer is mostly transitional cell carcinoma.
Schistosoma haematobium infection is a risk factor for bladder cancer.
It usually results in squamous cell carcinoma of the bladder. In
Western countries, the most common risk factor is smoking. A 10
pack-year smoking history or more is considered sufficient to put
patients at risk.
Question 223
• Question 223: A 65-year-old female presents to the hospital with a one-month history of vague
abdominal pain. The pain is worse in the right lower quadrant, and she feels that there is a mass that
was not there previously. She has no other associated symptoms, and her vitals are stable. On
examination, there is a palpable mass in the right lower quadrant that is mildly tender to palpation.
There are no peritoneal signs. The provider orders lab work, a urinalysis, and an abdominal computed
tomography (CT) scan. The patient returns in one week for her results. The patient does not have an
elevated white cell count or CA-125, kidney function is normal, and her urinalysis is unremarkable. Her
CT scan shows mild periappendiceal fat stranding with a dilated appendix 17 cm in diameter. It
contains a homogenous low-attenuating cystic structure, which measures 4 cm. What is the patient at
risk of developing?
• Choices:
• 1. Carcinoid syndrome
• 2. Peritonitis
• 3. Appendiceal rupture
• 4. Pseudomyxoma peritonei
Answer: 4
• Answer: 4 - Pseudomyxoma peritonei
• Explanations: This case is more likely a mucocele from a mucinous
neoplasm, most likely mucinous adenocarcinoma which has a propensity
for developing pseudomyxoma peritonei. Pseudomyxoma peritonei is a
clinical syndrome most often caused by a mucinous neoplasm of the
appendix. It leads to the spread of mucin cells in the peritoneum and serosa
of abdominal and pelvic organs. It commonly spreads to the pelvis and
paracolic gutters and grows slowly without distant metastasis. Imaging
studies would reveal mucinous ascites, peritoneal soft-tissue implants,
omental caking, and involvement of the gastrointestinal tract and ovaries.
The extent of disease is classified by the peritoneal cancer index (PCI), and
the results guide treatment
• and help give a prognosis. Appendicitis is the typical culprit for
causing an appendiceal rupture. The risk slowly increases after 36
hours from symptom onset, with a high risk after 48 hours. The
clinical scenario does not match the presentation of acute
appendicitis in symptoms, lab values, or imaging results. This is a
picture of a mass that developed over a longer period of time with a
very large mucocele evidenced by the CT scan.
Question 224
• Question 224: Mutation of what gene is frequently associated with
colon cancer?
• Choices:
• 1. K-Ras
• 2. Myc
• 3. Her
• 4. P52
Answer: 1
• Answer: 1 - K-Ras
• Explanations: Colon cancer is associated with mutation of the K-Ras gene.
Factors that suggest a genetic contribution to CRC include (1) strong family
history of CRC or polyps; (2) multiple primary cancers with CRC; (3) the
existence of other cancers with known syndromes causing an inherited risk,
such as endometrial cancer; and (4) early age at diagnosis. Hereditary CRC is
commonly inherited in an autosomal dominant pattern. Two syndromes are
autosomal recessive patterns (MYH-associated polyposis and NTHL1).
Additional factors coupled with diet, alcohol consumption, family history,
use of nonsteroidal anti-inflammatory drugs, cigarette smoking, colonoscopy
with removal of adenomatous polyps, and physical activity, may influence
the development of
• adenomatous polyps and risk of CRC. At least three computer models
estimate the probability that an individual affected with cancer carries
a pathogenic variant in a mismatch repair (MMR) gene associated
with Lynch syndrome (LS), the most common inherited CRC
syndrome. These include the MMRpro, MMRPredict, and
PREMM1,2,6 prediction models. Individuals with a risk of 5% or
greater on any models should be considered for referral for genetic
evaluation and testing.
Question 225
• Question 225: While volunteering in the first aid tent at a race, a runner collapses.
There is a medical health professional attending the event. Under Good Samaritan
laws, in which of the following situations should the provider expect liability
protection?
• Choices:
• 1. The patient dies from complications of an attempted chest tube that the patient did
not require
• 2. The patient dies after receiving intravenous fluids because of underlying congestive
heart failure the provider did not know about
• 3. The patient is treated to the best of the provider's abilities and billed via insurance
• 4. The patient is not treated by the provider out of fear of communicable disease, and
the patient later dies
Answer: 2
• Answer: 2 - The patient dies after receiving intravenous fluids because of
underlying congestive heart failure the provider did not know about
Explanations: Good Samaritan laws were originally written to protect
physicians from liability when rendering care outside of their usual clinical
setting. Good Samaritan laws give liability protection against “ordinary
negligence”. Ordinary negligence is defined as the failure to act as a
reasonably prudent person. Good Samaritan laws protect a caregiver for
the failure to exercise such care as the great mass of mankind ordinarily
exercises under the same or similar circumstances. Good Samaritan laws do
not protect from Gross negligence. This can be described as a conscious
and voluntary disregard of the need to use reasonable care, which is likely
to cause foreseeable grave injury or harm to a person.
Question 226
• Question 226: A 50-year-old female has a 2-centimeter thyroid
nodule. Fine needle aspiration is done, and she is diagnosed with
papillary carcinoma. Laboratories are ordered. Which of the following
is the most appropriate next step in management?
• Choices:
• 1. Thyroidectomy
• 2. Ultrasound of the neck.
• 3. Radiation therapy with iodine-131
Answer: 2
• Answer: 2 - Ultrasound of the neck.
• Explanations: Neck ultrasound to evaluate for lymph node
involvement and the extent of the disease can be done by
experienced radiologist instead of neck CT. Surgery will follow the CT
scan. Iodine-131 is used after surgery for metastatic disease. Papillary
carcinoma is the most common thyroid malignancy and is thyroid-
stimulating hormone-sensitive. Thyroid suppression with high-dose
levothyroxine at 2.2 to 2.5 micrograms/ kg is recommended.
Question 227
• Question 227: A concerned father brings in his 2-year-old for evaluation of his low heart rate. While
playing doctor with his toddler yesterday, he noted his child's pulse to be low. Oddly, his toddler has
stopped running as much and prefers to walk, holding an adult's hands. If left to walk on his own, his
gait is quite unsteady and looks almost as though he is learning to walk again. Of note, he started
walking at 12 months of age. On examination, the child's heart rate is 54 beats per minute. His
temperature is 98.5 degrees Fahrenheit, respiratory rate is 35 beats per minute, and blood pressure is
110/ 65 mmHg. When the provider tests his gait, he seems unsteady and looks for something to hold
onto to help him walk properly. On his ophthalmologic examination, his retinal vessel margins are
blurry. The provider suspects an intracranial mass. Which of the following statements is true about the
type of tumor most likely suspected?
• Choices:
• 1. The peak incidence is at 20 years of age
• 2. It typically arises from the vermis and the roof of the fourth ventricle
• 3. Hydrocephalus is usually seen late in the course of the disease
• 4. High tyrosine-protein kinase C receptors expression may be an indicator of poor prognosis
Answer: 2
• Answer: 2 - It typically arises from the vermis and the roof of the fourth
ventricle
• Explanations: Medulloblastomas are invasive embryonal tumors of the
cerebellum and tend to metastasize along the craniospinal axis. Tumor
growth starts along the floor of the fourth ventricle and occupies the
entirety of the ventricle, subsequently invading both the cerebellar
vermis and the brainstem, causing ataxia and signs of hydrocephalus
early on. The peak incidence of medulloblastoma is in the first decade
of life, with an annual incidence of 0.5 per 100,000 children. The
mainstay of treatment for medulloblastoma is maximal surgical
resection followed by chemotherapy and whole neuroaxis radiation.
Question 228
• Question 228: A 62-year-old man with non-Hodgkin lymphoma comes to the
physician because of a 2-week history of progressive shortness of breath and lower
extremity edema. He received his third course of chemotherapy 5 weeks ago.
Physical examination revealed S3 heart sounds, crackles bilaterally lower based lung
fields, and 4 + lower extremity edema. Which of the following is the mechanism of
action of the agent most likely responsible for this patient's clinical status?
• Choices:
• 1. Inhibition of topoisomerase II
• 2. Anti-TNF
• 3. Calcineurin inhibitor
• 4. Microtubule polymerization
Answer: 1
• Answer: 1 - Inhibition of topoisomerase II
• Explanations: These patient symptoms are most likely due to doxorubicin.
Doxorubicin is an anthracycline, which is a class of drugs known to be
cardiotoxic. The primary mechanism of action of doxorubicin involves the
drug’s ability to intercalate within DNA base pairs, causing breakage of DNA
strands and inhibition of both DNA and RNA synthesis. Doxorubicin inhibits the
enzyme topoisomerase II, causing DNA damage and induction of apoptosis. The
left ventricular ejection fraction should be assessed before and regularly during
and after doxorubicin treatment. Patients with severe myocardial insufficiency
or severe arrhythmia should not be administered these drugs. Myocardial
damage occurs with cumulative doses of doxorubicin. The risk of
cardiomyopathy is further increased with concomitant cardiotoxic therapy.
Question 229
• Question 229: A male undergoes a needle biopsy of a mass in the
neck. The pathologist mentions that there are Reed-Sternberg cells.
The patient may have which of the following?
• Choices:
• 1. Sarcoidosis
• 2. Tuberculosis
• 3. HIV
• 4. Hodgkin lymphoma
Answer: 4
• Answer: 4 - Hodgkin lymphoma
• Explanations: Reed-Sternberg cells are giant cells usually seen in
Hodgkin lymphoma. Reed-Sternberg cells are large and bilobed or
resemble an owl's eye. Reed-Sternberg cells are CD 30 and CD 15
positive. Rarely reed Stenberg cells may be seen in infectious
mononucleosis and in carbamazepine associated adenopathy.
Question 230
• Question 230: A 43-year-old male patient with a past medical history
significant for smoking and recently diagnosed renal cell carcinoma presents
with waddling gait and severe bilateral hip pain. Radiographs show
generalized osteopenia and regions of banded sclerosis (stress fractures) at
the femoral necks. Which lab value is the most suggestive of a disease
process with increased bone turnover?
• Choices:
• 1. Hypercalcemia
• 2. Hypocalciuria
• 3. Elevated alkaline phosphatase (ALP)
• 4. Hypophosphatemia
Answer: 3
• Answer: 3 - Elevated alkaline phosphatase (ALP)
• Explanations: Many biochemical markers, including bone ALP, are now
available. They are specific and sensitive in assessing disease
processes that involve the bone formation and resorption. In long-
term osteomalacia, decreased urine calcium excretion can be seen,
however, this is not necessarily suggestive of bone turnover. Serum
calcium can be increased or decreased in diseases of increased bone
turnover. Serum phosphate can be increased or decreased in diseases
of increased bone turnover. In renal osteodystrophy, one would
expect hyperphosphatemia. However, due to renal wasting in
osteomalacia, hypophosphatemia is expected.
Question 231
• Question 231: A patient has medullary carcinoma of the thyroid.
What lab values are most likely in the initial phase of the disease?
• Choices:
• 1. Low calcium, low phosphate
• 2. High calcium, low phosphate
• 3. Low calcium, high phosphate
• 4. Normal calcium, normal phosphate
Answer: 1
• Answer: 1 - Low calcium, low phosphate
• Explanations: Medullary thyroid carcinoma produces calcitonin.
Calcitonin can lower serum calcium and prevent reabsorption of
phosphate from the renal tubules. When any patient has MTC, it is
vital to rule out MEN syndrome. MTC often occurs with
hyperparathyroidism and pheochromocytoma. However, over time
the kidneys become resistant to the effects of calcitonin and both
calcium and phosphate return to normal.
Question 232
• Question 232: A 55-year-old female with rheumatoid arthritis
presents with dry, itchy eyes and a dry mouth. What neoplasia is she
susceptible to in the future?
• Choices:
• 1. Mucoepidermoid carcinoma of the salivary gland
• 2. Lymphoma
• 3. Leukemia
• 4. Adenoid cystic carcinoma of the lacrimal gland
Answer: 2
• Answer: 2 - Lymphoma
• Explanations: Patient with rheumatoid arthritis is at risk for
developing Sjogren syndrome. Patients with Sjogren syndrome have
40 times the incidence of malignant lymphoma of the general
population. There is no increased risk for the other neoplasias listed.
Immune complexes may deposit in skin, joints and other organs
resulting in a systemic vasculitis.
Question 233
• Question 233: Which illness is most likely characterized by a
translocation between chromosomes 9 and 22?
• Choices:
• 1. Acute lymphoblastic leukemia
• 2. Chronic myelogenous leukemia
• 3. Follicular lymphoma
• 4. Burkitt lymphoma
Answer: 2
• Answer: 2 - Chronic myelogenous leukemia
• Explanations: Chronic myelogenous leukemia has an 85 percent
incidence of translocation of chromosome 9 and 22. It involves the c-
abl proto-oncogene being fused to the her gene on 22, forming the
Philadelphia chromosome. The translocation of chromosome 9 and 22
is also seen in acute lymphoblastic leukemia. Burkitt lymphoma
involves chromosomes 8 and 14, while Follicular lymphoma involves
chromosomes 14 and 18.
Question 234
• Question 234: A 78-year-old female presents to her provider complaining of abnormal vaginal
discharge over the past three months. Her physical examination reveals BP of 122/ 78 mmHg, and a
pulse is 79 beats per minute, a temperature of 37 C, and a regular respiratory rate. Abdominal
examination reveals a distended abdomen with positive shifting dullness, coincident with ascites. CT
scan confirms the finding of ascites and shows an omental mass. She undergoes surgical staging and
is diagnosed with stage IV uterine serous carcinoma. She then receives systemic chemotherapy and
achieves a complete remission. She has several abnormal laboratory values before surgery, which
returned to normal following her treatment. Which of the following laboratory parameters would be
most indicative of developing recurrence if it is found to be abnormal in this asymptomatic patient
undergoing routine post-treatment surveillance?
• Choices:
• 1. FSH level
• 2. Serum creatinine
• 3. CEA (carcinoembryonic antigen) level
• 4. CA125
Answer: 4
• Answer: 4 - CA125
• Explanations: Uterine serous carcinoma is an aggressive uterine carcinoma
characterized by a higher propensity for the advanced disease at the time of
diagnosis. It is more likely to be found in older women. Women usually
present with watery or blood-tinged vaginal discharge. Therefore, careful
history taking is crucial to reach the diagnosis accurately. Uterine serous
carcinomas are very similar to serous ovarian carcinomas, with a propensity
to spread throughout the abdominal and pelvic cavities, where ascites and
peritoneal or omental carcinomatosis may be found on imaging. Patients
with these findings will also often have an elevated serum CA-125, similar to
patients with serous ovarian carcinoma. CA125 (cancer antigen-125) is the
abnormal lab value referred to in the question
• above. Following successful treatment for advanced uterine serous carcinoma,
CA125 will often return to normal, much like it does for serous ovarian
carcinoma. As such, CA125 is a vital lab parameter to follow because a rising
value usually indicates developing recurrence. CA125 levels will often begin
rising prior to the onset of new symptomatology or the appearance of
recurrence on imaging. Consequently, an abnormal CA125 is the laboratory
parameter that would be most indicative of recurrent uterine serous
carcinoma. CEA is a laboratory parameter that is more helpful for monitoring
patients with colon cancer. While hemoglobin and creatinine levels may be
abnormal in patients with cancer, an abnormality in one of these laboratory
values is non-specific. A rising CA125 in a patient with a history of advanced
uterine serous carcinoma most often indicates the development of recurrence.
Question 235
• Question 235: Which of the following is the most common type of
cancer in men age 15 to 35?
• Choices:
• 1. Testicular cancer
• 2. Bladder cancer
• 3. Renal cancer
• 4. Prostate cancer
Answer: 1
• Answer: 1 - Testicular cancer
• Explanations: The most common type of cancer in men age 15 to 35 is
testicular cancer. Testicular cancers usually present as a firm
nontender testicular mass. These cancers usually present as a painless
mass in the scrotum or a heaviness sensation. Testicular cancers are
very sensitive to chemotherapy even when they have
spread.However, there is always a risk of cancer in the contralateral
testes in 2-3 decades.
Question 236
• Question 236: A patient presents with weight loss and nausea. She
has lost 10 lbs over 2 months and has altered bowel movements.
Work up reveals that she has a sigmoid colon cancer. What is the first
site where the colon cancer usually spreads?
• Choices:
• 1. Liver
• 2. Lymph nodes
• 3. Ureter
• 4. Rectum
Answer: 2
• Answer: 2 - Lymph nodes
• Explanations: Most people seem to forget that colon cancer is a slow
growing cancer that first involves the lymph nodes before it spreads
to the liver. General principles of surgery include removal of the
cancer with adequate margins including areas of lymphatic drainage.
Isolated hepatic or pulmonary metastases can be removed surgically.
Chemotherapy is sometimes done preoperatively or as adjuvant
therapy.
Question 237
• Question 237: A 6-year-old male is undergoing neuroimaging for the evaluation of an
intracranial tumor. On MRI, the tumor has a low T1 but high T2 signal with contrast
enhancement. The mass is seen located centrally in the posterior fossa originating
from the vermis. Profound obstructive hydrocephalus is noted likely due to the
effacement of the fourth ventricle by tumor extension. The tumor further abutts the
dorsal brainstem. Which of the following statements is accurate regarding the most
likely type of pediatric brain tumor in this child?
• Choices:
• 1. Medulloblastomas are the third most common malignant tumor in childhood
• 2. These are glial cell tumors
• 3. Most originate in the cerebellum or posterior fossa
• 4. On rare occasions, the fourth ventricle may occlude
Answer: 3
• Answer: 3 - Most originate in the cerebellum or posterior fossa
Explanations: Medulloblastomas are the most common primary CNS
malignancy of childhood. These are embryonal neuroepithelial
tumors. They arise from the cerebellum in the posterior fossa most
commonly. Occlusion of the fourth ventricle often occurs with
symptoms of headache, nausea, vomiting, and gait ataxia prominent.
A combination of resection, radiation, and chemotherapy is used, but
a 5-year prognosis depends on the subgroup of the medulloblastoma
and associated cytogenetic aberrations, ranging from 40% (group 3
MB with myc amplification) to greater than 90% (WNT
medulloblastoma).
Question 238
• Question 238: Which of the following is a major problem with regular
screening programs for cancer?
• Choices:
• 1. Not enough enrollment
• 2. Overdiagnosis
• 3. Screening only works if started at an early age
• 4. Screening only benefits the elderly
Answer: 2
• Answer: 2 - Overdiagnosis
• Explanations: The major problem with regular screening tests for
cancer is that false positives and overdiagnosis are common. False
positives and overdiagnosis can lead to unnecessary procedures and
complications for the patient. Further, both the monetary and
emotional cost for the patient can be enormous, especially if the
lesion turns out to be benign. The risks and benefits of screening
should be discussed with patients before scheduling.
Question 239
• Question 239: What is a true statement regarding Pap smears?
• Choices:
• 1. Pap smears should be repeated annually for all patients
• 2. Pap smears are sensitive but not specific for the detection of high-
grade squamous intraepithelial lesion
• 3. Pap smears are an effective screening test for the detection of
endometrial carcinoma
• 4. Pap smears should be initiated at age 21 years
Answer: 4
• Answer: 4 - Pap smears should be initiated at age 21 years
Explanations: The American College of Obstetricians and
Gynecologists and the United States Preventive Services Task Force
(USPSTF) recommend cervical cancer screening be performed on all
women starting at age 21 years. They should be done every 3 years if
normal. According to the USPSTF, women age 30 to 65 years can
lengthen the screening interval with a combination of cytology and
human papillomavirus (HPV) testing every 5 years. For women 30 to
65 years who want to lengthen the screening interval, recommended
screening includes high-risk human papillomavirus (hrHPV) testing
alone or a combination of cytology and hrHPV testing every 5 years.
Question 240
• Question 240: An immunocompromised patient says, "I have been
exposed to the shingles. Is there anything I can do to avoid infection?"
Which of the following is the best response?
• Choices:
• 1. There are antiviral medications that can be administered
• 2. The varicella-zoster immunoglobulin can prevent infection
• 3. One cannot be protected from infection once exposed
• 4. Should one become infected, there are analgesics to relieve the
pain
Answer: 2
• Answer: 2 - The varicella-zoster immunoglobulin can prevent infection
Explanations: People who have been exposed to shingles can receive the
varicella-zoster immunoglobulin within 72 hours of exposure to be
protected from the infection. Antiviral medications such as famciclovir
are given to treat clients who have shingles. Antivirals will not prevent
the shingles. Analgesics are available to treat the pain associated with
shingles; however, analgesics will not prevent shingles. Chickenpox or
varicella is a contagious disease caused by the varicella-zoster virus
(VZV). The virus is responsible for chickenpox (usually primary infection
in non-immune hosts) and herpes zoster or shingles (following
reactivation of latent infection). Chickenpox results in a skin rash that
forms
• small, itchy blisters, which scabs over. It typically starts on the chest, back, and face
then spreads. It is accompanied by fever, fatigue, pharyngitis, and headaches which
usually last five to seven days. Complications include pneumonia, brain
inflammation, and bacterial skin infections. The disease is more severe in adults than
in children. Symptoms begin ten to 21 days after exposure, but the average
incubation period is about two weeks. Chickenpox is a worldwide, airborne disease
that is spread by coughing and sneezing, and also by contact with skin lesions. It may
start to spread one to two days before the rash appears until all lesions are crusted
over. Patients with shingles may spread chickenpox to those who are not immune
through blister contact. The disease is diagnosed based on the presenting symptoms
and confirmed by polymerase chain reaction (PCR) testing of the blister fluid or
scabs. Tests for antibodies may be performed to determine if immunity is present.
Although reinfections by varicella may occur, these reinfections are usually
asymptomatic and much milder than the primary infection.
Question 241
• Question 241: Which imaging study is most sensitive in diagnosing
primary carcinoid tumor?
• Choices:
• 1. MR
• 2. Metaiodobenzylguanidine (MIBG) scintigraphy
• 3. Octreotide scintigraphy
• 4. FDG PET scintigraphy
Answer: 3
• Answer: 3 - Octreotide scintigraphy
• Explanations: Carcinoid tumors are well-differentiated tumors that are
endocrinologically active. They may originate in both the gastrointestinal tract and
respiratory tract. They express elevated amounts of 5-HIAA. Metastatic lesions are
often best seen with CT and MR, the former using multiphasic techniques. Primary
lesions can be found with detailed CT/ MR techniques, like CT enterography, but are
better found with nuclear medicine methods. Metaiodobenzylguanidine (MIBG) is a
structural analogue to norepinephrine, and 123-I labeled MIBG can detect
pheochromocytoma, neuroblastoma, and carcinoid tumors. MIBG is better at
detecting gastrointestinal rather than bronchial and thymic carcinoids. Indium-111
labeled somatostatin analogue octreotide (pentetreotide) can also detect primary
carcinoid tumors, with a sensitivity approaching 90%. FDG PET is less successful, but
F-18 DOPA PET imaging is a developing technique.
Question 242
• Question 242: According to the Ann Arbor staging system, what stage
is Hodgkin lymphoma with affected lymph nodes on both sides of the
diaphragm?
• Choices:
• 1. Stage I
• 2. Stage II
• 3. Stage III
• 4. Stage IV
Answer: 3
• Answer: 3 - Stage III
• Explanations: HL spreads by contiguity, affecting adjacent lymph
nodes in progression. The Ann Arbor staging system defines 4 stages
of increasing extension, from I to IV. Stages I and II are defined by
lymphadenopathy in one (e.g., cervical) or two (e.g., axillary and
mediastinal) lymph node regions on the same side of the diaphragm,
respectively. Stage III defines lymphadenopathy affecting at least one
lymph node region on each side of the diaphragm, for example,
cervical and mesenteric. Stage IV defines extra-lymphatic involvement
by HL, for example, bone marrow, bone, and lung.
Question 243
• Question 243: When thalidomide is combined with corticosteroids for
the treatment of multiple myeloma, what is a potential complication?
• Choices:
• 1. Pneumocystis pneumonia
• 2. Heart failure
• 3. Deep vein thrombosis
• 4. Hematuria
Answer: 3
• Answer: 3 - Deep vein thrombosis
• Explanations: The incidence of deep vein thrombosis is very high
when thalidomide is combined with dexamethasone for the
treatment of multiple myeloma. Other side effects of thalidomide
include constipation and fatigue. Thalidomide also can cause
pancytopenia. Thus, all patients must have regular blood work to
ensure that they are not developing neutropenia and leukopenia. It
carries a black box warning regarding its teratogenicity.
Question 244
• Question 244: What is one of the most significant risk factors for
breast cancer?
• Choices:
• 1. Age
• 2. Weight
• 3. Family history
• 4. Smoking
Answer: 3
• Answer: 3 - Family history
• Explanations: Breast cancer risk is very high in women who have
close relatives with this disorder. Having a first-degree relative like
mother, sister, or daughter with breast cancer doubles a woman's risk
of also having it. Having two first-degree relatives increases the risk by
five-fold. At least 3 percent of women with breast cancer have a
family member with this disorder.
Question 245
• Question 245: Which of the following drugs is used in the treatment
of rheumatoid arthritis?
• Choices:
• 1. Methotrexate
• 2. Paclitaxel
• 3. Adriamycin
• 4. Carboplatin
Answer: 1
• Answer: 1 – Methotrexate
• Explanations: Methotrexate depresses the immune system and is
used to treat rheumatoid arthritis and severe psoriasis. Methotrexate
is also used to treat cancer of the breast, skin, head and neck, and
lung. Methotrexate is usually given after other medications have been
tried without success. Methotrexate is the initial drug of choice for
the treatment of rheumatoid arthritis. The majority of patients
prescribed methotrexate for RA do benefit from it. Unfortunately,
over time it's effectiveness lessens.
Question 246
• Question 246: A 65-year-old female presented to the clinic with a mass in her
right breast. Examination showed a hard immobile mass. Excisional biopsy
was done and the sample sent for further testing. The patient was
subsequently diagnosed with hormone-positive breast cancer. She underwent
radical mastectomy for her breast cancer. In addition to the surgery, what is
the best adjuvant treatment for this patient?
• Choices:
• 1. Estrogen
• 2. Clomiphene citrate
• 3. No further treatment
• 4. Anastrozole
Answer: 4
• Answer: 4 – Anastrozole
• Explanations: Anastrozole is an aromatase inhibitor, indicated as
adjuvant chemotherapy in postmenopausal women with breast
cancer. Clomiphene citrate is used to induce ovulation in the
treatment of infertility. It has no role in the treatment of breast
cancer. High levels of estrogen in the body have been shown to be a
risk factor for breast cancer. No further treatment is not the correct
answer. It is recommended to use adjuvant chemotherapy in addition
to surgery in patients with breast cancer.
Question 247
• Question 247: A 65-year-old female presents to the clinic complaining of generalized weakness,
shortness of breath, and fatigue. She has a history of hypertension, breast cancer for which she
is receiving chemotherapy, diabetes type 2 that is non-insulin dependent, past tobacco use, and
iron deficiency anemia. Vital signs revealed a blood pressure of 100/ 62 mmHg, a pulse of 82/
min, and a respiratory rate of 22/ min. Physical exam was significant for crackles and decreased
breath sounds on auscultation of the lungs bilaterally but was otherwise unremarkable. After
review of her medications, it was decided that her chemotherapy agent would need to be
changed. Which of the following is the mechanism of action of the chemotherapy agent being
taken by this patient?
• Choices:
• 1. Inhibition of cell wall
• 2. Inhibition of p53
• 3. Inhibition of DNA topoisomerase II
• 4. Inhibition of microtubules
Answer: 3
• Answer: 3 - Inhibition of DNA topoisomerase II
• Explanations: Adriamycin, also known as doxorubicin, has a
mechanism of action of inhibiting DNA topoisomerase II. Doxorubicin
affects the DNA helix by affecting base-pair bonding and thereby
affecting the mechanism of action of DNA topoisomerase II.
Doxorubicin is known for causing congestive heart failure, as seen by
the physical exam (crackles and decreased breath sounds) and
presentation of this patient. Doxorubicin is contraindicated in patients
with a history of cardiac disease, congestive heart failure, or recent
myocardial infarction (within 30 days).
Question 248
• Question 248: A 24-year-old female with a history of rheumatoid arthritis presented with
diffuse abdominal discomfort for the past 6 months. She was previously treated for gastritis.
She denied jaundice. Upper GI endoscopy was unremarkable, and the colonoscopy was
unrevealing. Physical examination was remarkable for the tender lateral abdomen. Abdominal
computerized tomography revealed a well-defined and heterogeneously circumscribed 3 x 3.5
x2.5 cm mass in the tail of the pancreas without invasion to surrounding tissues. On further
evaluation, no other signs and symptoms suggestive of malignant tumor identified. What are
the common epidemiological determinants of this neoplasm?
• Choices:
• 1. Adenocarcinoma of pancreas mostly in non-Hispanic whites with no age variations
• 2. SPEN of the pancreas is seen mostly in young non-white women
• 3. Pancreatic adenocarcinoma is seen mostly in the elderly of either sex
• 4. SPEN of pancreas seen commonly in Men of elderly
Answer: 2
• Answer: 2 - SPEN of the pancreas is seen mostly in young non-white
women
• Explanations: The clinical vignette points solid and papillary epithelial
neoplasm (SPEN) that are commonly seen in younger age. These
neoplasms are rarely malignant. SPEN tend to present in young non-
white females around the 2nd or 3rd decades of age. The mean age of
the SPEN of pancreas presentation is 25 years. The findings described
are for benign neoplasm, not malignant. SPEN of the pancreas is rare
in older age groups and in men compared to young women.
Question 249
• Question 249: Tumor lysis syndrome is a clinical condition that is
associated with one of the following electrolyte abnormalities?
• Choices:
• 1. Hypernatremia
• 2. Hyponatremia
• 3. Hypercalcemia
• 4. Hypocalcemia
Answer: 4
• Answer: 4 - Hypocalcemia
• Explanations: Tumor lysis syndrome (TLS) is associated with
hypocalcemia. Hypocalcemia in TLS is mostly secondary to the
chelation of phosphorus. This condition is more potentially life-
threatening than hyperphosphatemia. Potential complication of
hypocalcemia are arrhythmias, tetany, seizure, and death. The level of
calcium might still be relatively low even after the normalization of
phosphorus level because of deficiency of 1, 25 vitamin D.
Question 250
• Question 250: Exposure to which of the following is most likely to
cause peritoneal mesotheliomas?
• Choices:
• 1. Arsenic
• 2. Garlic
• 3. Asbestos
• 4. Radiation
Answer: 3
• Answer: 3 – Asbestos
• Explanations: Peritoneal mesotheliomas are caused by exposure to
asbestos. Peritoneal mesothelioma has a very rapid downhill course
and is usually seen in middle aged men. While exposure to asbestosis
is the leading risk factor, other cases have been linked to radiation,
talc, or mica exposure. Only 50 percent of patients with peritoneal
mesothelioma have a prior history of asbestosis exposure. Simian
virus association remains controversial.
Question 251
• Question 251: What is the advantage of pegfilgrastim over filgrastim?
• Choices:
• 1. It is more potent than filgrastim
• 2. It lasts longer than filgrastim
• 3. It is less harmful than filgrastim
• 4. It is more efficacious than filgrastim
Answer: 2
• Answer: 2 - It lasts longer than filgrastim
• Explanations: Filgrastim and pegfilgrastim are granulocyte-colony stimulating
factors. Pegylation is a process of covalent attachment of polyethylene glycol
polymer chains to another molecule. Pegylation reduces renal clearance and
makes the drug last longer. Pegfilgrastim has a prolonged duration of effect
relative to filgrastim and a reduced renal clearance.
Question 252

• Question 252: Which of the following cells actively secretes a specific


idiotype of antibody?
• Choices:
• 1. Plasma cell
• 2. CD4 + cell
• 3. CD8 + cell
• 4. Mature B cell
Answer: 1
• Answer: 1 - Plasma cell
• Explanations: The plasma cell, with its well-developed rough
endoplasmic reticulum, is the producer of a specific, secreted
antibody. Plasma cells are the terminal differentiation of a B cell and
secrete immunoglobulin Mature plasma cells lack most common B
cell markers. They are identified by CD138, CD78, and the interleukin-
6 receptor. They also do not express CD45
Question 253
• Question 253 : A 10-year-old boy with a past medical history of neurofibromatosis type 1 is
brought to the clinic by his mother because of a persistent small neck mass. The mother states
that she first noticed it when he was sick. She initially assumed it was an enlarged lymph node
related to an illness, but it has persisted for the past six months. The patient says that the
swelling does not cause any pain or trouble swallowing. The physical exam is only notable for a
small mass on the left side of his neck — a subsequent ultrasound and MRI concern for a
tumor. A biopsy reveals small, round, blue cells with immunohistochemistry positive for demin
and myoglobin. The clinician is worried about a metastasis. What symptom would be
concerning for this?
• Choices:
• 1. Cough
• 2. Headache
• 3. Upper abdominal pain
• 4. Urinary frequency
Answer: 1
• Answer: 1 - Cough
• Explanations: The pathological identification of rhabdomyosarcoma is
sometimes difficult due to the similarity of this entity to the other
small, round, blue, cell tumors of bone and soft tissue in childhood.
The presenting signs and symptoms of rhabdomyosarcoma are
variable and depend on multiple factors including the site of origin,
the patient’s age, and the presence or absence of distant metastases.
Common rhabdomyosarcoma metastatic sites include lung, bone
marrow, and lymph nodes. A lung metastasis can present with a
cough. The overall survival of metastatic rhabdomyosarcoma patients
is low and typically does not exceed 25%.
Question 254
• Question 254: A 49-year-old female undergoes a breast biopsy.
Immunochemical staining is positive for HER2/ neu, but negative for
both estrogen and progesterone receptors. Which of the following is
the best treatment for this patient?
• Choices:
• 1. Radiation
• 2. Tamoxifen
• 3. Trastuzumab
• 4. Chemotherapy with cisplatinum
Answer: 3
• Answer: 3 - Trastuzumab Explanations: Lack of staining for estrogen
receptors indicates that the tumor will not respond to tamoxifen.
Positive HER2/ neu staining indicates that she can be treated with
herceptin (trastuzumab). Females who are positive for HER2 usually
have a poor prognosis. Radiation therapy is offered to patients to
prevent local recurrence.
Question 255
• Question 255: Which of the following is associated with the CD10
antigen?
• Choices:
• 1. Acute myeloid leukemia
• 2. Acute lymphocytic leukemia
• 3. Chronic myeloid leukemia
• 4. Hodgkin lymphoma
Answer: 2
• Answer: 2 - Acute lymphocytic leukemia
• Explanations: The CD10 antigen is also known as neprilysin or
membrane metalloendopeptidase. The CD10 antigen is a zinc
dependent enzyme that degrades many peptides and is often
associated with acute lymphoblastic leukemias. In some types of
cancers, the CD10 is over expressed. Mutation in the CD 10 antigen
has also been linked to Alzheimer disease.
Question 256
• Question 256: Where is the most common site of metastasis for
sarcomas?
• Choices:
• 1. Lymph node
• 2. Lung
• 3. Liver
• 4. Kidney
Answer: 2
• Answer: 2 - Lung Explanations: Soft tissue sarcomas most commonly
metastasize to the lungs.
Question 257
• Question 257: A 22-year-old female patient presents to the office for
yearly evaluation. Her physical examination reveals a 1 cm left thyroid
nodule that is tender and mobile. Which of the following features
should raise the suspicion for malignancy?
• Choices:
• 1. The young age of the patient
• 2. History of radiation exposure
• 3. Tenderness
• 4. The sudden appearance of the nodule
Answer: 2
• Answer: 2 - History of radiation exposure
• Explanations: A young patient with a single firm nodule, a prior
history of neck radiation, and a "cold" nodule shown on radioiodine
thyroid imaging is considered high suspicion for thyroid cancer. Fine
needle aspiration (FNA) biopsy is the recommended next step in the
management of this patient. In conjunction with ultrasound
evaluation, FNA forms the cornerstone for thyroid nodule evaluation,
representing by itself the most cost-effective diagnostic tool used in
the assessment of nodules of the thyroid. Even with a negative FNA,
with a history of irradiation, further evaluation, including
hemithyroidectomy, may be warranted.
Question 258
• Question 258: A 5-year-old African child is brought to the hospital with fatigue and
a soft-tissue mass anterior to the left masseter muscle. The child's cervical lymph
nodes have enlarged rapidly within the past month. Investigations for EBV are
ordered, which come back positive. A FISH (Fluorescence in situ hybridization) test
is ordered as a follow-up and reveals a t( 8;14) translocation. Based on the
presentation of the patient and the exam results, what should be the
pharmacological recommendation in such a case?
• Choices:
• 1. Low dose cyclophosphamide
• 2. Observation of the mass for 3 weeks
• 3. High dose cyclophosphamide
• 4. ABVD (adriamycin, bleomycin, vinblastine, and dacarbazine)
Answer: 3
• Answer: 3 - High dose cyclophosphamide
• Explanations: Cyclophosphamide is activated and metabolized in the
liver by a microsomal oxidase system to active alkylating metabolites.
Rapidly proliferating malignant cells are often susceptible to the
metabolites of cyclophosphamide. These metabolites are thought to
cross-link DNA in tumor cells. It is mainly used for lymphomas but also
is used for some types of leukemia. High dose is appropriate for anti-
neoplastic purposes, while low-dose is preferred for treatment of
juvenile idiopathic arthritis/ vasculitis.
Question 259
• Question 259: Which one of the following is the best way to reinforce
positive coping behaviors in patients with cancer?
• Choices: 1. Linking patients to support groups
• 2. Making decisions for the patient
• 3. Advising on side effects of drugs
• 4. Encouraging compliance with treatment regimens
Answer: 1
• Answer: 1 - Linking patients to support groups
• Explanations: Joining support groups is a positive reinforcement for
coping styles. Making decisions for the patient diminishes self-
confidence. All patients should be educated about possible side
effects of drugs and the benefits of compliance but these do not
reinforce positive coping. Having cancer is stressful. Support groups
help people cope with the emotional aspects of cancer by providing a
place to share their feelings and challenges. They also allow people to
learn from others facing similar situations.
Question 260
• Question 260: What organ is the origin of the most common non-
dermatologic malignant neoplasm in women between the ages of 40
and 65 years?
• Choices:
• 1. Breast
• 2. Pancreas
• 3. Leukemia
• 4. Uterus
Answer: 1
• Answer: 1 – Breast
• Explanations: Skin cancer is the most common cancer in the United
States. The most common internal malignancy in women is breast
cancer, and the most common in males is prostate cancer. Lung
cancer is the most common cause of cancer mortality in the United
States for both men and women. Cancer deaths per capita in the
United States have fallen significantly, dropping 20% overall in the last
two decades in almost all major categories. This is due to a
combination of improved treatments, improved screening, and early
detection. Heart attack and stroke remain responsible for larger
numbers of deaths each year.
Question 261
• Question 261: What is the 5-year survival rate for bone metastasis
from kidney cancer?
• Choices:
• 1. 1%
• 2. 10%
• 3. 25%
• 4. 50%
Answer: 2
• Answer: 2 - 10% Explanations: Bone metastasis from breast has a 20%
5-year survival rate. Bone metastasis from prostate has a 25% 5-year
survival rate. Bone metastasis from kidney has a 10% 5-year survival
rate.
Question 262
• Question 262: A 66-year-old man with a history of follicular non-Hodgkins
lymphoma is being treated with anthracycline therapy, interferon
adjuvant therapy, and supportive pain medications. Over the course of his
treatment, this patient develops suicidal ideation. Which of the following
medications is most likely responsible for this patient’s symptoms?
• Choices:
• 1. Interferon-alpha 2b
• 2. Doxorubicin
• 3. Hydroxycodone
• 4. Acetaminophen
Answer: 1
• Answer: 1 - Interferon-alpha 2b Explanations: Interferon-alpha drugs
are used to treat a number of cancers, including follicular non-
Hodgkins lymphoma, as adjuvant therapy, in combination with
anthracyclines. Interferon therapy has been associated with
depression and suicidal ideation. Doxorubicin is an anthracycline that
is used to treat many cancers, including follicular non-Hodgkins
lymphoma. However, it would not be associated with suicidal
ideation. Acetaminophen and hydrocodone are supportive pain
medications. They do not cause suicidal ideation.
Question 263
• Question 263: A 55 year-old female is evaluated for abdominal and back
pain. Blood work and physical exam are normal. Computed scan of
abdomen reveals a 6 cm cystic lesion in tail region of pancreas. Histology
reveals internal septations and papillary projections from its walls. The best
management for a patient with a cystadenoma of the tail of pancreas is:
• Choices:
• 1. Distal pancreatectomy
• 2. Cysto-jejunostomy
• 3. Percutaneous drainage
• 4. Primary radiotherapy and chemotherapy
Answer: 1
• Answer: 1 - Distal pancreatectomy
• Explanations: Cystic lesions of pancreas are rare. Serous cystadenoma
is more common than mucinous cystic neoplasms. The majority of
cystadenomas of the pancreas are large at presentation and grow
near the tail. These may be removed with a distal pancreatectomy.
When the tumor is located in the head of the pancreas, a Whipple
operation may be required. At surgery, it is important to make sure
that the surgical margins are free of disease. Some surgeons even
insert a scope and examine the pancreatic duct during surgery.
Question 264
• Question 264: A 16-year-old male patient with a history of attention deficit
hyperactivity disorder (ADHD) and eye cancer presents with a 3-month history of right
knee pain. His pediatrician obtained an x-ray of his knee, which demonstrates a
destructive lesion in the proximal tibial metaphysis. A biopsy specimen shows poorly
differentiated malignant cells with abnormal mitotic figures and high nuclear to
cytoplasmic ratio. Which of the following genetic abnormalities is correctly paired
with the most likely primary bone cancer in this case?
• Choices:
• 1. Osteosarcoma and pRB-1 mutation
• 2. Ewing Sarcoma and t (2;13) translocation
• 3. Chondrosarcoma and translocation of the RB gene
• 4. Osteosarcoma and t (11;22) mutation
Answer: 1
• Answer: 1 - Osteosarcoma and pRB-1 mutation
• Explanations: This question stem describes an adolescent male patient
with a history of retinoblastoma (eye cancer). Mutations in the tumor
suppressor genes p53 and retinoblastoma (RB) are associated with the
development of osteosarcoma. The most common primary bone cancer
in this age group is osteosarcoma. Genetic syndromes caused by germline
mutations (that results in the inactivation of tumor suppressor gene or
activation of oncogenes) leads to the development of osteosarcoma. For
example, Le Fraumeni syndrome is an autosomal dominant disorder that
is the result of the inactivation of the p53 tumor suppressor gene. In this
disorder, patients are susceptible to developing many
• types of cancer, including osteosarcoma. Similarly, a mutation in the
retinoblastoma (RB1) gene is also associated with osteosarcoma. The
translocation between the Ewing Sarcoma gene EWS and
transcription factor FLI t (11;22) is associated with Ewing sarcoma.
Commonly, Ewing sarcoma occurs in the diaphysis of long bones.
Ewing sarcoma is the second most common primary bone cancer in
adolescents. The translocation t( 2;13) is associated with
rhabdomyosarcoma. The etiology of chondrosarcoma is still under
investigation. Mutations in the IDH genes are associated with some
forms of chondrosarcoma.
Question 265
• Question 265: Carcinoma of the tongue most commonly occurs at
which anatomical site?
• Choices:
• 1. Tip of the tongue
• 2. Tongue dorsum
• 3. Lateral border
• 4. Within the tongue substance
Answer: 3
• Answer: 3 - Lateral border
• Explanations: The most common location for carcinoma of the tongue
is on the lateral border, followed by the tongue tip. The number one
risk factor for tongue cancer is tobacco, followed by alcohol. This
malignancy is associated with a high number of deaths globally. Like
all other areas of the aerodigestive tract, the most common histology
of tongue cancer is squamous cell carcinoma. Both erythroplakia and
leukoplakia are lesions which have a high potential to transform into a
malignancy.
Question 266
• Question 266: A 72-year-old male presents to the emergency department
with sudden onset seizure and confusion. He also had multiple falls at home
and noticed that his left leg was weaker. He has a strong smoking history and
has been recently diagnosed with lung cancer. He undergoes brain imaging
which is negative for stroke. Lumbar puncture shows cerebrospinal fluid
cytology positive for malignant cells. What is the current stage of his cancer?
• Choices:
• 1. I
• 2. II
• 3. III
• 4. IV
Answer: 4
• Answer: 4 - IV
• Explanations: Leptomeningeal carcinomatosis is a late-stage and
metastatic complication of various cancer types, it is classified as
stage IV disease. Breast and lung cancer and melanoma are most
common cancer types known to cause leptomeningeal involvement.
Diagnosis requires at least a high-quality MRI with contrast and
lumbar puncture with cerebrospinal fluid studies. Rarely, it may be a
presenting symptom of systemic cancer. Given the metastatic nature
of the disease, the prognosis of leptomeningeal carcinomatosis is very
poor.
Question 267
• Question 267: A person found working in a factory where aniline
exposure is common might develop what malignancy?
• Choices:
• 1. Prostate cancer
• 2. Benign prostate hypertrophy
• 3. Bladder cancer
• 4. Renal cell cancer
Answer: 3
• Answer: 3 - Bladder cancer
• Explanations: Smoking is the major risk factor for bladder cancer.
Other risk factors for bladder cancer include exposure to carcinogens
like naphthylamine and aniline dyes. Hairdressers are prone to
bladder cancer because of the extensive exposure to hair dyes.
Colored and bright fruits like carrots are thought to reduce the risk of
bladder cancer.
Question 268
• Question 268: Which of the following is a risk factor for developing
cervical cancer?
• Choices:
• 1. Pregnancy as a teenager
• 2. Use of diaphragms for contraception
• 3. Douching regularly
• 4. Precocious puberty
Answer: 1
• Answer: 1 - Pregnancy as a teenager
• Explanations: Early age of the first pregnancy is a risk factor for
developing cervical cancer. Multiple sexual partners, a history of
sexually transmitted infection, and low socioeconomic status are
other risk factors. Risks factors for HPV and cervical cancer include
age at first intercourse, multiple sexual partners, smoking, herpes
simplex, HIV, and oral contraceptive use. HPV is transmitted by skin to
skin contact including during sexual intercourse, hand to genital organ
contact, and oral sex.
Question 269
• Question 269: A 70-year-old gravida 1, para 1, female with BMI 21 kg/ m2 consented to
undergo a hysterectomy, salpingo-oophorectomy, with possible pelvic and aortic lymph
node dissection and omentectomy after an endometrial biopsy revealed complex atypical
hyperplasia. Her medical history is significant for hypertension, controlled with lisinopril.
She denies any past surgical history or contributory social history. Of the following, which
most closely approximates the percentage of patients with complex atypical hyperplasia
on a preoperative biopsy who will have endometrial cancer on final pathology at the time
of hysterectomy?
• Choices:
• 1. 5%
• 2. 10%
• 3. 40%
• 4. 60%
Answer: 3
• Answer: 3 - 40%
• Explanations: Complex atypical hyperplasia is felt to be the immediate
precursor lesion for many endometrioid endometrial carcinomas. 40%
of patients with complex atypical hyperplasia on a preoperative biopsy
will have endometrial cancer on final pathology at the time of
hysterectomy The presence of complex atypical hyperplasia increases
the risk that a patient will develop endometrial carcinoma. Patients
with complex atypical hyperplasia should be counseled regarding the
risk of a coexistent endometrioid adenocarcinoma of the endometrium.
For patients who have completed childbearing, definitive management
of complex atypical hyperplasia is a hysterectomy.
Question 270
• Question 270: A 22-year-old African American female presents with a
new mass in the upper outer quadrant of her left breast. It is mildly
tender and mobile. What is the next step in management?
• Choices:
• 1. Diagnostic mammogram of the left breast
• 2. Bilateral screening mammogram
• 3. Left breast ultrasound
4. Breast MRI
Answer: 3
• Answer: 3 - Left breast ultrasound
• Explanations: Because of the breast density in females less than 35
years of age, a breast ultrasound is the best test to assess a breast
mass. In young females, fibroadenoma is the most common diagnosis,
but other breast mass etiologies need to be ruled out. Ultrasound will
show echogenicity, borders, and size. Ultrasound also can be used for
performing a core needle biopsy if a minimally invasive tissue
diagnosis is needed.
Question 271
• Question 271: In which of the following situations does the release of
adrenocorticotropic hormone cause a high plasma cortisol?
• Choices:
• 1. Hypovolemia
• 2. Prolonged surgery
• 3. Cushing disease
• 4. Somatic pain
Answer: 3
• Answer: 3 - Cushing disease
• Explanations: Fluctuating levels of serum cortisol have been linked to
abnormal adrenocorticotropic hormone (ACTH) levels. Depression,
stress, hypoglycemia, fever, trauma, fear, and pain can result in
abnormal ACTH levels. Diagnostic tests used for Cushing disease are
salivary and blood serum cortisol, 24-hour urinary free cortisol,
dexamethasone suppression test, and bilateral inferior petrosal sinus
sampling. Multiple tests should be used. First-line treatment is
surgical resection of the ACTH-secreting pituitary adenoma. Pituitary
radiation or bilateral adrenalectomy may be needed for postoperative
persisting hypercortisolemia.
Question 272
• Question 272: Which of the following increases the risk of testicular
cancer?
• Choices:
• 1. Early age of sexual activity
• 2. History of chlamydia infection
• 3. Undescended testes
• 4. History of prostatitis
Answer: 3
• Answer: 3 - Undescended testes
• Explanations: Undescended testes increase the risk of developing
testicular cancer. Testicular cancer also is associated with trauma and
orchitis. Testicular cancer usually occurs in men between ages 15 and
40 years. When comparing whites and African Americans, the
incidence of testicular cancer is 5:
Question 273
• Question 273: What is the recommended treatment for a mediastinal
non-seminomatous tumor?
• Choices:
• 1. Surgery
• 2. Chemotherapy followed by surgery
• 3. Radiation therapy alone
• 4. Hormonal therapy
Answer: 2
• Answer: 2 - Chemotherapy followed by surgery
• Explanations: Mediastinal non-seminomatous germ cell tumors
usually are locally invasive with metastases. This makes surgical
resection of all disease impossible. Chemotherapy is with bleomycin,
cisplatin, and etoposide-based regimens. Alpha-fetoprotein and beta
HCG are followed to assess response to treatment. If complete
response is seen the patients are monitored. If not, a second course
of chemotherapy is done. If radiographic abnormality remains with
normal serum markers, surgery is performed. Non-seminomas are
rarely radiosensitive.
Question 274
• Question 274: Which of the following medications acts on serotonin
receptors and is recommended for vomiting as a complication of
chemotherapy ?
• Choices:
• 1. Metoclopramide
• 2. Ondansetron
• 3. Hydroxyzine
• 4. Prochlorperazine
Answer: 2
• Answer: 2 - Ondansetron
• Explanations: Ondansetron is a 5 HT3 inhibitor and is useful for
chemotherapy-induced vomiting and nausea. It is related to other
agents such as granisetron and dolasetron. It is used preoperatively to
reduce nausea and vomiting. Adverse effects include headache,
malaise, diarrhea, and dizziness.
Question 275
• Question 275: A patient with advance colon cancer is going home from
the hospital. He is depressed and tires quickly although he can still
ambulate. He declines to participate in most activities. Select the best
OT recommendations for after discharge.
• Choices:
• 1. Discharge the patient from OT services
• 2. Refer the patient for home OT
• 3. Discuss the possibility of hospice and hospice OT with the primary
physician
• 4. Recommend a set of home exercises
Answer: 3
• Answer: 3 - Discuss the possibility of hospice and hospice OT with the
primary physician
• Explanations: Patients with advanced cancer, not seeking further
aggressive treatment are best cared for by a hospice. Hospice OT
focuses on quality of life, adaptation, and locus of control. A
depressed terminal patient is unlikely to do home exercises.
Discontinuation of all OT services would hinder continuation of role
performance.
Question 276
• Question 276: Which of the following breast cancers will reveal a
marked dermal lymphatic infiltration by cancer?
• Choices:
• 1. Paget
• 2. Intraductal
• 3. Inflammatory
• 4. Medullary
Answer: 3
• Answer: 3 - Inflammatory
• Explanations: Inflammatory breast cancer is a very aggressive type of
cancer. It tends to present with a red swollen and tender breast. A
biopsy will reveal marked dermal lymphatic infiltration by cancer cells.
Other features of the cancer include tethering of skin, peau d'orange,
and nipple retraction.
Question 277
• Question 277: A 56-year-old woman with a history of stage IIIA breast cancer (T3
N1 M0) presents to the clinic for her first chemotherapy regimen cycle, consisting
of docetaxel, doxorubicin, and cyclophosphamide. After she is done receiving
treatment, she will finish her visit by having an on-body injector placed on the back
of her upper arm, which will release pegfilgrastim 27 hours later. Which of the
following potential adverse effects is most likely to be prevented by this regimen?
• Choices:
• 1. Neutropenia
• 2. Hyponatremia
• 3. Hepatotoxicity
• 4. Heart failure
Answer: 1
• Answer: 1 - Neutropenia
• Explanations: Neutropenia (possibly progressing to febrile neutropenia)
is a disease state that is experienced by some patients with cancer
receiving myelosuppressive chemotherapy, such as this patient who is
receiving docetaxel, doxorubicin, and cyclophosphamide, all of which
can cause bone marrow suppression. Pegfilgrastim leads to the
increased production and maturation of neutrophils. Due to its effect
on bone marrow, a common side effect experienced by patients taking
pegfilgrastim is bone pain. Patients should have their temperature
taken regularly while receiving myelosuppressive chemotherapy in
order to monitor for febrile neutropenia.
Question 278
• Question 278: Which of the following tests are not typically needed
when evaluating a patient with gestational trophoblastic disease?
• Choices:
• 1. Chest x-ray
• 2. Liver function tests
• 3. Thyroid stimulating hormone
• 4. Hemoglobin
Answer: 2
• Answer: 2 - Liver function tests
• Explanations: Liver function tests are not routinely obtained. However, they
should be done if the patient presents with hypertension. Patients with molar
pregnancies are at increased risk of developing pre-eclampsia. Thyroid
stimulating hormone must be obtained to rule out molar pregnancy induced
hyperthyroidism. This should resolve with evacuation of the uterus, but
patients may require beta-adrenergic blocking agents before anesthesia to
reverse effects of thyroid storm. Patients with molar pregnancies often
present with vaginal bleeding and are at significant risk for bleeding with
dilation and evacuation. Therefore a baseline hemoglobin must be obtained.
All patients with molar pregnancies should have a chest x-ray to rule out
extrauterine disease. The most common location of metastasis is the lung.
Question 279
• Question 279: Which of the following medications can be helpful to
reduce the negative impact of tumor lysis syndrome?
• Choices:
• 1. Aspirin
• 2. Allopurinol
• 3. Steroids
• 4. Leucovorin
Answer: 2
• Answer: 2 - Allopurinol
• Explanations: Massive killing of tumor cells usually leads to increased
uric acid and other electrolytes. Allopurinol given before
chemotherapy helps reduce hyperuricemia by inhibiting xanthine
oxidase. Metabolism of purine analog drugs may be reduced by
inhibition of xanthine oxidase. Caution should be taken when
administering purine analog chemotherapy agent and allopurinol.
Question 280
• Question 280: Which is true regarding the treatment of hydatidiform
mole?
• Choices:
• 1. Embolization of trophoblastic tissue is a risk during suction curettage
• 2. Most molar pregnancies pass spontaneously and do not require
evacuation of the uterus
• 3. It is important to follow beta-human chorionic gonadotropin levels
until they begin to fall
• 4. Most patients require chemotherapy as part of their management
Answer: 1
• Answer: 1 - Embolization of trophoblastic tissue is a risk during
suction curettage
• Explanations: Molar pregnancy is most safely treated with suction
curettage, which carries a risk of embolization. Beta-human chorionic
gonadotropin levels must be followed until the titer is zero for 3
weeks. If titers begin to rise, one should consider invasive
trophoblastic disease. Furthermore, patients during this monitoring
phase should be advised to undertake reliable contraception to
prevent pregnancy. Only 20% of patients with a molar pregnancy
require chemotherapy, almost exclusively in the high-risk complete
molar group.
Question 281
• Question 281: A 36-year-old gravida 2, para 2, female is diagnosed with endometrial cancer. She initially
presented to the emergency department with the passage of clots, heavy vaginal bleeding, and feeling
pre-syncopal. Her blood pressure was 87/ 56 mmHg, and pulse was 120 beats per minute. Her hemoglobin
was noted to be 7.6 g/ dl. She received 1 unit of packed red blood cells with symptomatic improvement.
Further evaluation reveals that she underwent menarche at the age of 15, and her past medical history is
significant for estrogen-receptor-positive, BRCA 1 and 2 negative, breast cancer diagnosed 4 years ago. She
underwent a bilateral mastectomy, bilateral salpingo-oophorectomy, and has been taking tamoxifen for
the last 46 months. Pelvic examination was concerning for blood coming from the cervical os without any
other masses or abnormalities. Pelvic ultrasound demonstrated a 20 mm heterogenous endometrial
stripe. What aspect of her history conveys the greatest risk for developing endometrial cancer?
• Choices:
• 1. Age less than 40 years old
• 2. Multiparous
• 3. Menarche at 15 years old
• 4. The use of tamoxifen
Answer: 4
• Answer: 4 - The use of tamoxifen
• Explanations: Although the peak age for endometrial cancer is 60
years, most clinicians agree that women who are 40 years of age or
older are at increased risk. Contrary to its anti-estrogenic effect on
breast tissue, tamoxifen has a stimulatory effect on the endometrium.
As a result, patients taking tamoxifen are at an increased risk of
developing endometrial cancer. Other risk factors include diabetes
mellitus, irregular menstrual cycles, and the presence of atypical
glandular cells (AGC) on a Pap smear. Use of oral contraceptive pills,
cigarette smoking, and multiparity are considered protective factors.
Question 282
• Question 282: Which of the following is true about the most
malignant neurogenic tumor of childhood?
• Choices:
• 1. It arises from the nerve sheath
• 2. Less than 5 percent are hormonally active and produce
catecholamines
• 3. Metastatic spread to liver and bone is common
• 4. Radiation therapy is the treatment of choice
Answer: 3
• Answer: 3 - Metastatic spread to liver and bone is common
• Explanations: The most malignant neurogenic tumor in childhood is a
neuroblastoma. It arises from the sympathetic ganglia cells and can
secrete catecholamines. The majority occur before 2 years of age and
occur exclusively in the paravertebral sulci. They are highly invasive
and metastatic. They present with an abdominal mass, back pain,
hypertension, or paraplegia. Various paraneoplastic syndromes are
associated with it. They can secrete VIP (watery diarrhea, abdominal
pain) and catecholamines. Treatment is with both surgery and
chemotherapy. Prognosis is poor with this tumor.
Question 283
• Question 283: A patient has been diagnosed with leukemia. The
patient is to start vincristine therapy. Your instructions to the patient
will include which of the following information?
• Choices:
• 1. This drug is taken by mouth as home therapy
• 2. This drug rarely affects coordination or mentation
• 3. This drug may not be taken if the patient is experiencing
chickenpox or shingles
• 4. This drug is not effective in the treatment of Hodgkin disease
Answer: 3
• Answer: 3 - This drug may not be taken if the patient is experiencing
chickenpox or shingles
• Explanations: Vincristine should not be administered if the patient has
chickenpox or shingles because the immunosuppressant effect of this
neoplastic medication may cause chickenpox or shingles to generalize
and become more severe. Vincristine is administered. Vincristine
commonly causes peripheral neuropathy (pathology of the nerves).
The immunosuppressant effect of vincristine has made it very useful
in the treatment of Hodgkin disease. Overuse of vincristine has led to
the development of resistance by overexpression of p-glycoprotein
pump.
Question 284
• Question 284: A 28-year-old gravida 0 female presents to a reproductive
endocrinologist's office desiring fertility. She was recently diagnosed with
polycystic ovarian syndrome (PCOS) based on chronic anovulation, signs of
hirsutism (facial hair), and an ultrasound that was significant for multiple follicles
(> 10) on each ovary. At her initial visit, her body mass index (BMI) was noted to
be 38 kg/ m2, and her heart rate was 120 beats per minute. All of her other vital
signs were within normal limits. Her past medical history is significant for type II
diabetes that is controlled with metformin 500mg twice daily. She denies any
past surgical history. Her family history is significant for hypertension,
dyslipidemia, and obesity in her mother and father and PCOS in her older sister.
Upon questioning, she received all childhood vaccinations, including the HPV
series as a teenager. A cycle was induced using the progesterone withdrawal
method. However, she continued to have heavy bleeding with the
passage of clots for four weeks. A pelvic ultrasound was performed and
demonstrated a diffusely thickened endometrium without any other anomalies.
An endometrial biopsy was collected, and the result was concerning for
endometrial adenocarcinoma. Which of the following is true regarding
endometrial adenocarcinoma?
• Choices:
• 1. Increased estrogen exposure without progesterone is not a risk factor
• 2. Age less than 50 years is a risk factor
• 3. Surgical management is the preferred treatment option
• 4. Chemotherapy and hormonal therapy are commonly used for stage 1
disease
Answer: 3
• Answer: 3 - Surgical management is the preferred treatment option
Explanations: The primary treatment for endometrial
adenocarcinoma is total abdominal hysterectomy and bilateral
salpingo-oophorectomy. Lymph node dissection is necessary if the
depth of invasion greater then or equal to 50% of the myometrium.
Unopposed estrogen use is a risk factor for endometrial cancer and
leads to endometrial hyperplasia. Age greater than 50 years is a risk
factor for endometrial cancer. This patient is 55 years old.
Chemotherapy and hormonal therapy may be used as palliative
therapy. Radiation therapy is used as an adjuvant in stage 1, 2, or 3
diseases or if the patient is a poor surgical candidate.
Question 285
• Question 285: A 68-year-old male presents with concerns over a 3 cm skin lesion on
his neck. He is a farmer by occupation and spends a lot of time in the sun. His
history was significant for melanoma 20 years ago. On examination, the lesion has
rolled borders and central ulceration. A punch biopsy of the edge of the lesion is
performed in the office, showing a basal cell carcinoma with a high mitotic index
and perineural invasion. Which of the following features would indicate a high risk
of local recurrence?
• Choices:
• 1. Ultraviolet light exposure
• 2. Perineural invasion
• 3. History of melanoma
• 4. Rolled borders with central ulceration
Answer: 2
• Answer: 2 - Perineural invasion
• Explanations: Exposure to ionizing radiation causes long-lasting
changes to the dermis. Basal cell carcinoma occurring in an irradiated
field has a high risk of recurrence. A basal cell carcinoma
demonstrating perineural invasion is a more aggressive lesion and has
a high risk of recurrence following excision. Immunocompromised
patients have a higher risk of basal cell carcinoma recurrence after
treatment. This is true in many types of malignancies. A gross finding
of rolled borders on a basal cell carcinoma does not increase its risk of
recurrence. Rolled borders do not have predictive value regarding
biologic behavior.
Question 286
• Question 286: For which of the following is carcinoembryonic antigen
(CEA) not used?
• Choices:
• 1. To screen for colorectal cancer
• 2. To assess tumor load in patients that have resectable tumors
• 3. To monitor a patient for recurrence after surgery
• 4. To follow a response to treatment
Answer: 1
• Answer: 1 - To screen for colorectal cancer
• Explanations: Carcinoembryonic antigen (CEA) is not used to screen
for colon cancer. It is used to determine prognosis in patients with
tumors that appear to be resectable. A high CEA level indicates worse
prognosis but has not been incorporated into staging. CEA should be
monitored every 3 months for patients with stage II or III disease. CEA
can be used to follow response to treatment for residual disease.
Question 287
• Question 287: A 65-year-old white female is in the intensive care unit following major
surgery to treat an invasive lymphoma. She has a past medical history positive for
hyperlipidemia and type 1 diabetes mellitus. The patient is given oral mercaptopurine, and
her serum drug levels are measured accordingly. What would be the relative serum drug
level of mercaptopurine in the presence of normal and decreased liver function?
• Choices:
• 1. Increased serum levels of mercaptopurine in both normal and decreased liver function
• 2. Appropriate serum levels of mercaptopurine in normal liver function and increased serum
levels of mercaptopurine in decreased liver function
• 3. Increased serum levels of mercaptopurine in normal liver function and appropriate serum
levels of mercaptopurine in decreased liver function
• 4. Decreased serum levels of mercaptopurine in normal liver function and increased levels
of mercaptopurine in decreased liver function
Answer: 2
• Answer: 2 - Appropriate serum levels of mercaptopurine in normal
liver function and increased serum levels of mercaptopurine in
decreased liver function
• Explanations: The severity of the first pass effect is different for every
drug. Mercaptopurine experiences the first-pass effect, and in the
presence of decreased liver function, its metabolism would be
decreased, resulting in increased serum levels of the drug. The
severity of the first-pass effect can vary among genders. The severity
of the first-pass effect can also vary among age groups.
Question 288
• Question 288: Which of the following is a mixed opioid agonist-
antagonist?
• Choices:
• 1. Naloxone
• 2. Meperidine
• 3. Methadone
• 4. Nalbuphine
Answer: 4
• Answer: 4 – Nalbuphine
• Explanations: Nalbuphine is a mixed agonist-antagonist opioid
medication with analgesic properties. Nalbuphine is an agonist at the
kappa opioid receptor and a partial antagonist at the mu opioid
receptor. Nalbuphine is an effective analgesic with a potency
comparable to morphine on a mg-to-mg basis, but with lower abuse
potential and less respiratory depressant effects. Nalbuphine can be
utilized to treat opioid-induced pruritus. In fact, nalbuphine is
superior in treating opioid-induced pruritus over diphenhydramine,
naloxone, or propofol in patients receiving neuraxial opioids for acute
pain related to surgery or labor.
Question 289
• Question 289: An adult patient is undergoing bone marrow
transplantation for leukemia. He has a history of frequent sores on
the lip. Which of the following would be most effective as prophylaxis
in this patient?
• Choices:
• 1. Clindamycin
• 2. Metronidazole
• 3. Prednisone
• 4. Acyclovir
Answer: 4
• Answer: 4 - Acyclovir
• Explanations: Herpes simplex in such a patient merits prophylaxis with acyclovir. In
most transplant centers, patients are screened for herpes simplex,
cytomegalovirus, and Epstein-Barr virus. The cause of the infection in most cases is
reactivation and not a de novo infection. Prednisone, clindamycin, and
metronidazole have no efficacy as antivirals. Certain risk factors place patients
undergoing bone marrow transplantation at high risk for infections including
whether the transplant is autologous or allogeneic, the patient's immune status,
the type of immunosuppressive regimen, and graft rejection. The most critical risk
factor is immune status. Most transplantation centers try to determine prior
evidence of infection with herpes virus so that the patient can be started on
prophylaxis. The risk of reactivation of herpes viruses is very high when the
immune system is suppressed.
Question 290
• Question 290: According to the National Lung Screening Trial, for
which population should lung cancer screening be done?
• Choices:
• 1. All adults between ages 55 and 80 years
• 2. Adults more than 40 years of age with a 20 pack-year smoking
history
• 3. Adults aged 55 to 80 years with a 30 pack-year smoking history
• 4. Adults less than 60 years of age who are active smokers and have
no other risk factors
Answer: 3
• Answer: 3 - Adults aged 55 to 80 years with a 30 pack-year smoking history
Explanations: All adults aged 55 to 80 years do not meet the criteria for a high-
risk population. As lung cancer screening involves exposure to possible harmful
radiation and there are potential other risks to screening, identifying a high-risk
population is essential. According to the National Comprehensive Cancer
Network (NCCN), there is utility in screening patients with a 20 pack-year history
of smoking. However, recommendations are for patients more than 55 years of
age with an additional risk factor such as family history, occupational exposure to
carcinogens, or personal history of chronic obstructive pulmonary disease
(COPD). According to the National Lung Screening Trial, patients aged 55 to 80
years with a 30 pack-year history of smoking who are active smokers or quit
within the last 15 years qualify for annual low-dose CT screening. Patients with a
life-limiting condition should not be screened.
Question 291
• Question 291: What is the rationale of using tumor markers in
patients with lung cell carcinomas?
• Choices:
• 1. They are good indicators of prognosis
• 2. They are useful screening tests
• 3. They are useful for diagnosis
• 4. They are used to monitor therapy
Answer: 4
• Answer: 4 - They are used to monitor therapy
• Explanations: No useful markers are available yet for prognosis,
screening, or diagnosis. Tumor markers used to monitor treatment
include neuron specific enolase, carcinoembryonic antigen,
cytokeratin 19 fragments, squamous cell carcinoma antigen, cancer
antigen CA 125, and tissue polypeptide antigen. However, not all labs
have the ability to measure these markers, and not all lung cancers
secrete them. Currently, imaging is the most sensitive method for
monitoring therapy.
Question 292
• Question 292: Cadmium has been linked to several different forms of
cancer. Which organ is the only one that is agreed upon by the
International Agency for Research on Cancer (IARC) as at risk for
carcinogenesis from cadmium exposure?
• Choices:
• 1. Prostate
• 2. Pancreas
• 3. Lung
• 4. Kidney
Answer: 3
• Answer: 3 - Lung
• Explanations: The research is torn as to whether chronic cadmium
inhalation causes pulmonary toxicity. It is, however, agreed upon that
exposure causes lung cancer. A dose-response relationship was
demonstrated between the estimated cumulative exposure to
cadmium and lung cancer risk. There is a positive association
observed for prostate and kidney cancers, but these studies have not
shown causation. Those that have a history of chronic cadmium
exposure should be screened for lung cancer.
Question 293
• Question 293: Treatment of acute lymphocytic leukemia can be with a
medication that is a structural relative of folic acid. Which of the
following is this medication?
• Choices:
• 1. Doxorubicin
• 2. 5-fluorouracil
• 3. Methotrexate
• 4. 6-mercaptopurine
Answer: 3
• Answer: 3 - Methotrexate
• Explanations: Methotrexate binds and inhibits dihydrofolate
reductase, preventing purine synthesis. De novo purine ring synthesis
is prevented by 6-mercaptopurine. Doxorubicin prevents the proper
functioning of DNA by intercalating between base pairs. dTMP
synthesis is prevented by 5-fluorouracil.
Question 294
• Question 294: Patients receiving initial treatment with doxorubicin
hydrochloride should be informed of which of the following?
• Choices:
• 1. Urine may be dark green for 1 to 2 days
• 2. Hair growth will resume 1 to 2 months after the drug is stopped
• 3. The drug may decrease the urine concentration of uric acid
• 4. Immunizations should not be received during drug therapy
Answer: 4
• Answer: 4 - Immunizations should not be received during drug therapy
Explanations: Patients receiving doxorubicin hydrochloride should not
receive immunizations since their immune systems will be compromised.
Myelosuppression, manifesting as leukopenia and neutropenia may be
dose-limiting and can result in serious infection, septic shock,
hospitalization, transfusion requirements, and death. Anemia and
thrombocytopenia may also occur. Doxorubicin therapy may cause
tumor lysis syndrome and hyperuricemia in patients with rapidly growing
tumors. It may be necessary to administer anti-hyperuricemic agents as
prophylaxis. Urinary alkalinization may be necessary. Doxorubicin is
contraindicated in patients with severe myocardial insufficiency or
severe arrhythmia.
Question 295
• Question 295: Which is true regarding IV promethazine administration
recommendations?
• Choices:
• 1. Administer in a hand vein
• 2. Do not dilute
• 3. Avoid extravasation
• 4. Administer quickly to avoid adverse effects
Answer: 3
• Answer: 3 - Avoid extravasation
• Explanations: Promethazine is a vesicant that can cause tissue
necrosis. Avoid extravasation. Promethazine should not be given in a
hand or wrist vein; use a large bore vein. Dilution and a slow
administration rate are recommended. Follow package insert
recommendations by the manufacturer for administration. If the
patient complains of burning or pain at the site, stop the
administration immediately and investigate further.
Question 296
• Question 296: Tonsillar carcinoma has most strongly been linked to:
• Choices:
• 1. Alcohol
• 2. Family history
• 3. Smoking
• 4. Human papilloma virus
Answer: 4
• Answer: 4 - Human papilloma virus
• Explanations: Cancer of the tonsils is more common in men. Cancer
of tonsils have been linked to alcohol and smoking. Recent evidence
indicates that HPV infection and tonsillar cancer has a strong
correlation. Tonsillar cancer often spread to lymph nodes in the neck
by the time diagnosis is made.
Question 297
• Question 297: Which of the following is a recognized use of
radionuclide ventriculography?
• Choices:
• 1. It can be used to diagnose arrhythmia
• 2. It is used prior to radiation therapy to obtain a baseline
• 3. It can detect cardiac metastases from hematogenous spread
• 4. It can assess the impact of potentially cardiotoxic
chemotherapeutic agents
Answer: 4
• Answer: 4 - It can assess the impact of potentially cardiotoxic
chemotherapeutic agents
• Explanations: If the LVEF of a patient on cardiotoxic chemotherapy is
calculated and it is less than 45 percent, some oncologists would
consider discontinuing the drug. Alternatively, if there is a prior LVEF
for comparison and the drop over time is more than 15 percent, this,
too, is an argument to discontinue therapy with that agent. Radiation
therapy typically has little untoward effect on the heart. Radionuclide
ventriculography has no role in detection of hematogenous mets or
arrhythmias.
Question 298
• Question 298: Which carcinoma most commonly causes generalized
pruritus?
• Choices:
• 1. Lung cancer
• 2. Pancreatic cancer
• 3. Renal cell carcinoma
• 4. Squamous cell skin cancer
Answer: 2
• Answer: 2 - Pancreatic cancer
• Explanations: Painful jaundice and pruritus are common symptoms of
pancreatic cancer. It is caused by conjugated bilirubin deposition in
the skin. Adrenocorticotropic hormone secreting lung tumors can lead
to skin darkening. Renal cell carcinoma usually presents with
hematuria, flank pain, and a mass.
Question 299
• Question 299: Which is true of renal cell adenocarcinoma?
• Choices:
• 1. Usually seen in third or fourth decade
• 2. Presents typically with gross hematuria
• 3. May mimic a renal stone
• 4. Frequently invades the renal artery
Answer: 2
• Answer: 2 - Presents typically with gross hematuria
• Explanations: The presence of renal cell adenocarcinomas are
generally silent. The classic triad of flank pain, hematuria, and flank
mass is rare. The most common presentation is gross hematuria,
weight loss, fever, hypertension, and flank pain. A CT scan is the
imaging study of choice and can help stage the cancer. A CT scan can
also differentiate from cystic and solid lesions. Bone scans can be
done in patients who have elevated alkaline phosphatase levels.
Question 300
• Question 300: A 65-year-old man with a past medical history of hypertension, diabetes mellitus,
and lung cancer comes to the clinic for a follow up of his PET-CT scan. He is feeling healthy and
does not have any symptoms of cough, weight loss, fever, and fatigue. His lung cancer has been
treated in stage 2 with chemotherapy. His last chemotherapy session was six months ago. His blood
pressure is 120/70 mmHg, the pulse is 60/min, respiratory rate 15/min with oxygen saturation of
92% on room air. His PET CT scan shows a new hyperintense lesion in the right lower lobe, which is
suspicious for new lung metastasis. There is no uptake of FDG by thoracic lymph nodes. The CT-
guided biopsy confirmed the presence of moderately differentiated adenocarcinoma. Based on
PET-CT scan findings, what is the next best course of action?
• Choices:
• 1. Trial of new chemotherapy
• 2. Referral to cardiothoracic surgery for wedge resection
• 3. Radiation
• 4. Endobronchial ultrasound
Answer: 2
• Answer: 2 - Referral to cardiothoracic surgery for wedge resection Explanations:
Referral to cardiothoracic surgery for wedge resection is the best course of
action. Presence of isolated metastasis with no involvement of the thoracic
lymph nodes, prolong disease-free interval from last treatment to appearance
of new metastasis are signs indicating that patients might benefit from surgical
intervention. Common surgeries performed include circumscribed local incision
via video-assisted thoracoscopy (VATS) or thoracotomy, segmentectomy, and
wedge resection. Pneumonectomy done for the central lesions is rarely done.
Post-op complications reported after surgery include: infection, stroke,
prolonged air leak, and pulmonary embolism. Radiation is less effective given
the higher dose required to eliminate cancer cells with a high likelihood of
injuring surrounding normal lung parenchyma.
Question 301
• Question 301: A routine mammogram in a 58-year-old female patient reveals
a vague asymmetric density in the upper outer quadrant of the right breast.
Upon clinical evaluation, there was no evidence of a palpable mass. A core
biopsy reveals the presence of lobular carcinoma in situ. Which of the
following should be the next step in the treatment process?
• Choices:
• 1. The patient needs a radical mastectomy
• 2. Further histological assessment for tumor biomarkers is required to
determine TNM staging and treatment regime
• 3. It is first treated with chemotherapy and radiation
• 4. The tumor is always negative for estrogen
Answer: 2
• Answer: 2 - Further histological assessment for tumor biomarkers is
required to determine TNM staging and treatment regime
Explanations: Radical mastectomy cannot be considered as the first
line of treatment if all biomarkers and histological staining have not
been received by the referring provider. A full workup is required to
determine the best treatment output. Cells of lobular carcinoma in
situ appear as single, detached cells and have a loss of expression of
E-cadherin. Lobular carcinoma in situ can be estrogen or progesterone
sensitive, and the hormonal relation is not as important as the
proliferation index marker.
Question 302
• Question 302: A 57-year-old male presents with worsening headaches that
awaken him at night. MRI shows a right frontal mass removed surgically.
Pathologic analysis showed a hemorrhagic, necrotic tumor with frequent
mitoses and highly pleomorphic cells. Despite radiation and chemotherapy,
the patient dies 8 months later. What is the most probable diagnosis?
• Choices:
• 1. Meningioma
• 2. Oligodendroglioma
• 3. Glioblastoma multiforme
• 4. Pilocytic astrocytoma
Answer: 3
• Answer: 3 - Glioblastoma multiforme
• Explanations: Glioblastoma multiforme is the most aggressive of the
astrocytomas. It is the most common primary intracranial neoplasm.
Pilocytic astrocytoma and oligodendrogliomas are malignant but grow
slowly. Meningiomas are benign.
Question 303
• Question 303: A 54-year-old female being treated for stage 3 ovarian cancer presents to her
oncologist for follow up after her second round of chemotherapy. The review of systems is negative,
except for decreased sensation in her hands and ringing in her ears over the last few days. On
physical exam, she is found to have decreased sensation to pinprick in her toes and fingertips
bilaterally. The rest of the physical exam is within normal limits. The physician notes that the patient
should be referred for audiometry for further testing. On review of her most recent labs, she is
found to have an elevated creatinine 25% over her baseline with electrolytes within normal limits.
Which medication is the patient most likely taking for treatment of her ovarian cancer?
• Choices:
• 1. Paclitaxel
• 2. Topotecan
• 3. Doxorubicin
• 4. Cisplatin
Answer: 4
• Answer: 4 - Cisplatin
• Explanations: Cisplatin can cause significant renal toxicity. It can also
cause ototoxicity manifested as tinnitus and may lead to occasional
deafness. Blood urea nitrogen, serum creatinine, creatinine
clearance, and serum electrolytes should be monitored in patients
receiving cisplatin therapy. Ototoxicity is cumulative in patients
receiving cisplatin therapy. Audiometric testing should be performed
at baseline and prior to each dose. Cisplatin therapy is
contraindicated in patients with preexisting renal impairment, hearing
impairment, and patients who have myelosuppression.
Question 304
• Question 304: What are the 3 histologic subtypes of nonsmall cell
lung cancer (NSCLC) in decreasing order of frequency?
• Choices:
• 1. Small cell carcinoma > adenocarcinoma > large cell carcinoma
• 2. Squamous cell carcinoma > small cell carcinoma > adenocarcinoma
• 3. Large cell carcinoma > squamous cell carcinoma > adenocarcinoma
• 4. Adenocarcinoma > squamous cell carcinoma > large cell carcinoma
Answer: 4
• Answer: 4 - Adenocarcinoma > squamous cell carcinoma > large cell
carcinoma Explanations: Adenocarcinoma accounts for approximately
50 percent of cases of NSCLC, followed by squamous cell carcinoma
for 35 percent, and large cell carcinoma for 15 percent. Nonsmall cell
lung carcinoma (NSCLC) represents about 80-90 percent of all lung
cancer while small cell lung carcinoma (SCLC) accounts for 10-20
percent. Adenocarcinoma tends to be peripherally located with a high
propensity to metastasize while squamous cell carcinoma tends to be
centrally located with a lower propensity to spread to the brain. EGFR
mutations occur in high frequency in adenocarcinoma, non smokers,
and Asians (30-70 percent).
Question 305
• Question 305: Which biochemical marker is used to evaluate
gestational trophoblastic disease?
• Choices:
• 1. Alpha-fetoprotein
• 2. Human chorionic gonadotropin
• 3. Alkaline phosphatase
• 4. Follicle stimulating hormone
Answer: 2
• Answer: 2 - Human chorionic gonadotropin
• Explanations: Human chorionic gonadotropin (hCG) is used to follow
gestational trophoblastic disease (GTD). After the evacuation of the
uterus in patients with GTD, beta-hCG is followed to zero to ensure
lack of malignant transformation. A starting beta-hCG greater than
100,000 is an independent risk factor for the development of
gestational trophoblastic neoplasia. Thyroid stimulating hormone and
hCG are similar in structure. Therefore, in patients with high starting
beta-hCG levels, the patient must be closely monitored for
thyrotoxicosis.
Question 306
• Question 306: A 72-year-old man presents to the clinic for a recent
onset of urinary hesitancy. On physical exam, he is found to have a hard
prostate. His PSA is also elevated on lab analysis. A prostate biopsy is
performed. Which of the following features is associated with the worst
prognosis in this patient?
• Choices:
• 1. Increased stroll space
• 2. Uniform glands
• 3. Small glands
• 4. Nuclear changes
Answer: 4
• Answer: 4 - Nuclear changes
• Explanations: This patient likely has prostate cancer given his physical
exam and elevated PSA. One of the most important prognostic
indicators is the Gleason score. The Gleason score is based on tumor
histology. The more well-differentiated the tumor is, the lower the
score; which is indicative of a better prognosis. Atypical nuclear
features are evidence of more advanced disease and are a poor
prognostic indicator.
:Question 307
• Question 307: A 55-year-old patient with metastatic colorectal cancer
with KRAS wild-type started treatment with cetuximab. Which
electrolyte may become abnormal in this patient?
• Choices:
• 1. Sodium
• 2. Magnesium
• 3. Chloride
• 4. Bicarbonate
Answer: 2
• Answer: 2 - Magnesium
• Explanations: Cetuximab is an epidermal growth factor receptor (EGFR) ) inhibitor
used for the treatment of colorectal cancer, metastatic, KRAS wild-type (without
mutation), and head and squamous cell neck cancer. Hypomagnesemia is
common and may be severe. EGFR blockade can interfere with magnesium
transport. EGFR is strongly expressed in the kidney. 70% of filtered magnesium is
reabsorbed by the ascending limb of the loop of Henle. When fatigue or
hypocalcemia is encountered during cetuximab therapy, serum magnesium level
be measured and replete as necessary. Symptoms may be rapidly ameliorated
with supplementation. The onset of electrolyte disturbance may occur within
days to months after initiation of treatment. Monitor magnesium, calcium, and
potassium during treatment and for at least eight weeks after completion.
Question 308
• Question 308: A 59-year-old male patient presents to the emergency department
with complaints of headaches of several months' duration. CT head revealed a
mass in the left temporal lobe. MRI revealed heterogeneous enhancement and
central necrosis. Diagnosis of glioblastoma multiforme (GBM) is made post-surgical
resection. The patient is scheduled for concomitant chemoradiotherapy. Which of
the following medications is recommended for him along with the therapy he is
most likely to receive?
• Choices:
• 1. Doxycycline
• 2. Clindamycin
• 3. Trimethoprim/sulfamethoxazole
• 4. Azithromycin
Answer: 3
• Answer: 3 - Trimethoprim/sulfamethoxazole
• Explanations: Temozolomide is given orally daily during radiation therapy.
Adjuvant treatment starts four weeks after radiotherapy, and it is given
for six cycles daily for five days in a 28-day cycle. Side effects include
leucopenia, thrombocytopenia, hepatotoxicity, nausea, constipation,
fatigue, etc. Pneumocystis jirovecii pneumonia prophylaxis is required
during treatment with temozolomide, and concomitant radiation therapy
as temozolomide typically causes CD4 T cell depletion. It is recommended
even if patients are not on steroids. The medications for PCP prophylaxis
include trimethoprim/sulfamethoxazole, atovaquone, and dapsone.
Empirical treatment for other infections is not recommended.
Question 309
• Question 309: Which of the following is true about the treatment of
Ewing sarcoma?
• Choices:
• 1. Surgery alone is adequate
• 2. Surgery followed by postoperative chemotherapy is adequate
• 3. Preoperative chemotherapy followed by surgery is adequate
• 4. Chemotherapy alone is adequate
Answer: 3
• Answer: 3 - Preoperative chemotherapy followed by surgery is
adequate
• Explanations: Ewing sarcoma is best treated with preoperative
chemotherapy followed by surgery. It is a painful, rapidly growing
tumor, seen in young adults. X-rays show an onion skin pattern. The
most important prognostic factor is absence or presence of metastatic
disease. In addition, if the tumor is in the distal extremities, this is a
favorable prognostic factor compared to the pelvis. Age younger than
15 also offers a favorable prognosis. Go to the next page if you knew
Question 310
• Question 310: A 65-year-old male with a history of hypertension, diabetes
mellitus, and colon cancer that was surgically removed ten years ago,
presents to the clinic for shortness of breath and a recent 15-pound weight
loss. A chest x-ray shows a 9 mm solid nodule in the left upper lobe with a
small pleural effusion on the same side. His only medications include lisinopril
and metformin. What is the next best step for this patient?
• Choices:
• 1. Left-sided thoracentesis
• 2. Left upper lobectomy
• 3. A full-body PET scan
• 4. Perform a CT scan at 6 months
Answer: 3
• Answer: 3 - A full-body PET scan
• Explanations: This patient's history of cancer, new weight loss, and
location of the pulmonary nodule in the upper lobe is highly
concerning for malignancy. PET scan would be able to determine if
there are any other lesions/nodules in the body, this is useful given
the patient's history of cancer and concern for metastasis. CT scan at
3 months or obtaining a biopsy of the nodule would be other
acceptable forms of management. Malignant appearing nodules need
to be dealt with quickly as
Question 311
• Question 311: Which one of the following chemotherapy agents
works by inhibiting tyrosine kinase activity?
• Choices:
• 1. Vincristine
• 2. Imatinib
• 3. Rituximab
• 4. Abciximab
Answer: 2
• Answer: 2 - Imatinib
• Explanations: Imatinib works by inhibiting the BCR-ABL gene that has
tyrosine kinase activity. This mutant gene is implicated as the cause
for chronic myelogenous leukemia. Imatinib is known to inhibit major
liver enzymes and may cause drug-drug interactions. Rituximab is a
monoclonal antibody for CD-20 B lymphocytes.
Question 312
• Question 312: A 1-year-old boy, the firstborn child of a non-consanguineous marriage
following a full-term normal delivery, is brought to the outpatient department with
swaying while walking and recurrent episodes of vomiting for the past one month. He
had started walking from the 10th month of age onwards, but now he has begun
developing difficulty even to walk on plain ground. On examination, there is no
neurological finding, except for ataxia. He undergoes a magnetic resonance imaging of
the brain, which shows an enhancing mass in the cerebellar vermis with fourth
ventricular extension and mild hydrocephalus. What is the most likely diagnosis?
• Choices:
• 1. Brain stem glioma
• 2. Dysgerminoma
• 3. Medulloblastoma
• 4. Ependymoma
Answer: 3
• Answer: 3 - Medulloblastoma
• Explanations: The patient's age, imaging results, and anatomical
location, as well as the extension to the fourth ventricle, all are more
suggestive of medulloblastoma. Ependymomas are the second most
likely posterior fossa lesions in this age group. Medulloblastomas tend
to show a more homogeneous enhancement while ependymomas
usually appear heterogeneous, due to internal cysts, calcifications,
and hemorrhage. Treatment for medulloblastomas is initially
maximum surgical removal of the lesion, the addition of radiation and
chemotherapy aims to increase disease-free survival.
Question 313
• Question 313: A young man is found to have a painless testicular mass that does
not transilluminate. After surgical removal, it is determined to be malignant and
radiation therapy is initiated. Select the most probable histologic findings.
• Choices:
• 1. Scattered immature neural elements with abnormal tissue from 3 germ layers
• 2. Tumor cells with clear cytoplasm that are large and distinct cell membranes in
a fibrous stroma with numerous lymphocytes
• 3. Mixed malignant syncytiotrophoblasts and cytotrophoblasts
• 4. Tumor cells that have abundant granular, eosinophilic cytoplasm and few
intracytoplasmic rhomboid crystals
Answer: 2
• Answer: 2 - Tumor cells with clear cytoplasm that are large and distinct cell
membranes in a fibrous stroma with numerous lymphocytes
• Explanations: Testicular tumors can be seminomas or nonseminomatous germ cell
tumors (NSGCT). Seminomas comprise about one-third of all germ cell tumors.
NSGCTs include yolk sac tumors, embryonal carcinomas, teratocarcinoma, immature
teratomas, mixed germ cell tumors and choriocarcinomas. Seminomas spread
lymphatically. Histologically, cells are large with clear cytoplasm. Seminomas are
extremely radiosensitive while NSGCTs are not and have a somewhat worse
prognosis. Despite this sensitivity, chemotherapy is frequently used for seminomas
as it tends to have fewer long term side effects. Syncytiotrophoblasts, which make
beta human chorionic gonadotropin (HCG), and cytotrophoblasts, which make alpha
fetoprotein (AFP), are found together in choriocarcinoma. The presence of any
abnormal AFP means that the tumor cannot be a pure seminoma.
Question 314
• Question 314: Which of the following medications can cause acute
dystonia?
• Choices:
• 1. Acetaminophen
• 2. Diphenhydramine
• 3. Prochlorperazine
• 4. Phenytoin
Answer: 3
• Answer: 3 - Prochlorperazine
• Explanations: Acute dystonia is often a complication of medications
such as phenothiazines and butyrophenones. This is due to the
dopamine antagonism. It can be seen with levothyroxine and some
antiepileptics including carbamazepine and phenytoin.
Prochlorperazine is a phenothiazine used commonly as an antiemetic.
It is also indicated to treat psychosis and can be used off-label to treat
migraine. It should be used with caution in children since it can cause
acute dystonic reactions, neuroleptic malignant syndrome, seizures,
and other severe adverse effects.
Question 315
• Question 315: What would not be seen in patients with N2 disease in
nonsmall cell lung cancer?
• Choices:
• 1. The 5-year survival after surgery is 25% to 30%
• 2. The size of the lymph nodes does not affect survival
• 3. Patients with a single node have higher survival rates than those
with multiple nodes
• 4. Survival is similar in patients without radiological evidence of N2
disease
Answer: 4
• Answer: 4 - Survival is similar in patients without radiological evidence
of N2 disease
• Explanations: The presence of N2 disease on a CT scan carries a
worse prognosis than discovering N2 disease at surgery. Certain
patients with N2 disease, such as a lymph node in the
aortopulmonary window, have a better prognosis. Historically,
patients with N2 disease are not offered surgery but treated with
radiation and chemotherapy. There have been trials with preinduction
chemotherapy followed by surgery in patients with N2 disease, but
distant relapse was common. Survival was only improved in patients
who responded to the chemotherapy.
Question 316
• Question 316: During which cell phase does bleomycin primarily act?
• Choices:
• 1. S phase
• 2. G1 phase
• 3. G2 phase
• 4. M phase
Answer: 3
• Answer: 3 - G2 phase
• Explanations: Bleomycin is a glycopeptides antibiotic that is used to
treat a variety of lymphomas. The drug works during the G2 phase of
the cell cycle and binds to iron in the oxidase. Bleomycin induces DNA
strand breaks and also may inhibit incorporation of thymidine into the
DNA strands. It is used in the treatment of Hodgkin lymphoma,
squamous cell carcinomas, and testicular cancer.
Question 317
• Question 317: The prognosis of melanoma is based on which of the
following parameters?
• Choices:
• 1. Size
• 2. Superficial spread
• 3. Depth
• 4. Change in color
Answer: 3
• Answer: 3 - Depth
• Explanations: Features that affect prognosis include tumor thickness,
in millimeters, and depth of penetration. Other features include type
of melanoma, ulceration, presence of lymphatic invasion, location,
and distant metastasis. When there are distant metastases, the
cancer is difficult to cure. Melanoma metastasis to the brain has the
worst prognosis.
Question 318
• Question 318: A patient is receiving total parenteral nutrition through
a central line. A small amount of milky fluid is noted to be leaking at
the site of the central line dressing. Which of the following should be
the initial action?
• Choices:
• 1. Stop the infusion and change the dressing
• 2. Secure the junction where lipids are connected
• 3. Place the patient in a Trendelenburg position
• 4. Change the dressing when the infusion is completed
Answer: 3
• Answer: 3 - Place the patient in a Trendelenburg position
Explanations: There is a break in the line that may cause air embolism.
The patient should be placed on their left side with the head lower
than the feet. The infusion should then be stopped and the dressing
changed. The break in the line is not at the junction but rather at the
IV site.
Question 319
• Question 319: What type of cells is most commonly affected by
chemotherapy?
• Choices:
• 1. T-cells
• 2. Hepatocytes
• 3. Mucosal cells
• 4. Melanocytes
Answer: 3
• Answer: 3 - Mucosal cells
• Explanations: Cancer chemotherapy works by inhibiting specific sites
in a cell cycle. Rapidly dividing cells are affected the most by anti-
cancer drugs. Hepatocytes can divide but are mostly in the resting
phase. Toxicity of mucosal cells leads to mucositis and diarrhea. Go to
the next page if you knew the correct answer, or click the link
image(s) below to further research the concepts in this question (if
desired).
Question 320
• Question 320: What is the current treatment strategy for a completely
resected, stage I non-small cell cancer?
• Choices:
• 1. Chemotherapy
• 2. Chemotherapy and radiation to the brain
• 3. Adjuvant immunotherapy
• 4. No therapy
Question 320
• Question 320: What is the current treatment strategy for a completely
resected, stage I non-small cell cancer?
• Choices:
• 1. Chemotherapy
• 2. Chemotherapy and radiation to the brain
• 3. Adjuvant immunotherapy
• 4. No therapy
Answer: 4
• Answer: 4 - No therapy
• Explanations: Stage I patients have a survival of 65% to 75% over 5
years. The major problem is recurrence; thus surveillance is required.
Adenocarcinomas usually present with distant recurrence and
squamous cell carcinomas usually present with local recurrence.
Patients who undergo lesser resections for stage I cancer have a
higher incidence of recurrence. These patients require follow up with
a chest x-ray, not with adjuvant therapy.
Question 321
• Question 321: A 50 year old female with chronic hepatitis B presents with
weight loss and jaundice. There is ascites and peripheral edema.
Transaminases are 4 times normal and alkaline phosphatase is 250 U/L.
Alpha fetoprotein is 7200 ng/mL and ultrasound shows a 4 x 5 cm mass in
the liver. CT guided biopsy was done. What is the most probable diagnosis?
• Choices:
• 1. Hepatocellular adenoma
• 2. Hepatocellular carcinoma
• 3. Metastatic colon cancer
• 4. Hydatid cyst
Answer: 2
• Answer: 2 - Hepatocellular carcinoma
• Explanations: Patients with chronic viral hepatitis are at increased risk
for hepatocellular carcinoma. The marked elevation of alpha
fetoprotein makes this a likely diagnosis. Hepatocellular adenoma is
most common in females on oral contraceptives. Needle biopsy of a
hydatid cyst can cause anaphylaxis and death.
Question 322
• Question 322: Which is a possible side effect of leuprolide (GnRH
analogue) therapy for prostate cancer?
• Choices:
• 1. Testicular tenderness
• 2. Urinary hesitancy
• 3. Increased libido
• 4. Gynecomastia
Answer: 4
• Answer: 4 - Gynecomastia
• Explanations: A possible side effect is gynecomastia which is breast
enlargement in males. Other side effects include gastrointestinal upset,
dizziness, decreased libido, emotional lability, hot flashes. GnRH analogs act
as inhibitors of gonadotropin release and are palliative treatments for
advanced prostate cancer. Leuprolide is an FDA-approved gonadotropin-
releasing hormone agonist that is used for the management of
endometriosis, uterine leiomyomata (also known as uterine fibroids),
treatment of central precocious puberty in children, and advanced prostate
cancer. Off-label uses include management of breast cancer, hormone
therapy for male to female transgender patients, premenopausal ovarian
suppression and management of paraphilia and hypersexuality.
Question 323
• Question 323: A 65-year-old male presents to a clinic with complaints of
difficulty starting and maintaining a steady stream, hematuria, and nocturia. He
denies any history of weight loss, fatigue, or dysuria. He has a strong family
history of prostate cancer. A rectal exam is done, which reveals a 2 cm hard
nodule. His prostate-specific antigen (PSA) is 7.5 ng/mL. Which of the following
investigation should be performed to confirm the diagnosis of prostate cancer?
• Choices:
• 1. Magnetic resonance imaging
• 2. Prostate Cancer Antigen 3 (PCA3)
• 3. Needle biopsy
• 4. Transrectal ultrasonography
Answer: 3
• Answer: 3 - Needle biopsy
• Explanations: The only test that can dependably and conclusively confirm a cancer diagnosis is still a
histologically positive prostate biopsy, which remains the recommended standard of care. Even after
the confirmation of the diagnosis, the first decision in managing prostate cancer is determining
whether any treatment at all is needed. Prostate cancer, especially low-grade tumors, often grows so
slowly that frequently no treatment is required. The only test which can determine the grade of the
disease is a histological biopsy. Even such patients, who refuse the invasive diagnostic procedures due
to prior bad experience or in whom certain medical conditions preclude doing biopsy, need to be fully
informed regarding the standard of care as well as the possibility of treatment complications and side
effects without the absolute assurance that they have a prostatic malignancy that is sufficiently
aggressive and dangerous to justify the therapy. Transrectal ultrasonography has a high false-positive
rate, which makes it unsuitable as a screening tool, although it may be used in directing prostatic
biopsy. Even in the experienced centers of excellence for MRI, the negative predictive value (NPV) has
been reported as low as 72% to 76%, meaning that a negative MRI report will miss about one in four
high-grade prostate cancers. PCA3 is best used to determine the need for a repeat biopsy after initial
unfavorable histology.
Question 324
• Question 324: Which is least important to include in follow-up
evaluations of a patient with primary non-small cell lung cancer after
potentially curative surgery?
• Choices:
• 1. CT scan of the chest
• 2. Liver ultrasound
• 3. Smoking cessation
• 4. Annual influenza vaccination
Answer: 2
• Answer: 2 - Liver ultrasound
• Explanations: Recurrence of non-small cell lung cancer usually occurs
in the lungs. Therefore, follow-up after potentially curative resection
should focus on the lungs. CT scans are recommended every 6 to 12
months for the first 24 months and then annually. Smoking cessation
and annual influenza vaccination are recommended in these patients.
Routine liver ultrasound is not recommended and only done in
patients with elevated liver enzymes.
Question 325
• Question 325: Which lung cancer is most commonly associated with
the SIADH?
• Choices:
• 1. Squamous cell
• 2. Carcinoid
• 3. Small cell
• 4. Adenocarcinoma
Answer: 3
• Answer: 3 - Small cell
• Explanations: SIADH is most common with small cell carcinoma,
occurring in 7% to 11% of patients with lung cancer. The
manifestations of hyponatremia include changes in mental status,
lethargy, or seizures. In some patients, there are no symptoms despite
very low sodium levels. In individuals who have a slow gradual decline
of sodium, there may be few or no symptoms. Squamous cell cancer
usually causes hypercalcemia. Small cell cancer also causes a
myasthenia gravis-like syndrome known as Eaton Lambert syndrome.
Question 326
• Question 326: A 52-year-old female undergoes a breast biopsy after a
mammogram showed a focus of microcalcifications in her left breast.
The biopsy pathology reveals lobular carcinoma in situ. Which of the
following statements is true?
• Choices:
• 1. The contralateral breast may be affected in 15% to 25% of cases
• 2. Urgent, modified mastectomy is indicated
• 3. If ignored, it will develop into inflammatory breast cancer
• 4. The tumor is usually negative for estrogen receptors
Answer: 1
• Answer: 1 - The contralateral breast may be affected in 15% to 25% of
cases
• Explanations: Lobular carcinoma in situ is a disorder caused by
abnormal cells in the lobules of the breast. Unlike ductal carcinoma in
situ, lobular carcinoma in situ is not associated with
microcalcifications. Lobular carcinoma in situ is usually an incidental
finding on core biopsy. In this case, it is also important to assess the
pathology report for the etiology of the microcalcifications. In 20% of
patients, lobular carcinoma in situ may be bilateral. Lobular carcinoma
in situ is something of a misnomer. It is a marker of increased lifetime
Question 327
• Question 327: A 42-year-old male presents to a clinic with complaint nocturia.
He says that for the last five days, he has to get up in the middle of the night
to urinate. He looks otherwise healthy. He denies any history of weight loss,
hematuria, or dysuria. The digital rectal exam of the prostate did not show
any abnormality of the prostate. What is the general recommendation for the
screening of prostate cancer?
• Choices:
• 1. Perform transrectal ultrasound
• 2. Do a sextant biopsy of the prostate
• 3. Offer prostate-specific antigen (PSA) test
• 4. Offer 4K genomic testing
Answer: 3
• Answer: 3 - Offer prostate-specific antigen (PSA) test
• Explanations: Screening options include the digital rectal exam and a prostate-specific antigen (PSA) blood
test. Such screenings may lead to a biopsy with some associated risks. Routine screening with a DRE and
particularly a PSA test has become very controversial, so there are some general recommendations for
screening tests of the prostate to avoid unnecessary testing that might do more harm than good. An initial
PSA test at 40 to 45 years of age is recommended because it is highly predictive of future prostate cancer
risk. Routine PSA screenings are recommended only in reasonably healthy men from 45 to 75 years of age
who wish it after a frank discussion of the benefits, limitations, and potential risks of screening. Routine PSA
screenings are not recommended in patients who would not accept treatment even if cancer was found and
in healthy men over age 75 with normal PSA levels up to that point, as they are not likely to benefit from
treatment. It is recommended in men with an abnormal digital rectal examination suggestive of cancer
regardless of age.
• Transrectal ultrasound has no role in prostate cancer screenings. A patient with a negative initial tissue
biopsy being considered for a repeat prostatic biopsy can best be further analyzed and risk-stratified by
tissue-based gene assay, the "4K" test, or PCA3. A prostate biopsy is the only confirmatory test that can
dependably and conclusively confirm a cancer diagnosis, but it is only performed in patients with a high
clinical suspicion of prostate cancer.
Question 328
• Question 328: A patient develops tremors, hyperglycemia,
hyperlipidemia, and cholelithiasis after being treated for Hodgkin
lymphoma. What agent is most likely responsible?
• Choices:
• 1. Mycophenolate
• 2. Azathioprine
• 3. Cyclosporine
• 4. Steroids
Answer: 3
• Answer: 3 - Cyclosporine
• Explanations: Cyclosporine has many side effects including gingival
hyperplasia, peptic ulcers, hyperlipidemia, dyspnea, and
hypertension. Rare side effects include hyperkalemia, pancreatitis,
and hyperglycemia. Some individuals may also develop gallstones and
hepatotoxicity. Cyclosporine can also cause a burning sensation at the
fingertips and make one prone to fungal infections.
Question 329
• Question 329: A 65-year-old with breast cancer has an elevated
calcium. What is the best initial treatment of hypercalcemia?
• Choices:
• 1. Insulin
• 2. Calcitonin
• 3. Vitamin D
• 4. Fluids
Answer: 4
• Answer: 4 - Fluids
• Explanations: Treatment for hypercalcemia is required if the patient is
symptomatic or if the calcium level is more than 14 mg/dL, even in
asymptomatic patients. The goals of treatment include increased
elimination of calcium from the extracellular fluid, reducing
gastrointestinal absorption, and decreasing bone resorption.
Immediate therapy is directed at restoring intravascular volume and
promoting calcium excretion in the urine. This is done with an
infusion of 0.9% saline at twice the maintenance rate until any fluid
deficit is replaced and diuresis occurs. The initial rate is 200-300
mL/hr.
Question 330
• Question 330: Which of the following chemotherapy agents works by
inhibiting tyrosine kinase activity?
• Choices:
• 1. Vinblastine
• 2. Imatinib
• 3. Rituximab
• 4. Anastrozole
Answer: 2
• Answer: 2 - Imatinib
• Explanations: Imatinib works by inhibiting the BCR-ABL gene that has
tyrosine kinase activity. This mutant gene is implicated as the cause
for chronic myelogenous leukemia. Imatinib is known to inhibit major
liver enzymes and may cause drug-drug interactions. Rituximab is a
monoclonal antibody for CD-20 B lymphocytes.
Question 331
• Question 331: A 63-year-old presents with weight loss, anorexia, and
general malaise. She has not been feeling well for the past 3 months.
Endoscopy reveals a fungating mass in the stomach. What is the most
likely diagnosis?
• Choices:
• 1. Adenocarcinoma
• 2. Squamous cell
• 3. Leiomyosarcoma
• 4. Lymphoma
Answer: 1
• Answer: 1 - Adenocarcinoma
• Explanations: Gastric adenocarcinoma accounts for 95% of malignant
tumors of the stomach. Lymphomas are less commonly followed by
leiomyosarcomas. Approximately 40% of gastric adenocarcinomas in
the antrum, 40% occur in the body, and only 15% occur in the fundus.
Early gastric cancer has no symptoms. By the time symptoms appear,
the disease is almost always incurable. Signs of metastatic spread may
include succussion splash, hepatomegaly, periumbilical metastases,
and enlarged cervical lymph nodes.
Question 332
• Question 332: Esophageal cancer tends to arise in the:
• Choices:
• 1. Mucosa
• 2. Submucosa
• 3. Adventitia
• 4. Serosa
Answer: 1
• Answer: 1 – Mucosa
• Explanations: Esophageal cancer tends to arise in the mucosa.
Subsequently it invades the submucosa and muscular layers. As the
tumor grows, it can invade the trachea, aorta and recurrent nerve.
The two most common risk factors include tobacco and alcohol.
Consumption of meat is also associated with a risk of esophageal
cancer. For adenocarcinoma, the two principal risk factors include
GERD and obesity.
Question 333
• Question 333: Which of the following statements regarding the
prognosis of cervical cancer is true?
• Choices:
• 1. The 5-year survival rate is approximately 70%
• 2. It is usually not detected until it is advanced and incurable
• 3. When detected early, the rate of cure is close to 100%
• 4. Whites have a worse prognosis
Answer: 3
• Answer: 3 - When detected early, the rate of cure is close to 100%
Explanations: If detected early and treated aggressively, cervical
cancer can be cured close to 100% of the time. African Americans,
Native Americans, and Hispanic Americans have a worse prognosis
because the cancer often is not detected early. An abnormal Pap
smear is the most common finding in a patient with cervical cancer. In
the United States, the incidence of cervical cancer has dropped, but it
is still a common cause of death among young women in developing
countries.
Question 334
• Question 334: A 6-year-old has been admitted to the neurocritical care unit for a new diagnosis
of posterior fossa brain tumor status post-resection. She is intubated, sedated, and paralyzed.
The neurosurgeons placed an extra-ventricular drain to help with the management of elevated
intracranial pressure (ICP). Her ICP monitor has been set at 20 and is draining 2-5 ml/hr. Her
heart rate is 85 beats per minute, blood pressure is 125/90 mmHg, respiratory rate is set at 25
breaths per minute on the ventilator, and FiO2 30% with an oxygen saturation of 95% as
measured by pulse oximetry. Histologic evaluation reports large tumor cells with abundant
cytoplasm, pleomorphic nuclei and prominent nucleoli. What is the most common cytogenetic
abnormality in this type of tumor?
• Choices:
• 1. Monosomy 6
• 2. Isochromosome 17q (i17q)
• 3. Loss of heterozygosity of 9q
• 4. A gain of chromosome X
Answer: 2
• Answer: 2 - Isochromosome 17q (i17q)
• Explanations: The most common cytogenetic abnormality in
medulloblastoma is i17q, wherein the short arm (p) is absent, and
there is a gain of genetic material from the long arm (q). In greater
than 50% of patients, deletions in the short arm have been reported,
resulting in a genotypic designation as 17pLOH, i.e., loss of
heterozygosity of 17p. Of note, the tumor suppressor gene, TP53, is
located on chromosome 17p. However, mutations in TP53 are a low-
frequency occurrence in medulloblastoma. Monosomy 6 is associated
with the WNT (wingless) medulloblastoma subgroup; LOH 9q is
associated with the SHH (sonic hedgehog) medulloblastoma subgroup.
Question 335
• Question 335: A 16-year-old man presents with asymptomatic swellings on the right side of the
neck for six months. He also complains of fever, night sweats, and significant weight loss over
this period. Vital signs are normal except for a temperature of 101 F (38.3 C). Physical
examination reveals rubbery, tender, submandibular, submental, cervical, and axillary lymph
nodes. Laboratory results show an erythrocyte sedimentation rate of 125 mm/hour. On
histopathological examination of lymph nodes, malignant cells resembling owl's eyes
appearance are seen in a background of scant lymphocyte infiltration. Immunohistochemical
evaluation of these cells reveals positive CD15 and CD30. What is the most appropriate next
step in management?
• Choices:
• 1. Bone marrow biopsy
• 2. Chemotherapy with ABVD (doxorubicin, bleomycin, vinblastine, and dacarbazine)
• 3. Positron emission tomography scan
• 4. Involved-site radiation therapy
Answer: 3
• Answer: 3 - Positron emission tomography scan
• Explanations: A positron emission tomography (PET) scan is an
important tool for initial staging and response assessment of patients
with Hodgkin lymphoma. A PET scan may have false-positive results in
sites of infection or inflammation. Thus, it is integrated with a
diagnostic computed tomography scan for the initial staging of
Hodgkin lymphoma. The Deauville criteria are used to interpret PET
scan for response assessment during and at the end of the treatment.
Positive PET results at the end of treatment are related to adverse risk
factors in patients with Hodgkin lymphoma.
Question 336
• Question 336: A 65-year-old male is diagnosed with hairy cell
leukemia after further workup from a routine CBC which showed
pancytopenia. Which of the following should prompt treatment?
• Choices:
• 1. A platelet count of 123,000/microliter
• 2. Profound fatigue
• 3. Hemoglobin of 12.1 grams/dL
• 4. A palpable spleen on exam
Answer: 2
• Answer: 2 - Profound fatigue
• Explanations: Moderate decreases in platelet count can be seen with hairy
cell leukemia, but treatment is not indicated by platelet count alone unless
it is under 100,000/microliter. Profound fatigue, symptomatic
splenomegaly, and significant cytopenias are all indications to start
treatment. Significant cytopenias include absolute neutrophil count less
than 1000/microliter, hemoglobin concentration less than 11 grams/dL, and
platelet count less than 100,000/microliter. Mild to moderate anemia can
be seen with hairy cell leukemia, however, treatment would not be
indicated by hemoglobin alone. Presence of a palpable spleen alone would
not necessitate treatment; however, symptomatic splenomegaly would.
Question 337
• Question 337: A patient with cervical cancer is receiving
brachytherapy with a radium implant. The implant has become
dislodged. What should the nurse do first?
• Choices:
• 1. Leave the patient alone and contact the radiation department
• 2. Use long-handled forceps to place the implant into a lead container
• 3. Wrap the implant in gauze and place it behind a lead apron
• 4. Get a dosimeter reading and call the physician
Answer: 2
• Answer: 2 - Use long-handled forceps to place the implant into a lead container
Explanations: Implants can be temporary or permanent. Promote periods of rest for the
client. These clients are placed in a private room. Visitors and non-essential personnel
are limited, and the duration of visitation is 10 to 30 minutes. All visitors must stay at
least 6 feet away from the client. All procedures are done as far away from the implant
as possible, and a lead shield is placed between the implant site and caregiver. Shoe
covers and gloves are put on before entering the room. Pregnant persons or anyone
under 18 years of age may not enter the room. Place signs on the chart and door per
policy to include radiation treatment start time, the total amount of activity implanted,
and the anticipated stop date. Radiation education is required before entry to the room.
Healthcare providers wear dosimeters if in the room If the implant becomes dislodged,
do not pick it up with hands. Lift with long-handled forceps and place in a lead container;
both are kept at the bedside. Call the radiation oncologist and radiation safety personnel
immediately. Document the time found, and no visitors are allowed to enter the room.
Question 338
• Question 338: Which of the following medications may be used to
improve appetite?
• Choices:
• 1. Testosterone
• 2. Megestrol acetate
• 3. Triamcinolone
• 4. Glucagon
Answer: 2
• Answer: 2 - Megestrol acetate
• Explanations: Megestrol acetate is used as an antineoplastic agent
and an appetite stimulant. It is a progesterone derivative that is
thought to have action at the pituitary causing an anti-luteinizing
effect. It is thought to antagonize the effects of catabolic cytokines. It
is indicated for AIDS related cachexia and used off label for cancer
related cachexia. It is used to treat breast and endometrial cancer.
Question 339
• Question 339: A multiple myeloma patient is receiving
cyclophosphamide. What measure can be taken to prevent
hemorrhagic cystitis?
• Choices:
• 1. Push one liter of fluids daily
• 2. Administer oxybutynin concurrently
• 3. Administer mesna
• 4. Push 4 liters of fluid daily
Answer: 3
• Answer: 3 - Administer mesna
• Explanations: Mesna alters the bladder-toxic breakdown products of
cyclophosphamide and ifosfamide. It is also desirable to infuse about
3 liters of intravenous fluids to assist in hemorrhagic cystitis
prophylaxis. Oxybutynin treats urinary incontinence and has no place
in this therapeutic scenario. One and 4 liters of intravenous fluids are
too little and too much, respectively.
Question 340
• Question 340: Which of the following is associated with retrovirus
infection?
• Choices:
• 1. Cervical neoplasia
• 2. Barrett metaplasia
• 3. Hepatitis C
• 4. Adult T cell leukemia
Answer: 4
• Answer: 4 - Adult T cell leukemia
• Explanations: Human T cell leukemia/lymphotropic virus type 1 may
cause several diseases, including adult T cell leukemia/lymphoma, a
rare T-cell cancer. It is usually a highly aggressive non-Hodgkin
lymphoma. There is irregular nuclear contour of the leukemic cells.
Visceral involvement, lytic bone lesions, skin lesions, and
hypercalcemia are frequent complications. Prognosis is poor.
Question 341
• Question 341: What is the most common histology bladder cancer
with Schistosoma haematobium infection?
• Choices:
• 1. Adenocarcinoma
• 2. Columnar cell
• 3. Squamous cell
• 4. Transitional cell
Answer: 3
• Answer: 3 - Squamous cell
• Explanations: Bladder cancer is most commonly transitional cell
carcinoma. Schistosoma haematobium infection is a risk factor for
bladder cancer. It usually results in squamous cell carcinoma of the
bladder.
Question 342
• Question 342: A 65-year-old woman presents to the hospital with complaints of fatigue
and pain in her right leg. Her vital signs are blood pressure of 139/84 mmHg, the pulse
of 84/minute, and a respiratory rate of 24 breaths/minute. The physical examination is
unremarkable. The patient’s laboratory values reveal calcium of 3.2 mmol/L, creatinine
of 180 umol/L, and hemoglobin of 11g/dL. CT scan shows osteolytic lesions in the right
femur. The patient receives induction therapy with a proteasome inhibitor and two
other chemotherapy agents. What is the most likely diagnosis in this patient?
• Choices:
• 1. Multiple myeloma
• 2. Breast cancer
• 3. Hairy cell leukemia
• 4. Ovarian
Answer: 1
• Answer: 1 - Multiple myeloma
• Explanations: Bortezomib is FDA-approved for use in the initial treatment of multiple myeloma in
combination with cyclophosphamide and dexamethasone. It is also FDA-approved for use in the treatment
of multiple myeloma in individuals who previously responded to bortezomib and relapsed at least 6 months
after the completion of the prior treatment. Also, it is FDA-approved for use in the treatment of mantle cell
lymphoma in individuals who have received at least one prior first-line treatment. Bortezomib is a
proteasome inhibitor. The proteasomal system plays a vital role in cellular protein turnover, which is
essential for the homeostasis of cells. Bortezomib reversibly binds to the chymotrypsin-like subunit of the
26S proteasome, resulting in its inhibition and preventing the degradation of various pro-apoptotic factors.
The accumulation will eventually activate the programmed cell death via caspase-mediated pathways in the
neoplastic cells that are usually dependent on suppression of pro-apoptotic pathways for their proliferation
and survival.
• Bortezomib has a narrow therapeutic window; an overdose can occur when doubling the dose. The patient
will usually present with marked thrombocytopenia and hypotension, which are very difficult to reverse.
There are reports of several fatal outcomes from bortezomib overdose. If the patient experiences an
overdose, close monitoring in the ICU is recommended. Aggressive hydration and maintenance of normal
body temperature are vital.
Question 343
• Question 343: Which is true about lung cancer?
• Choices:
• 1. Pleural malignancy is associated with asbestosis
• 2. Pancoast tumors involve the base of the lung
• 3. A small cell lung cancer is generally found in the periphery of the
lung
• 4. Adenocarcinoma is a type of small cell lung cancer
Answer: 1
• Answer: 1 - Pleural malignancy is associated with asbestosis
Explanations: About 80% of lung cancer is non-small cell and 20% is
small or oat cell. Adenocarcinoma is a non-small cell lung cancer is
not associated with smoking and is most commonly seen peripherally
at the gas exchanging lung surface. Asbestosis is associated with
mesothelioma. Most patients have fatigue and progressive dyspnea.
Pancoast tumors are located at the apex of the lung.
Question 344
• Question 344: A male with a lymphoma is treated with multiple
chemotherapeutic agents. One month later, he complains of
paresthesias in his legs that include numbness and tingling. Which
agent would most likely cause such symptoms?
• Choices:
• 1. Prednisone
• 2. Cyclophosphamide
• 3. Vincristine
• 4. Adriamycin
Answer: 3
• Answer: 3 - Vincristine
• Explanations: The patient has reversible peripheral neuropathy from
vincristine. The lymphoma is being treated with cyclophosphamide,
doxorubicin, vincristine, and prednisone (CHOP). Side effects of
vincristine include constipation, bedwetting, increase or decrease in
urination, stomach cramps, dizziness, painful urination, lack of sweating,
joint pain, lightheadedness, side back or low back pain, visual changes,
drooping eyelids, headache, jaw pain, difficulty controlling muscle
movement, numbness or tingling in toes or fingers, orchialgia, weakness,
leukopenia, thrombocytopenia, and mouth inflammation. The peripheral
neuropathy is often permanent and may present as a foot drop.
Question 345
• Question 345: A 28-year-old gravida 0 female was recently diagnosed with polycystic ovarian syndrome based
on chronic anovulation, signs of hirsutism (facial hair), and an ultrasound that was significant for multiple
follicles (>10mm) in each ovary. At her initial visit, her body mass index (BMI) was noted to be 38 kg/m2, and
her heart rate was 120 beats per minute. All of her other vital signs were within normal limits. Her past
medical history is significant for type II diabetes that is controlled with metformin 500 mg twice daily. She also
reveals that she is tobacco-dependent and has smoked a pack of cigarettes per day for 20-years. In addition to
metformin, she takes a combined estrogen/progesterone oral contraceptive pill daily. She denies any past
surgical history. Her family history is significant for hypertension, dyslipidemia, and obesity in her mother and
father and PCOS in her older sister. Upon questioning, she received all childhood vaccinations, including the
HPV series as a teenager. What aspect of her past medical history puts her at the greatest risk for developing
endometrial carcinoma?
• Choices:
• 1. Smoking status
• 2. Combine oral contraceptive use
• 3. Age
• 4. Obesity
Answer: 4
• Answer: 4 – Obesity
• Explanations: Higher parity, smoking, and the use of oral
contraceptives decrease the amount of estrogen and therefore, the
risk of endometrial cancer. The main factor that increases a woman's
risk of endometrial cancer is chronic, unopposed estrogen exposure.
Women who are obese have a high risk of uterine cancer because
fatty tissues convert androstenedione to estrogen. Unopposed
estrogen exposure is the leading risk factor for developing
endometrial cancer. Polycystic ovarian disease and nulliparity are also
a risk factor for endometrial cancer as they both lead to increased
unopposed estrogen exposure.
Question 346
• Question 346: A 16 year old was recently diagnosed with germ cell tumor of
the testis. He was started on a chemotherapy combinations containing
ifosfamide. The patient developed gross hematuria a week after completion
of chemotherapy and was subsequently diagnosed with hemorrhagic cystitis.
Which of the following steps would have prevented the above mentioned
complication?
• Choices:
• 1. Administration of mesna and aggressive hydration
• 2. Aggressive hydration
• 3. Slow administration of the drug
• 4. Rapid administration of the drug
Answer: 1
• Answer: 1 –
• Administration of mesna and aggressive hydration Explanations:
Ifosfamide therapy needs to be administered with mesna to prevent
hemorrhagic cystitis. Aggressive hydration includes either PO or IV
hydration with at least 2 liters or more per day. Ifosfamide's adverse
event profile also includes renal compromise, bone marrow
suppression, and vomiting, and hair loss. Ifosfamide is used to treat
cancers including testicular cancer, bladder cancer, ovarian and
cervical cancer, small-cell lung carcinoma, and osteosarcoma.
Question 347
• Question 347: Which prostaglandin derivative is used to treat opioid-
induced constipation?
• Choices:
• 1. Misoprostol
• 2. Lubiprostone
• 3. Prostacyclin
• 4. Sildenafil
Answer: 2
• Answer: 2 - Lubiprostone
• Explanations: Lubiprostone is a derivative of prostaglandin E1 and increases
secretion of fluid in the gastrointestinal tract. Lubiprostone stimulates the
cystic fibrosis transmembrane conductance regulator and type two chloride
channels located in the jejunum and colon. This results in the secretion of
water and sodium chloride into the colonic lumen. These actions result in
increased tone, enhanced peristalsis, and increased acceleration of the
small bowel and colonic transit times. Lubiprostone can increase the overall
frequency of bowel movements each week in patients with opioid-induced
constipation, compared to a placebo. Further, the drug can shorten the
mean time to a first bowel movement when compared to a placebo.
Question 348
• Question 348: Which of following forms of lung cancer is most likely
to present as a large, central mass without distant metastases?
• Choices:
• 1. Squamous cell carcinoma
• 2. Adenocarcinoma
• 3. Small cell carcinoma
• 4. Large cell carcinoma
Answer: 1
• Answer: 1 - Squamous cell carcinoma
• Explanations: Squamous cell lung cancer is most likely to present as a
large, central mass without distant metastases. About 30% of all lung
cancers are squamous cell carcinoma (SCC) and centrally located.
They may be associated with hypercalcemia. Centrally located tumors
may present with a dry cough, hemoptysis, wheezing, or post-
obstructive pneumonia. Bronchoscopy is invaluable in the diagnosis of
SCC. SCC tends to be locally invasive and positive lymph nodes are
common. Hence, a CT scan
Question 349
• Question 349: Which tumor is known to produce Schiller-Duval
bodies?
• Choices:
• 1. Seminoma
• 2. Leydig cell tumor
• 3. Medullary cancer
• 4. Endodermal sinus tumor
Answer: 4
• Answer: 4 - Endodermal sinus tumor
• Explanations: Schiller duval body is a small structure seen in
endodermal sinus tumors. (Yolk sac). These tumors are most
commonly testicular tumors in children. Schiller duval bodies are
pathognomic for yolk sac tumors. The bodies resemble a glomerulus.
Question 350
• Question 350: Select the most important risk factor in the
development of colorectal cancer.
• Choices:
• 1. Family history
• 2. Irritable bowel syndrome (IBS)
• 3. Hyperplastic polyps
• 4. Low fiber intake
Answer: 1
• Answer: 1 - Family history
• Explanations: Genetic factors seem to be the major risk factor for
developing colorectal cancer. Mutation of the adenomatous polyposis
coli gene has been shown to predispose to adenomatous polyps and
to colorectal cancer. Several other oncogenes have been implicated.
IBS, low fiber intake, and hyperplastic polyps are not predisposing
factors.
Question 351
• Question 351: A 32-year-old female presents to her primary care
provider complaining of a lump near her shoulder and neck. On
palpation, the provider notices a mass in the supraclavicular fossa; of
what could this be a sign?
• Choices:
• 1. Lymph-edema
• 2. Malignancy
• 3. Fracture of the clavicle
• 4. A pulmonary disease
Answer: 2
• Answer: 2 - Malignancy
• Explanations: The location of a cervical mass in the supraclavicular
fossa provides a strong indication of malignancy. A biopsy is
mandatory in the majority of patients presenting with a
supraclavicular mass. In cases of metastatic disease, the location of a
cervical mass helps identify the primary site. Histologic findings are
essential for further diagnostic steps, therapy, and prognosis.
Question 352
• Question 352: Reduced ability to taste food is a common complaint in
patients who take which drug?
• Choices:
• 1. Aspirin
• 2. Heparin
• 3. Bleomycin
• 4. Lithium
Answer: 3
• Answer: 3 - Bleomycin
• Explanations: Reduced ability to taste things is a rare disorder. The
complete inability to taste is known as ageusia. Causes of hypogeusia
include the drug bleomycin. Almost all cancer drugs can cause a
reduced ability to taste.
Question 353
• Question 353: A patient is admitted with pancreatic cancer and
jaundice. What should be the highest nursing priority?
• Choices:
• 1. Constipation
• 2. Nutrition
• 3. Skin integrity
• 4. Fear
Answer: 2
• Answer: 2 - Nutrition
• Explanations: Patients with pancreatic cancer often have marked
weight loss secondary to anorexia. The nurse should plan on offering
frequent high-calorie bland feedings. Pain medications and poor oral
intake can cause constipation. The patient may have fears of dying,
but the initial priority should be nutrition.
Question 354
• Question 354: A CT scan of the abdomen shows a solid
heterogeneously enhancing left renal mass without fat involvement,
with direct invasion to the left renal vein. What is the Robson stage?
• Choices:
• 1. Stage II
• 2. Stage IIIA
• 3. Stage IIIB
• 4. Stage IV
Answer: 2
• Answer: 2 - Stage IIIA
• Explanations: Stage I renal cell cancer by Robson staging criteria is
limited to the kidney. Stage II is extension to perinephric fat but
confined by Gerota fascia. Stage IIIa is extension to the renal vein.
Stage IIIb is nodal involvement. Stage IIIc is both nodal and venous
involvement. Note that venous involvement is a LOWER stage than
nodal involvement which implies a better prognosis. Stage IV a is
direct invasion of adjacent organs. Stage IVb is distant metastases.
Question 355
• Question 355: Bence Jones protein can be found in which of the
following?
• Choices:
• 1. Waldenstrom hypergammaglobulinemia
• 2. Multiple myeloma
• 3. Sarcoidosis
• 4. Acute lymphocytic leukemia
Answer: 2
• Answer: 2 - Multiple myeloma
• Explanations: Bence Jones protein are lambda light chains in the
urine. These light chains are found in multiple myeloma and are used
to monitor therapy. Today, beta 2 microglobulin is a strong predictor
of multiple myeloma outcomes. Plain x -rays remain the gold standard
for imaging newly diagnosed multiple myeloma.
Question 356
• Question 356: A 30-year-old patient is found to have myasthenia
gravis and an anterior mediastinal mass. What is the most likely
diagnosis?
• Choices:
• 1. Lymphoma
• 2. Teratoma
• 3. Thymoma
• 4. Metastatic lung cancer
Answer: 3
• Answer: 3 - Thymoma
• Explanations: Thymoma causes 50% of all masses of the anterior
mediastinum. Only 15% of patients with myasthenia gravis have a
thymoma. Myasthenia gravis may, but does not always, improve or
resolve with resection of the thymoma. Thymoma also is associated
with hypogammaglobulinemia and red cell aplasia. About 33% to 50%
of patients with a thymoma have no symptoms and the lesion is an
incidental finding on imaging.
Question 357
• Question 357: Which of the following best describes oxaliplatin?
• Choices:
• 1. Cell cycle specific cancer chemotherapy
• 2. Alkylating agent drug
• 3. Antimetabolite drug
• 4. G-2 cell cycle specific
Question 358
• Question 358: Which of the following chemotherapeutic agents is
known to cause significant nephrotoxicity?
• Choices:
• 1. Cisplatin
• 2. Dacarbazine
• 3. Methotrexate
• 4. Bleomycin
Answer: 1
• Answer: 1 - Cisplatin
• Explanations: Cisplatin is one of the alkylating agents that may cause
severe ototoxicity and nephrotoxicity. Blood urea nitrogen, serum
creatinine, creatinine clearance, and serum electrolytes should be
monitored closely in patients receiving cisplatin. Nephrotoxicity may
be potentiated by coadministration of cisplatin with aminoglycosides.
Cisplatin is contraindicated in patients with preexisting renal
impairment. Renal function must return to normal prior to
administering subsequent cycles.
Question 359
• Question 359: A 67-year-old man is receiving chemotherapy for
multiple myeloma. The patient develops peripheral neuropathy.
Which of the following is the patient most likely receiving for mulitple
myeloma treatment?
• Choices:
• 1. Thalidomide
• 2. Lenalidomide
• 3. Bortezomib
• 4. Dexamethasone
Answer: 3
• Answer: 3 – Bortezomib
• Explanations: Peripheral neuropathy is a dose-limiting toxicity for
proteasome inhibitors. Bortezomib inhibits 26S proteasome subunit
activity. The mechanism for bortezomib-induced toxicity is not clearly
identified. Treatment schedule, dosage, and preexisting peripheral
neuropathy are risk factors for bortezomib-induced peripheral
neuropathy.
Question 360
• Question 360: What is the most common malignancy of the female
genital tract?
• Choices:
• 1. Squamous cell carcinoma of the vagina
• 2. Squamous cell carcinoma of the cervix
• 3. Squamous cell carcinoma of the vulva
• 4. Endometrial adenocarcinoma
Answer: 4
• Answer: 4 - Endometrial adenocarcinoma
• Explanations: The most common malignancy of the female genital
tract is endometrial adenocarcinoma. In women under 50 years of
age, the most common malignancy of the female genital tract is
cervical squamous cell carcinoma. The most common malignancy of
the vulva is squamous cell carcinoma. The most common malignancy
of the vagina is squamous cell carcinoma.
Question 361
• Question 361: A 60-year-old African American smoker male was hospitalized for
abdominal pain and hematuria. Ab abdominal single phase CT contrasted
obtained showed a perfectly round, homogeneous, smoothly-marginated mass in
the kidney. Additionally, on the computerized tomography density measurement,
the mass is 90 Hounsfield unit. What is the next step in the management of this
patient?
• Choices:
• 1. This is a cyst induced by nifurtimox-eflornithine and no further workup required
• 2. Obtain a non-contrasted CT for further evaluation of tumor characteristic
• 3. This lesion is malignant and requires nephrectomy
• 4. This lesion has malignant potential and requires active surveillance.
Answer: 2
• Answer: 2 - Obtain a non-contrasted CT for further evaluation of tumor
characteristic
• Explanations: Studies have shown that all uniform homogeneous masses > 70
Hounsfield unit (HU) on nifurtimox-eflornithine combination therapy are
benign high-density cysts. However, patient does not have history using such
agent. If a lesion is > 70 HU but < 110 HU on contrast-enhanced computed
tomography (CECT), is not clear to differentiate between a high-density cyst
that didn't enhance or a renal cell carcinoma that did enhance. In this
context, obtaining delayed imaging would help. Non contrasted CT is needed.
Obtaining a non contrasted CT Studies have shown that all uniform
homogeneous masses > 110 HU on CECT are malignant renal cell carcinoma.
Delayed CT is needed prior to any interventions or treatments
Question 362
• Question 362: Which of the following statements is true about
bladder cancer?
• Choices:
• 1. It may be related to naphthylamine exposure
• 2. It is more common in non-smokers
• 3. It is more common in females
• 4. It is associated with C. sinensis
Answer: 1
• Answer: 1 - It may be related to naphthylamine exposure
Explanations: The majority of bladder cancers are due to an
environmental cause. Smoking is a major risk factor and is followed by
exposure to nitrosamines and naphthylamine. Bladder cancer is also
associated with industrial exposure to dyes, paint, leather, metallic
dust, and rubber. Coffee consumption does not increase bladder
cancer. C. sinesis is associated with liver and biliary cancer.
Question 363
• Question 363: A 75-year-old white man presents to dermatology with a 5-month
history of an ulcerated erythematous plaque on his forehead. Biopsy examination
reveals a dermal-based pleomorphic and atypical spindle cell tumor with numerous
atypical mitoses. By immunohistochemistry, the lesion expresses CD-10 and
procollagen and is negative for cytokeratin, CD34, SRY-Box 10 (SOX10), and
melanoma antigen recognized T cells–1 (MART-1). Which of the following is the
least likely characteristic of this disease?
• Choices:
• 1. Severe solar-induced damage to the skin is a risk factor
• 2. Erythematous dome-like growth is typical
• 3. These tumors often contain melanin pigment
• 4. Keratin pearls are typical findings
Answer: 3
• Answer: 3 - These tumors often contain melanin pigment
• Explanations: Atypical fibroxanthoma (AFX) is a spindle cell tumor that arises in sun-
exposed areas in the elderly. It clinically presents with an erythematous nodule and
histologically is strikingly atypical and pleomorphic. Before the clinical entity of
atypical fibroxanthoma was defined, many of the lesions were mistakenly termed
atypical spindle cell melanoma. AFX generally exhibits indolent behavior and excision
is curative. It must be differentiated from pleomorphic dermal sarcoma, which is
histologically similar but deeper and is more aggressive. Immunoperoxidase studies
are useful in clarifying the diagnosis. Atypical fibroxanthoma marks with antibodies
to CD10 and procollagen and is negative for cytokeratin and melanocyte markers.
Excluding melanoma and squamous cell carcinoma is crucial to making the diagnosis.
These tumors may contain hemosiderin pigment but not melanin, as could occur in
melanoma. Keratin pearls occur in squamous cell carcinoma but not AFX.
Question 364
• Question 364: What stage of testicular germ cell tumors is confined to
the testicle?
• Choices:
• 1. Stage I
• 2. Stage II
• 3. Stage III
• 4. Stage IV
Answer: 1
• Answer: 1 - Stage I
• Explanations: According to the Children’s Oncology Group staging
system for testicular germ cell tumors, stage I is confined to the
testicle. Stage II includes microscopic residual disease that is present
in the scrotum or high in the spermatic cord. Stage III includes
retroperitoneal lymph node involvement. Stage IV includes distant
metastatic deposits.
Question 365
• Question 365: A 73-year-old male presents to the emergency department
complaining of constipation and abdominal pain. On a computed tomography
scan, he is found to have a mass in the colon. A follow-up colonoscopy and
biopsy reveals adenocarcinoma. Initial workup and staging indicate the mass
to be T2N1M0. Which of the following is the most accurate characterization
of the patient's N classification?
• Choices:
• 1. 8-10 lymph nodes are positive for adenocarcinoma
• 2. The tumor is small and limited to the submucosa
• 3. There is evidence of tumor metastasis to the liver
• 4. 1-3 lymph nodes are positive for adenocarcinoma
Answer: 4
• Answer: 4 - 1-3 lymph nodes are positive for adenocarcinoma
Explanations: N refers to the degree of lymph node involvement, with
N1-N3 indicative of progressive nodal involvement. Answer 1 is more
indicative of N3 than N1 disease. N0 indicates no nodal involvement.
Tumor size and invasiveness is indicated by T, not N. Metastatic
disease (to the liver) is indicated by M, not N. The patient in this
scenario is M0, which indicates no metastatic disease. The TNM
scoring system is used for the staging of cancers. It has important
diagnostic and prognostic information.
Question 366
• Question 366: Which of the following chemotherapeutic agents has
been associated with a decreased rate of lung metastasis from
cutaneous angiosarcoma?
• Choices:
• 1. Etoposide
• 2. Vincristine
• 3. Azathioprine
• 4. Paclitaxel
Answer: 4
• Answer: 4 – Paclitaxel
• Explanations: Continuous chemotherapy with paclitaxel or docetaxel
for the treatment of cutaneous angiosarcoma has been shown to
improve the length of survival, particularly by decreasing the rate of
lung metastasis with multifocal tumors. Metastatic angiosarcoma lung
tumors have been successfully treated with docetaxel. According to
Yasuhiro Fujisawa et al. (2018), paclitaxel is recommended as the first-
line treatment. Docetaxel is recommended as a second-line therapy in
patients refractory to paclitaxel. Etoposide, vincristine, nor
azathioprine
Question 367
• Question 367: In a patient with polycythemia vera, which of the
following symptoms is unlikely?
• Choices:
• 1. Headache
• 2. Dizziness
• 3. Visual changes
• 4. Increased exercise endurance
Answer: 4
• Answer: 4 - Increased exercise endurance
• Explanations: Polycythemia vera is typically associated with
headaches, dizziness, and visual disturbances. The pathophysiology of
these symptoms involves hyperviscosity of the blood, owing to an
increased hematocrit. The long-term treatment of polycythemia vera
is preferentially done with regular therapeutic phlebotomy (250 - 500
ml) targeting a hematocrit of 45%. Cytoreductive oral chemotherapy
with hydroxyurea can be used as an adjunctive measure, especially
when leukocytosis and thrombocytosis are also present. Aspirin is
very useful in reducing the symptoms of hyperviscosity and the
associated risk of thrombosis.
Question 368
• Question 368: A 74-year-old male presents with a cutaneous horn on
the pinna of his left ear. Which malignant condition is most likely to be
seen upon histological examination?
• Choices:
• 1. Basal cell carcinoma
• 2. Squamous cell carcinoma
• 3. Malignant melanoma
• 4. Sarcoma
Answer: 2
• Answer: 2 - Squamous cell carcinoma E
• xplanations: Most cutaneous horns are benign. Squamous cell
carcinoma is the most likely malignant condition to be seen upon
histological examination. Advanced age and male sex are predisposing
factors for malignancy. It mostly appears in sun-exposed areas and
may be preceded by actinic keratosis. Squamous cell carcinoma can
be flat, nodular, and even plaque-like in some cases with significant
induration and subcutaneous spread noticeable on palpation.
Squamous cell carcinoma occasionally can be painful and tender, and
these may be signs of perineural invasion.
Question 369
• Question 369: Which of the following drug commonly causes cardiac
toxicity?
• Choices:
• 1. Daunorubicin
• 2. Cisplatin
• 3. Cyclophosphamide
• 4. Vinblastine
Answer: 1
• Answer: 1 - Daunorubicin
• Explanations: Myocardial toxicity is a potentially fatal complication of
daunorubicin therapy. This may occur when a total cumulative dosage
exceeds 400 to 550 mg per square meter in adults; 300 mg/square
meter in children older than 2 years old; and 10 mg/kg in children
under 2 years old. Obtain a baseline echocardiogram or MUGA scan
to measure cardiac ejection fraction. Instruct patients to expect
reddish-colored urine after receiving this medication.
Question 370
• Question 370: A patient with an occult squamous cell carcinoma of
the lung undergoes surgery. What is the most common long-term
problem with these patients?
• Choices:
• 1. They develop empyema
• 2. They develop metastases to the brain
• 3. They develop neck metastases
• 4. They develop new cancers
Answer: 4
• Answer: 4 - They develop new cancers
• Explanations: Patient with an occult squamous cell carcinoma of the
lung who undergo surgery generally present with new cancers and
need 6 to 12 months of surveillance. Surveillance includes
bronchoscopy and chest x-ray. These new cancers can occur anytime
up to 7 years postoperatively. The cancer may occur at the original
site or in a distant part of the lung.
Question 371
• Question 371: For which cancer is alpha fetoprotein a tumor marker?
• Choices:
• 1. Lung cancer
• 2. Liver cancer
• 3. Colon cancer
• 4. Breast cancer
Answer: 2
• Answer: 2 - Liver cancer
• Explanations: Alpha-fetoprotein is a tumor marker for hepatocellular
carcinoma. It can also be elevated in germ cell tumors.
Carcinoembryonic Antigen (CEA) and the KRAS gene are markers of
colon cancer. Some tumor markers of breast cancer include BRCA 1
and 2 genes, CA15-3/CA27.29, HER2, and estrogen receptor.
Question 372
• Question 372: What is the best route of drug administration during
ACLS and PALS?
• Choices:
• 1. Peripherally inserted large bore IV
• 2. Established subclavian central line
• 3. Emergent placement of a femoral central line.
• 4. Via radial arterial line
Answer: 2
• Answer: 2 - Established subclavian central line
• Explanations: If cardiac arrest occurs, the central line is the first choice during an emergency if already
established. Central lines are not recommended to be placed during resuscitation because of
interruptions in compressions, possible complications during insertion, and lack of sterile technique
during an emergency. Introduction of medications via central line does provide rapid delivery of
medications into central circulation versus the peripheral route. A large bore peripheral IV is an
appropriate second choice if a central line is present. However, fs no central line is available then the
peripheral introduction of ACLS medications is appropriate. If no central or peripheral IV access is
available then emergent placement of an intraosseous needle should occur, this route is preferred over
the endotracheal administration and can be used in both adult and pediatric populations. In the event
of a failed central, peripheral, or IO access then several of the basic ACLS drugs can be placed down the
endotracheal tube. The mnemonic LEAN can be used to remember which drugs can be safely placed
• The success of resuscitation is dependent on obtaining and maintaining reliable IV access immediately.
If reliable access cannot be initiated then, IO should be placed. Delivery of meds should not be
delayed, or outcomes may be poor.
Question 373
• Question 373: Bladder cancer typically presents with which symptom?
• Choices:
• 1. Suprapubic pain
• 2. Fever
• 3. Hematuria
• 4. Incontinence
Answer: 3
• Answer: 3 - Hematuria
• Explanations: The primary symptom of bladder cancer is hematuria.
The hematuria is often gross and painless but may be only
microscopic. Other symptoms of bladder cancer include dysuria and
urinary urgency and frequency. Bladder cancer is most closely
associated with smoking due to carcinogenic agents from cigarette
smoke being removed from the general circulation by the kidneys and
then being excreted via the urine, which is temporarily stored in the
urinary bladder. This exposes the bladder mucosal lining to
concentrated levels of highly carcinogenic agents, resulting in bladder
cancer.
Question 374
• Question 374: A 54-year-old male receives chemotherapy for a refractory lymphoma and
presents to the clinic for follow up. He has been struggling with nausea, vomiting, and
fatigue since initiating therapy but is in good spirits. He reports noticing a strange sound
while watching television that his wife did not notice although he doesn’t seem
concerned by this. His physical exam is notable for signs of mild dehydration and
generalized lymphadenopathy. His most recent labs revealed leukopenia but were
otherwise within normal limits. What is another common side effect of the most likely
offending agent?
• Choices:
• 1. Folate deficiency
• 2. Nephrotoxicity
• 3. Pulmonary fibrosis
• 4. Skin hyperpigmentation
Answer: 2
• Answer: 2 - Nephrotoxicity
• Explanations: This patient presents with side effects consistent with cisplatin
administraiton which may be utilized in refractory lymphoma cases. Tinnitus or an
occasional decreased ability to hear normal conversation are indications of
ototoxicity, which has been frequently observed. Abnormalities of audiometric
testing are more common, and hearing loss can be unilateral or bilateral.
Frequency and severity increase with repeated doses and may not be reversible,
but they mostly occur in the 4,000 - 8,000 Hz range. As ototoxicity of cisplatin
(cisplatinum) is cumulative, audiometric testing should be performed, if possible,
prior to initiation of therapy and at regular intervals thereafter, particularly if the
clinical symptoms of tinnitus or hearing impairment occur. Another common side
effect of cisplatin is nephrotoxicity. It may also cause nausea and vomiting,
myelosuppression, and neuropathy.
Question 375
• Question 375: A 45-year-old patient with recently diagnosed left-sided invasive ductal
carcinoma presents to the office to discuss treatment. History includes previous left-sided
lobular ductal carcinoma and is status post lumpectomy and radiation therapy three
years ago. Pathology from a core needle biopsy reveals the tumor to be invasive ductal
carcinoma, estrogen, and progesterone-receptor positive. It is 12 mm in size. On physical
exam, nipples are not inverted, and the mass is easily palpable in the upper outer
quadrant with no lymphadenopathy. Which of the following is the most appropriate next
step in the management of the patient?
• Choices:
• 1. Proceed with lumpectomy and sentinel node biopsy
• 2. Proceed with mastectomy and axillary lymph node dissection
• 3. Proceed with mastectomy and sentinel node biopsy
• 4. Proceed with modified radical mastectomy
Answer: 3
• Answer: 3 - Proceed with mastectomy and sentinel node biopsy Explanations: This
patient previously had breast cancer and received breast conservation surgery
with radiation treatment. Therefore, she would be contraindicated for additional
breast conservation surgery (i.e., lumpectomy) as she may no longer receive
additional radiation. A sentinel node biopsy is appropriate in this patient as this
stage does not necessitate an axillary node dissection. Other contraindications to
breast-conserving treatment include prior radiation therapy, pregnancy,
multicentric tumor (tumors in more than one quadrant of the breast), inability to
obtain clear margins (large tumor or invasion into adjacent structures), and
physical inability to tolerate radiation therapy. Relative contraindications include
multifocal tumors, connective tissue disease, and a large tumor to breast size ratio.
A modified radical mastectomy is appropriate in some cases, such as inflammatory
breast cancer. This patient does not require upfront modified radical mastectomy.
Question 376
• Question 376: A 68-year-old female with a past medical history significant for diabetes mellitus,
hypertension, and tobacco abuse, presents to the emergency department with abdominal pain,
weakness, and weight loss of approximately 15 pounds in the past two months. She states that
lately, she has had a decreased appetite, which is what she attributed her weight loss too. The
abdominal pain has acutely worsened and became more constant, prompting her to come to the
emergency department for further evaluation. She admits to having small, thin caliber bowel
movements every few days and occasionally notes that her stool is black. She has been taking iron
supplements in addition to her multivitamin because she thought her weakness was secondary to
her doctor telling her she is anemic at her last check-up over six months ago. She assumed the
iron was also causing her stool discoloration. She denies any bleeding per rectum, nausea, or
vomiting. She has never had an endoscopy or colonoscopy before. She denies any recent travel or
sick contacts. Her family history is significant for her mother's passing from ovarian cancer. She
drinks socially on the weekends and has been smoking a pack per day since she was 17. On exam,
her vitals are within normal limits. She appears pale and thin with a mildly distended abdomen. A
rectal exam shows grade 2 internal hemorrhoids. You decide to get a full set of labs, imaging, and
gastroenterology
gastroenterology consult as you suspect she may have colon cancer as
part of your differential diagnosis. At what age should she have gotten a
colonoscopy?
• Choices:
• 1. 21 years old
• 2. 25 years old
• 3. 40 years old
• 4. 50 years old
Answer: 4
• Answer: 4 - 50 years old
• Explanations: In the absence of risk factors, adults should undergo screening
for colon cancer starting at age 50. Recommended procedures include
colonoscopy, sigmoidoscopy, or high-sensitivity fecal occult blood testing.
Other screening options include a flexible sigmoidoscopy every five years
and a double-contrast barium enema every five years. If no polyps are found
in the first screening colonoscopy, then the next one should be done after
ten years. If the patient has a first degree relative with diagnosed colorectal
cancer or advanced adenoma before 60 years of age or two first degree
relatives diagnosed, then screening should begin at 40 years old or ten years
before the earliest diagnosis in the patient's family; whichever comes first.
Testing should be repeated every five years.
Question 377
• Question 377: In what age group is Ewing sarcoma most common?
• Choices:
• 1. Infants
• 2. Between 9 and 20 years
• 3. Middle age
• 4. Over 55 years

.
Answer: 2
• Answer: 2 - Between 9 and 20 years
• Explanations: Ewing sarcoma is a malignancy of under 20 years of age.
It is the second most common primarily malignant bone tumor in
those 10 to 20 years. 27% of cases occur before age 10 years, 64%
occur from 10 to 20 years, and 9% occur from 20 to 30 years. It is
extremely rare to see Ewing sarcoma in the elderly.
Question 378
• Question 378: An 82-year-old male patient presents with evidence of metastatic
lesions and extra-hepatic disease to the lung on CT imaging prior to evaluation
from you as his medical oncologist. He understands that he is very ill, however,
he is inquisitive of what would be his poor prognostic factors that would harm
his ability for a potential cure. Which of the following is considered the poorest
prognostic indicator for patients with metastatic disease?
• Choices:
• 1. Extrahepatic disease to the lung
• 2. CEA of 5 ng/mL
• 3. 2 metastatic lesions to the liver from a known colorectal primary tumor
• 4. Recurrence of disease following prior intervention for colon cancer
Answer: 4
• Answer: 4 - Recurrence of disease following prior intervention for
colon cancer
• Explanations: Extrahepatic disease to any organ other than lung is
considered a very poor prognostic indicator for long term outcome
CEA greater than 10 ng/mL is considered a very poor sign. Greater
than 6 colorectal liver metastatic lesions are a poor sign. Recurrence
of disease after prior treatment is the poorest prognostic indicator for
a cure.
Question 379
• Question 379: A 65-year-old female patient presents to the hospital with complaints of severe
headaches associated with nausea and vomiting on awakening for the past few months. She also
reports loss of weight and loss of appetite. She recalls having radiation therapy ten years ago for
some growth, but can not precisely remember the diagnosis. On examination, she is found to have
a heart rate of 100 beats/min, a blood pressure of 100/70 mmHg, respiratory rate of 20/min, and
95 % oxygen saturation on room air. Lab values, including CBC and metabolic profile, are within the
normal range. CT head shows an irregular mass-like lesion, which was not present in her previous
imaging four years ago. MRI revealed a 3 cm irregular mass in the left temporal lobe. Information
regarding which of the following is most likely to help in establishing a diagnosis in such patients?
• Choices:
• 1. Previous radiation
• 2. Prior surgeries
• 3. Family history
• 4. Smoking and alcohol history
Answer: 1
• Answer: 1 - Previous radiation
• Explanations: Glioblastoma multiforme is the most common
aggressive primary cranial nervous system tumor. Usually more
common in older men and African American race. Studies have shown
that a previous history of irradiation in early life is a risk factor for
GBM. Although, in most cases, risk factors are not identified. It is
important to take a careful history, including a prior history of
irradiation, to assess for risk factors. No convincing evidence exists
regarding associations of GBM with family history, smoking, or alcohol
history.
Question 380
• Question 380: A 64-year old female is planning to undergo systemic
chemotherapy treatment with a pyrimidine analog drug for the treatment of
gastric adenocarcinoma. Her oncologist warns her of some of the most
common side effects of this chemotherapy drug. Which of the following is
the most common side effect from treatment with this chemotherapy agent?
• Choices:
• 1. Anemia
• 2. Neutropenia
• 3. Diarrhea
• 4. Hypotension
Answer: 3
• Answer: 3 - Diarrhea
• Explanations: 5-fluorouracil (5-FU) is a pyrimidine analog used for the
treatment of various visceral malignancies. The most common side
effects of systemic 5-FU treatment include diarrhea, vomiting, nausea,
and dehydration. Severe adverse effects from 5-FU treatment include
neutropenia, thrombocytopenia, pulmonary embolism, anemia, as
well as secondary sepsis and pneumonia. The most frequently
reported side effect of systemic 5-FU is diarrhea.
Question 381
• Question 381: What is the drug of choice for chemotherapy induced
nausea and vomiting?
• Choices:
• 1. Marijuana
• 2. Ondansetron
• 3. Metoclopramide
• 4. Corticosteroids
Answer: 2
• Answer: 2 - Ondansetron
• Explanations: Ondansetron is a serotonin 5-HT3 receptor antagonist.
It is used to treat nausea and vomiting. Ondansetron decreases vagal
nerve activity and deactivates the vomiting center in the medulla.
Ondansetron does not have much effect on motion sickness as it has
no effect on dopamine receptors. Marijuana does work in some
patients but only for mild cases. It is rarely effective alone and it not
well tolerated because of dysphoria and sedation.

•.
Question 382
• Question 382: Which of the following describes radiation that is
placed internally for treatment?
• Choices:
• 1. Brachytherapy
• 2. Teletherapy
• 3. Stereotactic body radiotherapy
• 4. Stereotactic radiosurgery
Answer: 1
• Answer: 1 - Brachytherapy
• Explanations: Brachytherapy is used to describe internal radiation
treatment. Examples include intracavitary and interstitial therapy.
Teletherapy is another word for external beam radiation therapy,
which uses a radiation source outside the body and is the most
common type of radiation. Stereotactic body radiotherapy is a type
of external beam radiation, typically using three to five treatments for
high, ablative doses. Stereotactic radiosurgery is a type of external
beam radiation in which a high-dose, single fraction of radiation is
used for tissue ablation.
Question 383
• Question 383: Which of the following is the best treatment for
chemotherapy associated neutropenia?
• Choices:
• 1. Blood transfusion
• 2. Administering filgrastim
• 3. Strict hand washing
• 4. Bone marrow transplant
Answer: 2
• Answer: 2 - Administering filgrastim
• Explanations: Chemotherapy-induced neutropenia can be treated
with filgrastim. Blood transfusion is of no benefit. Hand washing
prevents complications of neutropenia (infections) but does not treat
the underlying neutropenia. Patients should be in reverse isolation
with strict hand washing and avoidance of fresh fruits and vegetables.
Bone marrow transplant is not an option in this case.
Question 384
• Question 384: A 65-year-old female with a past medical history of hypertension, diabetes
mellitus, and smoking presents to the hospital with the complaint of weakness in the medial
two fingers, numbness, and tingling on lateral two fingers and dorsum of the left hand. The
patient reports she had chronic smokers cough for many years but now reports feeling a change
in intensity of the cough as it is becoming more severe. She has a low-grade fever, and the
blood pressure is 130/70 mmHg, pulse 78/min, respiratory rate 20/min with oxygen saturation
of 90 % on room air. On physical exam, the patient is unable to flex the lateral two fingers, and
motor strength is 2/5 in the left hand compared with the right. What is the next best diagnostic
test to be done in this patient?
• Choices:
• 1. Chest x-ray
• 2. CT chest without contrast
• 3. MRI chest
• 4. Nerve conduction studies
Answer: 1
• Answer: 1 - Chest x-ray
• Explanations: The chest x-ray is the initial imaging, widely available to
rule out pulmonary lesions. Of note, small nodules may not be
apparent on a chest x-ray. Pancoast tumors are the lung tumors
occupying the apex of the lung and can be detected on a chest x-ray.
Pancoast tumors can present in the form of superior vena cava
syndrome or thoracic outlet syndrome by compression on the brachial
plexus resulting in motor and sensory changes in the upper extremity.
CT scan chest with contrast is ordered to evaluate the complete
extent of the tumor, rule out smaller metastasis not evident on
routine chest x-ray.
Question 385
• Question 385: A 31-year-old man presents to his provider over concerns of the
enlargement of both his breast. He is otherwise healthy and takes no medications. On
further evaluation, he is found to have elevated total testosterone and decreased
luteinizing hormone. A scrotal ultrasound demonstrates a 3 cm discrete mass in his
right testicle. He undergoes an inguinal orchiectomy. On surveillance imaging six
months following his surgery, a 4 cm retroperitoneal mass is identified. What is the
best treatment option for the patient?
• Choices:
• 1. Chemotherapy
• 2. Radiation therapy
• 3. Retroperitoneal lymph node dissection followed by adjuvant chemotherapy
• 4. Retroperitoneal lymph node dissection
Answer: 4
• Answer: 4 - Retroperitoneal lymph node dissection
• Explanations: Leydig cell tumors have a bimodal distribution, with an initial peak
in the prepubertal age group, 4 to 10, and then a second peak between the ages
of 30 and 60. They are derived from the same Leydig cells, which normally reside
in the interstitium of testicles and secrete testosterone in the presence of the
luteinizing hormone. Due to Leydig cells' hormonally active properties, they can
present with precocious puberty, breast tenderness, or gynecomastia. They are
generally benign tumors, with only 5-10% being considered malignant. Malignant
behavior of Leydig cell tumors is exhibited by metastasis to the retroperitoneal
lymph nodes, liver, lungs, and bone. If metastatic disease does occur, it responds
poorly to chemotherapy or radiation, leaving surgical resection as the only
treatment that can render patients disease-free.
Question 386
• Question 386: A chronic, white, indurated oral mucosal lesion is
noted. Which of the following is a premalignant disorder that must be
excluded?
• Choices:
• 1. Leukoplakia
• 2. Candidiasis
• 3. Lichen planus
• 4. Pachyderma oris
Answer: 1
• Answer: 1 - Leukoplakia
• Explanations: Leukoplakia is a whitish, indurated lesion of the oral mucosa that is
premalignant. A biopsy is necessary to establish the diagnosis. Oral premalignancy is
considered as an intermediate stage. It is classified as into two broad straplines:
premalignant lesions and premalignant conditions. The "premalignant lesion" is defined
as “a morphologically reformed tissue in which oral cancer is more likely to occur than
in its seemingly normal counterpart.” An example is a leukoplakia. A "premalignant
condition" is defined as “a generalized state associated with a significantly increased
risk of cancer.” An example is oral submucous fibrosis. Recently the World Health
Organization (WHO) considered premalignant lesions and conditions under a single
group of disorders known as potentially malignant disorders. Oral leukoplakia is a
potentially malignant disorder affecting the oral mucosa. It is defined as “essentially an
oral mucosal white lesion that cannot be considered as any other definable lesion.”
• Oral leukoplakia is a white patch or plaque that develops in the oral
cavity and is strongly associated with smoking. Risk factors comprise
all forms of tobacco use, including cigar, cigarette, beedi, and pipe.
Other synergistic risk factors include alcohol consumption, chronic
irritation, fungal infections such as candidiasis, oral galvanism due to
restorations, bacterial infections, sexually transmitted lesions like
syphilis, combined micronutrient deficiency, viral infections, hormonal
disturbances, and ultraviolet exposure.
Question 387
• Question 387: Osteoradionecrosis is thought to result from which of
the following alterations identified following radiation injury?
• Choices:
• 1. Hypotension, anemia, and hypercarbia
• 2. Hypovascularity, hypocellularity, and hypoxia
• 3. Tissue contraction
• 4. Cellular atypia
Answer: 2
• Answer: 2 - Hypovascularity, hypocellularity, and hypoxia
Explanations: Hypovascularity, hypocellularity, and hypoxia are
descriptors of the known condition of the tissues within the field
following radiation therapy. These factors result in an abnormal
response to injury. When injury exceeds the ability of the radiated
tissue to undergo normal reparative processes, osteoradionecrosis
develops. Hyperbaric oxygen therapy has been shown to promote
angiogenesis and can help restore the vascularity of tissues damaged
by radiation.
Question 388
• Question 388: A patient is treated with systemic chemotherapy for
breast cancer. Treatment involves a drug that binds and stabilizes the
tubule to confer cytotoxicity. Which of the following adverse events is
most likely to be life-threatening in this patient?
• Choices:
• 1. Ototoxicity
• 2. Dilated cardiomyopathy
• 3. Pancreatitis
• 4. Febrile neutropenia
Answer: 4
• Answer: 4 - Febrile neutropenia Explanations: This patient with breast
cancer is most likely being treated with docetaxel, adjunct chemotherapy
that is proven to improve survival in high-risk cases. Docetaxel is a second
generation taxane, which functions by binding and stabilizing beta-tubulin,
enhancing its proliferation, and preventing the normal function of the
mitotic spindle. This causes cells to be unable to complete the M phase of
their life cycle, leading to cell death. The major adverse effects of docetaxel
include diarrhea, skin reactions, and myelosuppression. A more serious
manifestation of myelosuppresion is febrile neutropenia. In order to
prevent this potentially life-threatening adverse event, patients should be
monitored for neutropenia, fever, and signs of infection throughout
treatment with docetaxel.
Question 389
• Question 389: How often should pap smears be performed?
• Choices:
• 1. Yearly in sexually-active women
• 2. Yearly until age 50, and then they can be stopped
• 3. Every 5 years for sexually-active women
• 4. Every 3 years until age 65 years
Answer: 4
• Answer: 4 - Every 3 years until age 65 years Explanations: The USPSTF
recommends screening for cervical cancer in women ages 21 to 65 years
with cytology (Pap smear) every 3 years. For women 30 to 65 years who
want to lengthen the screening interval, recommended screening includes
high-risk human papillomavirus (hrHPV) testing alone or a combination of
cytology and hrHPV testing every 5 years. The USPSTF recommends against
screening for cervical cancer in women younger than 21 years of age. The
USPSTF recommends against screening for cervical cancer in women older
than age 65 years who have had adequate prior screening and are not
otherwise at high risk for cervical cancer. The USPSTF recommends against
screening for cervical cancer in women who have had a hysterectomy with
removal of the cervix and do not have a history of a high-grade precancerous
Question 390
• Question 390: A 62-year-old male is receiving treatment for advanced
bladder cancer. After infusion, he experienced symptoms that lead to
elevated blood urea nitrogen (BUN) and electrolyte disturbance on a
follow up the metabolic panel. Pretreatment with which of the
following medications could have prevented these derangements?
• Choices:
• 1. Steroids
• 2. Ondansetron
• 3. Mesna
• 4. Leucovorin
Answer: 2
• Answer: 2 - Ondansetron
• Explanations: Cisplatin is a level four-emetogenic drug that can cause
severe nausea and vomiting. Ondansetron is a seratonin antagonist
that can effectively control nausea and vomiting. Excessive vomiting
can cause dehydrating leading to elevated BUN and hypochloremic
metabolic alkalosis with hypokalemia due to volume contraction, the
expulsion of hydrochloric acid, and potassium losses. The routine
basic metabolic panel can help identify these disturbances.
Leucovorin rescue is given for patients receiving methotrexate. Mesna
is given to combat the risk of hemorrhagic cystitis caused by
cyclophosphamide.
Question 391
• Question 391: A 43-year-old female patient presented to the emergency department with a 1-day history
of left-sided flank pain. A non-contrast abdominal computed tomography (CT) scan revealed a partially
obstructing right ureteral calculus. Incidentally found on non-contrast CT was a 6.5-cm right-sided adrenal
mass that measured 15 Hounsfield units (HU), heterogeneous with irregular margins and calcifications.
She denies any fever, chills, night sweats, anorexia, palpitations, panic attacks, headaches, or abdominal
pain. She reported an intentional 10-lb — weight loss over the preceding year. On the physical exam, she
is afebrile; hear rate 61 bpm; blood pressure 118/76 mmHg; respiratory rate, 15 breaths per min; oxygen
saturation, 96% on room air; and body mass index 35 kg/m2. She has a rounded face, dorsocervical fat
pad, abdominal striae, but no evidence of hirsutism, ecchymoses, or thinning of the skin. Which one of
the following is the most probable etiology of this patient’s incidentally discovered adrenal mass?
• Choices:
• 1. Pheochromocytoma
• 2. Adrenocortical adenoma
• 3. Adrenocortical carcinoma
• 4. Myelolipoma
Answer: 3
• Answer: 3 - Adrenocortical carcinoma
• Explanations: This patient presents an adrenal incidentaloma, which is an inapparent
adrenal masses discovered inadvertently in the course of diagnostic testing or
treatment for conditions not related to the adrenals. Adrenal masses are among the
most common tumors. Adrenal Incidentaloma is not a single pathological entity, but
instead comprise a spectrum of different entities that share the same path of
discovery and include both benign and malignant lesions arising from the adrenal
cortex, the medulla or being of extra-adrenal origin. Presence of primary adrenal
malignancy is more related to mass size, as Adrenocortical carcinoma represents 2%
of all tumors =4 cm in diameter, 6% of tumors with size 4.1-6 cm and 25% of the
tumors >6 cm. On the other hand, benign adrenocortical adenomas comprise 65% of
masses =4 cm, and 18% of masses >6 cm. Furthermore, Adrenocortical carcinoma
tends to vary in appearance with frequent heterogeneous enhancement because
Question 392
• Question 392: Cancer cachexia is the state of extreme loss of muscle
and adipose tissue. This is frequently seen in patients with advanced
cancer of the respiratory tracts and gastrointestinal tract. Which of the
following is true about this condition?
• Choices:
• 1. This status is similar to simple starvation or hunger
• 2. This status can be reversed by nutritional supplements
• 3. This status can be reversed by successful chemotherapy or
radiotherapy
• 4. This status is secondary to altered metabolism
Answer: 4
• Answer: 4 - This status is secondary to altered metabolism
• Explanations: In simple starvation or hunger, fat replaces glucose as the preferred
fuel to spare lean body mass and is not associated with systemic increases in
proinflammatory factors. Cancer cachexia cannot be reversed by nutritional
supplements. Patients with cachexia are less tolerant to systemic
chemotherapy/radiotherapy. Several metabolic alterations have been recognized
and have been regarded as the cause of cancer cachexia, which includes ATP
production mainly from glycolysis rather than from tricarboxylic acid (TCA) cycle,
futile metabolic cycles, and increased free fatty acids production in plasma. Cancer
cells are unable to produce ATP via TCA cycle because of activation of HIF
(hypoxia-inducible factor) by tumor suppressors and oncogenes, as well as
mutations in metabolic enzymes, especially isocitrate dehydrogenase 1 (IDH1) and
IDH2 and other TCA cycle enzymes.
Question 393
• Question 393: A 65-year-old woman has been diagnosed with a 4 cm
breast cancer. Which of the following is associated with a better
prognosis?
• Choices:
• 1. Expression of HER2/Neu
• 2. Invasion of dermal lymphatics
• 3. Positive estrogen and progesterone receptors
• 4. Presence of only 4 axillary nodes
Answer: 3
• Answer: 3 - Positive estrogen and progesterone receptors
Explanations: Estrogen receptors are expressed in 75% of breast
cancers. Women who have estrogen receptors generally have a better
prognosis than those who do not. Once hormone receptors are
identified, women can be treated with drugs like tamoxifen and
anastrozole. Tamoxifen is only used in postmenopausal women with
cancer.
Question 394
• Question 394: Which are the major type(s) of pulmonary papillomas?
• Choices:
• 1. Squamous cell papilloma, glandular papilloma, and mixed
papilloma
• 2. Squamous cell papilloma and glandular papilloma
• 3. Mixed papilloma only
• 4. Glandular papilloma only
Answer: 1
• Answer: 1 - Squamous cell papilloma, glandular papilloma, and mixed
papilloma
• Explanations: The three main types of pulmonary papillomas are
squamous cell, glandular, and mixed. Squamous cell papillomas are
the most frequent, benign polypoid lesions of the lung. Glandular
papillomas are less frequent than squamous cell papillomas. Mixed
papillomas contain features of both squamous cell papillomas and
glandular papillomas.
Question 395
• Question 395: A 38-year-old gravida 0 female with a past medical history that is significant for
premature ovarian insufficiency presents to her provider's office after her cousin was diagnosed with
uterine cancer. She is worried about her risk of developing uterine cancer and would like to learn
more about the disease. She denies any past surgical history and contributory family medical history
other than her cousin with uterine cancer. Her BMI is 21 kg/m2; her vitals are all within normal
limits; she has regular menstrual cycles and underwent menarche at age 14. Her last period was four
years ago, at the age of 34. After further evaluation at that time, she was diagnosed with premature
ovarian insufficiency. Which of the following is true about endometrial cancer?
• Choices:
• 1. Endometrial cancer most often presents as postmenopausal bleeding in the third and fourth
decades
• 2. Thirty percent of patients with postmenopausal bleeding, when investigated, will have
endometrial cancer
• 3. Uterine cancer is the most common cancer in women
• 4. A risk factor for endometrial cancer is late menopause
Answer: 4
• Answer: 4 - A risk factor for endometrial cancer is late menopause Explanations:
Risk factors for endometrial cancer include nulliparity and late menopause
(after age 52). Endometrial cancer is commonly diagnosed between the sixth
and seventh decades of life. Other risk factors for endometrial cancer include
diabetes, obesity, breast cancer, use of tamoxifen, advanced age, and high levels
of estrogen. Approximately 10 percent of postmenopausal bleeding, when
investigated, will have endometrial cancer. The majority of patients at risk for
uterine cancer have high levels of estrogens. An estimated 40% of all uterine
cancers are linked to obesity. Uterine cancer is the fourth most common cancer
of women but the most prevalent gynecologic cancer. Polycystic ovary
syndrome is associated with irregular menstrual cycles or anovulation. It is
associated with very high rates of uterine cancer for the same reasons as
obesity.
Question 396
• Question 396: Which of the following is the best modality for
diagnosing a cystic breast mass in a 35 year old female presenting
with a painful mobile lump of fewer than 6 months duration?
• Choices:
• 1. MRI
• 2. Mammogram
• 3. CT
• 4. Ultrasound
Answer: 4
• Answer: 4 – Ultrasound
• Explanations: Ultrasound is the best modality for detecting cystic
structures. Fibrocystic breast condition most commonly occurs in
women who are between 30 and 50 years of age. When typical
features of a simple breast cyst are seen with ultrasonography, no
further workup is required. Large cysts that are symptomatic may
warrant aspiration.
Question 397
• Question 397: In a 65-year-old with chronic myelogenous leukemia,
there is an abnormal reciprocal translocation of chromosome 9 and
which other chromosome?
• Choices:
• 1. Chromosome 7
• 2. Chromosome 10
• 3. Chromosome 11
• 4. Chromosome 22
Answer: 4
• Answer: 4 - Chromosome 22
• Explanations: Chronic myelogenous leukemia is an abnormal
reciprocal translocation of chromosomes 9 and 22. The ABL oncogene
relocates from the long arm of chromosome 9 to the BCR region of
the long arm of chromosome 22. The BCR-ABL fusion gene encodes a
protein with strong tyrosine kinase activity. The BCR-ABL fusion gene
is a hallmark of chronic myelogenous leukemia.
Question 398
• Question 398: A 65-year-old man, CEO of an Asian tech company, presented to the
hospital with recurrent episodes of epistaxis, otitis media, and nasal obstruction with
mild unilateral deafness. On examination, the patient had a right-sided firm neck mass
in the upper jugular region. On investigation, his labs were within normal limits.
However, the endoscopic evaluation showed an infiltrating mass over the posterior wall
of the nasopharynx involving and obscuring the opening of the right eustachian tube. A
biopsy was taken that showed a poorly differentiated clump of non-keratinized cells.
The patient was started on concurrent chemoradiotherapy, however surgical
intervention was decided. What was the most likely rationale for the decision?
• Choices: 1. Regression of the tumor after concurrent chemoradiation
• 2. Residual or recurrent disease
• 3. Involvement of internal carotid artery
• 4. Extensive soft-tissue infiltration of the parapharyngeal spaces
Answer: 2
• Answer: 2 - Residual or recurrent disease
• Explanations: The clinical presentation, coupled with investigations is
consistent with nasopharyngeal carcinoma (NPC). The malignancy is
more common in patients with Chinese descent, which show
susceptibility coupled with Epstein Barr virus (EBV) infections. NPC is
treated with radiotherapy for stages 1 and 2 whereafter it is coupled
with chemotherapy. Surgical intervention is considered in the residual
or recurring disease that has minimal parapharyngeal involvement
along with sparring of carotid vessels.
Question 399
• Question 399: Which metastatic brain malignancy is associated with
the worst prognosis?
• Choices:
• 1. Glioblastoma multiforme
• 2. Melanoma
• 3. Small cell lung cancer
• 4. Breast cancer
Answer: 2
• Answer: 2 - Melanoma
• Explanations: The combination of hemorrhagic transformation of
metastases and inherent mortality of the disease makes the prognosis
for metastatic melanoma worse than the other cancers mentioned.
Glioblastoma multiforme carries a poor prognosis as well but is not
metastatic. Brain metastases from primary cancers listed above have
average mortalities in less than a year. Choriocarcinomas and
glioblastoma multiforme are also associated with significant cerebral
hemorrhages.
Question 400
• Question 400: A 56-year-old female presents with sudden onset of severe abdominal pain
and nausea. A soft, tender mass in the right upper quadrant was noted on exam, as well as
diffuse abdominal tenderness. Other than tachycardia, her vital signs are within normal
limits. She is unable to produce an accurate medical history, but a laparotomy scar is seen
on physical exam. Laboratory values are normal except for mild elevation in lactate. After
abdominal CT to confirm the diagnosis of afferent loop syndrome, she is treated by the
endoscopic stent to relieve the obstruction. Which of the following most likely represents
her past medical/surgical history?
• Choices:
• 1. Pancreaticoduodenectomy (Whipple) for pancreatic cancer 2 weeks ago
• 2. Billroth I gastroduodenostomy for ulcer disease 4 years ago
• 3. Roux-en-Y gastric bypass for weight loss 8 months ago
• 4. Billroth II gastrojejunostomy for malignant gastric outlet obstruction 3 years ago
Answer: 4
• Answer: 4 - Billroth II gastrojejunostomy for malignant gastric outlet obstruction 3
years ago
• Explanations: Afferent loop syndrome (ALS) is an uncommon complication after
distal gastrectomy and gastrojejunostomy, the incidence being 0.3 to 1%. It is
caused by partial or complete obstruction of the afferent loop consisting of the
bypassed duodenum and a small portion of the jejunum. Management is
dependent on underlying etiology, but a general rule is that benign causes are
treated with surgery, while malignant causes are treated endoscopically or
percutaneously in the acute setting. Billroth II gastrojejunostomy constructed after
distal gastrectomy is by far the most common cause of afferent loop syndrome
(ALS). While surgery is becoming a less common treatment for ulcer disease,
gastric cancer remains a common indication for partial gastrectomy with Billroth II
or Roux-en-Y reconstruction.
• Given the history of endoscopic stent placement for treatment of
afferent loop syndrome, rather than surgical reconstruction, one can
infer the patient was either a poor surgical candidate or the obstruction
was caused by recurrent malignancy. ALS due to recurrent disease is best
treated by palliative intervention to relieve the acute obstruction with
delayed/deferred surgical management until after appropriate treatment
of the underlying malignancy. Endoscopic or percutaneous intervention
is preferred in the acute setting. Revisional surgery is the mainstay of
treatment for benign causes of afferent loop syndrome and most often
done by converting a Billroth II gastrojejunostomy to a Roux-en-Y
gastrojejunostomy with a short afferent limb.
Question 400
• Question 400: A 56-year-old female presents with sudden onset of severe
abdominal pain and nausea. A soft, tender mass in the right upper quadrant was
noted on exam, as well as diffuse abdominal tenderness. Other than tachycardia,
her vital signs are within normal limits. She is unable to produce an accurate
medical history, but a laparotomy scar is seen on physical exam. Laboratory
values are normal except for mild elevation in lactate. After abdominal CT to
confirm the diagnosis of afferent loop syndrome, she is treated by the endoscopic
stent to relieve the obstruction. Which of the following most likely represents her
past medical/surgical history?
• Choices:
• 1. Pancreaticoduodenectomy (Whipple) for pancreatic cancer 2 weeks ago
• 2. Billroth I gastroduodenostomy for ulcer disease 4 years ago
• 3. Roux-en-Y gastric bypass for weight loss 8 months ago
• 4. Billroth II gastrojejunostomy for malignant gastric outlet obstruction 3 years
Answer: 4
Answer: 4 - Billroth II gastrojejunostomy for malignant gastric outlet obstruction 3 •
years ago
Explanations: Afferent loop syndrome (ALS) is an uncommon complication after •
distal gastrectomy and gastrojejunostomy, the incidence being 0.3 to 1%. It is
caused by partial or complete obstruction of the afferent loop consisting of the
bypassed duodenum and a small portion of the jejunum. Management is
dependent on underlying etiology, but a general rule is that benign causes are
treated with surgery, while malignant causes are treated endoscopically or
percutaneously in the acute setting. Billroth II gastrojejunostomy constructed after
distal gastrectomy is by far the most common cause of afferent loop syndrome
(ALS). While surgery is becoming a less common treatment for ulcer disease,
gastric cancer remains a common indication for partial gastrectomy with Billroth II
.or Roux-en-Y reconstruction
Given the history of endoscopic stent placement for treatment of •
afferent loop syndrome, rather than surgical reconstruction, one can
infer the patient was either a poor surgical candidate or the obstruction
was caused by recurrent malignancy. ALS due to recurrent disease is best
treated by palliative intervention to relieve the acute obstruction with
delayed/deferred surgical management until after appropriate treatment
of the underlying malignancy. Endoscopic or percutaneous intervention
is preferred in the acute setting. Revisional surgery is the mainstay of
treatment for benign causes of afferent loop syndrome and most often
done by converting a Billroth II gastrojejunostomy to a Roux-en-Y
.gastrojejunostomy with a short afferent limb
Question 401
?Question 401: What is the most common site of an oral cancer lesion •
:Choices
Gingiva .1
Palate .2
Tongue .3
Buccal mucosa .4
Answer: 3
Answer: 3 - Tongue •
Explanations: Though any site may be affected, the lateral portion of •
the tongue is the most common site of an oral cancer lesion. Oral
cancer mostly occurs after the age of 40 years. The risk of oral cancer
is twice as high in men compared to women. Smoking and heavy
.alcohol consumption increase the risk for oral cancer
Question 402
Question 402: A patient develops endometrial carcinoma. Which one •
?of the following drugs has the patient most likely been taking
:Choices
Oral contraceptives .1
Progesterone .2
Bromocriptine .3
Tamoxifen .4
Answer: 4
Answer: 4 – Tamoxifen •
Explanations: Oral contraceptives decrease the risk of endometrial •
cancer. Tamoxifen increases the risk of endometrial cancer, as does
obesity, nulliparity, age over 35 years, and anovulatory cycles. There
are many risk factors for endometrial cancer including unopposed
estrogen. One disorder is polycystic ovarian disease. Tamoxifen does
increase the risk of cancer but the data are old and derived from
retrospective analysis. Based on these studies tamoxifen is not
.prescribed for long term therapy
Question 403
Question 403: Hyperuricemia is a known adverse effect resulting from •
massive cell death during chemotherapy for which allopurinol is co-
administered therapeutically. With which chemotherapy agent should
allopurinol be used very cautiously or not at all because it can
?potentiate the toxicity of that agent
:Choices •
Chlorambucil .1
Mercaptopurine .2
Daunorubicin .3
.Bleomycin .4
Answer: 2
Answer: 2 - Mercaptopurine •
Explanations: Allopurinol is a xanthine oxidase inhibitor. Xanthine •
oxidase is the enzyme responsible for purine degradation into uric
acid. Mercaptopurine also is metabolized by xanthine oxidase. Thus,
xanthine oxidase inhibition can increase serum levels of
.mercaptopurine and lead to toxicity
Question 404
Question 404: What is the most common subtype of Hodgkin
?lymphoma seen in the mediastinum
:Choices •
Nodular sclerosing .1
Lymphocyte predominant .2
Lymphocyte depleted .3
Mixed cellularity .4
Answer: 1
Answer: 1 - Nodular sclerosing •
Explanations: The mediastinum may have nodular lymphocyte- •
predominant Hodgkin lymphoma or any of the four types referred to
as classical Hodgkin lymphoma. Nodular sclerosing, mixed cellularity,
lymphocyte rich, and lymphocyte depleted are the four classic types
of Hodgkin lymphoma. The most prevalent type of Hodgkin
lymphoma seen in the mediastinum is the nodular sclerosing type
which occurs in 55% to 75% of patients. The lymphocyte predominant
.type is seen in about 40% of patients
Question 405
• Question 405: Thalidomide may work as an immunomodulatory agent
because of its ability to do which of the following?
• Choices:
• 1. Stimulate T helper cells
• 2. Reduce levels of tumor necrosis factor alpha
• 3. Increase bone marrow angiogenesis
• 4. Reduce T suppressor cells
Answer: 2
Answer: 2 - Reduce levels of tumor necrosis factor alpha •
Explanations: It is believed that most of thalidomide's immune effects •
are related to its ability to inhibit tumor necrosis factor (TNF) alpha. It
can be used for wasting syndrome in AIDS. This is thought to be due
to its activity against TNF alpha that is implicated in encouraging HIV
replication in infected lymphocytes. Thalidomide also is used to treat
multiple myeloma. Its effects may be due to its anti-angiogenic
.properties
Question 406
• Question 406: What is the most common mediastinal mass in
childhood?
• Choices:
• 1. Lymphoma
• 2. Thymoma
• 3. Neuroblastoma
• 4. Seminoma
Answer: 3
Answer: 3 - Neuroblastoma •
Explanations: Lesions of the posterior mediastinum dominate in •
childhood. Anterior mediastinal masses are common in adults.
Neurogenic tumors are most common in childhood. after the first
decade of life, the most common posterior mediastinal lesions are
neurofibromas, ganglioneuromas, and rarely schwannoma.
Neurogenic tumors of the posterior mediastinum often invade the
.spinal canal and may cause nerve compression
Question 407
• Question 407: Which one of the following is a late consequence of
cancer chemotherapy?
• Choices:
• 1. Pancytopenia
• 2. Sideroblastic anemia
• 3. Acute myeloid leukemia
• 4. Tumor lysis syndrome
.
Answer: 3
Answer: 3 - Acute myeloid leukemia •
Explanations: One late consequence of chemotherapy is occurrence •
of another cancer. Acute myeloid leukemia can occur after
chemotherapy. The incidence is less than 1%, and it usually occurs 1.5
to 3 years after treatment. Pancytopenia and tumor lysis syndrome
.are acute complications
Question 408
• Question 408: Which of the following medications is approved for the
treatment of advanced breast cancer?
• Choices:
• 1. Bortezomib
• 2. Tositumomab
• 3. Fulvestrant
• 4. Erlotinib
Answer: 3
• Answer: 3 - Fulvestrant
• Explanations: Fulvestrant competitively binds to estrogen receptors in
tumors and normal tissue. It decreases DNA synthesis by producing a
nuclear complex. Estrogen receptors are down regulated and tumor
growth is inhibited. This drug is used for the treatment of metastatic
hormone receptor positive breast cancer in postmenopausal women.
Question 409
• Question 409: For which of the following is a patient infected with the
HIV virus at the highest risk for developing?
• Choices:
• 1. Hodgkin
• 2. Kaposi
• 3. Meningioma
• 4. Hairy cell leukemia
Answer: 2
• Answer: 2 - Kaposi
• Explanations: Patients with HIV are at a very high risk for developing a
variety of cancers. The reason for the increased cancer is co-infection
with Epstein-Barr virus, Kaposi sarcoma-associated herpes virus, and
human papillomavirus. The most common cancer in patients with
AIDs is Kaposi sarcoma. The lesions appear as purplish nodules on the
skin but can affect any organ. High-grade B-cell lymphomas are the
next most common and often occur in the brain.
Question 410
• Question 410: A 68-year-old woman comes to the physician because of
palpable lymph nodes in the left axilla. She denies weight loss, and her vital
signs are within normal limits. Mammography is negative for breast malignancy.
A biopsy is performed on the left axillary lymph nodes. Histopathology shows a
malignancy with multiple sites with pyrimidine cross-linking forming pyrimidine
dimers. Which of the following is the most likely site of origin of this cancer?
• Choices:
• 1. Ovary
• 2. Colon
• 3. Skin
• 4. Breast
Answer: 3
• Answer: 3 – Skin
• Explanations: Pyrimidine dimers are formed when UV light induces the
formation of thymine or cytosine linkages. Pyrimidine dimers are used
clinically as a marker of UV-induced DNA damage. Pyrimidine linkages
usually are corrected by the nucleotide excision pathway. Pyrimidine
linkages are usually removed by the nucleotide excision repair pathway.
However, the repair mechanisms can become overwhelmed by UV
exposure leading to mutations and malignant growth. Individuals with
xeroderma pigmentosum lack nucleotide excision repair mechanisms,
which increases the risk of cancer after exposure to UV radiation.
Pyrimidine dimers are markers of UV damage. The biopsy findings are not
characteristic of ovarian, colon, or breast cancer.
Question 411
• Question 411: Which of the following is a common finding in a patient
with multiple myeloma?
• Choices:
• 1. Joint swelling
• 2. Bone pain
• 3. Polyradiculopathy
• 4. Hypertension
Answer: 2
• Answer: 2 - Bone pain
• Explanations: Bone pain is generally due to vertebral body
compression fractures. Osteopenia and osteolytic lesions are
frequently observed throughout the entire body. Myeloma bone
disease is caused by cytokines that stimulate osteoclastic activity and,
consequently, bone resorption. Bisphosphonates such as zoledronic
acid and pamidronate are very useful agents for treating bone disease
and hypercalcemia in multiple myeloma.
Question 412
• Question 412: For primary small cell carcinoma of the lung, what is
the most common site for recurrence?
• Choices:
• 1. Brain
• 2. Liver
• 3. Bone
• 4. Mediastinum
Answer: 4
• Answer: 4 - Mediastinum
• Explanations: Relapse of small cell carcinoma is most common at the
primary site, which is intrathoracic. Therefore, radiation to the thorax
is administered to these patients. If patients with small cell lung
cancer survive long enough, second primary lung cancers occur.
Indicators of poor prognosis include weight loss, disease recurrence,
and hyponatremia. Even patients with localized disease have only a
15% 5-year survival rate.
Question 413
• Question 413: What is the most common type of Hodgkin tumor?
• Choices:
• 1. Nodular sclerosis
• 2. Lymphocyte dominant
• 3. Lymphocyte depleted
• 4. Mixed cellularity
Answer: 1
• Answer: 1 - Nodular sclerosis
• Explanations: Sixty to 80 percent of patients with Hodgkin lymphoma
have the nodular sclerosing subtype. Often the first nodes affected
are in the mediastinum. Mixed cellularity Hodgkin lymphoma is seen
in 15 to 30 percent of patients. Five percent of patients have
lymphocyte-rich and only 1 percent have lymphocyte depleted.
Question 414
• Question 414: Which condition is most likely to elevate
carcinoembryonic antigen?
• Choices:
• 1. Pancreatitis
• 2. Hepatitis
• 3. Small cell lung cancer
• 4. Colorectal cancer
Answer: 4
• Answer: 4 - Colorectal cancer
• Explanations: Both benign and malignant (harmless and cancerous)
conditions can increase the CEA level. The most frequent cancer
which causes an increased CEA is cancer of the colon and rectum. CEA
levels are not ideal for screening as many disorders can cause
elevation. CEA levels are used to monitor recurrence of disease after
surgery.
Question 415
• Question 415: A 65-year-old female patient has a past medical history
significant for stage 3 colon cancer status post colectomy and currently on
chemotherapy. She presented with fever, dysuria, and an increased
frequency of urine. She was diagnosed with a urinary tract infection leading
to sepsis and was admitted. On examination, she has a high-grade fever, the
blood pressure is 100/70 mmHg, the heart rate is 100/min, and the
respiratory rate is 16/min. Mild suprapubic tenderness is also present, but
otherwise, her examination is unremarkable. Pertinent labs include WBC of
14,000, hemoglobin 12.0, platelets 120,000 and creatinine of 1.0. What is
the most appropriate option to be included in her prophylactic regime?
• Choices:
• 1. Warfarin
• 2. Intermittent pneumatic compression
• 3. Aspirin
Answer: 4
• Answer: 4 - Low molecular weight heparin (LMWH) Explanations: Cancer
patients have increased risk of a thrombotic event (DVT and PE) and they
are even at greater risk when hospitalized thereby warranting a method
of DVT prophylaxis unless there is a clear contraindication. Low molecular
weight heparin is the preferred method in patients with cancer if there
are no contraindications like increased risk of bleeding or renal
impairment. Enoxaparin 40 mg daily or 30 mg daily in patients with mild
renal impairment. Due to delayed antithrombotic action, warfarin is not
normally used in DVT prophylaxis in patients with cancer. Aspirin does not
have a role in DVT prophylaxis in patients with cancer. Intermittent
pneumatic compressions might help but cannot be used as a stand-alone
method.
Question 416
• Question 416: A 65-year-old woman with no significant medical risk factors presents to the
emergency department with slurred speech for the past four hours. Neurology evaluation
confirms the diagnosis of a stroke with a strong suspicion of a cardioembolic source. The
patient is referred for the placement of an internal loop recorder for the detection of atrial
fibrillation and a transesophageal echocardiogram (TEE). On the long-axis view of the TEE, a
30 mm cardiac mass is noted on the aortic side of the aortic valve. The patient is referred for
cardiac surgery for removal of the mass. Pathologic analysis of the cardiac mass shows
papillary growths with fibrous tissue core and a lining of thickened endothelium. Grossly it is
described as having the appearance of a sea anemone. Which of the following is the most
likely diagnosis?
• Choices:
• 1. Papillary fibroelastoma
• 2. Myxoma
• 3.Endocarditis
Answer: 1
• Answer: 1 - Papillary fibroelastoma
• Explanations: It is important to recognize that papillary fibroelastoma
(PFE) can lead to symptoms of a stroke. A PFE with a size > 9mm,
highly mobile, and independent motion place it at risk for
embolization. The location on this patient's TEE makes it more likely
to be a PFE as it typically presents on the aortic side of the aortic
valve. In addition, the gross pathology description of sea anemone-
like appearance is pathognomic for this tumor. Myxomas are also
commonly encountered primary cardiac tumors, however, the gross
pathology description does not match up, and the location is typically
not on the aortic valve.
Question 417
• Question 417: What is the third most common cause of cancer death
in females?
• Choices:
• 1. Lung
• 2. Ovarian
• 3. Colon
• 4. Breast
Answer: 3
• Answer: 3 – Colon
• Explanations: Lung cancer is the leading cause of death in females.
Followed by breast. Followed by colon cancer. Pancreas is the fourth
most common, followed by gynecological cancers.
Question 418
• Question 418: What is the screening test for pheochromocytoma?
Choices:
• 1. CBC
• 2. 24 hour urine adrenaline and hormone by-products
• 3. MRI
• 4. CT
Answer: 2
• Answer: 2 - 24 hour urine adrenaline and hormone by-products
Explanations: Pheochromocytoma is screened with urinary
metanephrine. Chromaffin cells in the adrenal gland produce
adrenaline and noradrenaline. Chromaffin cells develop this tumor,
and the by-products are excreted into the urine, where increased
levels indicate the presence of the tumor. Imaging tests are only done
after the biochemical testing is completed.
Question 419
• Question 419: By how much can oral contraceptives decrease the risk
of endometrial cancer?
• Choices:
• 1. 15 percent
• 2. 35 percent
• 3. 50 percent
• 4. 75 percent
Answer: 3
• Answer: 3 - 50 percent
• Explanations: Combined oral contraceptive (OC) use can decrease the
risk of endometrial cancer by 50 percent. This effect lasts for up to 20
years. Combined OC use can decrease the risk of ovarian cancer by 27
percent. The longer the duration of use the greater the risk reduction.
Question 420
• Question 420: A patient with non-small cell lung cancer presents with
bone metastasis. What is their 5-year survival rate?
• Choices:
• 1. Less than 5%
• 2. 10%
• 3. 40%
• 4. 60%
Answer: 1
• Answer: 1 - Less than 5%
• Explanations: The 5-year survival rate for bone metastasis from non-
small cell lung (NSCLC) is less than 5%. Osseous metastases represents
advanced stage IV disease. Th 5-year survival rate for individuals with
stage IA NSCLC is about 50%; stage IIA cancer is about 30%; and stage
IIIA is about 14%. Palliative care is offered to these patients, and the
least invasive procedures are recommended.
Question 421
• Question 421: What is the most appropriate therapy for
nonmetastatic renal cell carcinoma?
• Choices:
• 1. Chemotherapy
• 2. Radiation therapy
• 3. Radical nephrectomy
• 4. Nephrectomy
Answer: 3
• Answer: 3 - Radical nephrectomy
• Explanations: Radical nephrectomy with removal of the adrenal gland
and regional lymph nodes is treatment of choice. Metastatic disease
is seen in about a third of patients on initial diagnosis. Renal cell
carcinoma is resistant to chemotherapy and radiation therapy.
Question 422
Question 422: A 57-year-old woman with a past medical history of
essential hypertension and type 2 diabetes mellitus presents to the
clinic for follow up. She has failed treatments with platinum-based
chemotherapy for ovarian cancer. Treatment with a liposomal
formulation of a FDA-approved antibiotic chemotherapeutic agent is
considered. Which of the following tests is most appropriate to order
before initiating treatment with the new medication?Choices:
1. Bone density measurement
2. Tuberculosis skin testing
3. Bone marrow biopsy
4. Echocardiography
Answer: 4
• Answer: 4 - Echocardiography
• Explanations: The liposomal formulation of doxorubicin is FDA-
approved for the treatment of ovarian cancer in patients who have
failed platinum-based chemotherapy. Prior to starting doxorubicin,
clinicians have advised patients undergo a baseline assessment of
cardiac function by evaluating a echocardiography. The cardiotoxicity
associated with doxorubicin is typically reversible, with patients
experiencing complete recovery of cardiac function following
discontinuation of treatment. Cardiotoxicity is a known adverse affect
of doxorubicin that usually causes decline in left ventricular function.
Question 423
• Question 423: Which of the following is consistent with multiple
endocrine neoplasia type I (MEN1) in a patient with a gastrinoma?
• Choices:
• 1. Gastrinoma with a prolactinoma
• 2. Gastrinoma and diarrhea
• 3. Gastrinoma and hypocalcemia
• 4. Gastrinoma with low parathyroid hormone (PTH) levels
Answer: 1
• Answer: 1 - Gastrinoma with a prolactinoma
• Explanations: In patients with multiple endocrine neoplasia type I
(MEN1) there is a clustering of pituitary, parathyroid, and pancreatic
tumors. Patients with MEN1 have hypercalcemia. Patients with MEN1
can have hyperparathyroidism and increased parathyroid hormone
(PTH) and calcium levels. Diarrhea is a feature of a gastrinoma and
does not suggest MEN1.
Question 424
• Question 424: Which of the following is the most important
prognostic factor in head and neck cancer?
• Choices:
• 1. Tumor size
• 2. Tumor location
• 3. The presence of cervical lymphadenopathy
• 4. Response to radiation therapy
Answer: 3
• Answer: 3 - The presence of cervical lymphadenopathy Explanations: The
presence of cervical lymphadenopathy is the most important prognostic
factor in head and neck cancer. Metastasis to the regional lymph nodes
reduces the 5-year survival rate by 50% compared to that of patients with
early-stage disease. The American Cancer Society has reported that 40% of
patients with squamous carcinoma of the oral cavity and pharynx present
with regional metastases. Therefore, management of the cervical lymph
nodes is an important component in the overall treatment plan for patients
with squamous cell carcinoma of the head and neck. Characteristics of
regional nodes that affect prognosis include the size, number, and location
of pathologically positive nodes. Involvement of the lower cervical and
lower posterior triangle lymph nodes has a very poor prognosis.
Question 425
• Question 425: What is Stewart Treves syndrome?
• Choices:
• 1. Ewing tumor of the femur
• 2. Hodgkin tumor of the mediastinum
• 3. Angiosarcoma after chronic lymphedema
• 4. Infection of bile duct with C. sinensis
Answer: 3
• Answer: 3 - Angiosarcoma after chronic lymphedema
• Explanations: Stewart Treves syndrome is an angiosarcoma that
occurs in long standing lymphedema. The disorder is seen after radical
mastectomy and often occurs on one arm. Angiosarcoma is a vascular
tumor and is lethal. The tumor is not common. Because of more
conservative treatment of women with breast cancer, the prevalence
has definitely decreased. Once the tumor occurs, the prognosis is very
poor. Metastatic spread is very common early on and no treatment
usually works well.
Question 426
• Question 426: Which of the following chemotherapy agents is used
for treatment of chronic myelogenous leukemia?
• Choices:
• 1. Vinblastine
• 2. Imatinib
• 3. Rituximab
• 4. Anastrozole
Answer: 2
• Answer: 2 – Imatinib
• Explanations: Imatinib works by inhibiting the BCR-ABL gene that has
tyrosine kinase activity. This mutant gene is implicated as the cause
for chronic myelogenous leukemia. Imatinib is known to inhibit major
liver enzymes and may cause drug-drug interactions. Rituximab is a
monoclonal antibody for CD-20 B lymphocytes.
Question 427
• Question 427: Which of the following drugs is used to stimulate bone
marrow to produce more white blood cells?
• Choices:
• 1. Vincristine
• 2. Imatinib
• 3. Filgrastim
• 4. Abciximab
Answer: 3
• Answer: 3 – Filgrastim
• Explanations: A major side effect of many chemotherapy drugs is
bone marrow suppression. Filgrastim is often used to stimulate the
bone marrow in increase neutrophil production. The pegylated form
of filgrastim is often used in clinical practice to increase its half-life,
but pegfilgrastim is available as a subcutaneous medication only. Do
not administer filgrastim in the period 24 hours before and 24 hours
after cytotoxic chemotherapy administration.
Question 428
• Question 428: What is the best test to make the diagnosis in a patient
with suspected bladder cancer?
• Choices:
• 1. MRI
• 2. Cystoscopy
• 3. Angiogram
• 4. Urine analysis
Answer: 2
• Answer: 2 - Cystoscopy
• Explanations: The best method to evaluate a bladder tumor is under
direct visualization. One is able to biopsy tissues during cystoscopy. CT
scan is the next best option, after cystoscopy. Besides diagnosing
bladder cancer, cystoscopy also can be done to further investigate
chronic recurrent urinary tract infections, unexplained hematuria,
urinary tract stones, dysuria, or narrowing of the urinary tract, as well
as for bladder catheterization.
Question 429
• Question 429: Which tumor is often associated with Rathke pouch?
• Choices:
• 1. Melanoma
• 2. Craniopharyngioma
• 3. Pituitary adenoma
• 4. Nasal carcinoma
Answer: 2
• Answer: 2 - Craniopharyngioma
• Explanations: Craniopharyngioma is a neoplasm of Rathke pouch. The
tumor may occur anywhere along the migration path of Rathke Pouch
toward the adenohypophysis. Rathke pouch is a depression in the
roof of the developing mouth that gives rise to the anterior pituitary.
Craniopharyngiomas have benign histology but malignant behavior.
Question 430
• Question 430: Which of the following acts as a spindle poison in
treating ovarian cancers?
• Choices:
• 1. Cyclophosphamide
• 2. Methotrexate
• 3. Paclitaxel
• 4. Bleomycin
Answer: 3
• Answer: 3 – Paclitaxel
• Explanations: Paclitaxel is spindle poison and prevents microtubule
disassembly. Paclitaxel causes neuropathy and hypersensitivity
reaction during infusion, used in treatment of ovarian and breast
cancers. Paclitaxel is M phase specific in the cancer cell cycle.
Paclitaxel is a CYP3A4 (cytochrome P450) substrate which can be
important for drug interactions.
Question 431
• Question 431: A 16-year-old boy is brought to the hospital for fever and
persistent watery diarrhea. He has a history of acute lymphocytic
leukemia. He recently completed a course of amoxicillin for bacterial
pharyngitis. Which of the following organisms is most likely responsible
for the patient's condition?
• Choices:
• 1. Salmonella typhi
• 2. Clostridium difficile
• 3. Clostridium perfringens
• 4. Shigella flexneri
Answer: 2
• Answer: 2 - Clostridium difficile
• Explanations: Clostridium difficile associated disease (CDAD) is a
unique colon infection that is acquired almost exclusively in association
with antimicrobial use and the consequent disruption of normal colonic
flora. The most commonly diagnosed diarrheal illness acquired in a
hospital, CDAD results from the ingestion of spores of C. difficile that
vegetates, multiply and secrete toxins, causing diarrhea and
pseudomembranous colitis (PMC). Risk factors for CDAD include older
age, greater severity of underlying illness, gastrointestinal surgery, use
of electronic rectal thermometers, enteral tube feeding, and antacid
treatment. The use of proton pump inhibitors may also be a risk factor.
Question 432
• Question 432: In a 67-year-old female with a stage 1, T1N0M0 breast
cancer, what is the best treatment besides surgery?
• Choices:
• 1. Observation
• 2. Radiation
• 3. Chemotherapy
• 4. Tamoxifen
Answer: 1
• Answer: 1 – Observation
• Explanations: For a stage 1 breast cancer, there is no other treatment
that can improve prognosis. The prognosis in stage 1 breast cancer
ultimately is dependent on the histology of the cancer. Adjuvant
therapy has not been shown to improve survival in patients with stage
1 cancer. For stage 1, T1N0M0 breast cancer, surgery followed by
close follow up is the treatment of choice.
Question 433
• Question 433: A 66-year-old male with a history of chronic cigarette
smoking presents with weight loss and frequent chest infections. Chest
x-ray and CT scan reveal a centrally located lung mass. Laboratory tests
show hyponatremia. Which lung cancer is the most likely cause?
• Choices:
• 1. Small cell
• 2. Squamous
• 3. Adenocarcinoma
• 4. Bronchial carcinoid
Answer: 1
• Answer: 1 - Small cell
• Explanations: Small cell lung cancer has the strongest association with
cigarette smoking of all the bronchogenic cancers. There is a 99%
association with small cell, 98% with squamous cell, and 75% with
adenocarcinoma. Small cell lung cancer is the most common lung
neoplasia causing hyponatremia. Small cell cancer causes euvolemic
hyponatremia due to excessive production of antidiuretic hormone
(ADH) resulting in the syndrome of inappropriate ADH production
(SIADH). In many cases, the SIADH precedes the symptoms of the
cancer by many months.
Question 434
• Question 434: Which is the best treatment for severe nausea and
vomiting in a patient receiving chemotherapy?
• Choices:
• 1. Prochlorperazine
• 2. Promethazine
• 3. Ginger
• 4. Dolasetron
Answer: 4
• Answer: 4 - Dolasetron
• Explanations: Chemotherapy-induced nausea occurs at the chemoreceptor trigger zone at the
level of the fourth ventricle. Prochlorperazine is a dopamine agonist and may provoke dystonia
and somnolence. It is best utilized for metabolic-related nausea and vomiting such as with
tumor, uremia, and hepatic disease. Promethazine is an antihistamine. It is best utilized for gut
receptor nausea, motion-related nausea, and inner ear related nausea, and is relatively
ineffective at the level of chemoreceptor trigger zone where chemotherapy-induced nausea
occurs. Ginger is an oral, over-the-counter homeopathic option and is precluded from this
scenario by the method of administration being solely oral. The primary effect of ginger is one
of increased gastric emptying. It has been shown to be effective as an adjunct or
complementary alternative therapy for nausea nausea once vomiting has been controlled or in
the prevention of nausea. Dolasetron is a selective 5-HT3 receptor antagonist that functions at
the level of the fourth ventricle known as the chemoreceptor trigger zone. 5-HT3 receptor
antagonists as a class often are used to treat nausea and vomiting associated with
chemotherapy which occurs at the chemoreceptor trigger zone.
Question 435
• Question 435: A 50-year-old patient with an unknown medical and surgical history walks
into the clinic for the first time. She has not seen a provider in over ten years because of a
lack of insurance but has recently begun working again. She has been estranged from her
family for 30 years and is unsure of any pertinent family history at this time. Her physical
examination is unremarkable. The provider orders a full set of labs and refers her to a
gastroenterologist for a colonoscopy. According to the American Cancer Society guidelines
for colorectal cancer screening, at what age should this patient be recommended for a
colonoscopy?
• Choices:
• 1. She should have had a colonoscopy at the age of 40 because she does not know her
family history
• 2. She should have had a colonoscopy at the age of 45 based on the current guidelines
• 3. She should have a colonoscopy at the age of 50 based on the current guidelines
Answer: 2
• Answer: 2 - She should have had a colonoscopy at the age of 45 based on the
current guidelines
• Explanations: Based on the most recent American Cancer Society guidelines,
the age for colon cancer screening should begin at 45 as it will result in more
lives saved. Patients with inflammatory bowel disease are at increased risk for
colon cancer. Screening colonoscopies should be done 10 years after the
initial diagnosis. Screening should be started earlier in patients with a family
history of colon cancer, familial polyposis, Lynch syndrome, or history of
radiation to the abdomen or pelvis for other cancers. Colonoscopy should be
done more frequently in those with a personal history of polyps or colon
cancer. Patients should talk to their healthcare providers who can determine
the appropriate timing for colonoscopy as well as the frequency of screening.
Question 436
• Question 436: A 20-year-old male presents to the emergency department for altered
mental status. Per the patient’s mother, the patient suddenly started sweating and became
unresponsive. She reports that he does not have any medical history, surgical history or any
significant family history. He does not take any medications. The patient has a temperature
of 98.9 degrees Fahrenheit, the pulse is 110 beats per minute, his blood pressure is 110/60
mm hg, blood oxygen saturation is 100% and he is breathing 14 times a minute. On physical
examination, he appears lethargic. His heart, lung, and abdominal examination is normal.
His skin is moist. His blood glucose level is found to be 20 mg/dL. He is given a dextrose
injection intravenously. Within a few minutes, he regains his consciousness with no residual
deficits. He denies any complaints. His EKG and troponins are within normal limits. His urine
sulphonyl-urea level is zero. The c-peptide levels are 4 ng/ml. The CT scan of his brain was
done and was negative for any abnormalities. What is the most likely cause of the patient’s
symptoms?
• Choices:
• 1. Ischemic Stroke
• 2. Insulinoma
• 3. Administration of sulphonylurea drugs
• 4. Myocardial infarction
Answer: 2
• Answer: 2 – Insulinoma
• Explanations: In patients with insulinoma, there is excessive endogenous
production of insulin from the insulin-secreting beta cells of the pancreas.
Patients with insulinoma can present with sweating, palpitations, tremors,
confusion, altered mental status, behavioral changes, seizures, syncope as
well as coma. C peptide levels are typically elevated in patients with
insulinoma. The use of exogenous insulin is contraindicated in this setting.
Rapid reversal of symptoms with the administration of glucose goes against
ischemic stroke as a cause of the symptoms. No chest pains with normal EKG
and troponins rule out myocardial infarction. Since the patient does not take
any medications and the urine sulphonylurea level is zero, it rules out the
administration of sulphonylurea as a cause of the patient's symptoms.
Question 437
• Question 437: What is the preferable location for bone marrow biopsy
in an adult?
• Choices:
• 1. Sternum
• 2. Tibia
• 3. Femur
• 4. Iliac crest
Answer: 4
• Answer: 4 - Iliac crest
• Explanations: The iliac crest is the most common site for both
aspiration and biopsy. The iliac crest is safe and accessible with
minimal risk of hitting nerves or vessels. Occasionally the sternum can
be aspirated in teenagers and young adults. Bone marrow biopsy is
useful to rule out leukemia.
Question 438
• Question 438: What is the first step in the treatment of a 40-year-old
who is 7 weeks with a molar pregnancy?
• Choices:
• 1. Await fetal heart activity with a hand-held Doppler prior to making
any decisions
• 2. Treat with promethazine intramuscular for nausea and bed rest
• 3. Perform a chest x-ray and a uterine evacuation
• 4. After termination, advising the patient to wait 2 months prior to
attempting another pregnancy
Answer: 3
• Answer: 3 - Perform a chest x-ray and a uterine evacuation
• Explanations: Chest x-ray is necessary for all newly diagnosed molar pregnancies
as the site of initial metastasis is most often the lung. Patients at the extremes of
age are at highest risk of molar pregnancy. Standard treatment of a molar
pregnancy is uterine evacuation with suction curettage. Hysterectomy is an
option for patients who have completed having children. All patients with molar
pregnancy should be monitored with serial serum human chorionic gonadotropin
(hCG) testing values after evacuation to evaluate for post-molar gestational
trophoblastic neoplasia. Guidelines from the American College of Obstetricians
and Gynecologists recommends testing every week until non-detectable for 3
weeks, then every month for 6 months. If the hCG remains undetectable for 6
months, then the patient may resume trying to become pregnant.
Question 439
• Question 439: What is the mode of action of paclitaxel?
• Choices:
• 1. DNA synthesis inhibitor
• 2. Cell membrane instability
• 3. Microtubule poison
• 4. Antagonist of luteinizing hormone
Answer: 3
• Answer: 3 - Microtubule poison
• Explanations: Paclitaxel interferes with normal function of
microtubules breakdown. Recent evidence shows that paclitaxel can
induce apoptosis by binding to BCL2. It is used to treat ovarian,
breast, and lung cancer. Side effects include loss of appetite, change
in taste, brittle nails and pain in legs. Dexamethasone is usually given
prior to paclitaxel therapy.
Question 440
• Question 440: A 65-year-old female diabetic reports a 6-month
history of right upper quadrant pain decreased by leaning forward.
She has recurrent superficial thrombophlebitis and jaundice. What is
the most probable diagnosis?
• Choices:
• 1. Pancreatic carcinoma
• 2. Cirrhosis secondary to viral hepatitis
• 3. Carcinoma of the ampulla of Vater
• 4. Choledocholithiasis
Answer: 1
• Answer: 1 - Pancreatic carcinoma
• Explanations: Pancreatic carcinoma located in the head can cause
biliary obstruction. Trousseau sign is hypercoagulability associated
with pancreatic and lung cancer. Carcinoma of the ampulla of Vater
can cause jaundice but not thrombophlebitis. Cirrhosis or
choledocholithiasis could cause jaundice but not the other findings.
Question 441
• Question 441: Methotrexate is prescribed for gestational
trophoblastic disease. Which of the following should be prescribed
together to reduce methotrexate side effects?
• Choices:
• 1. Vitamin B6
• 2. Filgrastim
• 3. Leucovorin
• 4. Vitamin B12
Answer: 3
• Answer: 3 - Leucovorin
• Explanations: Leucovorin is added to methotrexate to reduce
methotrexate toxicity. Leucovorin is a derivative of tetrahydrofolic
acid. It is converted to other folic acid derivatives and therefore, its
vitamin activity is equivalent to that of folic acid. Filgrastim is a colony
activation factor which is used to prevent neutropenia in patients
receiving other types of chemotherapy. Vitamin B6 and B12 do not
reduce the toxicity of methotrexate.
Question 442
• Question 442: A 17-year-old female presents to a clinic with painful lesions in the perineal area.
There is no history of fever, malaise, or any previous history of genital ulcers. Detailed medical
history revealed that she has multiple sexual partners, and she often has unprotected sexual
intercourse. On physical examination, there are multiple non-tender pedunculated lesions on the
external surface of her vulva. A biopsy of the genital lesions revealed hyperplastic squamous
epithelium with koilocytes and acentric nuclei. The clinician prescribed her podophyllotoxin
solution 0.15% cream to apply it on the affected area. The clinician also advised her regular pap
smear tests as she is also at the risk of developing which of the following cancers?
• Choices:
• 1. Vulvar cancers
• 2. Anal cancer
• 3. Vaginal cancer
• 4. Cervical cancer
Answer: 4
• Answer: 4 - Cervical cancer
• Explanations: The HPV infection is virtually responsible for all cases of
cervical cancer in women, with HPV type 16 accounting for about 50%
of these. Cervical cancer is the fourth most common cancer in
women. HPV infection appears to be the cause of most cases of anal
cancer (about 90%). Some vulvar cancers have been linked to HPV
infections (29% to 43%), while vaginal cancer is associated with HPV
infections, about 70% of the time (HPV Types 16 and 18).
Question 443
• Question 443: What is the chemical associated with carcinoma of the
liver?
• Choices:
• 1. Asbestosis
• 2. Iron
• 3. Mechlorethamine
• 4. Aspirin
Answer: 2
• Answer: 2 - Iron
• Explanations: Individuals who develop hemochromatosis, especially in
the presence of cirrhosis, are more likely to develop liver cancer. Liver
cancer accounts for about 30 percent of iron related deaths in
patients with hemochromatosis. Another chemical that increases the
risk of liver cancer is aflatoxin. Rare causes of liver cancer include
primary biliary cirrhosis, anabolic steroids, and primary sclerosing
cholangitis.
Question 444
• Question 444: Which of the following is the treatment for a
microadenoma secreting excess prolactin?
• Choices:
• 1. Bromocriptine
• 2. Chlorpromazine
• 3. Lithium
• 4. Methotrexate
Answer: 1
• Answer: 1 - Bromocriptine Explanations: Dopamine inhibits prolactin
release. Because of its peripheral effects, dopamine is not used to
control hyperprolactinemia. Bromocriptine is effective in reducing
prolactin secretion. Bromocriptine activates dopaminergic receptors.
Question 445
• Question 445: What is the mechanism of action of radiation therapy
for the treatment of cancer?
• Choices:
• 1. Damage of DNA
• 2. Denaturation of proteins
• 3. Lysis of cell membranes
• 4. Disruption of cellular transport
Answer: 1
• Answer: 1 - Damage of DNA
• Explanations: Radiation therapy acts by damaging DNA. This causes
the cells to either stop dividing or die. The radiation damage is either
by direct or indirect ionization and is irreversible. The process
generates free radicals which prevent the DNA repair mechanisms
from working.
Question 446
• Question 446: A 45-year-old woman presents to the hospital with severe vomiting. Her vital
signs show blood pressure 135/85 mmHg, temperature 98.7 F, and pulse of 80/min. The
patient has a history of non-small cell lung cancer. A physical exam is unremarkable. The
patient is currently receiving adjuvant treatment with cisplatin and etoposide as well, as
ondansetron and dexamethasone. The clinician decides to add another highly effective
antiemetic medication. Which of the following is the most likely mechanism of action of the
newly added antiemetic?
• Choices:
• 1. GABA-A receptor agonist
• 2. Mu-1 receptor antagonist
• 3. Dopamine-2 receptor agonist
• 4. Neurokinin-1 receptor antagonist
.
Answer: 4
• Answer: 4- Neurokinin-1 receptor antagonist
• Explanations: Aprepitant is a substance P/neurokinin 1 receptor
antagonist. It prevents acute and delayed vomiting by acting on the
substance P/neurokinin 1 (NK1) receptor. It is indicated for
chemotherapy and prevention of postoperative nausea and vomiting
but can cause diarrhea in some patients. It can be used with 5-HT3
receptor antagonists and corticosteroids for chemotherapy-induced
vomiting.
Question 447
• Question 447: A 58-year-old white man who is a nonsmoker presents to the emergency room
for shortness of breath and twenty-pound weight loss for about 6 months. A computed
tomography (CT) scan of the chest with contrast shows a solitary lung nodule and radiological
description notable as ground-glass opacity in the left upper lobe up to 1.5 cm. He undergoes
diagnostic bronchoscopy and left-sided video-assisted thoracoscopic surgery. Pathology from
the pleural mass biopsy reveals lung primary tumor with lipid growth. The oncologist who
interpreted the pathology report counseled the patient that this tumor is associated with an
excellent prognosis, referring to which one of the following histopathology of the tumor?
• Choices:
• 1. Small cell lung cancer
• 2. Squamous cell cancer of the lung
• 3. Sarcomatoid carcinoma
• 4. Adenocarcinoma of lung
Answer: 4
• Answer: 4 - Adenocarcinoma of lung
• Explanations: The histologic description of lung adenocarcinoma was revised by the International
Association for the Study of Lung Cancer, American Thoracic Society, and European Respiratory Society
to provide uniform diagnostic terminology. The different histologic subtypes of lung adenocarcinoma
include lepidic, acinar, papillary, micropapillary, and solid pattern. Most adenocarcinomas of the lung
are a mixture of the different histologic patterns, but they should be classified by their predominant
pattern of growth. Lepidic growth adenocarcinoma is defined as tumor cell proliferation along the
surface of alveolar walls without stromal, pleural, or vascular invasion. Tumors that are less than 3 cm
and demonstrate pure lepidic growth are considered "adenocarcinoma in-situ" and carry a nearly
100% disease-free survival following complete surgical resection. Tumors which are less than or equal
to 3 cm in dimension and have less than or equal to 5mm of invasion are termed "minimally invasive
adenocarcinoma." Different histologic subtypes have different prognostic significance with lepidic
harboring the best course and micropapillary being the most aggressive pattern. On CT, lepidic growth
is often demonstrated by the ground-glass component of the lesion but may also have a semi-solid
appearance. Pure ground-glass nodules are almost always adenocarcinoma in-situ while partly solid
nodules frequently correspond to minimally invasive adenocarcinoma..
Question 448
• Question 448: Fine needle aspiration of a thyroid mass in a 52-year-
old female shows medullary carcinoma. Which of the following is
most likely to be elevated in her serum?
• Choices:
• 1. Thyroxine
• 2. Calcitonin
• 3. Alpha-fetoprotein
• 4. Parathyroid hormone
Answer: 2
• Answer: 2 - Calcitonin
• Explanations: Malignant transformation of the parafollicular C-cells in
the thyroid results in medullary carcinoma. These cells secrete
calcitonin before and after transformation. Elevated parathyroid
hormone would be seen with adenomas of chief cells of the
parathyroid. Alpha-fetoprotein is elevated in hepatocellular
carcinoma.
Question 449
• Question 449: What imaging technique should be used in a patient
with a parathyroid cancer with suspected paratracheal lymph nodes?
• Choices:
• 1. Ultrasound
• 2. X-ray
• 3. CT
• 4. Fluoroscopy
Answer: 3
• Answer: 3 - CT
• Explanations: The parathyroid glands are drained by lymphatics which
course into the paratracheal and deep cervical lymph nodes. Thus in a
patient with a malignancy of the parathyroid, it is important to assess
the deep cervical nodes and paratracheal nodes with a CT scan. The
paratracheal nodes cannot be assessed with a physical exam, chest x-
ray, fluoroscopy, or ultrasound. If the node is positive, the patient will
require a modified neck dissection.
Question 450
• Question 450: What is the most common malignancy of the kidney in
elderly individuals?
• Choices:
• 1. Transitional cancer of the bladder
• 2. Renal cell carcinoma
• 3. Wilms tumor
• 4. Angiomyolipoma
Answer: 2
• Answer: 2 - Renal cell carcinoma
• Explanations: Renal cell cancer is the most common primary renal
cancer in adults. Risk factors include age, male sex, smoking, and
exposure to benzene, cadmium, or trichloroethylene. Individuals with
Von Hippel-Lindau and tuberous sclerosis also develop renal cell
cancer. In these individual the cancer is bilateral. Renal cell cancers
have a propensity to invade the renal vein and inferior vena cava. The
lungs are the most common distant site of metastases. The typical
presentation includes hematuria, flank pain, and incidental findings.
Rarely, renal cancers release humoral factors like parathyroid
hormone, prolactin, and erythropoietin.
Question 451
• Question 451: A postoperative patient is receiving tamoxifen 20 mg twice a
day by mouth as adjunct therapy for breast cancer. When the patient asks
about the purpose of this drug, what is the best response?
• Choices:
• 1. This medication will destroy any cancer cells that may have escaped into
your system when you had surgery
• 2. This medication prevents breast cancer from growing
• 3. This medication will stimulate the body's own immune system to produce
substances that seek out and destroy abnormal cells
• 4. This medication causes more calcium to become deposited in your bones,
which helps prevent the spread of cancer
Answer: 2
• Answer: 2 - This medication prevents breast cancer from growing
Explanations: Tamoxifen prevents estrogen from binding to estrogen
receptors in the breast. Through downstream intracellular processes,
it slows cell cycling. Tamoxifen is an anti-estrogen agent that may be
used as an adjunct to surgery in postmenopausal women whose
breast cancers are estrogen-receptor positive. It also is used
prophylactically in certain patients who are at high risk for developing
breast cancer. Tamoxifen is not known to destroy cancer cells.
Tamoxifen does not cause calcium to deposit in bone, though it can
cause hypercalcemia.
Question 452
• Question 452: What is the most common cause of squamous cell
cancer of the bladder in developing countries?
• Choices:
• 1. Alcohol
• 2. Malnutrition
• 3. Schistosoma haematobium
• 4. Renal failure
Answer: 3
• Answer: 3 - Schistosoma haematobium Explanations: In
underdeveloped and developing countries, particularly in Africa and
the Middle East, squamous cell cancers of the bladder are very
common. The cause of squamous cell cancers in these countries has
been linked to the parasite Schistosoma haematobium. In developed
countries, the most common causes are indwelling catheters,
recurrent infections, and bladder stones. All treatment options appear
to lead to similar end-results regardless of the etiologic factors
implicated in bladder cancer.
Question 453
• Question 453: Which of the following gastric tumors responds best to
radiotherapy?
• Choices:
• 1. Carcinoid
• 2. Melanoma
• 3. Lymphoma
• 4. Leiomyoma
Answer: 3
• Answer: 3 - Lymphoma Explanations: The role of radiation is
controversial for most gastric cancers. For early stage lymphoma,
most oncologists recommend radiation. Investigators have found
improved survival compared to those who undergo surgery. Radiation
is also advocated as adjuvant therapy and palliation. However, its role
postoperatively is not known. Individuals at high risk for surgery may
experience a better quality of life following radiation therapy when
compared to surgery.
Question 454
• Question 454: What is the most common type of posterior
mediastinal neurogenic tumor in adults?
• Choices:
• 1. Pheochromocytoma
• 2. Carcinoid
• 3. Neurilemmoma
• 4. Neuroblastoma
Answer: 3
• Answer: 3 - Neurilemmoma
• Explanations: Almost all neurogenic tumors in adults evolve from the
nerve sheath. The two most common nerve sheath tumors that occur
in the posterior mediastinum are neurofibromas and neurilemmoma.
Mediastinal pheochromocytomas may present with hypertension and
excess secretion of catecholamines. Primary excision is recommended
for all neurogenic tumors in the early stages. If they involve the spinal
cord, a preoperative MRI and a neurosurgical consult should be
obtained.
Question 455
• Question 455: Which of the following risk factors are not linked to an
increased risk of endometrial cancer?
• Choices:
• 1. Asthenia, poor nutrition, smoking
• 2. Age over 40 years, nulliparity
• 3. Obesity, diabetes mellitus, hypertension
• 4. White race, family history of endometrial cancer
Answer: 1
• Answer: 1 - Asthenia, poor nutrition, smoking
• Explanations: Risk factors for endometrial cancer include age older than 40
years, obesity, diabetes mellitus, white race, nulliparity, hypertension, and
family history of endometrial cancer. The underlying cause of the most
common type of endometrial cancer is unopposed estrogen exposure.
Obesity is directly linked to increased circulating levels of estrogen. Diabetes
mellitus and hypertension are associated with obesity and consequently are
linked to an increased risk of endometrial cancer. Endometrial cancer
incidence increases with age. Also, because pregnancy interrupts estrogen
stimulation of the endometrium, nulliparity resulting in a relatively greater
number of ovulatory cycles in a woman's lifetime is associated with an
increased risk of endometrial cancer.
Question 456
• Question 456: The first anti-cancer drug in the vinca alkaloid class is
vincristine. In giving this drug to a patient with nephroblastoma, it is
probable that there will be an adverse reaction to the drug. Which of
the following is the most common adverse response to vincristine
treatment?
• Choices:
• 1. Myelosuppression
• 2. Peripheral neuropathy
• 3. Hypouricemia
• 4. Severe diarrhea and dehydration
Answer: 2
• Answer: 2 - Peripheral neuropathy
• Explanations: Despite a robust adverse event profile of its own,
vincristine is one of the few cytotoxic anti-neoplastic medications that
does not cause bone marrow suppression. It has been known to cause
peripheral motor and sensory neuropathy. Among its other side
effects are constipation and hyperuricemia. It also has
immunosuppressive properties, which are sometimes utilized
therapeutically.
Question 457
• Question 457: A 65-year-old male presents to the hospital complaining of shortness
of breath and bilateral leg swelling that has been progressively worsening over the
last week. He states that he was recently started on a new chemotherapy agent
one month ago for a newly diagnosed lymphoma. He has never been short of
breath before or has noticed any leg swelling and is otherwise in his usual state of
health. Which of the following agents was this patient most likely started on by his
care provider?
• Choices:
• 1. Vincristine
• 2. Doxorubicin
• 3. Cisplatin
• 4. Cyclophosphamide
Answer: 2
• Answer: 2 - Doxorubicin
• Explanations: Doxorubicin, also known as adriamycin, is known for
being cardiotoxic and a cause of congestive heart failure. The classic
symptoms of congestive heart failure exacerbation are shortness of
breath more than baseline and bilateral leg swelling or edema.
Doxorubicin is FDA approved for the treatment of leukemias and
lymphomas. Vincristine, cisplatin, and cyclophosphamide generally
are not known for causing new-onset congestive heart failure as seen
in this case.
Question 458
• Question 458: Which antibody has the most important receptors on
mast cells?
• Choices:
• 1. Immunoglobulin A (IgA)
• 2. Immunoglobulin G (IgG)
• 3. Immunoglobulin D (IgD)
• 4. Immunoglobulin E (IgE)
Answer: 4
• Answer: 4 - Immunoglobulin E (IgE)
• Explanations: Mast cells play an important role in allergic disorders.
The binding of IgE to mast cell's Fc receptors leads to complex
reactions with the release of potent mediators. Mast cells have
granules containing heparin and histamine. They are involved in
anaphylaxis and other allergic reactions. Mast cells are located in the
tissues and not the blood stream.
Question 459
• Question 459: Which of the following agents can be used to terminate
cell division and allow chromosome analysis?
• Choices:
• 1. Hydroxyurea
• 2. Bleomycin
• 3. Methotrexate
• 4. Colchicine
Answer: 2
• Answer: 2 - Bleomycin
• Explanations: Chromosomal analysis needs an agent that can arrest
mitosis during the metaphase period. Colchicine acts on tubulin
protein to arrest cell division. Methotrexate depletes folinic acid from
the cell. Bleomycin interferes with cell division in the G2 phase.
Question 460
• Question 460: A 65-year-old man comes to the office for a routine visit. He
says he is still smoking and does drink occasionally. Regarding his health, he
has no complaints except for the occasional acidity and reflux. On
examination, he has a 2x2 cm mass located on the posterior aspect of the
neck. The mass is hard and not very mobile. He denies any pain. What is the
most likely cause of the mass in his neck?
• Choices:
• 1. Thyroglossal duct cysts
• 2. Branchial cleft cyst
• 3. Parotid gland
• 4. Lymph nodes
Answer: 4
• Answer: 4 - Lymph nodes
• Explanations: Malignancies of the head and neck are on the increase. The
most common malignancies of the neck area include thyroid cancer, salivary
gland cancer, lymphomas, and sarcomas. The most common presentation for a
thyroid or salivary gland cancer is an asymptomatic nodule within the gland.
For those who smoke and drink alcohol, the risk for head and neck cancers is
high. Cancers of the oral cavity, larynx, and pharynx typically present with a
mass, which is often a lymph node. These lymph nodes are typically located
somewhere along the lateral and posterior aspects of the neck. Risk factors for
head and neck cancers include smoking, sun exposure, tobacco use, alcohol
use, poor dentition, and family history. Sometimes these individuals may also
present with a sore throat, weight loss, dysphagia, or change in voice.
Question 461
• Question 461: A 65-year-old female with a past medical history of
chronic myeloid leukemia on imatinib is being admitted for disseminated
Herpes zoster. Which transmission-based precautions should be
instituted?
• Choices:
• 1. Contact precautions
• 2. Droplet precautions
• 3. Airborne precautions
• 4. Contact and airborne precautions
.
Answer: 4
• Answer: 4 - Contact and airborne precautions
• Explanations: Patients with disseminated Herpes zoster infection
should be placed on airborne and contact precautions. Susceptible
healthcare workers should not be assigned to care for these patients
if immune healthcare workers are available. Patients with the active
infection with the same pathogen and no other infection may be
roomed together (cohorting). Airborne precautions require that the
door to the room be closed at all times, to allow for negative pressure
air changes.
Question 462
• Question 462: A patient develops congestive heart failure after
starting a chemotherapy drug. Which agent is most likely to cause this
complication?
• Choices:
• 1. Doxorubicin
• 2. Cyclophosphamide
• 3. Steroids
• 4. Vincristine
Answer: 1
• Answer: 1 - Doxorubicin
• Explanations: All anthracycline drugs gave the potential to cause
cardiac toxicity. Individuals with risk factors for heart disease must
undergo cardiac screening test before being treated with doxorubicin.
The cardiotoxicity is often irreversible and due to free radical damage.
Doxorubicin works by intercalating DNA.
Question 463
• Question 463: Which of the following is true of benign prostatic
hyperplasia and prostate carcinoma?
• Choices:
• 1. They arise from similar zones of the prostate gland
• 2. They arise from different zones of the prostate gland
• 3. They affect the population at similar prevalence
• 4. They are examples of a progressive disease from hyperplasia to
malignancy
Answer: 2
• Answer: 2 - They arise from different zones of the prostate gland
Explanations: Benign prostatic hyperplasia develops from the
periurethral glands at the site of the median or lateral lobes, whereas
the posterior lobe and peripheral zone are more prone to a cancerous
formation. Prevalence of benign prostatic hyperplasia and prostate
cancer both increase with age. Both may be found in 80% or more of
men older than the age of 80 years. Benign prostatic hyperplasia and
prostate cancer are not examples of a progressive disease from
hyperplasia to malignancy. Prostate-specific antigen (PSA) may be
elevated in both conditions but the free PSA percentage tends to be
higher in benign prostatic hyperplasia.
Question 464
• Question 464: Which of the following complications is specifically
associated with insertion of a central venous femoral catheter?
• Choices:
• 1. Central line-associated bloodstream infection
• 2. Bladder puncture
• 3. Guidewire embolism
• 4. Cardiac dysrhythmia.
Answer: 2
• Answer: 2 - Bladder puncture
• Explanations: In general, complications from central venous access
can be classified into early and late complications. A complication
specific to the femoral site is bladder puncture. Some complications
that can occur with insertion at all central venous access sites are
central line-associated bloodstream infections (CLABSIs), catheter
fragmentation resulting in a guidewire embolism, and cardiac
dysrhythmias. CLABSIs are the most common complication of central
venous catheter placement, resulting in significant morbidity and
mortality, as well as, increased healthcare costs.
Question 465
• Question 465: How often should follow up be scheduled after a
pneumonectomy for lung cancer?
• Choices:
• 1. Twice a year
• 2. Every month
• 3. Once every 12 months
• 4. Every other year
Answer: 1
• Answer: 1 - Twice a year
• Explanations: Patients must have follow-ups at least two times a year
after a pneumonectomy for lung cancer. In patients who have
undergone curative-intent surgical resection for lung cancer, it is
suggested that chest CT is performed every six months for the first
two to three years after resection. Annual follow-up is recommended
every year thereafter.
Question 466
• Question 466: What is the most common primary tumor of the liver?
• Choices:
• 1. Hepatoma
• 2. Hydatid cyst
• 3. Hamartoma
• 4. Hemangioma
Answer: 1
• Answer: 1 - Hepatoma
• Explanations: The most common primary tumor of the liver is a
hepatoma. Hepatomas tend to occur in people who have hepatitis B,
excess alcohol consumption, or those with chronic hepatitis C
infection. Hepatomas have been linked to exposure to aflatoxins and
prolonged alcohol consumption. The initial presentation is of
abdominal pain, weight loss, or jaundice.
Question 467
• Question 467: A 65-year-old woman presents with back pain and fatigue. She is found
to have a hematocrit of 30%. Her MCV is 96 fl, and RDW is normal at 13%. Her WBC is
4,400/mm3. Her platelet count is 120,000/mm3 and creatinine is 1.2 mg/dL. Iron
studies are done, which reveal a normal iron, normal TIBC, and normal saturation. Her
ferritin is normal at 150 ng/ml. Her total protein is elevated at 9.1 mg/dl, while her
albumin is 3.1 mg/dl. Her urine dipstick is positive for 1+ protein. Which of the
following tests is most likely to reveal the underlying etiology in this patient?
• Choices:
• 1. Soluble serum transferrin receptor level
• 2. Tissue transglutaminase and anti-endomysial antibodies
• 3. 24-hour urine for creatinine clearance
• 4. Serum protein electrophoresis
Answer: 4
• Answer: 4 - Serum protein electrophoresis
• Explanations: The presentation described here suggests multiple myeloma, which
should be evaluated with a serum protein electrophoresis. Diagnosis is not always
obvious, especially in older patients, given their increased comorbidities. Multiple
myeloma should always be suspected in an elderly patient with normocytic
anemia, bone pain and evidence of renal dysfunction. The patient has normal iron
levels, making the diagnosis of celiac disease unlikely. As a result, there is no use
in testing for celiac disease with tissue transglutaminase and anti-endomysial
antibodies. This patient has anemia of chronic disease, and this testing for serum
transferrin receptor will be elevated, but will not help in diagnosis. A serum
protein electrophoresis will be helpful in diagnosing plasma cell dyscrasia, such as
multiple myeloma with the presence of M spike.
Question 468
• Question 468: Which is true regarding liver cancer?
• Choices:
• 1. More common in females than males
• 2. Hepatocellular carcinoma (HCC) is the most common primary hepatic
tumor in adults
• 3. People of Asia and Africa have a decreased incidence of liver cancer
compared to the USA and Europe
• 4. Women of childbearing age on contraceptives have an increased risk
of hepatocellular carcinoma (HCC)
.
Answer: 2
• Answer: 2 - Hepatocellular carcinoma (HCC) is the most common
primary hepatic tumor in adults
• Explanations: Seen more commonly in males. Peoples of Asia and
Africa have increased incidence of HCC, occurring at younger ages
than USA counterparts associated with increased hepatitis B and C.
Women on contraceptives have an increased risk of hepatocellular
adenoma.
Question 469
• Question 469: What phase of the cell cycle is most resistant to
chemotherapeutic agents?
• Choices:
• 1. G1 phase
• 2. G0 phase
• 3. S phase
• 4. M phase
Answer: 2
• Answer: 2 - G0 phase
• Explanations: Most chemotherapy drugs act on actively dividing cells.
G0 phase is a dormant phase and cells are most resistant to
chemotherapy agents. G1 phase is a time for cell growth and
chemotherapy drugs can easily hamper this phase. S phase is the DNA
synthesis phase and can be inhibited by several drugs.
Question 470
• Question 470: To what location does breast cancer most commonly
metastasize?
• Choices:
• 1. Bone
• 2. Lung
• 3. Liver
• 4. Brain
Answer: 2
• Answer: 2 - Lung
• Explanations: Upon autopsy, the most common site for breast cancer
metastasis is the lung, followed by bone. The risk of metastasis
increases with the presence of positive lymph nodes and with a large
primary tumor. To date, there are no markers to detect metastases
from breast cancer. In general, estrogen receptor-positive breast
tumors have a predilection to metastasize to bone, whereas invasive
lobular cancers spread to the gastrointestinal tract and ovaries.
Question 471
• Question 471: A patient who is allergic to tartrazine is to begin
treatment for lung cancer with the drug uracil mustard. Which of the
following is most appropriate regarding the administration of this
medication?
• Choices:
• 1. Chart the allergy and give the uracil mustard
• 2. Withhold the uracil mustard and notify the physician of the allergy
• 3. Administer a test dose of uracil mustard
• 4. Give the uracil mustard as prescribed
Answer: 2
• Answer: 2 - Withhold the uracil mustard and notify the physician of
the allergy
• Explanations: The uracil mustard should be withheld and the
physician notified if the client is allergic to tartrazine (a dye found in
drug capsules). Uracil mustard is an alkylating agent, a derivative of
nitrogen mustard used in treating certain types of tumors. A test
dosage should not be given since you already know the patient is
allergic to the dye (tartrazine) found in the capsules. The uracil
mustard should not be given since the patient is allergic to the
tartrazine (dye found in the capsules).
Question 472
• Question 472: Which of the following women is least likely to develop
a second primary breast cancer?
• Choices:
• 1. Premenopausal women
• 2. Women with initial diagnosis of lobular carcinoma in situ
• 3. Women over the age of 80
• 4. Women with a BRCA gene

•.
Answer: 3
• Answer: 3 - Women over the age of 80
• Explanations: Any patient can develop a second primary cancer.
However, elderly patients may not live long enough to develop a
second primary cancer. Women with lobular carcinoma in situ can
develop cancer in the other breast in 30% of cases. The younger the
female, the higher the chance of developing a second cancer.
Question 473
• Question 473: An adult presents with painless jaundice and nausea.
She has lost 10 pound in 3 months. CT scan reveals a 2.5 cm solid
mass on the head of the pancreas. Which of the following treatment
options provides the best chance for long-term survival?
• Choices:
• 1. Pancreaticoduodenectomy
• 2. Gastrojejunostomy
• 3. Stenting of bile duct
• 4. Total pancreatectomy
Answer: 1
• Answer: 1 - Pancreaticoduodenectomy
• Explanations: The Whipple procedure or pancreaticoduodenectomy is a major
surgical operation, which involves removal of the head of pancreas, duodenum,
and stomach. Pylorus preserving pancreaticoduodenectomy is now gaining
popularity as it allows for normal gastric emptying. Mortality from surgery
approaches 100% at centers that do less than five of these operations a year. At
good centers, the mortality varies from 5-15 percent. Pancreaticoduodenectomy is
the only curative option for pancreatic cancer. Once distant metastases to the liver
and regional lymph nodes are ruled out by imaging, endoscopic ultrasound is
typically utilized to verify respectability. If a borderline resectable lesion is
discovered, neoadjuvant chemotherapy prior to pancreaticoduodenectomy is
appropriate and provides a higher rate of margin negative, node-negative
resection.
Question 474
• Question 474: A postmenopausal 68 year-old female, gravida 2, para 2. presents to the emergency
department with the passage of clots, heavy vaginal bleeding, and feeling pre-syncopal. Her blood
pressure was 86/55 mmHg, and pulse was 123 beats per minute. Her hemoglobin was noted to be
7.6 g/dl. She received 1 unit of packed red blood cells with symptomatic improvement. Further
evaluation reveals she underwent menarche at the age of 11 and menopause at the age of 60. She
is obese, but otherwise, her past medical, past surgical, family and social history are
noncontributory. Pelvic examination was concerning for blood coming from the cervical os without
any other masses or abnormalities. A pelvic ultrasound demonstrated at 7mm heterogenous
endometrial stripe. What is the most common type of uterine cancer in a woman of this age-group?
• Choices:
• 1. Serous carcinoma
• 2. Papillary serous carcinoma
• 3. Low-grade endometrioid carcinoma
• 4. Clear cell carcinoma
Answer: 3
• Answer: 3 - Low-grade endometrioid carcinoma
• Explanations: Endometrioid carcinomas compose approximately 83% of
uterine cancers. Most commonly, endometrioid cancers are low grade. More
virulent serous and papillary serous carcinomas compose 4-6% of
endometrial carcinomas and are considered high grade. High-grade
carcinomas necessitate pelvic and para-aortic lymph node dissection as well
as omentectomy. Clear cell carcinoma is another high-grade endometrial
carcinoma, and it composes approximately 1-2% of endometrial cancers. It
is essential to differentiate between type 1 and type 2 endometrial cancers
for the management and prevention of these cancers. Low-grade
endometrioid cancer is considered type 1 and is the most common type of
endometrial cancer.
Question 475
• Question 475: A 52-year-old male presents to his primary care provider for his first
routine colonoscopy. He has been avoiding the procedure due to fear of
discomfort. He also expresses concerns about polyps. A friend of his had a
colonoscopy a few years prior, and his physician found a polyp. His friend
subsequently underwent multiple procedures and medical treatments. He asks
what kind of polyp could potentially be cancerous. What type of polyp has the
highest risk of malignant transformation?
• Choices:
• 1. Tubular adenoma
• 2. Adenomatous polyp
• 3. Juvenile polyp
• 4. Villous adenoma
Answer: 4
• Answer: 4 - Villous adenoma
• Explanations: Adenomas are either tubular (having a branched glandular
structure), villous (having elongated patterns of abnormal glandular tissue),
or tubulovillous (having a mixture of both). Villous adenomas are most
common in the rectal area, are commonly faster growing, and have the
highest risk of malignant transformation. Thus, these should be followed up
with repeat colonoscopy within 3 years. They are more likely to be sessile,
velvety, or have the appearance of cauliflower. The risk of malignancy is
greatest for polyps greater than 2 cm in diameter. Villous has the highest
risk of becoming malignant, while tubulovillous has the second highest risk
of becoming malignant, and tubular has the lowest risk.
Question 476
• Question 476: Which of the following statements about adjuvant
chemotherapy for early stage breast cancer is the most accurate?
• Choices:
• 1. Only younger females benefit from adjuvant therapy
• 2. Adjuvant therapy is of benefit to all patients, irrespective of node
status
• 3. Adjuvant therapy is not given to patients who are positive for the
HER/NEU2 antigen
• 4. Adjuvant therapy significantly improves survival
Answer: 2
• Answer: 2 - Adjuvant therapy is of benefit to all patients, irrespective
of node status
• Explanations: Most clinical trials have shown that the benefit of
adjuvant therapy is modest in women with breast cancer. Only about
a 20% reduction in death has been shown. The benefits of adjuvant
therapy extend to women of all ages. Moreover, there are benefits for
both node-positive and node-negative women. The benefits of
adjuvant therapy do not depend on the presence of estrogen receptor
or on HER/NEU 2 positivity. Overall, adjuvant therapy is given to many
patients, but only 4% to 10% show any benefit.
Question 477
• Question 477: What is the most common site for bladder cancer
metastases?
• Choices:
• 1. Liver
• 2. Lungs
• 3. Lymph nodes
• 4. Bone
Answer: 3
• Answer: 3 - Lymph nodes
• Explanations: At the time of a bladder cancer diagnosis, more than
25% of patients have lymph node involvement. Other common sites
include bone, lung, liver, and peritoneum. Bladder cancer is the most
common malignancy of the urinary tract. It is nearly three times more
common in men than women. The median age at diagnosis is 65 years
of age.
Question 478
• Question 478: A 64-year-old patient visits his physician for an enlarging
mole on his shoulder that has gotten larger over the years, has lost its
symmetry and shows irregular borders. The physician suspects a
dermatologic malignancy and refers the patient to a specialist. Which of
the following is the most likely embryologic origin of the malignant cell?
• Choices:
• 1. Surface ectoderm
• 2. Neural crest
• 3. Mesoderm
• 4. Neural tube
Answer: 2
• Answer: 2 - Neural crest
• Explanations: The ectoderm undergoes gastrulation to form the
neural crest, neural tube, and epidermis. Melanocytes are not
derived from surface ectoderm, mesoderm, or neural tube.
Melanocytes are derived from the neural crest of the ectoderm. They
are responsible for skin pigmentation. They can become cancerous
and malignant as melanomas, and are commonly identified by the
ABCDEs (asymmetry, irregular borders, not a uniform color, diameter
greater than 6mm, evolving). The primary cause of melanoma is
exposure to UV light rays, particularly in those individuals with low
levels of skin pigment.
Question 479
• Question 479: A patient with a lymphoma is being treated with combination
chemotherapy. After a month, the patient is found to have tachycardia and
dyspnea on exertion. What is the most likely agent responsible?
• Choices:
• 1. Doxorubicin
• 2. Cyclophosphamide
• 3. Bleomycin
• 4. Methotrexate

•.
Answer: 1
• Answer: 1 - Doxorubicin
• Explanations: The 1st sign of a high resting pulse rate and dyspnea
should be cause for concern. Cardiotoxicity due to doxorubicin can be
permanent and fatal if not recognized. The overall toxicity depends on
cumulative dosage. All patients with cardiac disease must be cleared
before being started on anthracycline agents.
Question 480
• Question 480: Which of the following is a risk factor for developing
cervical cancer?
• Choices:
• 1. Human papillomavirus (HPV) infection
• 2. Monogamy
• 3. Late age of first intercourse
• 4. Chlamydia infection
Answer: 1
• Answer: 1 - Human papillomavirus (HPV) infection
• Explanations: Risk factors for cervical cancer include early age of first
intercourse, multiple sex partners, smoking, and infection with human
papillomavirus (HPV). A proper history and physical exam are
imperative in helping detect and diagnose cervical cancer. Cervical
cancer often is asymptomatic. Pap smears are used to screen cervical
cancer screening.
Question 481
• Question 481: A 27-year-old female patient presented with a six-month history of
bloodstained discharge. Anamnesis established unequivocally that the patient's mother
received drugs throughout the pregnancy. Physical examination revealed a palpable pelvic
mass that was about the size of a gravid uterus of 12 weeks, with visible tissue protruding
through the introitus. Magnetic resonance imaging showed a huge mass arising from the
cervix of a rudimentary uterus and pushing the pelvic organs toward the abdomen.
Histopathological examination of the biopsy specimen established the diagnosis of clear cell
adenocarcinoma. Which of the following drugs is associated with clear cell cervical cancer?
• Choices:
• 1. Diethylstilbestrol
• 2. Clomiphene
• 3. Tamoxifen
• 4. Progesterone
Answer: 1
• Answer: 1 – Diethylstilbestrol
• Explanations: Diethylstilbestrol is an estrogen derivative that has
been linked to clear cell cervical or vaginal adenocarcinoma.
Diethylstilbestrol caused cancers in daughters of women who took the
drug during pregnancy. Daughters need to have a pap smear and a
pelvic exam every year because the risk of cancer is lifelong. Clear
cells appear clear because they are rich in glycogen. Today,
diethylstilbestrol is not used.
Question 482
• Question 482: What is the most common sign of carcinomatous
meningitis?
• Choices:
• 1. Asymmetric reflexes
• 2. Neck stiffness
• 3. Low back pain
• 4. Muscle weakness
Answer: 1
• Answer: 1 - Asymmetric reflexes
• Explanations: Asymmetry reflex is the most common sign of
carcinomatous meningitis. It occurs in about 70 percent of the
patients. Neck stiffness is the least common of all. Malignancy from
the breast, lung, melanoma or even a primary brain cancer may cause
carcinomatous meningitis. The median survival is only a few weeks at
best.
Question 483
• Question 483: Which is true of follicular thyroid cancer?
• Choices:
• 1. It is more common in males
• 2. It is most common in the second or third decade of life
• 3. More likely to have metastases than papillary cancer
• 4. Needle biopsy is an excellent method for diagnosis
Answer: 3
• Answer: 3 - More likely to have metastases than papillary cancer
Explanations: Patients with follicular cancer are more likely to have
lung and bone metastases. The bone metastases are usually
osteolytic. Follicular cancer is more common in females of all ages,
and there is a weak association with body mass index. Fine-needle
aspiration results should be evaluated with caution. False negatives
and false positives occur at about 6% to10% of cases. It is sometimes
difficult to differentiate follicular cancer from follicular adenoma
based on cytology. A thyroid biopsy is recommended when a follicular
cancer is suspected. The presence of colloid rich follicles lined by flat
follicular cells is often seen.
Question 484
• Question 484: Which of the following vaccines is contraindicated for
patients undergoing chemotherapy?
• Choices:
• 1. MMR vaccine
• 2. Pneumococcal vaccine
• 3. Hepatitis B vaccine
• 4. DTP vaccine
Answer: 1
• Answer: 1 - MMR vaccine Explanations: Patients on chemotherapy
have a compromised immunity and need immunization. Live
attenuated vaccines such as MMR are contraindicated.
Pneumococcal, hepatitis B, and influenza vaccines are indicated.
Opportunistic infections are suspected if fever continues to be high.
Question 485
• Question 485: Why might a patient be given allopurinol along with a
course of chemotherapy?
• Choices:
• 1. Protects against hepatotoxicity
• 2. Lessens myelosuppressive side effects
• 3. Counter possible hyperuricemia that can accompany a major cell
death burden
• 4. Protects host cells against chemotherapy toxicity
Answer: 3
• Answer: 3 - Counter possible hyperuricemia that can accompany a
major cell death burden
• Explanations: Hyperuricemia is associated with chemotherapy
regimens as an outcome of massive cell death. This cell death can
result in cellular purine degradation which leads to the formation of
uric acid. This purine conversion is facilitated by xanthine oxidase, the
enzyme inhibited by allopurinol. Hyperuricemia can lead to renal
damage or gout.
Question 486
• Question 486: T-helper cells are the cause of which neoplasm?
• Choices:
• 1. Intraepidermal carcinoma
• 2. Merkel cell carcinoma
• 3. Mycosis fungoides
• 4. Letterer-Siwe disease
Answer: 3
• Answer: 3 - Mycosis fungoides
• Explanations: Mycosis fungoides is a lymphoma of T-helper cells that
starts on the skin. It often has the appearance of eczema or psoriasis.
Intraepidermal carcinoma of the penis is squamous cell carcinoma in
situ. Letterer-Siwe disease has a proliferation of Langerhans cells
associated with histiocytosis X.
Question 487
• Question 487: Which of the following is the most common genetic
abnormality associated with Burkitt lymphoma?
• Choices:
• 1. Trisomy 13
• 2. Defect on chromosome 4
• 3. Translocation between chromosomes 9 and 22
• 4. Translocation between chromosomes 8 and 14
Answer: 4
• Answer: 4 - Translocation between chromosomes 8 and 14
Explanations: Trisomy 13 (Patau syndrome) is associated with
mutations in AML1/RUNX1 and increased expression of FLT3. Thus,
individuals with Patau syndrome are at increased risk of developing
acute myeloid leukemia (AML). Huntington chorea is linked to a
defect on chromosome 4. Chronic myeloid leukemia is related to a
translocation between chromosomes 9 and 22 (Philadelphia
chromosome). Burkitt lymphoma is associated with a translocation
between chromosomes 8 and 14.
Question 488
• Question 488: What is the most common cause of metastasis causing
epidural spinal cord compression?
• Choices:
• 1. Prostate
• 2. Lung
• 3. Lymphoma
• 4. Breast
Answer: 4
• Answer: 4 - Breast
• Explanations: The most frequent origin of metastasis resulting in
epidural spinal cord compression is the breast. This causes about a
fourth of epidural metastasis. Other origins include prostate, lung,
and malignant lymphoma. Each result in about 10% to 15% of cases of
epidural spinal cord compression.
Question 489
• Question 489: Which of the following autosomal dominant disorders
is associated with acoustic neuromas and pheochromocytomas?
• Choices:
• 1. Neurofibromatosis type 2
• 2. Gardner syndrome
• 3. Xeroderma pigmentosa
• 4. Gorlin syndrome
Answer: 1
• Answer: 1 - Neurofibromatosis type 2 Explanations:
Neurofibromatosis type 2 is a genetic disorder consisting of multiple
nerve tumors. Some nerve tumors are harmless, but others grow
large and compress the nerves. In neurofibromatosis type 2,
individuals may develop acoustic neuromas by 20 years of age.
Multiple schwannomas may also develop. Pheochromocytomas are
found with an increased frequency in these individuals.
Question 490
• Question 490: A 60-year-old female undergoes a hysterectomy, salpingo-oophorectomy,
pelvic and aortic lymph node dissection and omentectomy for a malignant uterine
neoplasm. The pathology report demonstrates a primary uterine carcinosarcoma
(formerly known as a malignant mixed mesodermal tumor). Metastatic disease is noted in
several lymph nodes and the omentum. Given the high-grade, metastatic, endometrial
carcinoma, she underwent postsurgical adjuvant therapy with external-beam radiation to
the entire pelvis. Which of the pelvic tissue is the most radiosensitive?
• Choices:
• 1. Ovarian tissue
• 2. Vaginal tissue
• 3. Rectal tissue
• 4. Bladder tissue
Answer: 1
• Answer: 1 - Ovarian tissue
• Explanations: Of the pelvic tissues listed, ovarian tissue is the most
radiosensitive at 2500 rads. The other structures can tolerate 5,000 to
20,000 rads. In patients who are so treated, moderate to severe
intestinal complications are common and last for more than three
months. Patients should be counseled that radiation therapy will likely
result in prompt cessation of ovarian function. Premenopausal
women should be made aware of the likelihood of menopausal
symptoms during and after treatment.
Question 491
• Question 491: Which of the following breast cancers has a tendency
to occur in both breasts?
• Choices:
• 1. Intraductal
• 2. Inflammatory
• 3. Lobular
• 4. Paget
Answer: 3
• Answer: 3 – Lobular
• Explanations: Lobular invasive cancer accounts for about 10% of all
breast cancers. Studies show that there is an increased risk of breast
cancer in the other breast. Treatment options depend on bilateral
mastectomy, tamoxifen, or close follow-up if there is a carcinoma on
site. Lobular cancers tend to be positive for estrogen and
progesterone receptors.
Question 492
• Question 492: After colorectal cancer, what is the next most common
cancer in patients with Lynch syndrome?
• Choices:
• 1. Gastric
• 2. Endometrial
• 3. Ovarian
• 4. Transitional cell
Answer: 2
• Answer: 2 - Endometrial
• Explanations: After colorectal cancer, patients with Lynch syndrome
also are prone to endometrial cancer. At least 40% of these patients
develop endometrial cancer by age 70. Colorectal cancer in patients
with Lynch syndrome occurs at an early age and is also characterized
by an increased risk of endometrial cancer. Other cancers that occur
include stomach, ovary, small bowel, pancreas, prostate, skin and liver
malignancies, In addition, in patients with lynch syndrome one may
also find synchronous colorectal tumors (primary cancer diagnosed
within 6 mo of each other).
Question 493
• Question 493: Which lung cancer commonly is associated with
hypercalcemia?
• Choices:
• 1. Small cell
• 2. Squamous cell
• 3. Adenocarcinoma
• 4. Carcinoid
Answer: 2
• Answer: 2 - Squamous cell
• Explanations: Humoral hypercalcemia of malignancy usually is due to
the production of a parathyroid-like hormone protein by the tumor.
Hypercalcemia most commonly is associated with squamous cell
carcinoma. Small cell cancer generally produces SIADH.
Adenocarcinoma does not cause the release of paraneoplastic factors.
Carcinoid secretes serotonin that causes carcinoid syndrome.
Question 494
• Question 494: A 66-year-old woman is in the middle of chemotherapy
treatments for an aggressive gastric cancer. She has a BMI of 17 kg/m2.
Which of the following is the most appropriate indication to start a full
liquid diet in this patient?
• Choices:
• 1. To ensure the patient does not have clear-colored bowel movements
• 2. To ensure the patient has a faster recovery time
• 3. To reduce the mortality rate
• 4. To ensure the patient can have their diet supplemented with immune-
enhanced nutritional support
Answer: 4
• Answer: 4 - To ensure the patient can have their diet supplemented with
immune-enhanced nutritional support
• Explanations: One added benefit of a full liquid diet is the higher amount of
calories and the added amount of nutrients provided. Bozzetti et al. showed
that the greatest post-operative complications in patients with gastric cancer
included weight loss, advanced age, earlier surgery involving the pancreas,
and lower serum albumin. Increasing amounts of nutritional support reduced
postoperative complications. The best form of nutritional support for those
patients was immune-enhanced, but results showed an increased protective
effect against complications with an increase in nutritional support. Full liquid
diets have been linked to fewer post-operative complications, NOT to faster
recovery time or mortality rate reduction.
Question 495
• Question 495: Which of the following precludes surgery in a patient
with a non-small cell cancer?
• Choices:
• 1. Pulmonary hyper osteoarthropathy
• 2. Hypercalcemia
• 3. Chest wall invasion
• 4. Cytology positive pleural effusion
Answer: 4
• Answer: 4 - Cytology positive pleural effusion
• Explanations: Patients with pleural effusion must undergo a thorough
evaluation to rule out a metastatic disease. Besides cytology of the
pleura, a thoracoscopy may be warranted. If the pleural fluid is
negative and the pleura has no masses, the patient is resectable. Any
pleural masses during video-assisted thoracoscopic surgery, upgrades
to stage 4, which is not a surgical disease.
Question 496
• Question 496: Widespread cancer was diagnosed in a 48-year-old
Asian immigrant. Serologic studies revealed a high level of alpha-
fetoprotein. What is the primary carcinoma diagnosis that favors this
laboratory finding?
• Choices:
• 1. Breast
• 2. Colon
• 3. Lung
• 4. Liver
Answer: 4
• Answer: 4 – Liver
• Explanations: Alpha-fetoprotein (AFP) is elevated in about 70% of
those with hepatocellular carcinoma. It also is elevated when a
metastatic disease affects the liver. Patients with cirrhosis and chronic
hepatitis B or C are followed with AFP for early detection of
hepatocellular carcinoma. Elevation of AFP should prompt radiologic
imaging.
Question 497
• Question 497: What is the best test to initially stage stomach cancer?
• Choices:
• 1. Endoscopy
• 2. Abdominal CT (neutral oral contrast agent plus IV contrast)
• 3. Endoscopic ultrasound
• 4. Double contrast upper GI series
Answer: 3
• Answer: 3 - Endoscopic ultrasound
• Explanations: Gastric cancer staging is based on the T,M,N system, with T
staging based on degree of wall involvement. T staging is best assessed with
endoscopic US. Endoscopy alone (without biopsy) cannot assess level of tumor
invasion (through lamina propria, through muscularis propria, through serosa).
Nodal and Metastases staging is best done with PET/CT. PET increases
sensitivity in detection of involved but not enlarged lymph nodes while also
increasing specificity in evaluation of enlarged but not involved lymph nodes. It
also An optimally performed double contrast UGI (which is seldom performed
any longer in the USA but still available in Japan, where gastric cancer is more
common and screening exams desired) can detect, non-invasively, even
minimal superficial gastric cancers, but does not play a role in staging.
Question 498
• Question 498: Which is true of elevation of liver function tests in
patients on total parental nutrition?
• Choices:
• 1. Usually not associated with any abnormalities
• 2. Is only seen in pregnancy
• 3. Is only seen in liver failure
• 4. Is only seen in patients with alcohol use disorder
Answer: 1
• Answer: 1 - Usually not associated with any abnormalities
Explanations: Serum hepatic enzyme concentrations are poor markers
of TPN and are often associated with fatty liver accumulation and liver
disease. Hepatic fat accumulation can occur without significant
changes in hepatic enzyme levels. Hepatic dysfunction can be
manifested by abnormal liver enzyme values, the most common way
to identify TPN-associated liver disease in the clinical setting for
further workup. Hepatic dysfunction, manifested by abnormal liver
enzyme values and intrahepatic cholestasis, is common in children
and adults receiving long-term TPN.
Question 499
• Question 499: A patient is receiving the myelosuppressive drug
etoposide phosphate. When does the patient's greatest risk for
infection occur?
• Choices:
• 1. The first week of administration
• 2. Seven to 14 days following administration
• 3. Fourteen to 28 days following administration
• 4. Six to 8 weeks following administration
Answer: 2
• Answer: 2 - Seven to 14 days following administration
• Explanations: The greatest risk of infection will occur within 7 to 14
days following the administration of the myelosuppressive drug
etoposide phosphate. Myelosuppressants inhibit bone marrow
function, which places patients receiving these agents at risk for
infection. Etoposide is a topoisomerase inhibitor and alters DNA. It is
used to treat many solid cancers. The drug is a derivative of
podophyllotoxin that was once widely used to treat warts.
Question 500
• Question 500: A 65-year-old female patient with no past family history of breast
carcinoma, presented with a lump in the left breast for three months. Physical
examination showed a well-circumscribed, ulcerative mass, in the right upper outer
quadrant of the left breast measuring 5×5 cm and was associated with foul smelling
bloody discharge. A left axillary lymph node was palpable measuring 3×2 cm.
Imaging studies did not disclose distant metastases. What is the most accurate
TNM stage of this patient?
• Choices:
• 1. Stage IIB
• 2. Stage IIIA
• 3. Stage IIIB
• 4. Stage IIIC
Answer: 3
• Answer: 3 - Stage IIIB
• Explanations: T4b: in case of cutaneous ulceration, we classify the
breast carcinoma as T4b. N1: Since an ipsilateral axillary lymph node
was palpable measuring 3x2 cm. M0: Imaging studies did not disclose
distant metastases. Overall, we classify this tumor as T4bN1M0, which
corresponds to stage IIIB. Stage IIIB breast carcinomas correspond to
T4, N0 or N1 or N2 and M0.
Question 501
• Question 501: Which medication is used to treat metastatic colon
cancer?
• Choices:
• 1. Adalimumab
• 2. Abciximab
• 3. Bevacizumab
• 4. Muromonab
Answer: 3
• Answer: 3 - Bevacizumab
• Explanations: Bevacizumab is a recombinant, humanized monoclonal
antibody. It binds to vascular endothelial growth factor inactivating it.
This inhibits microvascular growth retarding the growth of all tissues.
It is indicated for metastatic colorectal cancer, metastatic renal cancer,
glioblastoma, and advanced nonsquamous, non-small cell lung cancer.
It was indicated for breast cancer but later it was revoked as it proved
not to extend life or delay tumor growth.
Question 502
• Question 502: During the examination of a patient with a neck mass,
you ask the patient to swallow. What is the primary purpose for this
maneuver?
• Choices:
• 1. To determine if the mass is connected to the blood vessels
• 2. To determine if the mass is connected to the foramen cecum
• 3. To determine if the mass is the thyroid
• 4. To determine if the mass is a cancer
Answer: 3
• Answer: 3 - To determine if the mass is the thyroid
• Explanations: It is difficult to demarcate the borders of the thyroid
gland because it does move position in relation to swallowing. When
a mass moves with swallowing, you can be sure it is associated with
the thyroid. The pretracheal fascia runs on both sides of the lateral
neck and encloses the thyroid gland. Other masses in the neck area
do not move with swallowing. In any case, an ultrasound should be
ordered to confirm any thyroid pathology.
Question 503
• Question 503: A 65-year-old man presents to the clinic with a recent
onset of urinary hesitation. Upon physical exam, his prostate feels
hard and nodular. A PSA sample is taken and is 13 ng/mL (reference
range: <4 ng/mL). Which of the following factors is the most
important factor in determining the prognosis of his condition?
Choices:
• 1. Evidence of nodal metastasis
• 2. Fever
• 3. Histologic features
• 4. Elevated PSA
Answer: 3
• Answer: 3 - Histologic features
• Explanations: This patient likely has prostate cancer given his physical
exam and elevated PSA. The Gleason score is one of the most
important prognostic determinants of prostate cancer and is largely
determined by histologic features. Radiologic evidence of metastasis
is important in the staging of prostate cancer. Tumor staging is
important; however, the histologic features are more important. Fever
is a feature of prostatitis. Prostatitis presents with a "boggy prostate."
Question 504
• Question 504: For superficial bladder cancer, what is the
chemotherapeutic agent of choice?
• Choices:
• 1. 6-Mercaptopurine
• 2. Allopurinol
• 3. Bacillus Calmette-Guerin vaccine
• 4. Methotrexate
Answer: 3
• Answer: 3 - Bacillus Calmette-Guerin vaccine Explanations: Bacillus
Calmette-Guerin (BCG) causes an immune response and destroys
transitional cancer cells in the collecting system and bladder. BCG
vaccine is indicated for carcinoma in situ (CIS) of the bladder and
papillary cancer after transurethral resection of the prostate.
Immunotherapy with BCG is very effective for stage T1 and CIS
tumors. The intravesical instillation of BCG vaccine is undertaken
about 3 to 4 weeks following endoscopic resection of all visible
papillary tumors or bladder biopsies.
Question 505
• Question 505: A patient is receiving high dosages of methotrexate to
treat osteosarcoma. The drug leucovorin (folinic acid) has also been
prescribed. What is the purpose of leucovorin in this situation?
Choices:
• 1. To facilitate the action of methotrexate
• 2. To destroy malignant cells
• 3. To serve as a palliative treatment
• 4. To minimize methotrexate toxicity
Answer: 4
• Answer: 4 - To minimize methotrexate toxicity
• Explanations: The drug leucovorin is given to minimize methotrexate toxicity,
such as bone marrow suppression and renal damage. When leucovorin is
administered to minimize the toxic effects of methotrexate, the treatment is
referred to as the leucovorin rescue. Methotrexate is an antimetabolite
antineoplastic drug. It exerts its action by interfering with the formation of
the reduced, or active, form of folic acid in the body. Leucovorin (folinic acid)
is not an antineoplastic drug and will not facilitate the action of
methotrexate. Leucovorin (folinic acid) is the calcium salt of folic acid that is
used to antagonize the effects of methotrexate. It does not destroy cancer
cells. Leucovorin (folinic acid) is not used in the palliative treatment of
osteosarcoma.
Question 506
• Question 506: Which of the following would provide the best
assessment of a Pancoast tumor?
• Choices:
• 1. Frontal upright chest x-ray
• 2. High-resolution CT with contrast
• 3. MRI with contrast
• 4. Bone scintigraphy
Answer: 3
• Answer: 3 - MRI with contrast
• Explanations: Pancoast tumor refers to an apical lung cancer invading lower cord
of brachial plexus and sympathetic chain. Invasion of neurovascular bundle is best
demonstrated with MRI. Coronal and sagittal planes are most optimal to visualize
brachial plexus and fat around it, subclavian vein and artery, neural foramina, and
osseous structures at the level of the tumor. Chest x-ray often fails to
demonstrate early tumor due to significant overlap between normal structures in
apical region. In more advanced stages, it does not provide detailed assessment
of neoplastic infiltration either, beyond destroyed osseous structures. Standard
CT chest with contrast is used for detection of enlarged intrathoracic lymph nodes
in Pancoast tumor. Nodal involvement however is best assessed with PET/CT.
Bone scintigraphy has no advantage over MRI for bony involvement.
Question 507
• Question 507: Which of the following gene mutations is implicated in
familial or hereditary breast cancer?
• Choices:
• 1. RET proto-oncogene
• 2. RAS gene
• 3. BRCA gene
• 4. P53 gene
Answer: 3
• Answer: 3 - BRCA gene
• Explanations: BRCA gene mutations are found in most familial breast
cancer cases. The RET proto-oncogene is highly associated with
medullary cancer in the thyroid. RAS and P53 gene mutations are
implicated in colon cancer. The two commonly described BRCA
phenotypes are BRCA 1 and BRCA 2. Both have a strongly increased
likelihood of early breast cancer development, such that breast
cancer MRI screening is appropriate.
Question 508
• Question 508: What is the purpose of administering mesna with
cyclophosphamide?
• Choices:
• 1. Augments the action of cyclophosphamide
• 2. Prevents a complication of cyclophosphamide
• 3. Reduces rapid clearance from the body
• 4. Changes the drug into the active form
Answer: 2
• Answer: 2 - Prevents a complication of cyclophosphamide
Explanations: Cyclophosphamide induced hemorrhagic cystitis is
common. Mesna prevents this complication. Forced diuresis is also
helpful. Cyclophosphamide can also cause bladder cancer.
Question 509
• Question 509: A 65-year-old woman presents to the office with a recent diagnosis of right-
sided 1.2 cm invasive ductal carcinoma, which is grade 2, estrogen, and progesterone-
receptor positive, and HER2neu receptor-negative. She mentions that she has been doing
some reading and is strongly considering a mastectomy. Which of the following best
explains the difference in outcomes between breast-conserving surgery and mastectomy
to this patient?
• Choices:
• 1. Both require adjuvant radiation but vary in the necessity for chemotherapy
• 2. A positive margin after breast-conserving surgery necessitates a mastectomy
• 3. Local recurrence rates are similar when comparing breast-conserving surgery and
adjuvant radiation to simple simple mastectomy
• 4. Overall survival is similar when comparing breast-conserving surgery and adjuvant
radiation to simple mastectomy
Answer: 4
• Answer: 4 - Overall survival is similar when comparing breast-conserving surgery and
adjuvant radiation to simple mastectomy
• Explanations: Overall survival is similar when comparing breast-conserving surgery
and adjuvant radiation to simple mastectomy. Recurrence rates, however, are higher
in breast-conserving surgery. Recurrence rates are higher in breast-conserving surgery
when compared to mastectomy. This is why adjuvant radiation is often required when
undergoing breast-conserving surgery; as without adjuvant radiation, recurrence rates
are unacceptably high. This is why adjuvant radiation is often required when
undergoing breast-conserving surgery; as without adjuvant radiation, recurrence rates
are unacceptably high. Often, mastectomy does not require adjuvant radiation,
particularly in early-stage tumors, such as in this patient. A positive margin after
breast-conserving surgery does not necessitate a mastectomy. Patients may undergo
re-excisional lumpectomy if breast size and margin status are amenable.
Question 510
• Question 510: A 65-year-old smoker is seen because of an ulcerating
mass on his tongue. Biopsy establishes that this is cancer but he
declines surgical intervention. Which of the following statements
regarding cancer of the tongue is TRUE?
• Choices:
• 1. Carcinomas of the tongue are best treated by irradiation alone
• 2. Metastases to the preauricular lymph nodes is common
• 3. Cancer of the tongue most commonly occurs near the frenulum
• 4. Local invasion may result in hypoglossal or lingual nerve dysfunction
Answer: 4
• Answer: 4 - Local invasion may result in hypoglossal or lingual nerve
dysfunction
• Explanations: Cancers of the tongue commonly present on the lateral or
ventral surface of the tongue. Metastases to the submandibular or cervical
nodes are common. Local invasion may result in tongue deviation due to
involvement of the hypoglossal nerve or decreased sensation due to
involvement of the lingual nerve. Treatment is surgical wide local excision of
the tumor and neck dissection as needed. In patients without nodal
metastases, elective neck dissection is indicated for more invasive tumors. In
contrast to therapeutic nodal dissections, which are performed for clinically
evident lymph node metastases, elective nodal dissections are performed
for staging purposes.
Question 511
• Question 511: In which of the following tumors is human chorionic
gonadotrophin levels NOT elevated?
• Choices:
• 1. Seminoma
• 2. Choriocarcinoma
• 3. Islet cell tumors
• 4. Thymoma
Answer: 4
• Answer: 4 - Thymoma
• Explanations: The beta subunit of human chorionic gonadotrophin is
secreted by several malignancies. Malignancies with elevated levels of
HCG include seminoma, choriocarcinoma, and hydatidiform mole.
Some islet cell tumors also secrete a variety of hormones of which
one of them is HCG. Thymomas do not secrete HCG but do secrete a
variety of other peptides.
Question 512
• Question 512: A 65-year-old woman was recently diagnosed with stage 4 breast cancer.
After discussion with her family and oncology team, they decide to pursue an aggressive
chemotherapy regimen. The patient was coordinated to meet with a palliative care
provider in order to closely monitor her symptoms and discuss potential medication
regimens for symptomatic relief. The palliative provider notes that the patient is
scheduled to begin a number of chemotherapeutic agents, including cisplatin. What is
the most dangerous well-documented side effect of the class of medication she is likely
to be prescribed as a result of this regimen?
• Choices:
• 1. Dyspnea on exertion
• 2. QT prolongation
• 3. Anorexia
• 4. Refractory headache
Answer: 2
• Answer: 2 - QT prolongation
• Explanations: Administering antiemetics before chemotherapy is the
most effective way to reduce nausea. The 5-HT3 receptor antagonists
are the drugs of the first choice for chemotherapy-induced emesis; QT
prolongation is among the most feared side effect of heavy usage of
antiemetics. The drugs should be started at least 30 minutes before
the chemotherapy and continued for several days after the
chemotherapy is discontinued. In some cases, dexamethasone is
combined with the 5-HT3 antagonists, especially in patients receiving
cisplatin-containing cancer drugs.
Question 513
• Question 513: A 65-year-old male patient presented to the physician's office
for the routine wellness examination. Detailed history reveals that he is a
current smoker with a 40 pack-year history of smoking. His family history is
positive for rectal cancer in his father and lund cancer in his uncle. He looks
anxious about his chances of having lung cancer and is willing to quit
smoking. What is the best way to screen him for lung cancer?
• Choices:
• 1. Low-dose helical CT scanning
• 2. Chest x-ray
• 3. Sputum cytology
• 4. There is currently no effective screening test for lung cancer
Answer: 1
• Answer: 1 - Low-dose helical CT scanning
• Explanations: Lung cancer causes the highest number of cancer deaths in the
United States. As such, it is a prime target for a screening test to diagnose
asymptomatic lung cancer when it can be treated effectively. Low-dose
helical CT scanning is currently considered the best screening test to diagnose
asymptomatic lung cancer at a stage when it can be effectively treated. It is
recommended for persons age 55 to 74 with a history of smoking at least 30
pack-years, of, if a former smoker, has quit within the past 15 years. Chest x-
ray and sputum cytology have not been found to be effective for lung cancer
screening.
• Smoking cessation is the most effective way to prevent lung cancer from
developing.
Question 514
• Question 514: Which of the following drugs can be used to treat
chronic myelogenous leukemia?
• Choices:
• 1. Adalimumab
• 2. Tositumomab
• 3. Bevacizumab
• 4. Imatinib
Answer: 4
• Answer: 4 - Imatinib
• Explanations: Imatinib is used for Philadelphia chromosome-positive
(Ph+) chronic myeloid leukemia (CML) in chronic phase. Ph+ acute
lymphoblastic leukemia can also be treated with this condition.
Imatinib is an inhibitor of Bcr-Abl tyrosine kinase. This is the product
of the Philadelphia chromosome in patients with CML so inhibition
blocks proliferation and causes apoptosis.
Question 515
• Question 515: What is the most likely cause of unilateral bloody
nipple discharge in a 36-year-old female?
• Choices:
• 1. Fibroadenoma
• 2. Intraductal papilloma
• 3. Fat necrosis
• 4. Intraductal cancer
Answer: 2
• Answer: 2 - Intraductal papilloma
• Explanations: Intraductal papilloma is relatively common and is
usually unilateral. Intraductal papillomas are the most common cause
of bloody nipple discharge in women in their third to fourth decade of
life. Intraductal papilloma cannot be seen on a mammogram and
require a ductogram. Surgical excision of an intraductal papilloma is
recommended as it is a high-risk lesion.
Question 516
• Question 516: A 65-year-old male with acute lymphocytic leukemia is
admitted for thrombocytopenia. What is the appropriate room
assignment?
• Choices:
• 1. Semi-private room, so the patient will not feel alone
• 2. Semi-private room, so if the patient bleeds the roommate can call the
nurse
• 3. Private room, so the patient will not be exposed others to bodily fluids
• 4. Private room to protect the patient from infections from a roommate
Answer: 4
• Answer: 4 - Private room to protect the patient from infections from a
roommate
• Explanations: Patients with acute lymphocytic leukemia have an
impaired immune system. They should be placed in private rooms and
all food should be cooked. Patients are treated with chemotherapy.
The patient in the clinical scenario most likely will require platelet
transfusion to treat the thrombocytopenia.
Question 517
• Question 517: A 66-year-old female recently underwent a colonoscopy and was
found to have a large mass that could not be removed by the gastroenterologist.
She was referred to a surgeon and subsequently had that portion of her colon
removed and sent to pathology. She comes into the office weeks later for her
follow up visit, where the provider discusses the final pathology results showing
adenocarcinoma. Which tumor marker can we use to evaluate for recurrence of her
disease?
• Choices:
• 1. Alpha-fetoprotein (AFP)
• 2. CA 125
• 3. CA 19-9
• 4. Carcinoembryonic antigen (CEA)
Answer: 4
• Answer: 4 - Carcinoembryonic antigen (CEA)
• Explanations: CEA marker is present in colon cancer. This glycoprotein
is made in the gastrointestinal tract during fetal development, and its
manufacture ceases at birth. CEA levels are ideally used to assess the
recurrence of colon cancer. CEA levels are also increased in many
other cancers, so one cannot use them to assess for the presence of a
malignancy. However, once colon cancer has been resected, any
increase in CEA levels is indicative of recurrence.
Question 518
• Question 518: Tumors and cancers arising from which of the following
sites are difficult to eradicate by chemotherapy?
• Choices:
• 1. Cancers arising from bone marrow cells
• 2. Tumors arising from bone
• 3. Tumors of epithelial origin
• 4. Tumors arising from lymphoid cells
Answer: 2
• Answer: 2 - Tumors arising from bone
• Explanations: Solid tumors are usually hard to eradicate with
chemotherapy because most of the cells are in go-phase and probably
have a larger tumor burden. Rapidly dividing cells succumb early to
the effects of chemotherapy. Malignancies arising from bone marrow,
epithelial cells, as well as lymph nodes are relatively easy to eradicate.
Question 519
• Question 519: A patient with metastatic breast cancer is confused and
has a calcium level of 14.9 mg/dL (8.4-10.2). What is the first step in
management?
• Choices:
• 1. IV furosemide
• 2. IV bisphosphonate
• 3. Normal saline bolus
• 4. Subcutaneous calcitonin
Answer: 3
• Answer: 3 - Normal saline bolus
• Explanations: Adequate hydration with normal saline is the first step
in managing hypercalcemia. IV fluids increase intravascular volume
and reduce serum calcium levels. Loop diuretics can be used after
adequate hydration. Starting with diuretics would raise the relative
concentration of calcium. Thus, loop diuretics should be used
cautiously only after IV fluids. Bisphosphonates will lower the calcium
over two to four days. They do not exhibit their action immediately;
therefore, faster-acting agents like calcitonin need to be used in
emergent cases. Intravenous calcitonin can be used in emergencies
when saline and loop diuretics are not adequate.
Question 520
• Question 520: Internal radium implant treatment for cervical cancer
mandates which of the following?
• Choices:
• 1. A low-residue diet
• 2. Plenty of fluids
• 3. Daily perineal care
• 4. Warm compresses to the abdomen
Answer: 1
• Answer: 1 - A low-residue diet
• Explanations: A low-residue diet is necessary to prevent bowel
distention and to decrease the number of bowel movements the
patient has. Bowel movements are difficult while the implant is in
place and all movement should be minimized to prevent
dislodgement. Patients are on strict bed rest while the implant is in
place and no perineal care should be performed.
Question 521
• Question 521: A female is treated for stage II breast cancer. One year
later, she has no symptoms. Which of the following is the most
appropriate follow-up?
• Choices:
• 1. CT of the chest, abdomen, and pelvis to rule out metastatic disease
• 2. Laboratories including complete blood count and liver functions
• 3. Mammograms yearly at the minimum
• 4. Nuclear medicine bone scan to rule out metastatic disease
Answer: 3
• Answer: 3 - Mammograms yearly at the minimum
• Explanations: Six months after completion of treatment a
mammogram should be done and yearly thereafter. CT, bone scan,
and laboratory follow-up have not been shown to improve survival.
Symptomatic patients should have appropriate testing, such as a bone
scan for bone pain.
Question 522
• Question 522: A 31-year-old man with Hodgkin disease is treated with
the ABVD chemotherapy regimen. What does the B stand for in this
chemotherapy regimen?
• Choices:
• 1. Bleomycin
• 2. Busulfan
• 3. Bismuth
• 4. Bromocriptine
Answer: 1
• Answer: 1 - Bleomycin
• Explanations: AVBD chemotherapy regimen stands for adriamycin
(doxorubicin), bleomycin, vinblastine, and dacarbazine. Bleomycin can
cause skin and lung toxicity. Bleomycin forms a complex with oxygen
and iron. This bleomycin-oxygen-iron complex then binds to DNA
which causes DNA scission by free radical production. Bleomycin has
important antineoplastic activity against squamous carcinoma of the
cervix, lymphomas, and testicular cancers.
Question 523
• Question 523: Which is true about the side effects of chemotherapy?
• Choices:
• 1. Nausea and vomiting after chemotherapy are usually managed by
dose reduction
• 2. Renal toxicity is best prevented by vigorous hydration during
treatment
• 3. Chemotherapy-induced alopecia is usually irreversible
• 4. Opioids should be avoided in patients with severe mucositis
Answer: 2
• Answer: 2 - Renal toxicity is best prevented by vigorous hydration
during treatment
• Explanations: Nephrotoxicity is a common side effect of certain
chemotherapeutic agents. Vigorous hydration and use of diuretics to
maintain a high urine output are essential to prevent renal toxicity.
Hyperuricemia can result in tumor lysis syndrome and uric acid
nephropathy. The urinary output should always be monitored during
chemotherapy.
Question 524
• Question 524: Which of the following drugs is used for treatment of
Non-Hodgkin lymphoma?
• Choices:
• 1. Adalimumab
• 2. Fulvestrant
• 3. Bortezomib
• 4. Tositumomab
Answer: 4
• Answer: 4 - Tositumomab
• Explanations: Tositumomab is indicated for relapsed or refractory
CD20 positive, low-grade, follicular, or transformed non-Hodgkin
lymphoma (NHL). It is a murine IgG2a lambda monoclonal antibody. It
binds to the CD20 antigen that is present in greater than 90 percent
of B-cell NHLs. It may cause antibody-dependent cellular cytotoxicity,
complement-dependent cytotoxicity, and apoptosis. Administration
results in depletion of CD20 positive cells.
Question 525
• Question 525: What is the ideal therapy in a 44-year-old male with
newly diagnosed Hodgkin lymphoma localized to the mediastinum?
• Choices:
• 1. Radiation with chemotherapy
• 2. Surgery
• 3. Radiation only
• 4. Immune therapy
Answer: 1
• Answer: 1 - Radiation with chemotherapy
• Explanations: Hodgkin lymphoma is a curable disease. The treatment
protocol must be followed. The standard treatment of Hodgkin
lymphoma is a combination of radiation and chemotherapy. PET scan
is recommended to screen these patients for recurrence. A positive
PET scan correlates highly with recurrent disease. Relapses are known
to occur and are treated with salvage chemotherapy. Those with
refractory disease are offered allogeneic bone transplantation.
Question 526
• Question 526: What is the most important prognostic factor for
melanoma?
• Choices:
• 1. Size
• 2. Color
• 3. Border
• 4. Depth
Answer: 4
• Answer: 4 - Depth
• Explanations: Melanoma arises from melanocytes, melanin-producing
cells. The depth of the melanoma is the most important prognostic
factor. Tumor in regional lymph nodes or distant metastases is poor
prognostic indicator. The anatomic site also has a prognostic role. The
trunk and face have a worse prognosis than extremity.
Question 527
• Question 527: A 16-year-old male presents to your clinic with complaints of four months of
unprovoked nose bleeds and difficulty breathing out of his left nare. He has no other
complaints and reports no significant medical history. He mentions a family history of colon
cancer in his uncle and grandfather. He is well-appearing with an unremarkable physical
exam. Magnetic resonance imaging (MRI) is performed showing a highly vascular mass in
the posterior nasal cavity and sphenopalatine foramen without intracranial extension. Given
the likely diagnosis and the family history, what is the next best imaging study to order that
does not involve the head and neck?
• Choices:
• 1. Computed tomography (CT) chest
• 2. No other imaging is recommended
• 3. MRI abdomen
• 4. Barium enema
Answer: 4
• Answer: 4 - Barium enema
• Explanations: Nasopharyngeal angiofibroma has been shown to have an association with
familial adenomatous polyposis (FAP) and Gardner syndrome, with altered expression of the
APC gene. A double-contrast barium enema is the best imaging tool to evaluate for
innumerable colonic polyps seen in FAP. FAP has a high risk of malignant transformation of
the colonic polyps, with early onset of colon cancer, usually in the 4th-5th decades, and
demonstrates an autosomal dominant inheritance pattern. Patients with FAP will often
undergo a prophylactic total colectomy around 20 years of age, so with the history of
multiple generations with colon cancer and a new diagnosis of nasopharyngeal
angiofibroma, FAP should be considered and evaluated for. A barium enema is the imaging
test of choice. Other imaging such as abdominal and pelvic CT can be helpful to look for
other imaging features such as desmoid tumors or metastatic disease, but this should be
done

• so with the history of multiple generations with colon cancer and a new diagnosis of
nasopharyngeal angiofibroma, FAP should be considered and evaluated for. A barium enema
is the imaging test of choice. Other imaging such as abdominal and pelvic CT can be helpful
to look for other imaging features such as desmoid tumors or metastatic disease, but this
should be done as supplemental imaging.
Question 528
• Question 528: When are adenomatous polyps associated with
increased cancer risk?
• Choices:
• 1. When there is tubular histology
• 2. When there is active bleeding
• 3. When the polyp is larger than 2 cm
• 4. When the polyp is pedunculated
Answer: 3
• Answer: 3 - When the polyp is larger than 2 cm
• Explanations: Increased risk of colon cancer occurs when the polyp is
greater than 2 cm and the histology reveals a villous adenoma. Sessile
rather than pedunculated polyps are more likely to be malignant.
Polyps and malignancy are not related to bleeding or age. Familial
polyposis disorders are associated with cancer at a young age.
Question 529
• Question 529: A patient is receiving nitrogen mustard, vincristine,
procarbazine, and prednisone (MOPP chemotherapy regimen) to treat
Stage III Hodgkin disease. Which of the following should be monitored
in this patient prior to receiving chemotherapy?
• Choices:
• 1. Liver function
• 2. Respiratory status
• 3. White blood cell values
• 4. Gastrointestinal status
Answer: 3
• Answer: 3 - White blood cell values
• Explanations: The administration of MOPP chemotherapy is based on
white blood cell values. This combination of drugs can cause bone
marrow suppression. Nitrogen mustard is an alkylating agent. Vincristine
sulfate is a vinca alkaloid that inhibits mitosis. Procarbazine hydrochloride
is an alkylating agent whose mechanism of action is not completely
understood. Prednisone is a corticosteroid hormonal drug. Prednisone is
used more than any other hormone in treating malignancies. Liver
function studies are not used to guide the administration of the MOPP
regimen. Side effects of the MOPP regimen are mainly hematologic and
do not include adverse respiratory or gastrointestinal findings.
Question 530
• Question 530: A 57-year-old male with a history of hyperthyroidism, diabetes mellitus, and
hypertension presents to the physician's office with several months history of weight loss.
The patient states that he recently noticed blood-streaked stools and also noticed the
change in the caliber of the stools. Furthermore, he also states that his father was diagnosed
with colon cancer at the age of 46. Moreover, the colonoscopy performed on his last visit
revealed a mass, which was positive for adenocarcinoma when biopsied. He is scheduled for
surgical resection of the mass. Which of the following can increase the risk of radiation
enteritis if he undergoes post-operative radiation therapy?
• Choices:
• 1. Family history
• 2. Age
• 3. Comorbid conditions like diabetes mellitus, hypertension
• 4. Presence of autoimmune conditions
Answer: 3
• Answer: 3 - Comorbid conditions like diabetes mellitus, hypertension
Explanations: Development of radiation enteritis has been shown to be
dependent on factors including radiation dose, past abdominal surgery,
body mass index, comorbid conditions like diabetes mellitus and
hypertension, and concurrent chemotherapy. Statistics show that a large
number of patients undergoing surgery for radiotherapy-induced gut
damage die from their original cancer within 2 years. Whereas without
cancer recurrence, the five-year survival is approximated to be 70%.
Ongoing studies have shown an increased risk of malignancy as a late
consequence of pelvic radiation Patients are advised to maintain
adequate hydration and avoid foods that cause discomfort. This can vary
in different people.
Question 531
• Question 531: What should be the immediate next step in the case of
an elderly female who is admitted for pyelonephritis and started on IV
antibiotics who becomes tachycardic and hypotensive who is then
found to have a fever and to be somnolent with tachypnea?
• Choices:
• 1. Administer a bolus of IV fluids
• 2. Administer an albuterol nebulizer treatment
• 3. Order blood cultures
• 4. Administer a stress dose of IV hydrocortisone
Answer: 1
• Answer: 1 - Administer a bolus of IV fluids
• Explanations: This case is suggestive of septic shock and the first step
should be to treat her hemodynamic stability first with volume
resuscitation in an attempt to counteract the fluid loss due to capillary
extravasation. A total of 4-6 L of fluid may be required in such cases
and should be bolused at 500 ml increments or 10ml/kg in children.
An isotonic fluid such as normal saline (0.9%) or lactated ringers
should be used. Hypotension and tachycardia are signs of reduced
circulatory volume and IV pressors such as norepinephrine and stress
dosed hydrocortisone may be necessary necessary if hemodynamic
instability persists despite aggressive fluid resuscitation.
Question 532
• Question 532: Which is a typical finding in a female with inflammatory
breast cancer?
• Choices:
• 1. Significant dermal lymphatic infiltration by cancer
• 2. Marked granulocytic infiltration
• 3. Bilateral breast involvement
• 4. Palpable mass that is usually centrally located
Answer: 1
• Answer: 1 - Significant dermal lymphatic infiltration by cancer
Explanations: The diagnosis of inflammatory breast cancer is clinical.
A patient may present with rapidly progressing, tender, and enlarged
breast. The classic history is the invasion of skin dermal lymphatics by
the cancer. The obstructed lymph channels produce erythema,
warmth, edema, and peau d'orange. The physical exam will reveal a
warm breast with no underlying mass. Crusting and retraction may be
present.
Question 533
• Question 533: When a patient with lung cancer has associated
mediastinal lymphadenopathy (metastasis), how does the staging
change if a hilar metastasis is contralateral to the original lesion,
rather than ipsilateral?
• Choices:
• 1. N1 to N2
• 2. N2 to N3
• 3. N0 to N3
• 4. N3 to N4
Answer: 2
• Answer: 2 - N2 to N3
• Explanations: Metastatic disease found in the contralateral
mediastinum compared to the original lesion is more concerning and
is a bad prognostic indicator. TNM staging is critical for determining
treatment for patients. Spiculated and rapidly growing lymph nodes
are less likely to be benign. Guidelines for following mediastinal
lymph nodes can be found on the American College of Radiology
website.
Question 534
• Question 534: Pulmonary function tests should be routinely
performed in patents taking which of the following medications?
Choices:
• 1. Cyclophosphamide
• 2. 5-flourouracil
• 3. Methotrexate
• 4. Bleomycin
Answer: 4
• Answer: 4 - Bleomycin
• Explanations: Pulmonary fibrosis is a known complication of the drug
bleomycin. Bleomycin pulmonary toxicity is dose related. Amiodarone
is another drug commonly associated with this condition. Routine
lung function tests should be done in all patients taking these
medications. Methotrexate can cause acute and chronic interstitial
pneumonia but routine pulmonary function tests are not done.
Question 535
• Question 535: A 65-year-old man with a recent diagnosis of Non-
Hodgkin lymphoma, who recently started chemotherapy, presents to
the clinic with complaints of oral ulcerations. He reports having trouble
chewing as the lesions are painful. Which of the following is most likely
cause of patient’s symptoms?
• Choices:
• 1. Poor oral hygiene
• 2. Chemotherapy side effect
• 3. Lichen planus
• 4. Systemic lupus erythematosus
Answer: 2
• Answer: 2 - Chemotherapy side effect
• Explanations: Common toxicities associated with chemotherapy
agents is mucositis. Chemotherapy affects active cells, so it tends to
affect cells that are rapidly multiplying like mucosa. Thorough buccal
cavity assessments and avoidance of the use of commercial
mouthwashes in patients with mucositis can help decrease patient
discomfort. All the other factors can cause ulcerations, but since this
patient started chemotherapy recently, most likely causing his
ulceration.
Question 536
• Question 536: Which of these tests is most sensitive test to localize a
pheochromocytoma?
• Choices:
• 1. Abdominal CT
• 2. Abdominal MRI
• 3. 123-Iodine MIBG
• 4. Arteriogram
Answer: 3
• Answer: 3 - 123-Iodine MIBG
• Explanations: The nuclear study with MIBG is the most sensitive test to identify
pheochromocytoma. This is because 10 percent are extra-adrenal in location. MIBG imaging
is a functional study to search the entire body rather than an anatomic study to detail the
anatomy of the adrenal glands. If restricted to only adrenal pheochromocytomas, MIBG is
less sensitive than MRI/CT, failing to detect about 10 percent of the lesions. To say a study is
the most sensitive is not to say it should be ordered first. Since 90 percent of
pheochromocytomas are adrenal in origin, an abdominal CT or MRI should begin the workup
after clinical/lab evidence of pheochromocytoma is found. Onlywhen the suspected pheo is
shown not to be adrenal in origin is MIBG needed. I-123 studies are best for detection and
minimize patient radiation dose. I-131 exams can be therapeutically helpful. Both abdominal
MRI and abdominal CT are excellent modalities for assessing the adrenal glands, both to see
and to characterize adrenal masses. Classically, pheochromocytomas are very intense
("bright") on T2 weighted images and MRI is more specific in the diagnosis than CT.
Question 537
• Question 537: Which of the following medications is a serotonin
receptor antagonist?
• Choices:
• 1. Paroxetine
• 2. Enalapril
• 3. Alprostadil
• 4. Ondansetron
Answer: 4
• Answer: 4 - Ondansetron
• Explanations: Ondansetron is a potent antiemetic used in
chemotherapy-induced nausea and vomiting. It works be blocking
5HT (serotonin) receptors in the chemoreceptor trigger zone.
Paroxetine is a selective serotonin reuptake inhibitor and does not
block any receptors. Alprostadil is a prostaglandin analog that is used
in pulmonary arterial hypertension.
Question 538
• Question 538: Which of the following should be offered or included in
survivorship education of all patients after mastectomy?
• Choices:
• 1. Reconstructive surgery
• 2. Support groups
• 3. Volunteer opportunities
• 4. Home health care
Answer: 2
• Answer: 2 - Support groups
• Explanations: Information about community resources such as
support groups may help a patient cope with her illness and her
changes in body image. All women should be taught about arm
exercises to prevent lymphedema and shoulder immobility. Women
post mastectomy must also be told about their options for breast
reconstruction because losing a breast can cause loss of self-esteem.
If the patient has a drain, she must be educated about the drainage
system and keeping the wound clean.
Question 539
• Question 539: Which of the following drugs can results in loss of knee
tendon reflexes?
• Choices:
• 1. Bleomycin
• 2. Daunorubicin
• 3. Mercaptopurine
• 4. Vincristine
Answer: 4
• Answer: 4 – Vincristine
• Explanations: Vincristine vinca alkaloid commonly causes peripheral
neuropathy. Paresthesias and numbness are common with higher
doses. Loss of reflexes and foot drop may occur in severe cases.
Constipation is another common complication seen with vincristine
use.
Question 540
• Question 540: Which chemotherapy drug often is associated with
peripheral neuropathy?
• Choices:
• 1. Isoniazid
• 2. Vinblastine
• 3. Doxorubicin
• 4. Vincristine
Answer: 4
• Answer: 4 - Vincristine
• Explanations: Vincristine commonly is associated with peripheral
neuropathy. Isoniazid is not a chemotherapy agent. The main side effects
of vincristine are chemotherapy-induced peripheral neuropathy, hair loss,
hyponatremia, and constipation. Chemotherapy-induced peripheral
neuropathy may be severe, and in some patients, may be a reason to
reduce or avoid using vincristine. The progressive and enduring
symptoms include tingling, numbness, pain, and hypersensitivity to cold,
which begins in the hands and feet and progresses to the arms and legs.
Foot drop is one of the first signs. Individuals with a family history of foot
drop or Charcot-Marie-Tooth disease should avoid vincristine.
Question 541
• Question 541: What is the most common malignancy causing inferior
vena cava syndrome?
• Choices:
• 1. Melanoma
• 2. Renal cell carcinoma
• 3. Colon cancer
• 4. Liver adenocarcinoma
Answer: 2
• Answer: 2 - Renal cell carcinoma
• Explanations: Inferior vena cava syndrome is due to obstruction of the
inferior vena cava (IVC). The obstruction can be due to tumor
thrombus in the renal vein extending into the IVC. Renal cell cancer
has a propensity for invading the renal vein and causing intraluminal
thrombosis. Caval thrombosis is also seen secondary to hepatomas
and adrenal cortical carcinomas. Compression of the inferior vena
cava can also occur with a gravid uterus and with Budd-Chiari
syndrome.
Question 542
• Question 542: A 72-year-old female with lung cancer has nausea,
vomiting, and then lethargy. She is arousable, inattentive, and
disoriented. There are diminished reflexes and proximal weakness.
What is the most likely etiology?
• Choices:
• 1. Hyponatremia
• 2. Hypernatremia
• 3. Hypercalcemia
• 4. Hypocalcemia
Answer: 3
• Answer: 3 - Hypercalcemia
• Explanations: 5% of patients with lung cancer develop hypercalcemia.
It can be secondary to a paraneoplastic syndrome or metastatic bone
disease. There is a decrease in membrane excitability that causes the
symptoms. Convulsions and coma are possible.
Question 543
• Question 543: A 68 year old male undergoing chemotherapy for lung
cancer presents with complaint of headache, facial swelling, nausea
and dizziness; exam reveals normal vital signs, normal heart and lung
sounds, with dilated neck veins and facial edema; his diagnosis?
• Choices:
• 1. Malignant pericardial effusion
• 2. Superior vena cava syndrome
• 3. Tumor lysis syndrome
• 4. Pancoast tumor
Answer: 2
• Answer: 2 - Superior vena cava syndrome
• Explanations: Superior vena cava syndrome occurs with compression of the SVC by
tumor or enlarged mediastinal nodes causing venous engorgement of the head, neck
and upper trunk; treatment is with diuretics, glucocorticoids and further treatments
aimed at the obstructing lesion (radiation, chemotherapy, stent or surgery).
Malignancies may metastasize to the pericardium leading to tamponade; physical
examination findings include hypotension, jugular venous distention, distant heart
sounds, and pulsus paradoxus. Tumor lysis syndrome is a group of metabolic
abnormalities resulting from rapid hypocalcemia, hyperuricemia, and renal failure just
hours after receiving treatment (allopurinol prophylaxis prior to chemotherapy helps
prevent hyperuricemia). A Pancoast tumor (superior sulcus tumor) is a tumor at the
lung apex that may cause compression of the subclavian artery, brachiocephalic vein,
phrenic nerve, recurrent laryngeal nerve, vagus nerve, or the sympathetic ganglion
resulting in a Horner's syndrome.
Question 544
• Question 544: Which is the most common hormone or hormone-like
substance produced by pulmonary carcinoma?
• Choices:
• 1. Adrenocorticotropic hormone (ACTH) or ACTH-like substance
• 2. Antidiuretic hormone
• 3. Gonadotrophin
• 4. Parathormone
Answer: 1
• Answer: 1 - Adrenocorticotropic hormone (ACTH) or ACTH-like
substance
• Explanations: Nearly all the endocrine syndromes are associated with
oat cell carcinoma but the most common is Cushing syndrome. The
syndrome comes on suddenly and may be fulminant. The same
syndrome may be associated with bronchial carcinoid. The presence
of a paraneoplastic syndrome bears no correlation to prognosis. Once
the cancer is removed or treated, the syndrome resolves.
Question 545
• Question 545: A 37-year-old female presented with persistent left upper abdominal quadrant
(LUQ) discomfort for four months. She denied any change in bowel habits or weight loss. The
physical examination revealed mild epigastric tenderness. Routine laboratory tests, including
tumor marker levels, were within the normal range. Abdominal ultrasonography (US)
reported a heterogeneous semisolid mass with cystic components. Abdominal computerized
tomography (CT) revealed a well-defined 12×10×8 cm mass involving the pancreatic tail and
was heterogeneous and abuts splenic vein. It was tentatively diagnosed solid papillary cystic
tumor of the pancreas. What is the next step in the management?
• Choices:
• 1. Systemic chemotherapy
• 2. Surgery
• 3. Locoregional Radiation
• 4. Watchful waiting.
Answer: 2
• Answer: 2 - Surgery
• Explanations: Solid papillary tumors of the pancreas are commonly benign,
and chemotherapy is not indicated unless there is malignant
transformation. SPEN stands for Solid and Papillary Epithelial Neoplasm
(SPEN) of the pancreas. The tumor has also been referred with other
names, including solid pseudopapillary tumor of the pancreas, solid
pseudopapillary neoplasm. These are indolent and benign neoplasms and
primarily managed by surgery that offers five-year survival of up to 95%.
Solid papillary cystic tumors are not amenable for radiation. Primary
treatment is surgery. Solid papillary cystic tumors of the pancreas are
commonly benign and curative by surgery alone. Rarely they and watch is
not the recommended strategy.
Question 546
• Question 546: Which malignancy occurs most in a pregnant woman?
• Choices:
• 1. Cervix
• 2. Ovary
• 3. Breast
• 4. Colon
Answer: 1
• Answer: 1 – Cervix
• Explanations: Cervical cancer is the most common gynecologic
malignancy in pregnancy. The treatment of cervical cancer during
pregnancy is very challenging because one also has to consider the
fetus at the same time. In the last decade, preservation of the
pregnancy and treatment has become more common. It is important
to know that pregnancy by itself does not influence the cervical
lesions and progression of the disease is not common.
Question 547
• Question 547: Intra-epidermal (in-situ) squamous cell carcinoma of
the penis can progress to which type of cancer?
• Choices:
• 1. Nodular melanoma
• 2. Basal cell carcinoma
• 3. Squamous cell carcinoma
• 4. Adenocarcinoma
Answer: 3
• Answer: 3 - Squamous cell carcinoma
Question 548
• Question 548: A 17-year-old female presents to the office with complaints
of vision abnormalities. She has no history other than appendectomy one
year ago. On evaluation, the healthcare provider discovers a loss of
peripheral vision. The provider suspects a benign tumor of the pituitary
gland. Which imaging test will most likely confirm this diagnosis?
• Choices:
• 1. Brain computed tomography (CT) scan
• 2. Chest radiograph
• 3. Brain magnetic resonance image (MRI)
• 4. Ultra-sound
Answer: 3
• Answer: 3 - Brain magnetic resonance image (MRI)
• Explanations: Most pituitary tumors are slow-growing, benign tumors.
Depending on the size, they are divided into microadenoma, which is
tumors less than 10 mm in size and macroadenoma, which is greater
than 10 mm. MRI is the recommended standard of care for pituitary
adenoma pressing on cranial nerves. A chest x-ray is performed if
there is concern about metastasis to the lungs.
Question 549
• Question 549: Which of the following is a common side effect of
methotrexate treatment?
• Choices:
• 1. Mucositis
• 2. Pancytopenia
• 3. Alopecia
• 4. Peripheral neuropathy
Answer: 1
• Answer: 1 – Mucositis
• Explanations: Methotrexate depletes cells off their reduced folic acid
and inhibits DNA replication. The most common complication with
chronic use is mucositis. This occurs due to inhibition of cell
proliferation in tissues with rapid cell turnover. Bone marrow
depression and pneumonitis are other common complications.
Question 550
• Question 550: Which of the following is most commonly used to treat
metastatic prostate cancer?
• Choices:
• 1. Surgery
• 2. Leuprolide
• 3. Spironolactone
• 4. Steroids
Answer: 2
• Answer: 2 – Leuprolide
• Explanations: GnRH is released in pulse mode and continuous GnRH
release results in hypogonadism. Continuous administration of
leuprolide causes hypogonadal state. Metastatic prostate cancer is
treated with leuprolide to decrease androgenic stimulation of growth
of tumor mass. Spironolactone has anti androgenic properties but it is
not indicated to treat metastatic prostate cancer.
Question 551
• Question 551: A 66-year-old male presents with a lump in his right
parotid gland. Ultrasound-Guided fine needle aspiration of the lump is
planned. He is subsequently diagnosed with the most common benign
type of parotid tumor. What kind of cells does this tumor originate
from?
• Choices:
• 1. Squamous and epithelial
• 2. Epithelial and myoepithelial
• 3. Columnar and epithelial
• 4. Myoepithelial and columnar
Answer: 2
• Answer: 2 - Epithelial and myoepithelial
• Explanations: The most common benign type of parotid tumor is a
pleomorphic adenoma. It accounts for 70-80% of all salivary gland
tumors. Pleomorphic adenomas are mixed tumors and usually arise
from epithelial and myoepithelial cells. The incidence of such tumors
is 2-3 per 100,000 population. It is usually found in the superficial lobe
of the parotid gland and the most common surgical treatment is
superficial parotidectomy.
Question 552
• Question 552: A patient undergoes video-assisted thoracoscopic
surgery for a solitary nodule. The pathologist reports that the frozen
section is positive for a small cell carcinoma. The margins are all clear.
What is the next step in management?
• Choices:
• 1. Observation
• 2. Chemotherapy followed by radiation
• 3. Mediastinoscopy for staging
• 4. Lobectomy followed by chemotherapy
Answer: 4
• Answer: 4 - Lobectomy followed by chemotherapy
• Explanations: Selected patients with stage I small cell carcinoma can
be treated with lobectomy, followed by chemotherapy. This treatment
approach has a 5-year survival approaching 60%. Radiation should be
administered early in treatment to prevent local recurrence. For
patients with extensive disease, the treatment is palliative and end of
life options must be discussed with the patient.
Question 553
• Question 553: Which is true about cervical cancer? Choices:
• 1. Cervical cancer rates are increasing in North America
• 2. The first symptom is usually postcoital vaginal bleeding
• 3. Most women who acquire human papillomavirus go on to develop
cervical cancer
• 4. The Pap smear is diagnostic for cervical cancer
Answer: 2
• Answer: 2 - The first symptom is usually postcoital vaginal bleeding
Explanations: Cervical cancer is on the decline in North America
because of screening with Pap smears. Most women present with
postcoital vaginal bleeding, dysuria, or vaginal discomfort. Human
papillomavirus infection is common in sexually active women but only
a small percentage of them develop cancer. Pap smear is only a
cytological study and definitive confirmation of cancer requires a
biopsy.
Question 554
• Question 554: Which is the most likely cancer of the renal collecting
system for a North American male between the ages of 60 and 80
years?
• Choices:
• 1. Wilms tumor
• 2. Adenocarcinoma
• 3. Transitional cell cancer
• 4. Squamous cell cancer
Answer: 3
• Answer: 3 - Transitional cell cancer
• Explanations: In North America, the most common type of bladder
cancer is transitional cell carcinoma. The median age at diagnosis of
bladder cancer is 65 years. Males are three times more likely to
develop bladder cancer than females. Bladder cancer is more
common in whites than African Americans, but African Americans
tend to have a poorer prognosis. Bladder cancer has the highest rates
of recurrence compared to any other malignancy. Squamous cell
carcinoma is the second most common cell type associated with
bladder cancer and is more common in industrialized countries.
Question 555
• Question 555: A patient with bone cancer is being treated with long-acting
morphine. He presents with vague complains of abdominal distension,
nausea, hard stools, and straining. You feel that he may have developed
opioid-induced constipation. You decide to use the Bristol stool scale to
classify his constipation. This tool uses what criteria to assess constipation?
• Choices:
• 1. Stool size
• 2. Stool color
• 3. Stool form
• 4. Stool weight
Answer: 3
• Answer: 3 - Stool form
• Explanations: The Bristol Stool Form Scale can be used to classify
stool form into seven categories. Types 1 and 2 indicate constipation.
Types 3 and 4 are considered normal. Type 5 indicates a lack of fiber
in one's diet. Types 6 and 7 indicate diarrhea. The Bristol stool scale
also is sensitive to alterations in intestinal transit time caused by
medications such as opioids or laxatives.
Question 556
• Question 556: Which of the following drugs acts on topoisomerase II?
• Choices:
• 1. Methotrexate
• 2. Cyclophosphamide
• 3. Azathioprine
• 4. Etoposide
Answer: 4
• Answer: 4 – Etoposide
• Explanations: Etoposide is an antineoplastic that is a topoisomerase II
inhibitor and a plant alkaloid. It is used for glioblastoma multiforme,
Kaposi sarcoma, lung cancer, testicular cancer, Ewing sarcoma,
lymphoma, and nonlymphocytic leukemia. Etoposide is usually used in
combination with other cancer drugs. It has many side effects include
anorexia, hair loss, diarrhea, and pancytopenia. When administered
intravenously, a slow drip is given as it can drop blood pressure.
Question 557
• Question 557: Radiation is cataractogenic at which single radiation
dose?
• Choices:
• 1. 10 rad
• 2. 50 rad
• 3. 100 rad
• 4. 200 rad
Answer: 2
• Answer: 2 - 50 rad
• Explanations: Cataract formation can occur with a dose as small as 50 rads, or 0.5
Gray. This is not to say that cataracts always form at 50 Rad exposure. It is to say
they don't form with less radiation exposure. Cataracts are an example of a
nonstochastic effect. They have a threshold below which they do not occur.
Above the threshold higher doses increase the severity not the likelihood of
occurrence. Other nonstochastic (or deterministic) effects include gonadal
dysfunction and bone marrow dysfunction. For doses greater than 50 rads
increasing dose increases the likelihood of cataracts. Stochastic effects are effects
that have no threshold and which increase in probability, not the severity,
increased exposure. Examples include cancer induction and genetic effects. So
increasing the radiation dose increases the likelihood of cancer, not the severity
of cancer induced.
Question 558
• Question 558: Which lymphoma is endemic to Sub-Saharan Africa and
typically associated with Epstein-Barr virus infection?
• Choices:
• 1. Hodgkin lymphoma
• 2. Kaposi sarcoma
• 3. Follicular lymphoma
• 4. Burkitt lymphoma
Answer: 4
• Answer: 4 - Burkitt lymphoma
• Explanations: Burkitt lymphoma is endemic to New Guinea and
equatorial Africa. Almost all cases Sub-Saharan Africa are associated
with Epstein-Barr virus while sporadic cases of Burkitt lymphoma only
show Epstein-Barr virus in 20% of patients. Burkitt lymphoma is a
high-grade lymphoma derived from B cells with 85% of patients
showing translocation t (8;14)(q24;q32). It is theorized that malaria
infection makes individuals more prone to developing Burkitt
lymphoma.
Question 559
• Question 559: A patient has cancer and is to receive a combination of
fluorouracil (5-FU) and leucovorin calcium. He is also receiving
allopurinol. When teaching the patient about 5-FU, you will include
which of the following dermatological manifestations?
• Choices:
• 1. Tearing, conjunctivitis, and blurred vision
• 2. Darkening of the skin along the veins and dark coloration of the nails
• 3. Urticaria
• 4. Bone marrow suppression
Answer: 2
• Answer: 2 - Darkening of the skin along the veins and dark coloration
of the nails
• Explanations: Fluorouracil (5-FU) may cause darkening of the skin
along the veins and dark coloration of the nails. Rash and urticaria are
side effects of leucovorin. Allopurinol may cause bone marrow
suppression, which is not a dermatological manifestation. Some
patients may need an antihistamine and even a topical steroid to
prevent the itching and dryness that follows the IV injection of 5
fluorouracil.
Question 560
• Question 560: A 64-year-old female with metastatic breast cancer is
receiving treatment with capecitabine develops by intense, painful
erythema of the palms and soles. What is the most likely diagnosis?
• Choices:
• 1. Acral erythema
• 2. Erythema multiforme
• 3. Herpes zoster infection
• 4. Graft-versus-host disease
Answer: 1
• Answer: 1 - Acral erythema
• Explanations: Acral erythema is also known as palmar-plantar
erythrodysesthesia, palmoplantar erythrodysesthesia, hand-foot syndrome,
and Burgdorf reaction. This is an adverse event caused by many classic
chemotherapeutic agents and the newer molecular targeted therapies. It is
characterized by intense, painful erythema of the palms and soles, that can
progress to the formation of vesicles or bullae. In the case of capecitabine,
an oral fluoropyrimidine that is converted in vivo to 5-FU providing
prolonged tissue exposure, acral erythema is dose-related. One of the most
common treatment-related adverse events when treated with capecitabine
is the hand-foot syndrome. The incidence is around 60 percent.
Question 561
• Question 561: Pleuropulmonary blastoma are which of the following?
• Choices:
• 1. A carcinoma
• 2. A childhood cancer of the thorax
• 3. An endocrine disorder
• 4. A metabolic disease
Answer: 2
• Answer: 2 - A childhood cancer of the thorax
• Explanations: Pleuropulmonary blastoma is a childhood cancer of the
thorax. According to the 2015 World Health Organization
classification, pleuropulmonary blastoma is considered sarcoma.
There are three subtypes of pleuropulmonary blastoma: cystic, mixed,
and solid. Pleuropulmonary blastomas typically are observed before 2
years of age.
Question 562
• Question 562: A patient is given a chemotherapeutic agent. Later he
develops pins and needles feeling in his legs. Which of the following
most likely caused this adverse effect?
• Choices:
• 1. Chlorhexidine
• 2. Cyclophosphamide
• 3. Prednisone
• 4. Vincristine
Answer: 4
• Answer: 4 – Vincristine
• Explanations: Vincristine is well known to cause peripheral
neuropathy. Foot drop is often the first symptom of vincristine
therapy. Other nerve problems with vincristine include spinal nerve
demyelination, intractable pain, and ascending paralysis. Other
common adverse effects of vincristine include alopecia, nausea,
vomiting, weight loss, and constipation.
Question 563
• Question 563: Which of the following breast cancers has thread-like
strands of carcinoma cells, one cell thick through a fibrous matrix?
• Choices:
• 1. Infiltrating lobular carcinoma
• 2. Tubular carcinoma
• 3. Intraductal carcinoma
• 4. In situ lobular carcinoma
Answer: 1
• Answer: 1 - Infiltrating lobular carcinoma
• Explanations: The carcinoma cells of infiltrating lobular carcinomas
are in single file groups called "Indian file" cells. ILC is characterized by
bland, small, round cells that have scant cytoplasm. They usually
infiltrate the tissue in a single file. Lobular cancer usually does not
damage the adjacent anatomic structures or initiate a significant
connective tissue response. Because of the distinctive biology and
growth pattern and biology, lobular carcinomas is often easily
diagnosed on palpation. However, Lobular carcinoma is also known to
be bilateral, multicentric and multifocal, which makes breast
conservation therapy unlikley.
Question 564
• Question 564: Mutations in which gene are associated with Lynch
syndrome, also known as hereditary non-polyposis colorectal cancer?
• Choices:
• 1. APC
• 2. P52
• 3. MMR
• 4. Kras
Answer: 3
• Answer: 3 - MMR
• Explanations: The MMR gene is most likely responsible for Lynch
syndrome. Synchronous and metachronous colorectal tumors are
common in patients with hereditary non-polyposis colorectal cancer.
Lynch syndrome is an inherited condition that increases the risk of
colon cancer and other cancers. Lynch syndrome is also known as
hereditary nonpolyposis colorectal cancer (HNPCC). Lynch syndrome
is the most common inherited syndrome that can increase the risk of
colon cancer. Lynch syndrome causes approximately 3 out of every
100 colon cancers. Lynch syndrome causes colon cancer at an earlier
age than in the general population.
Question 565
• Question 565: A patient who is undergoing chemotherapy complains
of tingling, paresthesias, and pain in her legs at night. Examination
reveals evidence of peripheral neuropathy. Which medication may
she need to stop taking?
• Choices:
• 1. Taxol
• 2. Bleomycin
• 3. Cisplatinum
• 4. Vincristine
Answer: 4
• Answer: 4 – Vincristine
• Explanations: Vincristine is an alkaloid used to treat a variety of
cancers. It is a mitotic inhibitor. Vincristine prevents formation of
mitotic spindle and acts during metaphase. The vinca alkaloids affect
all rapidly dividing cells. Side effects of vincristine include peripheral
neuropathy, hyponatremia, constipation, and hair loss. Vincristine can
also cause a foot drop and this may be a reason to discontinue
therapy.
Question 566
• Question 566: A 59-year old female presents complaining of swelling and
redness in her right breast. Two weeks ago, she was prescribed a corticosteroid
cream for the itching but the redness and swelling still persist. On physical exam,
the right breast appears swollen, red, and the skin appears dimpled but there is
no mass palpated. The axillary exam is negative. A biopsy reveals the presence
of a malignancy. This type of breast cancer is automatically staged at what level?
• Choices:
• 1. 1
• 2. 2
• 3. 3
• 4. 4
Answer: 4
• Answer: 4 – 4
• Explanations: Any breast cancer that extends to the chest wall or skin
is automatically staged T4. The diagnosis of inflammatory breast
cancer is only made with a full thickness skin biopsy. In less than 10%
to 20% of cases, a mass is palpated. The initial treatment is
chemotherapy but the prognosis is poor. Go to the next page if you
knew the correct answer, or click the link image(s) below to further
research the concepts in this question (if desired).
Question 567
• Question 567: Cancer antigen 125 (CA 125) was investigated to be a
tumor-associated antigen in treating what type of cancer?
• Choices:
• 1. Ovarian cancer
• 2. Non-small cell lung cancer
• 3. Pancreatic adenocarcinoma
• 4. Colorectal carcinoma
Answer: 1
• Answer: 1 - Ovarian cancer
• Explanations: CA 125 is a great tumor marker monitoring the
treatment of ovarian cancer and was investigated to target ovarian
cancer. CA 125 is approved by the US FDA for monitoring therapy
response in women with known epithelial ovarian carcinoma. The
normal value for CA 125 is less than 35 U/mL The gene for CA 125 is
called MUC16
Question 568
• Question 568: Which of the following is a complication of tumor lysis
syndrome?
• Choices:
• 1. Hypersthenuria
• 2. Hyperuricosuria
• 3. Hyperglycemia
• 4. Hypernatremia
Answer: 2
• Answer: 2 – Hyperuricosuria
• Explanations: Acute hyperuricosuria is a common complication of
tumor lysis syndrome. The syndrome rarely contributes to gout or uric
acid stones. Chronic hyperuricosuria is often associated with gout and
uric acid stones.
Question 569
• Question 569: A 65-year-old male patient presents to the hospital complaining of shortness of
breath and bilateral leg swelling that has been worsening for the last four days. He states he has
been compliant with his medication for congestive heart failure (CHF) that occurred as an
adverse effect of a chemotherapy agent he took about one year ago for leukemia. He states his
leukemia is now in remission, and his medical history is only significant for CHF and
hypertension. On examination, he is afebrile, heart rate is 86/min, respiratory rate is 16/min,
and blood pressure is 108/62 mmHg. On auscultation, diffuse crackles are heard in the bilateral
lung fields, and there is pitting edema in the lower extremities bilaterally. Which of the following
toxicities are also noted in the chemotherapy agent this patient was taking?
• Choices:
• 1. Loss of vision
• 2. Hepatotoxicity
• 3. Renal failure
• 4. Neuropathy
Answer: 2
• Answer: 2 - Hepatotoxicity
• Explanations: Adriamycin is known to cause cardiomyopathy,
neutropenia during acute treatment, and hepatotoxicity. The liver
function tests (LFTs) should be closely monitored in patients taking or
who have taken adriamycin due to the risk of hepatotoxicity.
Adriamycin is contraindicated in patients with bilirubin levels of > 5.0
mg/dL or severe hepatic impairment; dose adjustments are required
for all patients with liver problems or elevated bilirubin. Adriamycin
dosage does not need to be corrected based on the renal function. It
is also not known to cause vision loss or significant neuropathy.
Question 570
• Question 570: A patient experiences a sudden feeling of warmth over
the face, neck, and chest. The skin is reddened and they experience
diffuse sweating. Which of the following most like caused her
symptoms?
• Choices:
• 1. Spironolactone
• 2. Estrogen
• 3. Tamoxifen
• 4. Somatostatin
Answer: 3
• Answer: 3 - Tamoxifen
• Explanations: Tamoxifen has mild side effects like irregular menses,
hot flashes, bone and joint aches, swelling feet, and headache.
Tamoxifen is an estrogen receptor antagonist, preventing the growth
of some cancer cells. Tamoxifen is sometimes used to treat
gynecomastia, bipolar disorder, and a temporary form of castration in
sex offenders. Tamoxifen has been linked to endometrial cancer in
some women, and its use is limited to less than five years. Tamoxifen
can improve the serum lipid profile but has not been shown to reduce
the risk of ischemic heart disease in women.
Question 571
• Question 571: What is the best treatment for adenoid cystic
carcinoma of the trachea?
• Choices:
• 1. Preoperative radiation and surgery
• 2. Surgery and postoperative radiation
• 3. Surgery
• 4. Surgery followed by chemotherapy
Answer: 2
• Answer: 2 - Surgery and postoperative radiation
• Explanations: Adenoid cystic cancer (cylindroma) is a very slow-
growing tumor that always extends beyond the visible confines. The
cancer spreads submucosally and perineurally. Incidence is equal in
both sexes and is not related to smoking. Lymph node involvement
occurs in 10% of patients. Surgery followed by radiation is the
treatment for this cancer. Adenoid cystic carcinoma has a high
incidence of recurrence. Due to its submucosal spread, postoperative
radiation is administered.
Question 572
• Question 572: A postmenopausal 68-year-old female gravida 2, para 2 female presented to the emergency
department with the passage of clots, heavy vaginal bleeding, and feeling pre-syncopal. Her blood pressure
was 87/56 mmHg, pulse was 120 beats per minute, respiration was 18 breaths per minute, and the
temperature was 98 Fahrenheit. Her hemoglobin was noted to be 7.6 g/dl. She received 1 unit of packed red
blood cells with symptomatic improvement. Further history reveals she underwent menarche at the age of
11 and menopause at the age of 60. She is obese, but otherwise, her past medical, past surgical, family and
social history are noncontributory. Pelvic examination was concerning for blood coming from the cervical os
without any other masses or abnormalities. Further evaluation with pelvic ultrasound demonstrated a 7 mm
heterogenous endometrial stripe. Follow up with endometrial biopsy was concerning for high-grade
endometrial carcinoma,and she underwent a total hysterectomy, bilateral salpingo-oophorectomy,
omentectomy, and aortic and pelvic lymph node dissection. The final pathology report was significant for
papillary serous endometrial carcinoma. In addition to metastasizing via the lymphatic system, what is
another route by which papillary serous endometrial carcinoma can metastasize?
• Choices:
• 1. Papillary serous carcinomas are low-grade and do not typically metastasize.
• 2. Hematogenously
• 3. Transit through the fallopian lumens
• 4. Direct spread through tissues of adjacent organs
Answer: 3
• Answer: 3 - Transit through the fallopian lumens
• Explanations: Papillary serous carcinomas are high grade and
commonly metastasize via the lymphatic system and through the
fallopian lumens to disseminate throughout the pelvis and abdomen.
Hematogenous spread is most commonly associated with
choriocarcinoma not papillary serous carcinoma. Direct tissue spread
is more common with cervical cancer. Low risk, type 1 endometrioid,
carcinomas tend to remain confined to the uterus and therefore have
a favorable prognosis.
Question 573
• Question 573: A patient is about to undergo their first cycle of chemotherapy. They
mention concern about their aunt who said during her chemotherapy regimen years ago,
she developed severe nausea and vomiting one night prior to her next round of
chemotherapy. This patient is describing a well-documented classification of nausea and
vomiting. There are many researched options for approaching this situation. However, out
of all the treatment options, what is the most effective means of preventing this
classification of nausea and vomiting?
• Choices:
• 1. Acupuncture throughout the chemotherapy regimen
• 2. A routine antiemetic regimen enacted before the patient's chemotherapy treatment
begins
• 3. Positive visualization of symptom resolution
• 4. Eating small meals in the days leading up to chemotherapy treatments
Answer: 2
• Answer: 2 - A routine antiemetic regimen enacted before the patient's
chemotherapy treatment begins
• Explanations: Anticipatory nausea and vomiting are common in patients
receiving chemotherapy. The best prevention for developing anticipatory nausea
and vomiting is to prevent nausea and vomiting in general during chemotherapy
treatment by enacting a routine antiemetic regimen prior to starting
chemotherapy. Acupuncture and positive visualization are also treatments for
anticipatory nausea and vomiting; however, they are implemented to stop the
symptoms after they start.

• Delayed chemotherapy-induced nausea and vomiting are also common in


patients given corticosteroids and opioid analgesia.
Question 574
• Question 574: According to the American Cancer Society, yearly
screening mammograms for women are recommended no later than
what age?
• Choices:
• 1. 40 years of age
• 2. 45 years of age
• 3. 50 years of age
• 4. 55 years of age
Answer: 2
• Answer: 2 - 45 years of age Explanations: According to the American
Cancer Society (ACS), women 40 to 44 years of age should have the
choice to start annual breast cancer screening with mammograms.
They also recommend that women age 45 to 54 should have
mammograms every year. At 55 years of age, women can choose to
switch to mammograms every 2 years or continue yearly screening.
The ACS also recommends that screening continue as long as a
woman is in good health and is expected to live 10 more years or
longer.
Question 575
• Question 575: A 22-year old sexually active female comes in for her first pap
smear. She is hesitant about the procedure and wants to know if it will screen
for cervical cancer. You describe the procedure to her and its limitations.
Which of the following is true regarding the Pap smear?
• Choices:
• 1. It is nearly 100% sensitive for detecting cervical dysplasia
• 2. There is almost no intraobserver and interobserver variability in reading the
slide
• 3. If the Pap smear is negative, then there is no risk for cervical cancer
• 4. Nearly 50% of new cervical cancers occur in women who have never had a
pap smear
Answer: 4
• Answer: 4 - Nearly 50% of new cervical cancers occur in women who
have never had a pap smear
• Explanations: The Pap smear has a sensitivity for detecting cervical
dysplasia that ranges from 30% to 90%. There is significant
intraobserver and interobserver variability in reading the slide. Pap
smear has a sensitivity of 55.4% for detecting cervical neoplasia. Over
half of women diagnosed with cervical cancer have never or rarely
had a Pap smear.
Question 576
• Question 576: Which tumor markers are elevated with pure testicular
seminomas?
• Choices:
• 1. Beta-human chorionic gonadotropin (beta-hCG)
• 2. Alpha-fetoprotein
• 3. Lactate dehydrogenase (LDH)
• 4. Beta-hCG and LDH
Answer: 4
• Answer: 4 - Beta-hCG and LDH
• Explanations: Serum beta-human chorionic gonadotropin is elevated
in about 20% of testicular seminomas. Serum lactate dehydrogenase
is elevated in 30% to 80% of men with pure seminoma and in 60% of
those with non-seminomatous tumors. Alpha-fetoprotein is not
elevated in pure seminomas.
Question 577
• Question 577: A female reports having gastric upset and easily
bruised skin. She is also found to have significant bone thinning and
labile mood. Which of the following is the most likely cause?
• Choices:
• 1. She is undergoing premature menopause
• 2. She is on long-term corticosteroid therapy
• 3. She engages in excessive exercise
• 4. She is taking too much vitamin K
Answer: 2
• Answer: 2 - She is on long-term corticosteroid therapy
• Explanations: Side effects of steroids include hyperglycemia, insulin
resistance, peptic ulcer, and diabetes mellitus. Other side effects
include osteoporosis, anxiety, colitis, hypertension, erectile
dysfunction, amenorrhea, and retinopathy. Steroids can severely
affect mood- which includes depression, psychosis, and even rage.
The side effects of corticosteroids are not immediate and may take a
few weeks or months to develop.
Question 578
• Question 578: An off-duty clinician is riding his bike. He comes across
another biker who has fallen off his bike and has an obvious deformity to
his wrist. Which of the following is most accurate about this situation?
• Choices:
• 1. The clinician is legally obligated to splint the wrist and arrange to
follow up for the victim
• 2. The clinician may help the victim even if he refuses aid
• 3. The clinician is protected by Good Samaritan laws only if he is trained
in this specific area of medicine
• 4. The clinician has a moral obligation to help the victim
Answer: 4
• Answer: 4 - The clinician has a moral obligation to help the victim
Explanations: Most western countries recognize the moral duty to
stop and treat rather than a legal requirement. Few jurisdictions have
created an affirmative duty for a medical professional to provide care
in the absence of an established patient relationship. A few states
require healthcare professional to stop and render aid during an
emergency. This only applies if this can be accomplished without
placing themselves in danger. Generally, if the victim is unconscious
and not responsive, a good samaritan can help them on the grounds
of implied consent. If the person is conscious and can reasonably
respond, a would-be rescuer should ask permission first.
Question 579
• Question 579: Why would a patient with a history of pulmonary
impairment not be given bleomycin sulfate?
• Choices:
• 1. Pulmonary fibrosis
• 2. An increase in vital capacity
• 3. Pulmonary alkalosis
• 4. Pneumothorax
Answer: 1
• Answer: 1 - Pulmonary fibrosis
• Explanations: Patients with a history of pulmonary impairment should
not receive bleomycin sulfate since pulmonary fibrosis may occur.
There may be a decrease, not increase, in vital capacity due to the
possibility of pulmonary fibrosis. Pulmonary acidosis, not alkalosis, is
likely to occur since carbon dioxide may build up. Pneumothorax is
associated with a perforation through the chest wall. It is a collection
of air or gas in the pleural cavity.
Question 580
• Question 580: A 42-year-old female patient presented to the clinic with mild temporoparietal pain,
tinnitus, denasalization of voice, blood-stained sputum, and progressive unilateral deafness. The
patient further explained that she underwent a kidney transplant at the age of 27 years. On
examination, the patient had a blood pressure of 130/80mmHg with a pulse of 94/min. On further
examination, multiple unilateral neck masses involving the upper jugular and the supraclavicular
lymph nodes were observed. On investigation, the patient had slightly elevated levels of creatinine
with complete blood count in the normal range. Imaging studies showed a poorly circumscribed
mass extending from the posterior nasopharyngeal wall, and consequently, treatment was started.
Which of the following is the most likely instigating cause of the pathology?
• Choices:
• 1. Alcohol abuse
• 2. Tobacco intake
• 3. Radiation exposure
• 4. Infection with Epstein Barr virus
Answer: 4
• Answer: 4. Infection with Epstein Barr virus Explanations: The history
of organ transplant is suggestive of immunosuppression, which incurs
repeated EBV infections. The presence of trotters triad is highly
consistent with nasopharyngeal carcinoma (NPC). Trotter’s triad is a
combination of conductive deafness, palatal palsy, and
temporoparietal pain. The most common nodal involvement is upper
jugular or apical nodes of the posterior triangle. The supraclavicular
nodal involvement signifies advanced disease as they are the last to
get involved.
Question 581
• Question 581: What is the most appropriate intervention in the case
of an adult female with metastatic cancer who is complaining of
severe pain despite IV morphine every four hours?
• Choices:
• 1. Order a patient controlled analgesia (PCA) pump with a NSAID
• 2. Change the analgesic to fentanyl
• 3. Add a scheduled benzodiazepine
• 4. Give a morphine bolus and start patient controlled analgesia
Answer: 4
• Answer: 4 - Give a morphine bolus and start patient controlled
analgesia
• Explanations: To obtain adequate pain control, a bolus of opioids
should be given and then maintained with a morphine PCA pump.
PCA pumps are important for maintenance of pain control but the
small doses are not adequate to achieve pain relief without a bolus.
Another option is to start the patient on a NSAID and titrate till pain
relief is obtained. One may want to change the morphine to a long
acting formula. It is vital to provide adequate analgesia for break
through pain.
Question 582
• Question 582: Hoarseness may be the first sign of which of the
following?
• Choices:
• 1. Laryngeal carcinoma
• 2. Asthma
• 3. Vasomotor rhinitis
• 4. Chronic obstructive pulmonary disease (COPD)
Answer: 1
• Answer: 1 - Laryngeal carcinoma
• Explanations: Hoarseness may be the first sign of laryngeal carcinoma.
Other clinical manifestations of laryngeal carcinoma include halitosis,
painful swallowing, dyspnea, and a palpable lump in the neck. The
most common type of laryngeal cancer is squamous cell carcinoma.
Total or partial laryngectomy are the treatments of choice for
laryngeal carcinoma.
Question 583
• Question 583: A 55-year-old male with a 75 pack-year history of
smoking presents to discuss screening options to identify lung cancer
early. Which of the following statements is the best advice to give
him?
• Choices:
• 1. Annual low-dose CT scan
• 2. Annual chest radiograph
• 3. No screening exam has been shown to improve cancer mortality
• 4. Annual sputum cytology
Answer: 1
• Answer: 1 - Annual low-dose CT scan
• Explanations: Low-dose CT scanning has been shown to be of benefit
for patients between 55 and 74 years of age with a 30 pack-year
history of smoking. Chest radiographs and sputum cytology have not
shown to be an effective means of screening. Because the majority of
lung nodules are benign, patients may undergo unnecessary testing
including biopsies. Screening for lung cancer also requires clinical
judgment.
Question 584
• Question 584: What is the most common type of bladder cancer after
infection with Schistosomiasis haematobium?
• Choices:
• 1. Transitional cancer
• 2. Squamous cell
• 3. Adenocarcinoma
• 4. Carcinoma in situ
Question 584
• Question 584: What is the most common type of bladder cancer after
infection with Schistosomiasis haematobium?
• Choices:
• 1. Transitional cancer
• 2. Squamous cell
• 3. Adenocarcinoma
• 4. Carcinoma in situ
Answer: 2
• Answer: 2 - Squamous cell
Question 586
• Question 586: A Pancoast tumor typically presents with which of the
following findings?
• Choices:
• 1. Horner syndrome
• 2. Digital clubbing
• 3. Ectopic hormone secretion
• 4. Tracheal deviation
Answer: 1
• Answer: 1 - Horner syndrome
Question 587
• Question 587: A 70-year-old female undergoes a hysterectomy, salpingo-
oophorectomy, pelvic and aortic lymph node dissection and omentectomy for a
malignant uterine neoplasm. The final pathology report demonstrates a primary
serous endometrial carcinoma. Metastatic disease is noted in several lymph nodes and
the omentum. Which of the following is the most prevalent genetic mutation
identified in uterine serous carcinomas?
• Choices:
• 1. Phosphatase and tensin homolo
• g (PTEN)
• 2. Breast Cancer gene 1 (BRCA)
• 3. Tumor protein 53 (p53)
• 4. Breast Cancer gene 2 (BRCA2)
Answer: 3
• Answer: 3 - Tumor protein 53 (p53)
• Explanations: Uterine serous carcinomas behave in an aggressive fashion and
are more likely to present with advanced disease. p53 mutations are
associated with 93-100% of serous carcinomas whereas p53 expression is
normal in both clear cell and high-grade endometrioid carcinomas. p53 is a
tumor suppressor gene, and a point mutation or frame-shift mutation in p53
is associated with more advanced and aggressive endometrial cancers. p53 is
a tumor suppressor gene commonly mutated in ovarian serous
• carcinomas.Uterine serous carcinomas behave much like ovarian serous
carcinomas and are treated with similar chemotherapeutic regimens. Like
serous ovarian cancers, uterine serous carcinomas have an increased rate of
mutations in p53.
Question 588
• Question 588: A 47-years-old female with the past medical history significant for well-
controlled hyperthyroidism, diabetes mellitus, and hypertension came to her primary
care clinic for the gradually worsening symptoms of hoarseness, cough, episodic
palpitations, dysphagia and dilated pupils for the past two months. She says that the
symptoms used to be intermittent but have now started getting worse are constantly
present. An MRI scan showed a mass in her neck, pushing the internal carotid artery
anteromedially and internal jugular vein posterolaterally. Which of the following is the
expected diagnosis on the lesion biopsy?
• Choices:
• 1. Carotid body paraganglioma
• 2. Glomus vagale
• 3. Papillary thyroid cancer
• 4. Jugular-tympanic paragangliomas
Answer: 2
• Answer: 2 - Glomus vagale
• Explanations: Carotid body mass is generally located at the carotid bifurcation and displaces the
external carotid artery anteromedially and internal carotid artery posterolaterally. Glomus
vagale is the rare but most common solitary tumor of post-styloid parapharyngeal space (PPS),
which causes hoarseness and mass effect due to vocal cord paralysis. It pushes the internal
jugular vein posterolaterally and the internal carotid artery anteromedially. CT scan and MRI are
very sensitive for the diagnosis due to the increased vascularity of this tumor. Treatment is
usually surgical resection but carries a high risk of deficits in the tenth cranial nerve distribution.
Although symptoms of hoarseness, cough, episodic palpitation, dysphagia, and dilated pupils
can be seen in papillary thyroid cancer, the course of symptoms, location of the tumor,
and presentation directs towards glomus vagale paraganglioma.
• Absence of pulsatile tinnitus, in this case, points towards glomus vagale rather than jugular-
tympanic paragangliomas, which can be diagnosed early due to classic symptoms of pulsatile
tinnitus with or without conductive hearing loss.
Question 589
• Question 589: What is the second most common cause of cancer
death in females?
• Choices:
• 1. Lung
• 2. Ovarian
• 3. Colon
• 4. Breast
Answer: 4
• Answer: 4 - Breast
• Explanations: Lung cancer is the leading cause of death in females.
The second leading cause of death in females is breast cancer. The
next leading cause of death in females is colon cancer.
Question 590
• Question 590: Which is the most common malignant lesion of the
spleen?
• Choices:
• 1. Metastatic cancer
• 2. Lymphoma
• 3. Adenocarcinoma
• 4. Sarcoma
Answer: 2
• Answer: 2 - Lymphoma
• Explanations: Lymphomas are the most common malignant spleen
lesions and mimic an abscess. Lymphomas of the spleen may be
either one large lesion or numerous discrete, small masses.
Metastases do occur in the spleen and include masses from
melanomas, breasts, lungs, the colon, the prostate, and the uterus. A
variety of cysts occur in the spleen and present with left upper
quadrant discomfort and pain. These cysts are easily diagnosed based
on CT scan features and they can even be aspirated.
Question 591
• Question 591: A 65-year-old man presents with nausea and vomiting after surgery
for right knee replacement. His vital signs include a blood pressure of 139/86
mmHg, a temperature of 98.8 F, a pulse of 79 beats per minute, and a respiratory
rate of 21 breaths per minute. The physical exam notes the normally healing
surgical incisions on the right knee. The patient is administered an anti-emetic
medication for treatment. Which of the following is the most likely mechanism of
action of the anti-emetic medication?
• Choices:
• 1. Ventroposterolateral thalamic nucleus activator
• 2. Chemoreceptor trigger zone blocker
• 3. Substantia nigra activator
• 4. Raphe nucleus blocker
Answer: 2
• Answer: 2 - Chemoreceptor trigger zone blocker
• Explanations: Prochlorperazine blocks dopamine receptors (D2) in the
chemoreceptor trigger zone and is indicated to treat nausea and
vomiting associated with surgery, chemotherapy, radiation, and other
conditions. It is not recommended for nausea and vomiting associated
with pregnancy. It is also indicated to treat psychosis. Off label, it is
used to treat migraines. At low doses, about 2% of patients
experience extrapyramidal side effects, but at high doses, it can be as
much as 40%. Neuroleptic malignant syndrome, tardive dyskinesia,
acute dystonia, and parkinsonism are some of the significant adverse
effects.
Question 592
• Question 592: Which of the following is a risk factor for prostate
cancer?
• Choices:
• 1. A blood relative affected with the disease
• 2. Smoking
• 3. Alcohol use disorder
• 4. Benign prostatic hyperplasia
Answer: 1
• Answer: 1 - A blood relative affected with the disease
• Explanations: Having a first-degree relative with prostate cancer is a
risk factor. The cause of prostate cancer is unclear. The primary risk
factors are obesity, age, and family history. Prostate cancer is
uncommon in men under 45, but incidence increases with age. The
average diagnostic age is 70. Men who have first-degree family
members with prostate cancer have double the risk of getting the
disease. Risk increases with an affected brother more than an affected
father. Males with high blood pressure are more likely to develop
prostate cancer. Prostate cancer is associated with lack of exercise.
Question 593
• Question 593: Which drug is cardiotoxic?
• Choices:
• 1. Methotrexate
• 2. Aspirin
• 3. Esomeprazole
• 4. Doxorubicin
Answer: 4
• Answer: 4 – Doxorubicin
• Explanations: The most common cause of cardiac toxicity in cancer
patients is treatment with chemotherapy drugs called anthracyclines.
Doxorubicin is a frequently prescribed anthracycline. The most
dangerous side effect of doxorubicin is cardiomyopathy, causing
congestive heart failure. The drug dexrazoxane is used to mitigate
doxorubicin's cardiotoxicity. Doxorubicin can also cause typhlitis, an
acute life-threatening infection of the bowel. Some patients may
develop skin eruptions on the palms of the hand or soles of the feet,
swelling, pain, and erythema. Due to these side effects and its red
color, doxorubicin has earned the nickname 'red devil' or 'red death.'
Question 594
• Question 594: Which is a known side effect of doxorubicin?
• Choices:
• 1. Cardiotoxicity
• 2. Pulmonary fibrosis
• 3. Pulmonary hypertension
• 4. Renal failure
Answer: 1
• Answer: 1 - Cardiotoxicity
• Explanations: Doxorubicin is known to have a cardiotoxic effect.
Doxorubicin is known to cause heart damage by induction of free
radicals. The cardiotoxicity may occur immediately or even be delayed
for a few years. Patients must undergo an ECHO or a MUGA scan prior
to starting doxorubicin.
Question 595
• Question 595: A 16-year-old boy with Down syndrome is brought to the clinic because of
fever, fatigue, and generalized malaise for the past few days. On examination, his
conjunctiva is pale, and gum hypertrophy is present. Several bruises and petechiae are
present on his lower limbs. Abdominal examination reveals an enlarged spleen. Full blood
count shows Hb 8.6 g/dL, platelet count 66 × 109/L, WBC count 120 × 109/L, and
absolute neutrophil count (ANC) 480 cells/µL. Peripheral smear shows blast cells in
abundance. Bone marrow aspirate: 30% blasts with red needle-like cytoplasmic
inclusions. What is the most likely diagnosis?
• Choices:
• 1. Acute lymphoblastic leukemia (ALL)
• 2. Chronic lymphocytic leukemia (CLL)
• 3. Acute myeloblastic leukemia (AML)
• 4. Chronic myeloid leukemia (CML)
Answer: 3
• Answer: 3 - Acute myeloblastic leukemia (AML)
• Explanations: Children with Down syndrome and other inherited conditions
are predisposed to the development of myeloid proliferation and are at an
increased risk of developing leukemia. The patients with AML present with
the symptoms of cytopenias, including pallor, fever, lethargy, petechiae, easy
bruising, and recurrent infections. Gum hypertrophy and bleeding may occur
from the infiltration of blast cells. White blood cell counts vary, but the ANC is
often < 500 cells/µL. Myeloblasts may be seen in peripheral smears. A bone
marrow aspirate with a biopsy that shows =20% myeloblasts is diagnostic for
AML. The presence of Auer rods (red needle-like structures composed of
azurophilic granular material in the cytoplasm of myeloblasts) is another
diagnostic feature seen in AML.
Question 596
• Question 596: Which of the following is best to increase appetite in
patients with cancer who are receiving palliative or hospice care?
• Choices:
• 1. Increasing carbohydrate in the diet
• 2. Controlling nausea and vomiting and adding dexamethasone
• 3. Providing adequate pain relief
• 4. Tetrahydrocannabinol (THC) or megestrol acetate
Question 597
• Question 597: Which of the following drugs is an alkylating agent?
Choices:
• 1. Bleomycin
• 2. Vincristine
• 3. Tamoxifen
• 4. Busulfan
Answer: 4
• Answer: 4 – Busulfan
• Explanations: Busulfan is an alkylating agent that is cell cycle-phase
nonspecific. Its predominant actions are against granulocytes. It
alkylates thiol groups and causes nucleoprotein cross-linking. It can
cause severe bone marrow depression during the 2nd or 3rd week.
Monitoring is necessary. Busulfan resistance can develop.
Question 598
• Question 598: Which medication decreases the toxicity of
cyclophosphamide to the bladder?
• Choices:
• 1. Naloxone
• 2. Sodium bicarbonate
• 3. Furosemide
• 4. Mesna
Answer: 4
• Answer: 4 - Mesna
• Explanations: Cyclophosphamide can cause hemorrhagic cystitis
necessitating discontinuation of the medication. Most patients will
have resolution of symptoms. Microhematuria can persist long after
discontinuation. Up to 55% of patients in who continue the medication
have persistent symptoms. Mucosal inflammation and extensive
chronic bleeding can result in long-term cystitis, irreversible bladder
fibrosis, bladder contraction, and bladder cancer. High dose mesna is
uroprotective against acrolein. Mesna has a sulfhydryl group that binds
acrolein in collecting system and detoxifies it. Mesna is also used with
ifosfamide to protect the bladder. It can be given PO, IV, or SQ.
Question 599
• Question 599: A young male with anterior mediastinal mass has elevated
levels of both HCG and AFP. CT scan of the chest reveals a large mass
extending into the right chest. The patient was administered
multimodality chemotherapy for two months. The levels of both HCG and
AFP returned to normal. What is the next step in therapy?
• Choices:
• 1. Debulking surgery
• 2. Debulking surgery followed by radiation
• 3. Further chemotherapy
• 4. Observe
Answer: 1
• Answer: 1 - Debulking surgery
• Explanations: The patient has a mediastinal germ cell tumor. If the
tumor markers return to normal, a CT scan is done and all residual
tissue is removed. This residual tissue is generally a teratoma or scar
tissue. Occasionally the CT scan may not show residual tissue and
surgery is not required.
Question 600
• Question 600: Which characteristic of a lymph node is suspicious for
malignancy?
• Choices:
• 1. Cystic nature
• 2. Hardness
• 3. Redness
• 4. Presence of fluid
Answer: 2
• Answer: 2 - Hardness
• Explanations: Lymph nodes associated with a malignancy are usually
2-3 cm and feel very hard to palpation. The lymph nodes may be
adhered to the underlying tissues. If the cause is an infection, the
lymph nodes may be tender to palpation and there may evidence of
erythema in the surrounding area. Other physical findings that may
raise suspicion of a systemic disorder include presence of
splenomegaly.
Question 601
• Question 601: Which type of breast cancer has the best prognosis?
Choices:
• 1. Paget's
• 2. Inflammatory breast cancer
• 3. Mucinous
• 4. Medullary
Answer: 3
• Answer: 3 – Mucinous
• Explanations: Mucinous is a rare type of breast cancer often seen in
the 7th decade of life. It may present with a palpable mass with rare
calcifications. Mucin production is the hallmark and is usually ER and
PR positive. These cancers over express glycoproteins MUC 2 and
MUC 6. Mucinous cancers have an excellent prognosis with a 80%
survival at 10 years.
Question 602
• Question 602: A 36-year-old male is evaluated for the fatigue of 3 months duration. He
admits to easy bruising, rash over his lower extremities, and frequent respiratory infections.
He has rheumatoid arthritis, schizophrenia, and history of Rocky Mountain Spotted Fever
when he was 5 years old requiring antibiotics. Medications are chlorpromazine and gold. He
works at a paint manufacturing company. Temperature is 38°C (100.4°F), blood pressure is
100/58 mmHg, pulse rate is 94/min and regular, respiratory rate is 17/min, and oxygen
saturation is 92% on ambient air. Physical examination reveals a gentleman in no acute
distress. There are conjunctival pallor and mild oozing of blood at gingival mucosa. There
are rales in the left lower lung field. The stool is guaiac negative. Petechiae is present over
bilateral lower extremities. Chest x-ray demonstrates consolidation in the left lower lung.
Results of laboratory studies are notable for hemoglobin 5.5 g/dL, platelets 50000, and
white blood cell count 1000 with 30% neutrophils. The peripheral smear is negative for
blasts or schistocytes. Bone marrow biopsy demonstrates paucity of cells with fat
infiltration. What would be the most common cause of death in this patient?
• Choices:
• 1. Cerebral ischemia
• 2. Thrombosis
• 3. Heart failure
• 4. Infection
Answer: 4
• Answer: 4 – Infection
• Explanations: In patients with aplastic anemia, there is also anemia
secondary to decreased red blood cell production. This can cause cerebral
ischemia secondary to global hypoperfusion. This is not the most common
cause of death in patients with aplastic anemia. Thrombosis is not the most
common cause of death in patients with aplastic anemia. Anemia in the
setting of aplastic anemia can lead to high-output heart failure as the heart
tries to increase cardiac output to perfuse the body and wears out. This is
not the most common cause of death. Aplastic anemia often results in
morbidity and mortality secondary to infection. This is in part secondary to
leukopenia. Thrombocytopenia can result in bleeding, another cause of
death.
Question 603
• Question 603: A 2-year-old male is brought by his mother to the healthcare provider for a well-
child visit. He can speak in 2-to-4 word sentences and has 50% intelligible language. He is
coloring with crayons that he holds with his fingers and can run and jump with both feet.
Recently, however, the mother says that he has been bumping into furniture at home. He also
often trips and falls. He woke up the last two mornings and vomited, clear, non-bilious fluid. He
has been afebrile and has had normal bowel and bladder function. On exam, his height, weight,
and head circumference are within the 50% percentile for age. His respiratory rate is 35/min,
heart rate is 75/min, and his blood pressure is 115/75 mmHg. What is the next best step in the
evaluation of this patient?
Choices:
• 1. Refer to neurology for full neurologic assessment
• 2. MRI with and without contrast
• 3. Prescribe anti-emetics and observe for the next 3 days
• 4. Diagnostic lumbar puncture
Answer: 2
• Answer: 2 - MRI with and without contrast
• Explanations: While this male is developing normally, his recent symptoms are concerning for elevated
intracranial pressure leading to morning emesis from pressure buildup. The most crucial step to take at
this point is to rule out a CNS tumor with an MRI. Contrast that is rim-enhancing might delineate the
abnormal blood supply for the tumor. The most common malignant brain tumor of childhood is
medulloblastoma. It is subdivided into wingless (WNT), sonic hedgehog (SHH), group 3, and group 4
tumors. At this early stage in diagnosis, the subtype cannot be determined simply by MRI. Instead,
cytogenetic studies, including methylation profiling, copy number variation, and immunohistochemistry,
help with subgrouping. While toddlers are learning to navigate their surroundings and are prone to falls,
this child's symptoms suggest that he's either having a hard time with depth perception or with his
coordination, both suggesting a posterior fossa, cerebellar-type tumor.

• Simply observing with conservative management (anti-emetics) is not a reasonable course of action given
the high risk of missing occult malignancy. While a neurologist is an essential partner in the management
of this patient, imaging studies are indicated given the patient's symptoms. Moreover, without imagining,
doing a lumbar puncture in a patient with intracranial hypertension can lead to herniation and death.
Question 604
• Question 604: Methotrexate, commonly used for cancer
chemotherapy, competitively inhibits what enzyme to reduce the
levels of tetrahydrofolate?
• Choices:
• 1. Dihydrofolate reductase
• 2. Thymidylate synthase
• 3. Pteridine reductase
• 4. Methionine synthase
Answer: 1
• Answer: 1 - Dihydrofolate reductase
• Explanations: Methotrexate involves the competitive inhibition of
dihydrofolate reductase, which decreases intracellular tetrahydrofolate.
Thymidylate synthase is involved in the reductive methylation of dUMP
to dTMP. Inhibition of this enzyme will not decrease intracellular
tetrahydrofolate. Pteridine reductase is involved in the reduction of 5,
6, 7, 8-tetrahydrobiopterin (THBP) to biopterin. While this enzyme is
involved in folate metabolism, inhibition of this enzyme will not
decrease intracellular tetrahydrofolate. Methionine synthase catalyzes
the regeneration of methionine from homocysteine. Inhibition of this
enzyme will not decrease intracellular tetrahydrofolate.
Question 605
• Question 605: According to the American Cancer Society guidelines,
when should an average-risk, 45-year-old woman have a screening
mammogram?
• Choices:
• 1. Beginning at 50 years of age
• 2. Every 2 years
• 3. Every year
• 4. Every 3 years
Answer: 3
• Answer: 3 - Every year
• Explanations: According to the American Cancer Society, women 40 to
44 years of age should have the choice to start annual breast cancer
screening with mammograms. They also recommend that women age
45 to 54 should get mammograms every year. Women 55 years of age
and more should switch to mammograms every 2 years or can
continue yearly screening. Screening should continue as long as a
woman is in good health and is expected to live 10 more years or
longer.
Question 606
• Question 606: A 44 year old male complains of sudden loss of hearing
and a persistent headache. A few weeks prior he had been having
episodes of dizziness and vertigo. A CT scan revealed a mass in the
internal acoustic meatus. If this is a primary tumor, which is most likely?
• Choices:
• 1. Melanoma
• 2. Schwannoma
• 3. Glioblastoma multiforme
• 4. Meningioma
Answer: 2
• Answer: 2 - Schwannoma
• Explanations: Because of the high number of nerves, the internal
acoustic meatus is often prone to nerve malignancies like
schwannomas and neuromas. These malignancies are best visualized
by CT or MRI. The individuals often present with ear pain, dizziness,
vertigo, or loss of hearing. Surgical excision can provide a cure but
hearing loss may be permanent. Stereotactic radiotherapy is another
option.
Question 607
• Question 607: You are doing a resection of a thymoma and the tumor
has partially invaded the phrenic nerve. How do you proceed?
• Choices:
• 1. Resect the tumor with the phrenic nerve
• 2. Leave the phrenic nerve alone, and give postoperative
chemotherapy
• 3. Perform an interposition nerve graft to the phrenic nerve
• 4. Give radiation therapy
Answer: 1
• Answer: 1 - Resect the tumor with the phrenic nerve
• Explanations: One phrenic nerve can be resected with a tumor.
Weaning will be difficult, but it is possible. As complete a resection as
possible should be done. The innominate vein can also be excised if it
is involved with the tumor. This is now considered a stage 3 cancer
and postoperative radiation should be considered to prevent local
recurrence.
Question 608
• Question 608: Which adverse effect is associated with doxorubicin?
Choices:
• 1. Pulmonary fibrosis
• 2. Cardiac toxicity
• 3. Otic toxicity
• 4. Gout
Answer: 2
• Answer: 2 - Cardiac toxicity
• Explanations: While an effective agent against a number of
malignancies, doxorubicin is well known for its cardiotoxicity. This
cardiotoxicity is cumulative and irreversible. The prevailing theory is
that this toxicity results from oxidative stress in cardiac myocytes and
can lead to congestive heart failure. Doxorubicin is used in the
treatment of certain leukemias, Hodgkin lymphoma, and other
cancers.
Question 609
• Question 609: Why is it inadvisable to use a peripheral vein as the
administration site for total parenteral nutrition (TPN)?
• Choices:
• 1. They are unable to accept the large gauge needles required for
TPN administration
• 2. Peripheral veins are prone to rupture from the osmotic pressure of
TPN fluids
• 3. They are prone to irritation and inflammation from TPN fluids
• 4. The risk of embolism is too great
Answer: 3
• Answer: 3 - They are prone to irritation and inflammation from TPN
fluids
• Explanations: Most TPN fluids are both hypertonic and hyperosmotic.
Peripheral venous vasculature lacks sufficient blood flow to dilute and
render the TPN non-irritating to the tunica intima of the vessel. This
can lead to phlebitis, occlusion, and pain. Exceptions are mildly
hypertonic solutions with less than 10% dextrose.
Question 610
• Question 610: A 65-year-old male presents to the hospital complaining of central chest
pain for one week. He also complains of shortness of breath (SOB) associated with his
chest pain that has been worsening over the same timespan. He has a medical history
significant for COPD, hypertension, hyperlipidemia, lymphoma in remission for the
past two years, and diabetes type 2 with a recent HbA1c of 9.2%. He states he has not
been compliant with his medications for the past month due to insurance issues but
was previously taking a chemotherapeutic agent for his lymphoma. He currently takes
aspirin, atorvastatin, amlodipine, hydrochlorothiazide, men's multivitamins daily, and
insulin. On exam, vitals are within normal limits, and there are bilateral decreased
breath sounds on auscultation of the lungs, the cardiovascular exam is unremarkable,
and his skin is dry but normal in color. On chest x-ray, there is bilateral vascular
congestion and decreased aeration throughout the lung fields. Which of the following
indicates cardiotoxicity due to the medication he was previously taking for his
lymphoma?
• Choices:
• 1. Elevated cardiac troponin levels
• 2. T-wave inversions in V1, V2, and V3 leads
• 3. Decreased left ventricular ejection fraction
• 4. Prolonged QRS complex in leads V1 and V2
Answer: 3
• Answer: 3 - Decreased left ventricular ejection fraction
• Explanations: Due to the cardiotoxic profile of adriamycin, baseline left
ventricular ejection fraction (LVEF) should be done prior to starting Adriamycin
and should be monitored routinely using an echocardiogram. An LVEF drop of
>10% from baseline is considered to indicate cardiotoxicity from adriamycin
and immediate intervention by physicians and the healthcare team is required.
An alternative to echocardiogram monitoring is radionuclide ventriculography
such as multigated acquisition angiogram (MUGA) or equilibrium radionuclide
angiogram (ERNA) scans although less frequently used. Echo monitoring in
patients being treated with or have previously been treated with adriamycin is
important to continually monitor their cardiac reserve. Troponin levels and
ECG abnormalities are not specific for adriamycin toxicity.
Question 611
• Question 611: A 45-year old female has a pituitary mass. What
hormone is most commonly secreted by this mass?
• Choices:
• 1. Growth hormone
• 2. Thyroid hormone
• 3. Prolactin
• 4. Adrenocorticotropic hormone
Answer: 3
• Answer: 3 – Prolactin
• Explanations: The hormone most often secreted by tumors of the
pituitary gland is prolactin. Prolactinomas are asymptomatic. A
prolactinoma is classified based on its size. A microprolactinoma is
less than 10 millimeters in diameter. A macroprolactinoma is greater
than 10 millimeters in diameter. Bromocriptine usually is the
treatment of choice for a prolactinoma. The drug has withstood the
test of time and is relatively safe.
Question 612
• Question 612: Of which of the following is Bowen disease an in situ
presentation?
• Choices:
• 1. Benign metaplasia
• 2. Basal cell carcinoma
• 3. Squamous cell carcinoma
• 4. Malignant melanoma
Answer: 3
• Answer: 3 - Squamous cell carcinoma
• Explanations: Bowen disease is squamous cell carcinoma in situ. The
potential for considerable lateral growth is great. Bowen disease is
most common in the sun-exposed areas of fair-skinned persons. The
highest incidence is in the geriatric population. In about 3% of people,
squamous cell carcinoma will progress to invasive squamous cell
cancer.
Question 613
• Question 613: Which of the following neoplastic drug often used to
treat seminomas works by inhibiting topoisomerase II?
• Choices:
• 1. Cisplatin
• 2. Ciprofloxacin
• 3. Etoposide
• 4. Cyclophosphamide
Answer: 3
• Answer: 3 - Etoposide
• Explanations: Etoposide is an anti-cancer drug that inhibits
topoisomerase II. The drug is widely used to treat many solid cancers
and often given in combination with other drugs. When given
intravenously, it can lower blood pressure. Patients should be
adequately hydrated when taking etoposide to prevent kidney
damage.
Question 614
• Question 614: The most common area of metastases is to which part
of the eye?
• Choices:
• 1. Choroids
• 2. Pupil
• 3. Canals of Schlemm
• 4. Iris
Answer: 1
• Answer: 1 - Choroids Explanations: The frequency of choroidal
metastasis is estimated to be approximately 2% to 7%. If all
intraocular metastases are considered, this number is approximately
20%. Tumors from the breast and lung may present with metastases
to the eye. The majority of patients with choroidal metastases are
asymptomatic. Choroidal metastases can cause retinal detachment.
Question 615
• Question 615: Which risk factor has the greatest influence on the
development of breast cancer?
• Choices:
• 1. Prior radiation therapy
• 2. Poor diet
• 3. Family history
• 4. Age
Answer: 4
• Answer: 4 - Age Explanations: Age has the greatest influence on the
development of breast cancer. Most breast cancers are diagnosed
after 50 years of age. Exposure to endogenous and exogenous
estrogen also affects risk. Approximately, 10% of cases are associated
with germline mutations such as BRCA1 and BRCA2. BRCA mutation
carriers carry a 40% to 85% lifetime risk of developing breast cancer
and a 25% to 65% lifetime risk of developing ovarian cancer.
Question 616
• Question 616: Which of the following antineoplastic drugs is most
likely responsible for peripheral neuropathy?
• Choices:
• 1. Bleomycin
• 2. Cisplatin
• 3. Cyclophosphamide
• 4. Vincristine
Answer: 4
• Answer: 4 - Vincristine Explanations: Vincristine is a common drug
that is implicated in peripheral neuropathy. Numbness and tingling
sensations are common with higher doses. Paralytic ileus can also
develop in some patients. Cisplatin causes peripheral neuropathy less
frequently. Bleomycin is associated with pulmonary toxicity.
Question 617
• Question 617: According to the American Cancer Society, at what age
should prostate cancer screening initially be considered for those at
average risk of developing prostate cancer?
• Choices:
• 1. 30 years of age
• 2. 40 years of age
• 3. 50 years of age
• 4. 60 years of age
Answer: 3
• Answer: 3 - 50 years of age Explanations: In men without risk factors for
prostate cancer, the American Cancer Society (ACS) currently recommends that
screening procedures initially be considered at the 50 years of age. The ACS
further recommends that the decision to undergo screening be made by the
patient after thorough discussion with a healthcare provider. Screening should
consist of a prostate-specific antigen (PSA) blood test and a digital rectal
examination. As of May 2018, the U.S. Preventive Services Task Force states "For
men aged 55 to 69 years, the decision to undergo periodic prostate-specific
antigen (PSA)–based screening for prostate cancer should be an individual one."
In 2013 the American Urological Society recommended against screening men
under 40 years of age and 40 to 54 years of age with average risk. They
recommend a discussion between the provider and patient for men 55 to 69
years of age.
Question 618
• Question 618: Which hormone stimulates gastrin secretion in patients
with a gastrinoma?
• Choices:
• 1. Somatostatin
• 2. Prolactin
• 3. Secretin
• 4. Pancreatic polypeptide
Answer: 3
• Answer: 3 - Secretin Explanations: Secretin stimulation of gastrin is a
very useful provocative test in patients with a gastrinoma.
Somatostatin inhibits gastrin secretion. Prolactin promotes lactation
in the breast and does not stimulate gastrin. Pancreatic polypeptide
has no effect on gastrin secretion.
Question 619
• Question 619: A 72-year-old male patient was diagnosed with glioblastoma
multiforme and underwent surgical resection. He is planned to be
commenced on concurrent radiotherapy and chemotherapy. What is the
mechanism of action of most common systemic therapy recommended for
treatment according to the national comprehensive cancer network (NCCN)
guidelines?
• Choices:
• 1. Inhibition of protein synthesis
• 2. Vascular endothelial growth factor inhibitor
• 3. Nucleic acid breakdown and apoptosis
• 4. Epidermal growth factor receptor inhibitor
Answer: 3
• Answer: 3 - Nucleic acid breakdown and apoptosis Explanations:
Mechanism of action of Temozolomide is through alkylation
(methylation) of DNA, leading to ds DNA breaks and apoptosis.
Temozolomide is used commonly in GBM as systemic therapy. It crosses
the blood-brain barrier and is an antineoplastic/alkylating agent.
Temozolomide is a prodrug that is non enzymatically converted to the
active drug. Available in oral and IV formulation with the same
bioavailability. The mechanism of action of bevacizumab is VEGF
receptor inhibition. The mechanism of action of lomustine is by
inhibition of protein synthesis. Clinical trials are done using combination
therapy with temozolomide and lomustine, but convincing evidence is
lacking.
Question 620
• Question 620: What agent has been proven partially effective as an
adjuvant treatment for colorectal cancer in a patient with
unresectable metastases?
• Choices:
• 1. Bevacizumab
• 2. Cyclophosphamide
• 3. Methotrexate
• 4. Aspirin
Answer: 1
• Answer: 1 - Bevacizumab Explanations: The goal of drug therapy is to
down-stage the tumor, induce remission, and reduce mortality.
Bevacizumab is recommended for all patients with colon cancer and
metastases. In some patients, it may extend survival for a few
months. Bevacizumab is a murine-derived monoclonal antibody that
inhibits angiogenesis and inhibits vascular endothelial growth factor.
Local recurrence is common with rectal cancer. Most patients see
recurrence within the first year after surgery.
Question 621
• Question 621: A female has been found to have a 3 cm cancerous
breast lesion. At surgery, no lymph nodes are identified. The mass is
found to be positive for HER2/Neu and negative for both estrogen and
progesterone receptors. What is the best treatment?
• Choices:
• 1. Radiation therapy
• 2. Chemotherapy
• 3. Trastuzumab
• 4. Mastectomy
Answer: 3
• Answer: 3 - Trastuzumab Explanations: Most common breast cancer is
infiltrating ductal cell. If the lesion is localized to the breast, surgery
may be curative. Chemotherapy is administered only if axillary lymph
nodes are positive. For localized tumors, radiation therapy may be
administered to prevent local recurrence. Positive HER2/neu staining
indicates that the cancer may respond to trastuzumab. If estrogen
receptor staining is positive, Tamoxifen can be administered. Patients
who have HER2 usually have a poor prognosis than patients who are
positive for the estrogen receptors.
Question 622
• Question 622: A 70-year-old male former farmer presents with a 4-month history of
abdominal pain and weight loss. Physical examination is remarkable for an enlarged
liver. A right upper quadrant ultrasound confirms a 10cm hyperechoic zone in the
left hepatic lobe. A liver biopsy is performed, and pathology reveals a highly
vascular area with blood-filled cavities without any evidence of cirrhosis. He has
never worked in an industrial setting. Which of the following chemicals is most
likely responsible for the development of the liver tumor found in this farmer?
• Choices:
• 1. Arsenic
• 2. Trichloroethylene
• 3. Methylene chloride
• 4. Carbon tetrachloride
Answer: 1
• Answer: 1 - Arsenic Explanations: Inorganic arsenic has a multitude of industrial and
agricultural uses. It is naturally occurring and has contaminated groundwater sources in
developing countries, notably Bangladesh and parts of Taiwan. Chronic inorganic arsenic
exposure has been associated with a variety of cancers to include hepatic angiosarcoma.
Hepatic angiosarcoma is a very rare hepatic malignancy (<1% of all liver malignancies)
with pathologic features notable for vascular invasion of the portal vein and hepatic vein
branches. Other potential toxic causes of hepatic angiosarcoma include thorium and vinyl
chloride. Hepatic angiosarcoma may have a latent period of 20+ years. Thus, exposures to
arsenical pesticides or arsenic-contaminated groundwater much earlier in life may still be
responsible for tumor development. Hepatic angiosarcoma has a poor prognosis with a
life expectancy of merely 6-12 months after diagnosis.
• Trichloroethylene, methylene chloride, and carbon tetrachloride are all chlorinated
industrial solvents that may result in hepatotoxicity with repeated use. However, none of
these agents is associated with the development of hepatic angiosarcoma.
Question 623
• Question 623: Vincristine is administered to a patient with a
lymphoma. How does it act?
• Choices:
• 1. Inhibiting T cells
• 2. Inhibiting purine synthesis
• 3. Inhibiting microtubule formation
• 4. Inhibiting IL2 formation
Answer: 3
• Answer: 3 - Inhibiting microtubule formation Explanations: Vincristine
inhibits microtubule formation into mitotic spindles causing cell death
at metaphase. This affects not only the fast growing malignant cells,
but also cells of the bone marrow and GI tract. It is made from the
vinca plant by coupling vindoline and catharanthine. Side effects
include peripheral neuropathy, constipation, hyponatremia, and hair
loss.
Question 624
• Question 624: A 65-year-old man with metastatic prostate cancer to
bone presents with bone pain. He has had no pain relief with aspirin.
The clinician decides to choose a drug from the next level of the
analgesic ladder. Which side effect of the next most appropriate drug is
independent of tolerance development?
• Choices:
• 1. Respiratory depression
• 2. Miosis
• 3. Pruritis
• 4. Addiction
Answer: 2
• Answer: 2 - Miosis Explanations: According to the World Health
Organization's (WHO) pain ladder for cancer pain relief in adults,
prompt oral administration of medication is warranted. Nonopioids,
such as aspirin and acetaminophen, should be given first. If necessary,
mild opioids, such as codeine, should be given next. If mild opioids are
unable to bring adequate pain relief, then strong opioids, such as
morphine, should be given until the patient is free of pain. Tolerance
does not develop to miosis and constipation. The WHO also
recommends adjuvant medications "to calm fears and anxiety" as
needed.
Question 625
• Question 625: For individuals with average risk factors, flexible
sigmoidoscopy screening should begin at what age?
• Choices:
• 1. 40 years
• 2. 50 years
• 3. 65 years
• 4. 70 years
Answer: 2
• Answer: 2 - 50 years Explanations: The USPSTF recommends screening for colorectal
cancer (CRC) using fecal occult blood testing, sigmoidoscopy, or colonoscopy, in adults,
beginning at age 50 years and continuing until age 75 years. The American Cancer
Society recommends colonoscopy every 10 years, double-contrast barium enema every
5 years, or virtual colonoscopy every 5 years. Any positive test requires colonoscopy.
Screening of persons with risk factors should begin at an earlier age, depending on the
situation. Digital rectal examinations should begin at 40 years of age. The major
complication associated with flexible sigmoidoscopy is intestinal perforation, which
occurs in < 0.05 percent of cases. Polyps are usually described as sessile, broad based,
or pedunculated. Polyps are usually classified histologically into hyperplastic
• and adenomatous types. If adenomatous polyps are noted, the patient should undergo
colonoscopy to rule out synchronous polyps located more proximally in the colon.
Question 626
• Question 626: Which drug frequently is used in combination with
cisplatin for testicular cancer? Choices:
• 1. Bleomycin
• 2. Cisplatin
• 3. 5-fluorouracil
• 4. Leuprolide
Answer: 1
• Answer: 1 - Bleomycin Explanations: Multimodality chemotherapy is
often used to treat testicular cancer. Carboplatin is often combined
with bleomycin and etoposide for the maximal kill of cancer cells. The
average survival for testicular cancer is about 95 percent. Bleomycin
binds to the ferrous ions in the DNA and generates free radicals,
which subsequently results in DNA breakdown.
Question 627
• Question 627: What is the most common histology of bladder
carcinoma?
• Choices:
• 1. Adenocarcinoma
• 2. Columnar cell
• 3. Squamous cell
• 4. Transitional cell
Answer: 4
• Answer: 4 - Transitional cell Explanations: Bladder cancer is mostly
transitional cell carcinoma. Schistosoma haematobium infection is a
risk factor for bladder cancer. It usually results in squamous cell
carcinoma of the bladder. In Western countries, the most common
risk factor is smoking. A 10 pack-year smoking history or more is
considered sufficient to put patients at risk.
Question 628
• Question 628: A 65-year-old male presents to a clinic with complaints of
difficulty in urination and hematuria for the past several weeks. He also has a
history of weight loss and fatigue. On the digital rectal exam (DRE), the prostate
is enlarged, nodular, and rock hard. His prostate-specific antigen (PSA) is 40
ng/ml. A magnetic resonance imaging (MRI) scan of the pelvis was advised.
Which of the following MRI finding is an indication of a stage III disease?
• Choices:
• 1. Regional node involvement based on node size
• 2. Evidence for distant spread
• 3. Evidence for extracapsular extension
• 4. Evidence for intracapsular confinement
Answer: 3
• Answer: 3 - Evidence for extracapsular extension Explanations: T1 and T2
cancers are limited to just the prostate and are considered "localized." T3
cancer has spread outside the prostatic capsule but has not reached the
rectum or bladder. Extracapsular spread distinguishes stage III from stage II
prostate cancer. T3 tumor extends through the prostatic capsule but does
not invade into the prostatic apex or beyond the capsule. T3a tumor
involves either unilateral or bilateral extracapsular extension. T3b tumor
invades seminal vesicle(s). Regional node involvement would represent
Jewett stage D1, stage IV prostate cancer. The distant spread would
represent Jewett stage D2, stage IV prostate cancer. Intracapsular
confinement represents Jewett stage B, stage II prostate cancer. Stage I
prostate cancer is by definition not seen at all on imaging.
Question 629
• Question 629: Which cell is the originator of a carcinoid?
• Choices:
• 1. Clara cell
• 2. Golgi body
• 3. Kulchitsky cell
• 4. Basal cell
Answer: 3
• Answer: 3 - Kulchitsky cell Explanations: Kulchitsky cells give rise to
carcinoids. Kulchitsky cells also are referred to as enterochromaffin
cells. They are a type of enteroendocrine and neuroendocrine cell.
These cells are found in aggregates along the gastrointestinal tract,
appendix, small bowel, and colon. They primarily secrete serotonin,
which acts as a paracrine hormone. In carcinoid syndrome, excess
secretion of serotonin is responsible for diarrhea, facial flushing, and
abdominal cramps.
Question 630
• Question 630: Tamoxifen works by which of the following
mechanisms?
• Choices:
• 1. Inhibiting androgen synthesis
• 2. Decreasing secretion from pituitary gland
• 3. Blocking estrogen receptors
• 4. Blocking progesterone receptors
Answer: 3
• Answer: 3 - Blocking estrogen receptors Explanations: Tamoxifen is an
oral medication used to treat and prevent breast cancer. Its active
metabolite, hydroxytamoxifen, is an antagonist of the estrogen
receptor in the breast. It has mixed agonist and antagonist actions in
other tissues. It prevents bone loss because of its mixed actions.
While it has beneficial effects on lipid profiles, it has not been shown
to reduce the risk of coronary events.
Question 631
• Question 631: In people who have had a liver transplant and are
chronically immunosuppressed, which of the following is the most
common cancer?
• Choices:
• 1. Melanoma
• 2. Lymphoma
• 3. Gallbladder cancer
• 4. Pancreatic adenocarcinoma
Answer: 2
• Answer: 2 – Lymphoma
• Explanations: Lymphomas are by far the most common cancer in
immunocompromised patients with a liver transplant. Kaposi sarcoma
does occur, but the rates are low. There is some evidence that
immunocompromised individuals may also be prone to an increased
incidence of colon cancers. Patients who have had a renal transplant
are most prone to skin cancer.
Question 632
• Question 632: A 53-year-old Hispanic woman is brought to the emergency
department after an episode of seizures. It started with an involuntary
movement in the right hand lasting for 2–3 minutes, dizziness followed by an
episode of generalized tonic-clonic seizures witnessed by her husband. She
reports having a headache while recovering from the postictal phase. She has a
history of enucleation of the right eye 23 years ago. The postsurgical history of
the enucleated eye revealed a mushroom-shaped choroidal melanoma. She is a
non-smoker and worked in a rubber factory. Vitals are stable on presentation.
Labs are within normal limits. CT chest shows large left hilar lymphadenopathy
with 3–4 soft tissue density nodules in the left lower lobe, the largest
measuring 4 cm. The liver shows a 2 cm hypodense lesion in segment 8. What is
the most likely site of metastasis of the patient's past disease?
• Choices:
• 1. Liver
• 2. Lung
• 3. Brain
• 4. Bone
Answer: 1
• Answer: 1 - Liver
• Explanations: The liver is the most common site of metastasis from
ocular melanoma (93%). Patients without liver metastases or with the
liver being not the first site of metastases have better survival. The
lung is the second most common site of metastasis, with 24%. The
bone is the third common site, with a 16% rate of occurrence. The
majority of patients with metastatic disease have metastases in
multiple sites.
Question 633
• Question 633: A 69-year-old with advanced prostate cancer has
elected to undergo medical castration with leuprolide. The FDA
recommends that patients who receive leuprolide should be closely
monitored for which of the following?
• Choices:
• 1. Gout
• 2. Fasting blood glucose
• 3. Testicular atrophy
• 4. Creatine kinase
Answer: 2
• Answer: 2 - Fasting blood glucose
• Explanations: Administration of leuprolide can induce diabetes mellitus
and increase the risk of heart disease. Thus, all patients need regular
fasting blood glucose levels and should be educated on decreasing the
risk factors for heart disease. Other blood work should include levels of
cholesterol, serum testosterone, and triglyceride levels. Testosterone is
checked to make sure it reaches castrate levels. Long-term leuprolide
therapy can also lead to osteopenia and osteoporosis. This initially can
be treated with calcium citrate supplements and vitamin D.
Consideration then should be given to adding an oral or injectable
bisphosphonate or a rank ligand inhibitor like denosumab.
Question 634
• Question 634: In which medication category is oxaliplatin correctly
categorized?
• Choices:
• 1. Anxiolytic agent
• 2. Antineoplastic agent
• 3. Anti-seizure medication
• 4. Atypical antipsychotics
Answer: 2
• Answer: 2 - Antineoplastic agent
• Explanations: Oxaliplatin is an antineoplastic agent used primarily to
treat colorectal cancer. It often is given with folinic acid and 5-
fluorouracil. It is thought to work by inhibiting DNA synthesis in
cancer cells. It is a platinum-containing compound, like cisplatin and
carboplatin.
Question 635
• Question 635: What is the most common side effect of anti-
endothelial growth factor receptor (EGFR) agents?
• Choices:
• 1. Gastrointestinal distress
• 2. Hair loss
• 3. Fatigue and muscle weakness
• 4. Skin rash
Answer: 4
• Answer: 4 - Skin rash
• Explanations: Anti-EGFR therapies have been approved for treatment
of some cancers, including lung cancer and colon cancer. Anti-EGFR
monoclonal antibodies (cetuximab, panitumumab, erlotinib, and
gefitinib) act as competitive inhibitors of EGFR ligand binding. EGFR
tyrosine kinase inhibitors (gefitinib, erlotinib, brigatinib, lapatinib) are
small molecules that bind and inhibit the EGFR intracellular tyrosine
kinase, which prevents further downstream activation. A side effect
of both of these medications is papulopustular eruptions, seen in 90%
of patients.
Question 636
• Question 636: A 65-year-old man presents to the hospital with complaints of fatigue,
night sweats, and unexplained bruising. He also notes that he has had 12 pounds of
unintentional weight loss in the past two months. Physical examination is generally
unremarkable. Routine labs are ordered and yield a leukocyte count of 66/microliter
(reference range: 4.5-11.0/microliter). A blood smear shows numerous myeloid cells of
varying maturity; subsequent genetic analysis reveals a 9:22 chromosomal translocation.
The most appropriate therapy to start in this patient belongs to which class of drugs?
• Choices:
• 1. BCR-ABL tyrosine-kinase inhibitor
• 2. Humanized monoclonal antibody against VEGF-A
• 3. A random polymer of 4 amino acids found in myelin basic protein
• 4. A synthetic version of N-acetylgalactosamine-6-sulfatase
Answer: 1
• Answer: 1 - BCR-ABL tyrosine-kinase inhibitor
• Explanations: Given the patient's history and routine lab work, it is likely that this patient has
leukemia. Clinical suspicion of leukemia combined with genetic analysis results of a 9:22
chromosomal translocation narrows the differential towards chronic myeloid leukemia (CML).
An appropriate therapy to start this patient on is Imatinib which is a BCR-ABL tyrosine kinase
inhibitor. Imatinib is a powerful targeted therapeutic drug for several types of cancer. In addition
to the prescribing physician, pharmacists and nurses are responsible for educating the patient
on the potential side effects of the drug. While most side effects occur in the first two years of
use, many patients will be on chronic therapy for many years, so it is essential to stay vigilant in
monitoring for side effects.
• A humanized monoclonal antibody against VEGF-A is bevacizumab, which is typically used as a
treatment for colorectal cancer. A random polymer of 4 amino acids found in myelin basic
protein is glatiramer acetate, which is typically used as a treatment for multiple sclerosis. A
synthetic version of N-acetylgalactosamine-6-sulfatase is Elosulfase alfa, which is typically used
as a treatment for Morquio Syndrome.
Question 637
• Question 637: What is the mode of action of paclitaxel?
• Choices:
• 1. It inhibits purine synthesis
• 2. It causes leaks in the cell membrane
• 3. It prevents the breakdown of microtubules
• 4. It blocks the use of folic acid
Answer: 3
• Answer: 3 - It prevents the breakdown of microtubules
• Explanations: Paclitaxel interferes with the normal function of
microtubule breakdown. Recent evidence shows that paclitaxel can
induce apoptosis by binding to Bcl-2. Paclitaxel is used to treat
ovarian, breast, and lung cancer. It is also used in drug-eluting stents.
Side-effects include loss of appetite, changes in taste, brittle nails, and
pain in extremities. Dexamethasone usually is given prior to paclitaxel
therapy.
Question 638
• Question 638: A female patient has a BRCA1 gene mutation. After
breast cancer, what is the next most common cancer that she may
develop?
• Choices:
• 1. Endometrial cancer
• 2. Cervical cancer
• 3. Ovarian cancer
• 4. Pancreatic cancer
Answer: 3
• Answer: 3 - Ovarian cancer
• Explanations: Women and men with a BRCA1 gene mutation are at
an increased risk of developing breast cancer and other epithelial
malignancies. In women, the next most common cancer that can
develop is ovarian cancer. Men with the BRCA gene mutations also
are predisposed to an increased risk of developing prostate cancer.
Other cancers that occur in women with a BRCA1 gene mutation
include cervical, endometrial, or pancreatic cancer.
Question 639
• Question 639: A patient receiving intravenous daunorubicin
chemotherapy for Kaposi sarcoma has extravasation of his infusion
site. What should be done?
• Choices:
• 1. Continue the infusion and take corrective action after completion
• 2. Stop the infusion and apply direct manual pressure
• 3. Continue the infusion but apply a warm compress to the infusion
site
• 4. Stop the infusion and apply a cold compress.
Answer: 4
• Answer: 4 - Stop the infusion and apply a cold compress.
Explanations: In any event, the infusion should be stopped promptly.
For daunorubicin, cold compress over the extravasation site is the
correct course of action. Some agents would require a warm
compress (e.g., vincristine). Direct pressure would further exacerbate
damaged tissue at the infusion site.
Question 640
• Question 640: What is the most serious complication of multiple
myeloma?
• Choices:
• 1. Fractures
• 2. Hypercalcemia
• 3. Renal failure
• 4. Spinal cord compression
Answer: 4
• Answer: 4 - Spinal cord compression
• Explanations: Spinal cord compression is a very serious complication
of multiple myeloma. Close to 20% of patients develop cord
compression after the onset of multiple myeloma. Pathological
fractures are also common in patients with multiple myeloma.
Multiple myeloma can also cause kidney injury via deposition of
amyloid or direct tubular injury.
Question 641
• Question 641: What is the best treatment for a patient with
chemotherapy induced nausea and vomiting?
• Choices:
• 1. Diphenhydramine
• 2. Ondansetron
• 3. Ginger
• 4. Dexamethasone
Answer: 2
• Answer: 2 - Ondansetron
• Explanations: Ondansetron is a 5HT3 receptor antagonist. It is often
used to treat chemotherapy-induced nausea and vomiting. For
chemotherapy-induced vomiting, it is often combined with aprepitant
and dexamethasone. It is also widely used in the ER to treat nausea
from gastroenteritis. The drug is also used to treat emesis in post
surgery patients.
Question 642
• Question 642: Tobacco smoking has been linked to which salivary
gland tumor?
• Choices:
• 1. Pleomorphic adenoma
• 2. Cystic adenoid
• 3. Warthin tumor
• 4. Lymphoma
Answer: 3
• Answer: 3 - Warthin tumor
• Explanations: Smoking appears to be a risk factor for development of
Warthin tumor. The Warthin lesion is bilateral in about 10-20 % of
cases. It is only found in the major salivary glands. Warthin is a benign
lesion and transformation into cancer is not possible. The majority of
patients have a normal survival rate after excision but there is a
recurrence rate of about 5%. The Warthin lesion is a soft, smooth and
well-encapsulated mass. It may contain multiple cysts.
Question 643
• Question 643: A 4-year-old was diagnosed with a brain tumor at the age of 2 and is being
seen by the provider for evaluation of constitutional symptoms. He had undergone
subtotal surgical resection of the tumor, whole axis neuroradiation, and 4-drug
chemotherapy. His MRI from 6 months ago showed minimal tumor burden within the
posterior fossa. He has now begun to have night sweats, fevers, and weight loss over the
past four months. He frequently complains of his body, hurting and refuses to play
games with the other kids. What is the most common site for extraneural metastases
with this type of malignancy?
• Choices:
• 1. Bone
• 2. Lymph nodes
• 3. Liver
• 4. Lungs
Answer: 1
• Answer: 1 – Bone
• Explanations: Extraneural metastases in pediatric medulloblastomas
are an infrequent occurrence at about 7%. The most frequent sites of
extraneural metastasis in children include bone, 78%; lymph nodes,
33%; liver, 15%; and lungs, 11%. The average time for development
following maximal surgical resection is 20 months. Survival in these
cases is dismal and in most cases, can be less than six months.
Question 644
• Question 644: A patient with colon cancer is prepared for elective
resection. Which of the following decreases the risk of postoperative
infection?
• Choices:
• 1. Preoperative antibiotics with activity against aerobes and anaerobes
• 2. No antibiotics
• 3. Postoperative antibiotics with activity against aerobes and anaerobes
over the next 72 hours.
• 4. Options 1 and 3
Answer: 1
• Answer: 1 - Preoperative antibiotics with activity against aerobes and
anaerobes
• Explanations: Broad-spectrum antibiotics with activity against aerobes
and anaerobes may be effective in cases where such flora is
anticipated. Administering antibiotics for more than 12 hours after an
elective operation should be avoided to prevent bacterial resistance.
The combination of preoperative and postoperative antibiotic
administration is unnecessary. Today because of drug resistance,
many surgeons do not prescribe preoperative antibiotics during
colonic cleansing.
Question 645
• Question 645: Which of the following agents has cardiac toxicity as its
limiting factor?
• Choices:
• 1. Bleomycin
• 2. Taxol
• 3. Asparaginase
• 4. Doxorubicin
Answer: 4
• Answer: 4 – Doxorubicin
• Explanations: Doxorubicin is an anthracycline antibiotic used to treat
a variety of cancers. Doxorubicin works by intercalating DNA. The drug
is administered intravenously and has several side effects. Cumulative
doses can cause cardiac side effects which can lead to CHF and dilated
cardiomyopathy. The damage to the heart is due to free radical.
Question 646
• Question 646: What is the most common presentation of a patient
with uterine cancer?
• Choices:
• 1. Pain
• 2. Vaginal bleeding
• 3. Dysuria
• 4. Painful intercourse
Answer: 2
• Answer: 2 - Vaginal bleeding
• Explanations: Vaginal bleeding after menopause should always be a
concern for the presence of uterine cancer. Advanced cases,
especially patients who develop uterine clear cell or papillary cell
histologies, may present with bloating, abdominal pain, and other
features of a metastatic disease. Other presenting symptoms of
uterine cancer include pain, purulent genital discharge, changes in
bowel habits, weight loss, and incontinence. Less than 5% of women
are asymptomatic, and the diagnosis is made after workup of an
abnormal Pap test.
Question 647
• Question 647: You have a patient with breast cancer who is being treated with
dual therapy of trastuzumab and tamoxifen. Her sister also had breast cancer and
received trastuzumab, but she is unfamiliar with tamoxifen and wants to know
what role it plays in her cancer therapy. Which of the following is your response?
• Choices:
• 1. Your particular cancer is stimulated by estrogen, and tamoxifen inhibits your
body's production of estrogen
• 2. This drug is an immune stimulant that helps your immune system attack cancer
cells
• 3. This drug potentiates your trastuzumab therapy through direct enzymatic action
• 4. This drug counteracts the side effects of trastuzumab therapy
Answer: 1
• Answer: 1 - Your particular cancer is stimulated by estrogen, and
tamoxifen inhibits your body's production of estrogen
• Explanations: Certain cancers are estrogen receptor positive and are
stimulated by the presence of estrogen. Tamoxifen inhibits estrogen
production. Tamoxifen is sometimes abused by bodybuilders because
it can inhibit estrogen production. When excessive doses of anabolic
steroid is consumed, it undergoes aromatization and a large
percentage is converted to estrogen. Tamoxifen is taken daily for five
years for breast cancer. Serious side effects include stroke, blood clots
and uterine cancer.
Question 648
• Question 648: What is the most common type of breast cancer?
Choices:
• 1. Invasive ductal
• 2. Inflammatory
• 3. Invasive lobular
• 4. Mucinous
Answer: 1
• Answer: 1 - Invasive ductal
• Explanations: Seventy five to eighty percent of breast cancers are
invasive ductal carcinoma. It arises from terminal duct epithelial cells.
Invasive lobular carcinoma accounts for 10 percent of all breast
cancers. Mucinous or colloid breast cancer is only seen in 1 to 2
percent of patients.
Question 650
• Question 650: Microscopic examination of a skin biopsy of malignant
melanoma shows very large, visible nucleoli. This most likely signifies
that the cells are making large amounts of which of the following?
Choices:
• 1. Immunoglobulins
• 2. Golgi apparatus
• 3. Ribosomes
• 4. New DNA
Answer: 3
• Answer: 3 - Ribosomes
• Explanations: Ribosomal RNA production and packaging into
ribosomes occur in the nucleolus. Large nucleoli indicate a high rate
of ribosome production and thus an increased rate of production of
proteins. Large nucleoli occur with rapidly replicating malignancies
and with benign conditions, indicating an increased rate of production
of proteins. Microscopic examination of the melanoma can answer
questions about the behavior of the tumor based on the size of the
nucleolus.
Question 651
• Question 651: A 65-year-old female comes to a clinic complaining about recent weight loss
and difficulty in swallowing food. She states that she has lost over 20 pounds over the last
three weeks and cannot seem to gain any weight no matter how much she eats. She has no
heat or cold intolerance but does complain her neck feels swollen from time to time. Upon
palpation of her neck, no masses are palpable on the thyroid. Neck sonography is
performed, and a midline neck mass is observed. Her family history is impeccable, and so is
her past medical history. Resection of the mass and biopsy are performed, and the results
are pending. Which of the following is most commonly associated with this mass?
• Choices:
• 1. Anaplasic Thyroid Carcinoma
• 2. Hurthle cell thyroid carcinoma
• 3. Papillary thyroid carcinoma
• 4. Medullary thyroid carcinoma
Answer: 3
• Answer: 3 - Papillary thyroid carcinoma
• Explanations: Papillary thyroid carcinoma is the most commonly
associated cancer to thyroglossal duct cyst carcinoma, with over 80%
of the cases. Other carcinomas, such as Hurthle cell thyroid
carcinoma, is less commonly associated with thyroglossal duct cyst
carcinoma. Anaplasic thyroid carcinoma is not as frequently found as
papillary thyroid carcinoma, in thyroglossal duct cyst carcinoma.
Medullary thyroid carcinoma is not as frequently found as papillary
thyroid carcinoma, in thyroglossal duct cyst carcinoma.
Question 652
• Question 652: Mucinous nonneoplastic cysts often can be confused with mucinous cystic neoplasms
and intraductal papillary mucinous neoplasms (IPMNs). How are mucinous nonneoplastic cysts
different than mucinous cystic neoplasms and IPMNs?
• Choices:
• 1. Mucinous cystic neoplasms have ovarian type stroma whereas mucinous nonneoplastic cysts do not.
Mucinous nonneoplastic cysts lack ductal communication whereas IPMNs do have ductal
communication
• 2. Mucinous cystic neoplasms have ovarian type stroma whereas mucinous nonneoplastic cysts do not.
IPMN lack ductal communication whereas mucinous nonneoplastic cyst do not
• 3. Mucinous cystic neoplasms do not have ovarian type stroma whereas mucinous nonneoplastic cysts
do. Mucinous nonneoplastic cysts lack ductal communication whereas IPMNs do have ductal
communication
• 4. Mucinous cystic neoplasms do not have ovarian type stroma whereas mucinous nonneoplastic
nonneoplastic cysts do. IPMN lack ductal communication whereas mucinous nonneoplastic cyst do not
Answer: 1
• Answer: 1 - Mucinous cystic neoplasms have ovarian type stroma whereas
mucinous nonneoplastic cysts do not. Mucinous nonneoplastic cysts lack
ductal communication whereas IPMNs do have ductal communication
Explanations: Mucinous cystic neoplasms are large, mucin-producing cysts
that do not communicate with the ductal system. The finding of ovarian
type stroma is diagnostic for this lesion. Mucinous nonneoplastic cysts are
mucin-producing cyst without malignant potential. Mucinous nonneoplastic
cysts are differentiated from mucinous cystic neoplasm by lack of ovarian-
type stroma and from intraductal papillary mucinous neoplasms (IPMNs) by
lack of ductal communication. Differentiating these cysts are of critical
importance because IPMNs and mucinous cystic neoplasms have malignant
potential and may require surgical resection.
Question 653
• Question 653: A breast biopsy is performed on a 38-year-old female.
The biopsy shows atypical hyperplasia and mild fibrocystic changes.
What is her cancer risk?
• Choices:
• 1. Zero
• 2. Impossible to say
• 3. Slightly increased
• 4. Unchanged
Answer: 3
• Answer: 3 - Slightly increased
• Explanations: Atypical hyperplasia is not a malignancy, but it can be a
precursor to the development of breast cancer. Anyone who has been
diagnosed with atypical hyperplasia should undergo routine screening
and surveillance. When atypia is discovered, the patient should be
closely followed. For women at high risk, monitoring every 3 to 6
months is recommended. Tamoxifen has been used with some
success.
Question 654
• Question 654: Which of the following is a common sexual
complication after radiation therapy in women?
• Choices:
• 1. Amenorrhea
• 2. Decreased libido
• 3. Increased libido
• 4. Vaginal dryness
Answer: 4
• Answer: 4 - Vaginal dryness
• Explanations: Vaginal dryness is a common complication after
radiotherapy. Sexual interest can decrease due to different reasons.
Premature menopause can result if the ovaries receive radiotherapy.
Estrogen replacement therapy can help relieve symptoms.
Question 655
• Question 655: A 36-year-old male is evaluated for the fatigue of 3 months duration. He
admits to easy bruising, rash over his lower extremities, and frequent episodes of
respiratory infections. He had a prior history of rheumatoid arthritis, schizophrenia, and
history of Rocky Mountain Spotted Fever. His current medications include chlorpromazine
and gold. His vital signs include temperature is 38°C (100.4°F), blood pressure is 100/58
mmHg, pulse rate is 94b/min, and regular, respiratory rate is 17/min, and oxygen saturation
is 92% on ambient air. Physical examination reveals conjunctival pallor and mild oozing of
blood at gingival mucosa and rales in the left lower lung field while the stool is guaiac
negative. Also, petechiae are present over bilateral lower extremities. On further
investigation, chest x-ray demonstrates consolidation in the left lower lung. Also, the results
from laboratory studies are notable for hemoglobin 5.5 g/dL, platelets 50000, and white
blood cell count 1000 with 30% neutrophils. The peripheral smear is negative for blasts or
schistocytes. Bone marrow biopsy demonstrates the paucity of cells with fat infiltration.
Which of the following statements are true regarding this patient's current condition?
• Choices:
• 1. A bone marrow transplant from a matched sibling donor offers the
worst survival
• 2. If no bone marrow transplant HLA match or elderly patient,
immunosuppressant treatment should not be used
• 3. Recombinant human G-CSF does not stimulate
proliferation/differentiation in hematopoietic cells; recombinant
human erythropoietin stimulates hematopoiesis
• 4. If untreated, most patients will not recover bone marrow function
spontaneously
Answer: 4
• Answer: 4 - If untreated, most patients will not recover bone marrow
function spontaneously
• Explanations: If untreated, most patients will die within a few months
from bleeding or infection. Androgens are also used, in addition to
immunosuppressants. Human G-CSF can increase granulocytes
counts, leading to increased neutrophils and decreased risk of
infection. Very few patients recover spontaneously from aplastic
anemia.
Question 656
• Question 656: Onion skinning on a bone radiograph is indicative of
which condition?
• Choices:
• 1. Osteogenic sarcoma
• 2. Chondroma
• 3. Ewing sarcoma
• 4. Osteoma
Answer: 3
• Answer: 3 - Ewing sarcoma
• Explanations: Ewing sarcoma is common in children and often occurs
in long bones and the pelvis. Typical features of presentation include
fever, pain, weight loss, anemia, and leukocytosis. Radiographs may
show onion skinning due to successive layers of periosteal
development. This may be present with osteogenic sarcoma as well,
but it is more likely seen in Ewing sarcoma. Other features on
radiography may include a moth-eaten appearance and soft tissue
mass with saucerization.
Question 657
• Question 657: A patient with cancer is being treated with
chemotherapy. Which of the following antineoplastic drugs used for
the treatment of germ-cell tumors works by inhibiting topoisomerase
II?
• Choices:
• 1. Cisplatinum
• 2. Ciprofloxacin
• 3. Etoposide
• 4. Cyclophosphamide
Answer: 3
• Answer: 3 - Etoposide
• Explanations: Etoposide is an immunosuppressant drug used to treat
a variety of cancers. The drug is also known as VP-16 and is often
given in combination with other cancer drugs to reduce toxicity. VP-16
works by inhibiting topoisomerase 11. After administration of VP-16,
one must drink lots of fluid to prevent damage to the bladder and
kidneys. Side effects include alopecia, hypotension, metallic taste, and
bone marrow suppression.
Question 658
• Question 658: A 65-year-old woman presents with unexplained fever, drenching night
sweats, and significant weight loss over the past five months. On physical
examination, vital signs are normal. There is no peripheral lymphadenopathy.
Computed tomography (CT) scan reveals hepatosplenomegaly and retroperitoneal
lymphadenopathy. Histopathological examination reveals malignant cells with
abundant slightly basophilic cytoplasm and a distinctive bi-nucleate morphology
resembling owl's eyes appearance. Which of the following is the origin of these cells?
• Choices:
• 1. Cytotoxic T cell
• 2. Dendritic cell
• 3. Germinal center B cell
• 4. Macrophage
Answer: 3
• Answer: 3 - Germinal center B cell
• Explanations: The diagnosis of lymphocyte-depleted classic Hodgkin
lymphoma requires morphological evidence of Hodgkin/Reed-
Sternberg cells with scant lymphocyte infiltration under microscopic
examination. Classic diagnostic Reed-Sternberg cells are malignant
cells with abundant slightly basophilic cytoplasm and a distinctive bi-
nucleate morphology resembling owl's eyes appearance.
Mononuclear variants are called Hodgkin cells. These cells originate
from germinal center B cells. Immunohistochemical evaluation of
these cells is usually positive for CD15, CD30, PAX5, and negative for
CD3, CD20, CD45, and CD79a.
Question 659
• Question 659: A 66-year-old male presents after a small mass was
discovered on colonoscopy. Initial workup reveals carcinoma in situ.
What is the most likely TNM classification for this patient?
• Choices:
• 1. T1N1M0
• 2. TisN0M0
• 3. T0N0M0
• 4. TisN3M1
Answer: 2
• Answer: 2 - TisN0M0
• Explanations: T is used to classify tumors based on size and
invasiveness. N is for regional lymph node spread, and M indicates the
presence of metastases. Carcinoma in situ is indicated by Tis.
Carcinoma in situ is non-invasive. Therefore nodal status should be
N0. T0 indicates no evidence of tumor, not carcinoma in situ. T1 is
more indicative of a small, minimally invasive tumor. Carcinoma in situ
is non-invasive and non-metastatic. The TNM scoring system is used
for the staging of cancers. It has important diagnostic and prognostic
information.
Question 660
• Question 660: Breast cancers are generally found in what location?
Choices:
• 1. Lower-inner quadrant
• 2. Upper-inner quadrant
• 3. Lower-outer quadrant
• 4. Upper-outer quadrant
Answer: 4
• Answer: 4 - Upper-outer quadrant
• Explanations: The most frequent location of breast cancer is the
upper-outer quadrant. Breast cancer can, however, involve any
quadrant of the breast. Breast cancer also can develop in the axillary
tail, or tail of Spence, which also contains breast tissue. This relates
only to breast cancer in women. Men compose only 1% of patients
with breast cancer, but they have no predilection to the upper-outer
quadrant. Male breast cancer tends to be periareolar.
Question 661
• Question 661: A 61-year-old woman presents with severe nausea and vomiting. Her
vital signs are blood pressure of 131/82 mmHg, the temperature of 98.8 F, a pulse
of 75 beats per minute, and a respiratory rate of 24 breaths per minute. The
physical exam is unremarkable. The patient was recently diagnosed with breast
cancer, and she is receiving treatment with methotrexate, 5-fluorouracil, and
cyclophosphamide. Which of the following would provide the best management for
this patient?
• Choices:
• 1. Diphenhydramine
• 2. Rolapitant
• 3. Amifostine
• 4. Mesna
Answer: 2
• Answer: 2 – Rolapitant
• Explanations: Rolapitant is a substance P/neurokinin 1 receptor
antagonist. It prevents acute and delayed vomiting by acting on the
substance P/neurokinin 1 (NK1) receptor. It is indicated for
chemotherapy and prevention of postoperative nausea and vomiting
but can cause diarrhea in some patients. It can be used with 5-HT3
receptor antagonists and corticosteroids for chemotherapy-induced
vomiting.
Question 662
• Question 662: Leuprolide treatment is often used for which of the
following types of cancer?
• Choices:
• 1. Liver cancer
• 2. Renal cell carcinoma
• 3. Prostate cancer
• 4. Hodgkin disease
Answer: 3
• Answer: 3 - Prostate cancer
• Explanations: Gonadal stimulation is dependent on a pulse release of
GnRH from the hypothalamus. Leuprolide is a GnRH agonist that
reduces pituitary stimulation if given in continuous dose. Flutamide is
an androgen receptor block that is frequently given along with
leuprolide. Both medications are useful to ablate the androgenic
stimulation of prostate cancer.
Question 663
• Question 663: A 65-year-old female presents to the emergency department
complaining of pain in the lower abdomen. She denies any vaginal discharge but
says she has constant pain in the lower abdomen. This has been ongoing for a few
months. She has lost weight and has no appetite. Ultrasound reveals that she has
a large mass in the ovary. Biopsy reveals that she has a Krukenberg tumor. Which
of the following represents the stage is Krukenberg tumor in this patient?
• Choices:
• 1. I
• 2. II
• 3. III
• 4. IV
Answer: 4
• Answer: 4 - IV
• Explanations: Krukenberg tumor is metastatic disease to the ovary.
The most common primary site for this tumor is stomach followed by
colon, rectum, and breast. Upper and lower gastrointestinal
endoscopy and imaging are needed to evaluate the primary source
for this tumor. Given the metastatic nature of this tumor, the
prognosis is extremely poor.
Question 664
• Question 664: A 69-year-old male presents to you for prostate-specific
antigen screening. He wants to know what the test will do for him and if it will
save his life. Which of the following is not true regarding prostate-specific
antigen screening?
• Choices:
• 1. There continues to be controversy about the test
• 2. The lifetime chances of being diagnosed with prostate cancer is
significantly increased as a result of the screening test
• 3. The lifetime risk of dying from prostate cancer as a result of the test is
significantly decreased for all men
• 4. The screening test is recommended in males over the age of 55 years
Answer: 3
• Answer: 3 - The lifetime risk of dying from prostate cancer as a result of the test is
significantly decreased for all men
• Explanations: Like all screening tests for cancer, over-diagnosis is a common problem.
Especially with prostate cancer screening, the tests are not currently sensitive enough
and often lead to additional procedures even in men who ultimately are not diagnosed
with cancer. While the testing will identify prostate cancer, the risk of death is not
lessened in many men. Most of the cancers discovered are low-risk cancers that are
often monitored with active surveillance. About 25% of men on active surveillance will
have their cancer progress and require treatment. Ultimately, the decision about getting
a prostate-specific antigen (PSA) test should be made by the patient and family after
carefully considering the benefits and risks. Thus, providers should thoroughly inform
their patients before subjecting them to the PSA test. In general, do not test men who
do not reasonably have at least a 10-year life expectancy
• expectancy or are beyond age 70 years. However, a baseline PSA test
between 40 to 45 years of age has been suggested as a reasonably
reliable screening as those with a PSA level of less than one are
unlikely to develop significant prostate cancer over the next 25 years.
Also, while prostate cancer in patients less than 50 years of age is
uncommon, it is very aggressive and highly dangerous. The myriad
number of treatments and lack of good long-term trials have made it
difficult to decipher the data for a definitive answer to the PSA
controversy. All patients should be educated about the controversies
and let them make an informed decision about the test.
Question 665
• Question 665: Which of the following cancers is most common in
patients who undergo transplants or receive chemotherapy?
• Choices:
• 1. Colon cancer
• 2. Lung cancer
• 3. Skin cancer
• 4. Blood cancer
Answer: 3
• Answer: 3 - Skin cancer
• Explanations: Transplant recipients are at high risk for cancer because
of immunosuppression, chronic antigenemia, susceptibility to
oncogenic viruses, and use of potent drugs. Cancers of the colon,
lung, bladder, and prostate do occur. The risk of cancer is highest for
the skin, lip, and kidney, and for lymphomas. Surveillance is necessary
for all patients who undergo transplant. All suspicious lesions must be
biopsied.
Question 666
• Question 666: Which of the following is the best description of
paclitaxel?
• Choices:
• 1. Alkylating agent
• 2. Antimetabolite
• 3. Microtubule stabilizing agent
• 4. Antibiotic
Answer: 3
Answer: 3 - Microtubule stabilizing agent •
Explanations: Paclitaxel stabilizes microtubules. This interferes with •
the normal breakdown of microtubules during cell division. Paclitaxel
is used to treat cancers of the lung, breast, ovary, head, neck, and
advanced Kaposi sarcoma. It may distort mitotic spindles causing the
chromosomes to break. Paclitaxel has a black box warning for
.hypersensitivity reactions and bone marrow suppression
Question 667
• Question 667: Which of the following is the most common intracranial
tumor associated with neurofibromatosis type 1?
• Choices:
• 1. Astrocytoma
• 2. Meningioma
• 3. Oligodendroglioma
• 4. Optic nerve glioma
Answer: 4
• Answer: 4 - Optic nerve glioma
• Explanations: Optic nerve glioma is the most common intracranial
tumor associated with neurofibromatosis NF type 1. It occurs in about
10% to 15% of patients with NF 1. Optic nerve gliomas typically
present in childhood and often in the setting of NF type 1. The tumors
often are low-grade and indolent. Optic nerve gliomas rarely occur in
adults. The tumors usually are aggressive, and they are not associated
with NF type 1.
Question 668
• Question 668: A 65-year-old chronic male smoker presented with constant,
mechanical pain over the thoracolumbar region. On clinical evaluation, his
neurological status was normal. On detailed radiological evaluation, there was the
presence of a large lytic lesion at the T12 vertebral body. He also had previously
undergone treatment for pulmonary adenocarcinoma nine months ago. The Spinal
Instability Neoplastic Score (SINS) score was calculated to be 10. Which of the
following is the most appropriate management option?
• Choices:
• 1. Laminectomy
• 2. Thoracolumbar stabilization plus biopsy
• 3. Biopsy only
• 4. Wide en bloc excision plus anterior and posterior reconstruction
Answer: 2
• Answer: 2 - Thoracolumbar stabilization plus biopsy
• Explanations: Spinal metastases occur in 10% to 20% of patients with
malignancies, and 10% to 20% of these patients develop spinal or neural
element compression that may necessitate surgical intervention. There has
been a need for a systematic classification system to assess the need for
surgical stabilization to guide the therapeutic approaches in the
management of such lesions. In 2010, the Spine Oncology Study Group
(SOSG) devised the Spinal Instability Neoplastic Score (SINS) system. This
system assesses and scores the following six variables. The location of the
lesion, pain characteristics, type of bony lesion, radiographic spinal
alignment, vertebral body destruction, and involvement of posterolateral
elements. The scores for each variable are added to obtain the final score.
• The minimum and maximum scores are from 0 to 18. A score of 0 to 6
indicates stability. A score of 7 to 12 indicates indeterminate (or
impending) instability. A score of 13 to 18 indicates instability. Patients
with scores greater than 7 require surgical consultation. Another
important consideration in the decision-making process for spinal
metastasis includes the life expectancy of these patients. This aspect
has also had an assessment in other classification systems including
Tomita, Tokuhashi, and Oswestry systems.
Question 669
• Question 669: Which of the following is FDA approved to treat opioid-
induced constipation in adults?
• Choices:
• 1. Linaclotide
• 2. Methylnaltrexone
• 3. Plecanatide
• 4. Polyethylene glycol
Answer: 2
• Answer: 2 - Methylnaltrexone
• Explanations: Methylnaltrexone is FDA approved to treat opioid-
induced constipation (OIC) in adults. Lubiprostone and naloxegol also
are approved for treating OIC in adults. Methylnaltrexone is a
peripheral mu-opioid receptor antagonist. Methylnaltrexone has a
limited ability to cross the blood-brain barrier. Consequently, it does
not eliminate opioid-induced analgesia nor cause withdrawal
symptoms.
Question 670
• Question 670: Which malignancy is most often associated with
acanthosis nigricans?
• Choices:
• 1. Multiple myeloma
• 2. Melanoma
• 3. Gastric cancer
• 4. Inflammatory breast cancer
Answer: 3
• Answer: 3 - Gastric cancer
• Explanations: In older individuals, acanthosis nigricans may be
associated with a malignancy. The malignancies often associated with
acanthosis nigricans include adenocarcinomas of the gastrointestinal
tract, uterus, lung, breast, and ovary. 70 to 90% of patients have
adenocarcinomas of the gastrointestinal tract. 55-61% of malignant
acanthosis nigricans cases are secondary to gastric cancer. Almost
18% of malignant acanthosis nigricans cases predate the diagnosis of
malignancy. Approximately 61.3% of cases are diagnosed
simultaneously with the cancer manifestation.
Question 671
• Question 671: A 73-year-old male has had metastatic prostate cancer
for 9 months and has lost 13 kg. He has had some anorexia but is still
eating. Select the mediator that may be causing the weight loss.
Choices:
• 1. Interleukin 3
• 2. Tumor necrosis factor alpha
• 3. Complement 3a protein
• 4. Interferon alpha
Answer: 2
• Answer: 2 - Tumor necrosis factor alpha
• Explanations: Wasting syndrome or cachexia is frequently seen with
cancer. Cytokines associated with this are tumor necrosis factor alpha,
interleukins 1 and 6, and interferon. Increased insulin resistance is
part of the metabolic derangement Interleukin 3 stimulates the
differentiation of myeloid progenitor cells.
Question 672
• Question 672: A patient has a unilateral invasive vulvar carcinoma
that is 2 cm in diameter. The depth of invasion is 0.5 mm, and there is
no evidence of metastasis to lymph nodes. Which treatment is best?
Choices:
• 1. Chemotherapy
• 2. Radiation therapy
• 3. Wide, local excision
• 4. Radical vulvectomy and bilateral inguinal lymphadenectomy
Answer: 3
• Answer: 3 - Wide, local excision
• Explanations: Squamous cell carcinoma of the vulva is treated
surgically. Radiation therapy and chemotherapy may be used as
neoadjuvant treatments. This patient has stage IA vulvar cancer.
Wide, local excision is the best treatment, typically with at least 1 to
2-cm margins.
Question 673
• Question 673: Which gastric tumor shows the best response to
radiotherapy? Choices:
• 1. Sarcoma
• 2. Adenocarcinoma
• 3. Lymphoma
• 4. Leiomyoma
Answer: 3
• Answer: 3 – Lymphoma
• Explanations: Primary gastric lymphomas are not common. Often
lymphomas from elsewhere metastasize to the stomach. Most
patients with gastric lymphomas are in the sixth decade of life and
may present with epigastric pain, fatigue, weight loss, and malaise.
Diagnosis is done with endoscopy and biopsy. Most lymphomas are of
B cell origin. At least 60% of MALT lymphomas respond to radiation
therapy.
Question 674
• Question 674: Which of the following chemotherapy agent works by
disrupting base pairing? Choices:
• 1. Cyclophosphamide
• 2. Methotrexate
• 3. 5-fluorouracil
• 4. Bleomycin
Answer: 1
• Answer: 1 - Cyclophosphamide
• Explanations: Alkylating agents react with base pyrimidine and purine
base pairs and form dimers. This causes disruption of DNA replication.
Cyclophosphamide is an alkylating agent. Methotrexate and 5-
fluorouracil are antimetabolites. Bleomycin is a glycopeptide
antibiotic.
Question 675
• Question 675: Which of the following best defines "palliative care"?
Choices:
• 1. Care provided in an emergency or intensive care setting
• 2. Care provided while waiting for admission to a hospice
• 3. Care provided to relieve or ease the symptoms of a serious
condition or disease, regardless of the prognosis
• 4. Care provided to adults at end of life
Answer: 3
• Answer: 3 - Care provided to relieve or ease the symptoms of a
serious condition or disease, regardless of the prognosis
• Explanations: Palliative care may be administered in any setting.
Palliative care (PC) aims to relieve or ease the symptoms of a life-
threatening disease or disorder, regardless of the cure rate. PC may
occur throughout all stages of serious illness. Initiating PC early, at the
time of diagnosis, is acceptable and even preferred in most instances.
Palliative care puts relief first. Easing symptoms and stressors of
serious illnesses, regardless of the prognosis, is the essence of
palliative care. There are no age limits in palliative care.
Question 676
• Question 676: An 86-year-old male was begrudgingly brought to the clinic by his
daughter-in-law after his continuous complaints of fatigue ongoing the past few
months. He denies a medical history but has refused to see a physician in over 20
years since his wife passed away following an appendectomy that had gone awry.
Aside from the fatigue, his only complaints are mild back pain and stomach pain,
which has occurred on and off for years for which he takes non-steroidal anti-
inflammatory drugs (NSAIDs) with some relief. Physical examination is significant
for mild epigastric tenderness to palpation and + 1 pedal edema of the lower
extremities bilaterally. Testing of para-amino hippuric acid clearance (a conjugate
measure for renal plasma flow) and glomerular filtration rate (GFR) on lab results
helped the physician conclude that the patient’s disease involved a significant
decrease in the renal filtration fraction. Which of the following study results is
diagnostic of this patient’s underlying disease.
• Choices:
• 1. Echocardiography
• 2. SPEP (serum protein electrophoresis)
• 3. 24-hour urine collection
• 4. Upper GI endoscopy
Answer: 2
• Answer: 2 - SPEP (serum protein electrophoresis)
• Explanations: Echocardiography is an imaging technique used to assess for
congestive heart failure. Clinically speaking a patient with congestive heart failure is
considered hypervolemic. However, there is a reduction in effective circulating blood
volume leading to a significantly decreased renal plasma flow. Although the GFR is
also decreased in congestive heart failure, it is not to the extent of renal plasma flow,
which leads to the overall increase in the filtration fraction. A monoclonal spike (M-
spike) on serum protein electrophoresis suggests that the patient may have multiple
myeloma. The multiple myeloma is a disease in which abnormal plasma cells
produce a significant amount of abnormal antibodies, increasing the serum protein.
The filtration fraction (FF) is the fraction of renal plasma flow (RPF) filtered across
the glomerulus, estimated with the glomerular filtration rate (GFR). The equation to
calculate the filtration fraction is as follows. FF = GFR/RPF Para-amino hippuric
Question 677
• Question 677: Which condition causes very high levels of human
chorionic gonadotropin?
• Choices:
• 1. Yolk sac tumor
• 2. Granulosa cell tumor
• 3. Choriocarcinoma
• 4. Ectopic pregnancy
Answer: 3
• Answer: 3 - Choriocarcinoma
• Explanations: Choriocarcinoma is very sensitive to chemotherapy. It
has a tendency to spread very early to the lungs. Choriocarcinoma
presents with very high levels of human chorionic gonadotropin
(HCG). Ectopic pregnancy does not cause abnormally elevated levels
of HCG. Early in a normal pregnancy levels of HCG double every 48 to
72 hours. With an ectopic pregnancy, HCG levels do not rise in such a
predictable fashion.
Question 678
• Question 678: The most common presenting sign with chronic
myelocytic leukemia is which one of the following?
• Choices:
• 1. Lymphadenopathy
• 2. Splenomegaly
• 3. Hepatomegaly
• 4. Sternal tenderness
Answer: 2
• Answer: 2 – Splenomegaly
• Explanations: The most common presenting sign of chronic
myelocytic leukemia is splenomegaly. The majority of patients with
CML have no specific symptoms and the disease presentation is
insidious. The spleen size correlates with the WBC count; the higher
the WBC count, the larger the spleen. Hepatomegaly can also occur
but is not as pronounced as splenomegaly.
Question 679
• Question 679: How is radiation intensity related to distance?
• Choices:
• 1. Directly proportional
• 2. Inverse proportional
• 3. Exponential
• 4. Inversely exponential
Answer: 4
• Answer: 4 - Inversely exponential
• Explanations: Radiation intensity decreases with the square of
distance. Increasing distance decreases radiation, an inverse
relationship. This is known as the "inverse square law". This has
important implications in radiation safety. The most effective method
to decrease one's exposure to radiation is to simply step back from
the source. This law applies to radiation from all sources, both from x-
rays and from nuclear medicine radiopharmaceuticals.
Question 680
• Question 680: What is the first step in managing a febrile patient who
is neutropenic?
• Choices:
• 1. Chest radiograph
• 2. Blood cultures followed immediately by antibiotics
• 3. Blood cultures and wait for the results before beginning antibiotics
• 4. Repeat temperature measurement in 30 minutes
Answer: 2
• Answer: 2 - Blood cultures followed immediately by antibiotics
Explanations: Febrile neutropenia includes a temperature greater
than 101F and an absolute neutrophil count less than 500
cells/microliter. Empiric antibiotics should be started after taking
blood and urine cultures. Other investigations may follow after
initiating antibiotics and monitoring of the immune system status
must be carried out. If a fungal infection is suspected, appropriate
samples for culture and antifungal therapy should be initiated.
Question 681
• Question 681: What type of drug is paclitaxel?
• Choices:
• 1. NItrogen mustard
• 2. Antimetabolite
• 3. Vesicant
• 4. Antimicrotubular agent
Answer: 4
• Answer: 4 - Antimicrotubular agent
• Explanations: Paclitaxel is an antimicrotubular agent used in cancer
treatment. It inhibits cell replication by interfering with the mitotic
phase of cell proliferation. Paclitaxel is FDA approved for the
treatment of breast cancer, Kaposi sarcoma (AIDS-related), Non-small
cell lung cancer, ovarian cancer. Paclitaxel is administered
intravenously.
Question 682
• Question 682: A 75-year-old man presents to the clinic with a recent
onset of hematuria. Upon physical exam, he is found to have a hard
and nodular prostate. Which of the following factors is associated
with the best prognosis in this patient?
• Choices:
• 1. Gleason score of 5
• 2. PSA of 12 ng/mL (reference range: <4 ng/mL)
• 3. Prostate nodularity
• 4. Hematuria
Answer: 1
• Answer: 1 - Gleason score of 5
• Explanations: The Gleason score is a histologic assessment of prostate
cancer. The Gleason score takes into account several histologic
features. The lower the Gleason score, the more like normal tissue
the cells appear. The lower the Gleason score, the better the
prognosis.
Question 683
• Question 683: Which type of nerve block is most effective in relieving
pain from pancreatic cancer?
• Choices:
• 1. Celiac plexus nerve block
• 2. Superior hypogastric plexus block
• 3. Lumbar sympathetic block
• 4. Ganglion impar block
Answer: 1
• Answer: 1 - Celiac plexus nerve block
• Explanations: Afferent sensory neurons of the upper abdominal viscera travel with
sympathetic nerve fibers supplying the area. The sympathetic fibers arise from segments T5 to
T12, travel in the greater, lesser, and least splanchnic nerves, and synapse in the celiac ganglia
within the celiac plexus. The celiac plexus lies in the retroperitoneal space over the
anterolateral surface of the aorta, primarily at the T12-L1 vertebral level. Performing a
neurolytic celiac plexus nerve block yields moderate success in treating pancreatic cancer pain.
Superior hypogastric plexus block is used to treat sympathetically mediated pain in the pelvis
and is performed at the L5 level. A lumbar sympathetic block is performed at the L2 level and
is most commonly used to treat complex regional pain syndrome in the lower extremities.

• The ganglion impar is the solitary terminal ganglion of the sympathetic chain and is located at
the level of the sacrococcygeal joint. Blockade of this ganglion is also useful in treating lower
pelvic pain that is sympathetically mediated.
Question 684
• Question 684: What is the most common histology of oral cavity
cancer?
• Choices:
• 1. Melanoma
• 2. Basal cell
• 3. Squamous cell
• 4. Ameloblastic
Answer: 3
• Answer: 3 - Squamous cell
• Explanations: Oral cavity cancers are not uncommon and occur
globally. The major histopathology of oral cancer is squamous cell
cancers. Most of the squamous cell cancers arise from premalignant
conditions like leukoplakia, lichen planus, discoid planus, or
erythroplakia. In most cases, oral cavity cancers are diagnosed late
because of non-specific symptoms and lack of physician examination
of the oral cavity.
Question 685
• Question 685: How many more times potent is fentanyl than
morphine?
• Choices:
• 1. 100 times
• 2. 20 times
• 3. 50 times
• 4. 120 times
Answer: 1
• Answer: 1 - 100 times
• Explanations: Fentanyl is about 80 to 100 times more potent than
morphine. Calculations should include incomplete cross tolerance.
Fentanyl can be administered as a patch, IV, intranasal, lozenges or
sublingual. However, like all opiates it can also depress respiration and
cause respiratory arrest. Recently there have been many deaths in
children and liberal use of fentanyl is now being curtailed by the FDA
and DEA.
Question 686
• Question 686: At what age should routine Pap smears be done?
• Choices:
• 1. At 21 years of age regardless of prior sexual history
• 2. At age 18 years of age
• 3. At the onset of sexual activity
• 4. Three years after the onset of sexual activity
Answer: 1
• Answer: 1 - At 21 years of age regardless of prior sexual history
Explanations: The latest guidelines recommend a Pap smear be done
at 21 years of age regardless of prior sexual history. Abnormal cervical
cytology is very common in young women and in most cases resolves
without treatment. Further, women under 21 years of age account for
less than 0.1% of all cervical cancers. There is no evidence showing
that cervical cancer screening at a younger age improves morbidity or
mortality.
Question 687
• Question 687: A 65-year-old male patient presents for routine follow-up for his radical
prostatectomy, which was done seven years ago to treat his prostate cancer. He reports
that he had erectile dysfunction, but it resolved on its own. His charts from previous visits
show serum prostate-specific antigen (PSA) levels have been slowly rising over the past
few years. Labs are obtained, and the PSA level is currently 70 ng/ml. With the suspicion
of prostate cancer recurrence, you want to start a drug to lower his testosterone levels.
What is the mechanism of action of the drug that would be used to reduce the patient’s
testosterone to castration levels?
• Choices:
• 1. 5-alpha reductase inhibitor
• 2. PDE5 Inhibitor
• 3. GnRH analog
• 4. Synthetic HCG
Answer: 3
• Answer: 3 - GnRH analog
• Explanations: Leuprolide is an FDA-approved gonadotropin-releasing hormone agonist that is
used for the management of endometriosis, uterine leiomyomata (also known as uterine
fibroids), treatment of central precocious puberty in children, and advanced prostate cancer.
In advanced prostate cancer, leuprolide is used as a palliative treatment to help reduce
comorbidities and improve quality of life. Leuprolide lowers testosterone which in turn
minimizes the stimulatory effect it normally has on androgen-sensitive prostate cancer cells;
therefore causing them to undergo apoptosis. For the treatment of prostate cancer,
providers should monitor FSH and LH levels, serum testosterone levels, and prostate-specific
antigen levels. Fatigue, weakness, and urinary tract obstruction should be watched for during
initial treatment. Screening should be
• performed for diabetes prior to starting treatment as leuprolide can worsen glycemic
control. Blood glucose and HbA1c should be periodically measured throughout therapeutic
use. Electrocardiograms and electrolyte levels should be monitored occasionally as well.
Question 688
• Question 688: Which of the following would not be seen in stage II
non-small cell carcinoma?
• Choices:
• 1. T1 tumors have a better prognosis than T2 tumors
• 2. An increasing number of N1 nodes has a poor prognosis
• 3. Patients with hilar nodes have a better prognosis than those with
lobar nodes
• 4. Recurrence of both distant and local disease is common
Answer: 3
• Answer: 3 - Patients with hilar nodes have a better prognosis than
those with lobar nodes
• Explanations: Patients with stage II non-small cell cancer are treated
surgically. However, patients with hilar lymph nodes have a slightly
worse prognosis than those with lobar lymph nodes. Radiation usually
is not given to patients with stage I or stage II disease if the resection
is complete. Several trials have shown a decreased survival rate after
postoperative radiation in these patients. Some trials have shown a
slight survival advantage in stage II patients after surgery. Because
more than 50% of patients with stage II disease relapse,
chemotherapy is now offered to most patients.
Question 689
• Question 689: What is the most common form of soft tissue mass?
Choices:
• 1. Sarcoma
• 2. Adenoma
• 3. Melanoma
• 4. Lipoma
Answer: 4
• Answer: 4 – Lipoma
• Explanations: Lipoma is the most common soft tissue mass. Lipomas
are commonly slow growing. Lipomas greater than 10 cm are referred
to as "giant lipomas." Radiologic imaging prior to surgery may be
prudent for lipomas that feature giant size (greater than 10cm), rapid
growth, pain, fixation to underlying tissues, or location in deep
tissues, the thigh, or retroperitoneal space.
Question 690
• Question 690: From which location does pheochromocytoma
originate?
• Choices:
• 1. Adrenal cortex
• 2. Renal cortex
• 3. Adrenal medulla
• 4. Renal medulla
Answer: 3
• Answer: 3 - Adrenal medulla
• Explanations: Pheochromocytomas are chromaffin cell tumors derived
from neural crest. 90 percent are found in the adrenal medulla. They
produce catecholamines. Only 10% of pheochromocytomas are
malignant and cure can be obtained with surgery. Unlike other
sympathetic tissues pheochromocytomas are not innervated by the
sympathetic system and catecholamine release is autonomous.
Question 691
• Question 691: Epithelioid sarcoma lacks expression of which of the
following putative tumor suppressor genes?
• Choices:
• 1. Integrase interactor-1, INI-1/SMARCB1
• 2. p53
• 3. Mismatch repair proteins, MLH1/PMS2/MSH2/MSH6
• 4. Retinoblastoma, Rb
Answer: 1
• Answer: 1 - Integrase interactor-1, INI-1/SMARCB1
• Explanations: Lack of INI-1expression is seen in up to 90% of
epithelioid sarcomas. INI-1 deficient tumors include epithelioid
sarcoma, malignant extra-renal rhabdoid tumor, and renal medullary
carcinoma. INI-1/SMARCB1 is a component of a chromatin
remodeling complex necessary for global DNA transcription.
INI-1/SMARCB1 is located on chromosome 22q11.
Question 692
• Question 692: From which of the following does a
rhabdomyosarcoma originate?
• Choices:
• 1. Fat tissue
• 2. Subcutaneous tissue
• 3. Striated muscle
• 4. Smooth muscle
Answer: 3
• Answer: 3 - Striated muscle
• Explanations: Rhabdomyosarcomas are malignancies of connective
tissue, especially striated or skeletal muscle. Rhabdomyosarcomas can
occur in any part of the body. Embryonal rhabdomyosarcomas occur
in children and are often found either in the head or genital area.
Pleomorphic type malignancies occur in adults and anise in the
muscles of the extremities.
Question 693
• Question 693: What is the estimated 5-year survival for a patient with
stage one lung cancer?
• Choices:
• 1. 5%
• 2. 25%
• 3. 50%
• 4. 75%
Answer: 4
• Answer: 4 - 75%
• Explanations: The estimated 5-year survival for a patient with stage
one lung cancer averages about 75%. Stage two survival is about 50%.
Stage three survival drops to 30% or less. Stage four lung cancer has
less than a 5% survival rate.
Question 694
• Question 694: A 65-year-old male patient presents with unintentional weight loss
and occasional epigastric pain. During the physical examination, mild icterus is
noted. After appropriate laboratory evaluation, CT scan is ordered, which reveals a
7mm indeterminate hypodense lesion at the junction of extrahepatic bile ducts
with significant dilatation of bilateral intrahepatic bile ducts. No other mass
identified. Based on the history and imaging findings, which of the following is a
risk factor for the malignancy in this patient?
• Choices:
• 1. Cholangitis
• 2. Occupational exposure to toxic dyes
• 3. Hepatitis C
• 4. Chronic pancreatitis
Answer: 2
• Answer: 2 - Occupational exposure to toxic dyes
• Explanations: Above described CT findings are consistent with hilar
cholangiocarcinoma (Klatskin tumor). Most patients present with
biliary obstruction and unresectable disease. The 5-year survival with
this malignancy is reported to be very low. Underlying hepatic
disease, ulcerative colitis, exposure to toxic dyes, and liver flukes are
known risk factors. Other risk factors for cholangiocarcinoma include
choledochal cysts and Caroli disease. Chronic pancreatitis, cholangitis,
or hepatitis C infection are not known as risk factors for
cholangiocarcinomas.
Question 695
• Question 695: A 67-year-old female with chronic lymphocytic
leukemia develops hemolytic anemia. Which of the following is
appropriate nursing care?
• Choices:
• 1. Reverse isolation
• 2. Allow the patient to remain in her room and get rest
• 3. Encourage the patient to ambulate and socialize
• 4. Restrict fluids
Answer: 2
• Answer: 2 - Allow the patient to remain in her room and get rest
Explanations: Chronic lymphocytic leukemia is due to a monoclonal
proliferation of lymphocytes. Autoimmune hemolytic anemia can be a
complication. Patients affected will show fatigue and pallor.
Encouraging the patient to rest with limited ambulation is
appropriate.
Question 696
• Question 696: A 71-year-old female with a history of smoking
undergoes a CT scan, which reveals a 3-centimeter by 3-centimeter
peripheral lesion in the lung. Biopsy reveals tall columnar cells which
have invaded the normal lung tissue. What is the most likely diagnosis?
• Choices:
• 1. Carcinoid
• 2. Squamous cell
• 3. Adenocarcinoma
• 4. Metastatic breast cancer
Answer: 3
• Answer: 3 - Adenocarcinoma
• Explanations: Adenocarcinoma is a cancer of bronchial mucosal
glands and represents about one-third of all lung cancers. It is a
peripheral cancer and there are several subtypes. Bronchoalveolar
cancer is a distinct subtype which grows along the alveolar septate.
Adenocarcinomas are not associated with smoking and often develop
in scars. Bronchoalveolar carcinoma is notorious for presenting as
pneumonia or interstitial lung disease.
Question 697
• Question 697: A thirteen-year-old male with a known APC gene
mutation undergoes endoscopic evaluation. What findings would you
expect to identify during the exam?
• Choices:
• 1. Multiple adenomatous polyps
• 2. Mucosal outpouchings at the sigmoid colon
• 3. Large ulcerated colon mass
• 4. Stricture of the colon
Answer: 1
• Answer: 1 - Multiple adenomatous polyps
• Explanations: A mutation at the APC gene is related to familial
adenomatous polyposis (FAP). Patients will develop hundreds of
adenomatous colon polyps by adolescence. Endoscopic screening
should begin at age twelve. Colonic resection is performed for
colorectal cancer prevention. Without treatment, the risk of
developing colorectal cancer reaches 100%.
Question 698
• Question 698: A 65-year-old female with renal cancer presents with
dyspnea, fatigue, and dizziness. She is on warfarin for the treatment of
a left leg deep vein thrombosis. Her hematocrit is 22%, hemoglobin is
7.1 g/dL, platelets 109,000, PT 1.3, PTT 36, INR 1.45, and WBC 12,000.
What is the next step in management?
• Choices:
• 1. Transfuse packed red blood cells
• 2. Transfuse platelets
• 3. Transfuse fresh frozen plasma
• 4. Give vitamin K
Answer: 1
• Answer: 1 - Transfuse packed red blood cells
• Explanations: The most obvious problem with this patient is the
severely decreased hemoglobin. Transfusion usually is recommended
when the patient is symptomatic or the hemoglobin is less than 7.5
g/dL or the hematocrit is less than 25%. Transfusion of one unit of
packed red blood cells may raise the hemoglobin by 1.5 grams. Her
INR is not in the therapeutic range of 2 to 3 and should be addressed
once she is stabilized.
Question 699
• Question 699: Which of the following descriptions of an asymptomatic,
incidental pancreatic cyst would qualify for surveillance only?
• Choices:
• 1. The cyst is less than 3 cm, no solid component, and no main pancreatic
duct dilation
• 2. The cyst is greater than 3 cm, no solid component, and no main
pancreatic duct dilation
• 3. The cyst has a solid component and main pancreatic duct dilation
• 4. The cyst is less than 3 cm, no solid component, and main pancreatic
duct dilation
Answer: 1
• Answer: 1 - The cyst is less than 3 cm, no solid component, and no
main pancreatic duct dilation
• Explanations: Surveillance is recommended if a pancreatic cyst is less
than 3 cm, has no solid component, and there is no main pancreatic
duct dilation. If surveillance is initially chosen as the management
option, then an MRI should be done in 1 year and then every 2 years
after that until 5 years have passed. If surveillance has been done for
5 years and there is no change in the cyst, then surveillance can be
stopped. If during a surveillance MRI there is a change in the cyst,
then an endoscopic ultrasound-guided fine needle aspiration biopsy
(EUS-FNA) should be performed.
Question 700
• Question 700: A patient with a preexisting history of hypertension and atrial fibrillation
on propranolol presents to the emergency department with worsening jaundice,
shortness of breath, abdominal distention, tarry stools, and scleral icterus. Examination
reveals pulmonary crackles and hepatomegaly. He first noticed these symptoms several
months ago, but over the last few weeks, his symptoms have become worse. His blood
pressure is 70/50 mmHg, and his heart rate is 44/min. His CT chest and abdomen reveals
multiple metastases and moderate ascites. What type of cancer is most likely to explain
his vital signs?
• Choices:
• 1. Liver
• 2. Oral pharyngeal
• 3. Pulmonary
• 4. Rectal
Answer: 1
• Answer: 1 – Liver
• Explanations: Orally administered drugs pass through the portal circulation to the liver.
Propranolol is an example of a drug that undergoes extensive enterohepatic metabolism.
Patients with liver dysfunction will metabolize propranolol less and may be at risk of toxic
effects of propranolol even at therapeutic doses. Propranolol is not administered rectally.
However, rectally administered drugs that are
absorbed through the proximal rectum are subject to enterohepatic metabolism, while distal
rectal administration does not undergo first-pass metabolism by liver enzymes. Oral
pharyngeal cancer may affect the ability to absorb sublingual drugs; however, propranolol is
given via the intravenous or oral route only. Moreover, sublingual administration does not go
through the first-pass effect, and it is absorbed directly from the oral mucosa. The inhalational
route does not undergo the first-pass effect.
Question 701
• Question 701: What is the most common malignancy of the vagina?
Choices:
• 1. Basal cell carcinoma
2. Squamous cell carcinoma
3. Clear cell carcinoma
4. Sarcoma botryoides
Answer: 2
• Answer: 2 - Squamous cell carcinoma
• Explanations: Squamous cell cancer of the vagina is the most common
malignancy and typically occurs in women over the age of 60. Vaginal
Adenocarcinoma arises from the secretory glands and has a
propensity to spread. Sarcoma botryoides is a type of
rhabdomyosarcoma usually occurs in small children.
• Women who were exposed to diethyl stilbestrol are at an increased
risk for clear cell adenocarcinoma.
Question 702
• Question 702: Which of the following is the most important
management strategy to prevent central line insertion site infection?
Choices:
• 1. Daily dressing changes
• 2. Changing the line frequently
• 3. Aseptic technique for dressing changes
• 4. Avoid use of parenteral nutrition or lipid-based transfusions
Answer: 3
• Answer: 3 - Aseptic technique for dressing changes
• Explanations: Central line dressings do not need frequent changes. Gauze
based dressing may be changed every two days, and transparent dressing
may be changed every 5 to 7 days. More frequent dressing changes need onl
• Each lumen of the central line should be assigned a distinct purpose.
Frequent manipulation of the line may lead to higher catheter-based
complications. Prophylactic antibiotics have not been shown to reduce
central line insertion site infections. Aseptic technique during dressing
changes ensures that no new bacteria is introduced to the insertion site.
Furthermore, proper hand hygiene and the use of gloves for any line
manipulation
Question 703
• Question 703: Which of the following agent can be used as first-line
therapy in lowering an elevated serum calcium?
• Choices:
• 1. Mithramycin
• 2. Calcitonin
• 3. Bisphosphonates
• 4. Doxorubicin
Answer: 2
• Answer: 2 - Calcitonin
• Explanations: Calcitonin is a small peptide that is produced by C-cells
in the thyroid. The hormone acts by reducing calcium in the body.
Calcitonin reduces calcium by inhibiting absorption from the gut,
inhibiting osteoclastic activity, and inhibiting phosphate reabsorption
from the renal tubules.
• Subcutaneous calcitonin also has been used in individuals who have
mania and bipolar disorder. Calcitonin also is used as a tumor marker
for medullary cancer of the thyroid.
Question 704
• Question 704: A 51-year-old female has a breast cancer which is
staged at T2N1M1. What does this staging signify?
• Choices:
• 1. The tumor is greater than 6 cm
• 2. The tumor is attached to the chest muscles
• 3. Only axillary nodes are involved
• 4. There are metastases
Answer: 4
• Answer: 4 - There are metastases
• Explanations: A breast cancer that is more than 5 cm is a T3. N1
means that the cancer has spread to 1 to 3 lymph nodes in the axilla.
M1 means that there is distant spread, most likely to bone, lung, or
liver. The patient has a stage 4 cancer.
Question 705
• Question 705: What is the risk factor most commonly associated with
renal cancer?
• Choices:
• 1. Alcohol
• 2. Smoking
• 3. Schistosomiasis
• 4. Hypertension
Answer: 2
• Answer: 2 – Smoking
• Explanations: Smoking is the most common risk factor for renal
cancer. It is found in nearly 70% of individuals with renal cancer. In
men, greater than 20 pack-years of smoking was associated with a 2.3
fold increase in renal cancer risk. The correlation was less strong in
women. Obesity is another risk factor followed by hypertension.
• Other rare causes of renal cancer include tuberous sclerosis, von
Hippel-Lindau disease, and renal transplant.
Question 706
• Question 706: What is the mechanism of action of ondansetron?
Choices:
• 1. Stabilizing chemoreceptor trigger zone dopamine receptors
• 2. Slowing peristalsis
• 3. Blocking serotonin receptors
• 4. Blocking histamine receptors
Answer: 3
• Answer: 3 - Blocking serotonin receptors
• Explanations: Most antiemetic drugs work by stabilizing the
chemoreceptor trigger zone. Ondansetron is a serotonin (5HT3) receptor
blocker used for nausea and vomiting. Ondansetron works in both the
medulla oblongata and peripherally by reducing vagal nerve activity. Recent
studies question the safety of ondansetron in pregnancy. Some studies

• suggest ondansetron causes increased risk of severe birth defects and


injury to the mother. Its use in pregnancy is now considered off-label. The
only FDA-approved drug for treating nausea and vomiting in pregnancy is
doxylamine/pyridoxine.
Question 707
• Question 707: What is the most likely cause of vomiting, abdominal
cramping, and frequent urge to have a bowel movement after
radiotherapy?
• Choices:
• 1. Intestinal infection
• 2. Radiation enteritis
• 3. Release of chemicals from dying cancer cells in the intestine
• 4. Mucositis
Answer: 2
• Answer: 2 - Radiation enteritis
• Explanations: Radiation enteritis is an acute complication of
radiotherapy with symptoms of nausea, vomiting, abdominal
cramping, tenesmus, diarrhea, hematochezia, anorexia, fever, and
frequent urge to have a bowel movement. It is an unavoidable
adverse effect of radiotherapy. Its development is highly variable and
depends on the duration, dosage, and gut sensitivity to radiation.
• Rapidly dividing mucosal cells of the intestine are affected by the
radiation, which leads to the symptoms. A late complication of
radiotherapy can be intestinal strictures.
Question 708
• Question 708: What composes Bence Jones proteins?
• Choices:
• 1. Albumin
• 2. Heavy immunoglobulin chains
• 3. Immunoglobulin light chains
• 4. Tamm-Horsfall protein
Answer: 3
• Answer: 3 - Immunoglobulin light chains
• Explanations: Bence Jones proteins are diagnostic for multiple
myelomas. The proteins are immunoglobulin light chains produced by
the plasma cells. The light chains are usually kappa. Bence Jones
proteins also may be seen in patients with Waldenstrom
macroglobulinemia.
Question 709
• Question 709: Chemotherapy is least likely to cause which of the
following? Choices:
• 1. Pneumonia
• 2. Nausea
• 3. Constipation
• 4. Bleeding
Answer: 3
• Answer: 3 – Constipation
• Explanations: Chemotherapy often leads to nausea and vomiting.
Chemotherapy also suppresses the bone marrow. It leads to a higher
risk of infection, bleeding, and anemia. Diarrhea is more likely than
constipation.
Question 710
• Question 710: Which of the following can be used to screen for
colorectal cancer?
• Choices:
• 1. Single contrast barium enema
• 2. Plain abdominal radiographs
• 3. Abdominal CT scan
• 4. Double contrast barium enema
Answer: 4
• Answer: 4 - Double contrast barium enema
• Explanations: Double contrast barium enema can be used to screen
for colorectal cancer. CT scan can be used in the staging of colorectal
cancer but not screening. Plain abdominal radiographs are not used in
colorectal cancer screening. Single contrast barium enema can
diagnose diverticulitis but has been supplanted by CT scan.
Question 711
• Question 711: A benign tumor of the uterus is called?
• Choices:
• 1. Leiomyoma
• 2. Cyst
• 3. Lesion
• 4. Mass
Answer: 1
• Answer: 1 - Leiomyoma
• Explanations: Benign tumors in the uterus are called a leiomyoma or
fibroid. A fibroid is from smooth or involuntary muscle. Leio means
smooth. A leiomyoma is a benign smooth muscle tumor that rarely
becomes cancer (0.1%). They can occur in any organ, but are common
in the uterus, small bowel, and the esophagus.
Question 712
• Question 712: What is the most common histology of bladder
carcinoma?
• Choices:
• 1. Adenocarcinoma
• 2. Columnar cell
• 3. Squamous cell
• 4. Transitional cell
Answer: 4
• Answer: 4 - Transitional cell
• Explanations: Bladder cancer is mostly transitional cell carcinoma.
Schistosoma haematobium infection is a risk factor for bladder cancer.
It usually results in squamous cell carcinoma of the bladder.
Question 713
• Question 713: What is the most common presentation of
mesothelioma on chest radiographs?
• Choices:
• 1. Pneumothorax
• 2. Large pleural effusion
• 3. Unilateral pleural thickening
• 4. Mediastinal adenopathy
Answer: 3
• Answer: 3 - Unilateral pleural thickening
• Explanations: The most common presentation of mesothelioma is a
unilateral plaque-like or nodular pleural thickening. Pleural effusions
are common (about 75 percent), but pleural thickening is seen in
more than 90 percent. Calcified plaques are seen in 20 percent
(evidence that asbestos exposure is a risk factor).
• Unilateral pleural thickening is sensitive, but not specific. Most
examples of unilateral pleural thickening are not mesothelioma.
Question 714
• Question 714: From which of the following does gastric
adenocarcinoma arise?
• Choices:
• 1. Mucosa
• 2. Muscle
• 3. Lymphoid tissue
• 4. Connective tissue
Answer: 1
• Answer: 1 - Mucosa
• Explanations: The overall incidence of gastric cancer has declined in
most western countries but the numbers are increasing in developing
countries. Patients are usually seen in the 5th to 7th decades of life
and are most likely to be black or Native American. The majority of
stomach cancers are adenocarcinomas arising from the mucosa.
Other cancers include lymphomas, stromal tumors, and carcinoids.
Proximal stomach cancers have been attributed to Barrett esophagus.
Distal gastric cancers have been linked to H. pylori.
Question 715
• Question 715: Obtaining aminoglycoside levels in patients is very
important due to toxicities that may be experienced by a patient. Levels
need to be monitored especially in those who may have increased or
decreased clearance rates. When should peak aminoglycoside levels be
obtained?
• Choices:
• 1. Immediately after a dose is administered
• 2. 15 minutes after a dose is administered
• 3. 30 minutes after a dose is administered
• 4. 60 minutes after a dose is administered
Answer: 3
• Answer: 3 - 30 minutes after a dose is administered
• Explanations: Peak aminoglycoside levels should be obtained 30
minutes after a dose is administered. Trough levels should be
obtained immediately before a dose. Peak and trough levels should
be drawn around the third or fourth dose when approximately
• approximately steady state conditions have been established. The
renal function should always be obtained in patients receiving
aminoglycosides.
Question 716
• Question 716: Which is true of penile cancer?
• Choices:
• 1. More common in circumcised men
• 2. Usually arises from the corona of the glans penis
• 3. Is usually an adenocarcinoma
• 4. Never spreads
Answer: 2
• Answer: 2 - Usually arises from the corona of the glans penis
Explanations: Penile cancers can occur anywhere on the penis, but
the majority form on the corona and are superficial. Penile cancers
are usually squamous cell in origin and behave like squamous cell
cancer elsewhere on the skin.
• The biggest problem with penile cancer is that most men delay
seeking help when they find a lesion on the penis. Surgery is the
mainstay of treatment. Circumcision is an effective prophylactic
measure for penile cancer.
Question 717
• Question 717: Which of the following is an indication for
prochlorperazine?
• Choices:
• 1. Infection
• 2. Miscarriage
• 3. Nausea
• 4. Gastroesophageal reflux disease
Answer: 3
• Answer: 3 - Nausea
• Explanations: Prochlorperazine is indicated for treating nausea and
vomiting. Prochlorperazine is a phenothiazine dopamine (D2) receptor
antagonist. Using prochlorperazine increases the risk of a serious
movement disorder called tardive dyskinesia. The condition may remit
once the drug is stopped. Tetrabenazine can also be offered.
• Prochlorperazine carries of a risk of orthostatic hypotension due to
antagonism of the alpha1-adrenergic receptors. Thiazide diuretics may
cause profound hypotension when taken with prochlorperazine.
Metoprolol can also cause marked hypotension if it is used with
prochlorperazine. Monitor these patients closely for adverse effects.
Question 718
• Question 718: A 55-year-old male patient, with a newly diagnosed testicular
seminoma, presents to the emergency department with complaints of severe
nausea and vomiting. Less than 24 hours ago, he underwent cisplatin
treatment. The emergency department team advises that the patient takes an
NK1 receptor antagonist before his cisplatin treatment to control his vomiting.
This drug affects a chemical predominantly in what part of the brain?
• Choices:
• 1. Medulla
• 2. Midbrain
• 3. Pons
• 4. Hypothalamus
Answer: 1
• Answer: 1 – Medulla
• Explanations: Substance P and NK1 receptors are found in the gastrointestinal (GI) tract
and two areas in the medulla that control emesis called the area postrema and the nucleus
solitarius. These areas control the vomiting reflex and contain high levels of substance P.
Emetogenic chemotherapies, such as cisplatin, cause a release of substance P that binds to
NK1 receptors triggering emesis.
• NK1R antagonists, such as aprepitant, are believed to inhibit signaling of the area
postrema, lessening the severity of chemotherapy-induced emesis. While novel therapies
involving substance P and NK1R antagonists are currently undergoing clinical testing, the
biggest clinical impact of Substance P antagonists is as antiemetic agents, for example,
aprepitant, oral or intravenous, and its prodrug fosaprepitant (IV). These drugs are used for
the prevention of nausea and vomiting associated with chemotherapy drugs and are
administered before the chemotherapy treatment initiation. Aprepitant may also be used
for the treatment of chronic refractory pruritus.
Question 719
• Question 719: A 62-year-old male with a history of hypertension, diabetes mellitus, and stage III
chronic kidney disease presents to his primary care physician with a six-month history of facial swelling
and flushing, which is worse at the end of the day. He reports night sweats and unwanted weight loss
of 10 pounds. Vital signs reveal a blood pressure of 146/82 mm Hg, heart rate of 92 beats per minute,
respiratory rate of 16 breaths per minute, and temperature of 99.1 degrees Fahrenheit. He is
saturating well on ambient air. The physical exam is notable for facial edema and flushing, as well as
proptosis. Lung exam is significant for bibasilar crackles. Labs are remarkable for hemoglobin of 10.2
g/dl and a creatinine of 1.6 mg/dl. The rest of the labs are unremarkable. Chest radiograph shows a
widened mediastinum with bilateral pleural effusions. Which of the following is the next best step in
management?
• Choices:
• 1. Chest CT without IV contrast
• 2. Chest CT with IV contrast
• 3. MRI chest without IV contrast
• 4. MRI chest with IV contrast
Answer: 3
• Answer: 3 - MRI chest without IV contrast
• Explanations: CT chest with IV contrast is the imaging modality of choice for
evaluation of anterior mediastinal masses; however, it would be
contraindicated in this patient with a creatinine of 1.6 mg/dl. IV contrast is
not needed with MRI for evaluation of anterior mediastinal masses;
therefore, MRI without contrast is the imaging
• imaging modality of choice when there is a contraindication to IV contrast.
Anterior mediastinal masses can present with compression symptoms, such
as superior vena cava syndrome, which presents with facial swelling, flushing,
and proptosis. B-symptoms and pleural effusions in this 62-year-old male
suggest a possible diagnosis of Hodgkin lymphoma, which has a bimodal
distribution of incidence.
Question 720
• Question 720: A 55-year-old man undergoes a colonoscopy, which
shows a 10 cm tubular adenomatous polyp that required extensive
abdominal surgery to fully remove. After his abdominal surgery, he is
advised a clear liquid diet for 3 days. He has tolerated the diet well.
What is the next step in the dietary progression of this patient?
• Choices:
• 1. Nill per orum
• 2. Full liquid diet
• 3. Soft food diet
• 4. Regular diet
Answer: 2
• Answer: 2 - Full liquid diet
• Explanations: A patient who is prescribed a full liquid diet follows a
specific diet type requiring the intake of all liquids and semi-liquids, but
no forms of solid intake. The main indication for a full liquid diet is in
the perioperative period. In abdominal surgeries for patients regardless
of their immunosuppressive state, the typical dietary progression post-
operatively is one that begins with a clear liquid diet while recovering
from anesthesia, followed by the full liquid diet, then soft diet, and
finally, regular dietary intake. One added benefit of a full liquid diet is
the higher amount of calories and added amount of nutrients provided,
which have been linked to fewer postoperative complications.
Question 721
• Question 721: A 44-year-old male patient presented to the hospital with diplopia, mild deafness,
nasal obstruction, and cacosomia. On examination, he has a blood pressure of 110/70 mmHg,
heart rate of 80/min with a small nodular swelling in the apex of the posterior triangle of the
neck. On nasopharyngoscopy, a nasopharyngeal mass was observed, obscuring the choana, and
a biopsy was taken showing a non-keratinized clump of cells. His complete blood count, liver
function, and renal functions are all within a normal range. On imaging studies, MRI shows a
growth eroding the skull base and clivus with extension into the parapharyngeal spaces. Which
of the following lab investigations would be further warranted for the biopsy sample?
• Choices:
• 1. Immunostaining for Epstein Barr virus
• 2. Culture for tuberculosis
• 3. Cytological evaluation for human T-cell leukemia virus
• 4. Cytological evaluation for human papilloma virus
Answer: 1
• Answer: 1 - Immunostaining for Epstein Barr virus
• Explanations: The nasopharynx is a small, tubular structure above the soft palate that connects the nose to the
oropharynx. Cancers that arise from this area are usually squamous cell carcinomas that behave differently
than the other head and neck cancers. The most commonplace of origin is the fossa of Rosenmuller, which is
the pharyngeal recess. Virus-related oncogenesis is a common theme in head/neck cancers. Oropharyngeal
cancers are associated with human papillomavirus (HPV) infection, and nasopharyngeal cancers (NPC) are
associated with the Ebstein-Barr virus (EBV). The etiology of nasopharyngeal carcinoma is complex and is not
yet completely understood. Carcinogenesis is however known to be associated with high titers of EBV. Other
risk factors include a diet consisting of preserved foods containing nitrosamines and smoking. In some
epidemiologic studies, a family history of nasopharyngeal cancer has been established as a predisposing factor
as well. In areas where EBV is not endemic, like the United States, smoking and alcohol are found to be risk
factors. In some areas of the world, circulating EBV DNA is
• being used for screening and disease surveillance. Studies have shown that high circulating levels of EBV DNA is
associated with poorer response to treatment and higher rates of distant metastasis and mortality. The survival
of patients with nasopharyngeal cancer depends on the time of diagnosis and extent of local invasion. For those
patients with local disease, the prognosis is far. For patients with locally invasive disease, the prognosis is
guarded. With the use of radiation alone, there is a 40% survival, but when it is
Question 722
• Question 722: A patient is starting chemotherapy with doxorubicin.
Which one of the following is a teaching point for this medication?
Choices:
• 1. Alopecia is a side effect, but hair growth resumes immediately on
cessation of the drug
• 2. Supplement with folic acid during this treatment
• 3. Do not receive any vaccines during the course of doxorubicin
• 4. Urine will be more yellow than usual
Answer: 3
• Answer: 3 - Do not receive any vaccines during the course of
doxorubicin
• Explanations: Doxorubicin has an immunosuppressive effect.
Immunizations are contraindicated during the course of treatment
since their immune system is compromised. Doxorubicin works on
DNA by intercalation and does not interrupt folic acid synthesis. It can
turn urine red, and hair growth typically resumes after 3 to 5 months
of therapy cessation.
Question 723
• Question 723: A patient is receiving cisplatin following the removal of
the right testis. Which of the following is a toxic effect of this drug?
• Choices:
• 1. Permanent hearing loss
• 2. Abdominal pain
• 3. Hyperpigmentation
• 4. Wheezing and crackles
Answer: 1
• Answer: 1 - Permanent hearing loss
• Explanations: Ototoxicity is one of the toxic effects of cisplatin
manifested by hearing loss and tinnitus. Ototoxicity may be severe
and it is cumulative. Audiometric testing should be performed at
baseline and prior to each dose. Patients should be well hydrated
prior to and for 24 hours after administration. Cisplatin is
contraindicated in patients with hearing impairment, preexisting renal
impairment, or with myelosuppression.
Question 724
• Question 724: What is the 5-year survival rate with stage II non-small-
cell carcinoma?
• Choices:
• 1. 80%
• 2. 65% to 75%
• 3. 40% to 50%
• 4. 25%
Answer: 3
• Answer: 3 - 40% to 50%
• Explanations: The 5-year survival rate for stage I non-small-cell
carcinoma is 70% to 80%. The 5-year survival rate for stage II is 40% to
50%. The 5-year survival rate for stage III is 25%. People who stop
smoking after a lung cancer diagnosis improve their prognosis by
decreasing the cancer progression.
Question 725
• Question 725: By what mechanism does bleomycin cause lung
fibrosis?
• Choices:
• 1. Increasing capillary permeability
• 2. Free radicals
• 3. Reperfusion injury
• 4. Increasing calcium influx
Answer: 2
• Answer: 2 - Free radicals
• Explanations: Bleomycin causes generation of free radicals and leads
to lung damage. This may result in sensitivity to oxygen toxicity. The
proinflammatory cytokines IL-18 and IL-1beta have been implicated.
All patients with prior bleomycin exposure should be tested for lung
function prior to any general anesthesia. The reason is that they may
become
Question 726
• Question 726: Which of the following disorders is associated with
bilateral involvement and has an early onset?
• Choices:
• 1. Familial polyposis coli
• 2. Gorlin syndrome
• 3. Turcot syndrome
• 4. Lobular carcinoma
Answer: 4
• Answer: 4 - Lobular carcinoma
• Explanations: Lobular cancer is a breast tumor occurring in the
lobules. Invasive lobular cancer affects 5% of women. Lobular cancer
tends to occur in both breasts. While doctors initially believed that
both breasts have an equal risk of developing cancer, the ipsilateral
breast is at greater risk. Options for treatment are the same as for
intraductal cancer and may include bilateral prophylactic mastectomy.
Question 727
• Question 727: Are bone metastases MORE common in the axial or
appendicular skeleton?
• Choices:
• 1. Axial skeleton
• 2. Appendicular skeleton
• 3. Appendicular skeleton, but not distal to knee or elbow
• 4. It varies by primary cancer; for example, axial for prostate,
appendicular for lung
Answer: 1
• Answer: 1 - Axial skeleton
• Explanations: By far, the most metastases go to the axial skeleton:
spine, pelvis, thorax, and skull. If a met goes to the appendicular
skeleton, it is far more likely to be proximal to the knee and elbow.
Mets distal to these joints are very rare. Blastic versus lytic varies by
primary; actual met location does not.
Question 728
• Question 728: What is the maximum duration for which megestrol
acetate can be used at a time for the management of cancer anorexia
cachexia syndrome?
• Choices:
• 1. 8 to 12 weeks
• 2. 12 to 15 weeks
• 3. 4 to 6 weeks
• 4. 15 to 20 weeks
Answer: 1
• Answer: 1 - 8 to 12 weeks
• Explanations: The study duration in Cochrane systematic review by
Garcia et al. ranged from 2 to 24 weeks. The median trial duration
was 8 weeks. Assessments were carried out at 2 to 3 months of
treatment with megestrol acetate. Mortality was increased in
patients on megestrol acetate. However, the same finding should be
interpreted with caution as these are patients with advanced cancer
in whom the mortality is high. It is important to remember that
megestrol acetate will not be effective in those with refractory
cachexia.
Question 729
• Question 729: Which complaint is most common among patients
receiving radiation therapy?
• Choices:
• 1. Insomnia
• 2. Cough and difficulty breathing
• 3. Fatigue
• 4. Lower extremity swelling
Answer: 3
• Answer: 3 – Fatigue
• Explanations: Fatigue is a common compliant after radiation therapy.
Cough and difficulty breathing could be due to radiation pneumonitis.
Lower extremity swelling could be due to pericardial inflammation.
Mucositis is also a common acute toxicity of radiation therapy
Question 730
• Question 730: A 66-year-old female presents with vaginal bleeding for the
past 4 months. She initially thought this was due to postmenopausal changes,
but now she wants to make sure that nothing is wrong. She undergoes a
transvaginal ultrasound which reveals a thickened endometrium. Sampling of
the endometrium reveals the presence of a malignancy. The incidence of
endometrial cancer is least common in which population?
• Choices:
• 1. European women
• 2. Jewish women
• 3. African American women
• 4. American white women
Answer: 3
• Answer: 3 - African American women
• Explanations: Endometrial cancer incidence peaks in the sixth decade
of life. It is the fourth most common cancer in women in the United
States. It is least common in African American women and most
common in Jewish women. Ultrasound may reveal a thickened
endometrium or a polyp-like lesion. The usual presentation is
postmenopausal bleeding.
Question 731
• Question 731: Which of the following malignancies with bone
metastasis has the worst 5-year survival rate?
• Choices:
• 1. Lung
• 2. Breast
• 3. Thyroid
• 4. Prostate
Answer: 1
• Answer: 1 – Lung
• Explanations: Lung metastasis has the worst 5-year survival rate. The 5-year
survival rate is less than 5%. Bone metastases result from primary tumor
invasion to the bone. Bone-originating primary tumors such as
osteosarcoma, chondrosarcoma, and Ewing sarcoma are rare. Bone
metastases arise from epithelial tumors and form a solid mass of the bone.
Bone metastases cause a dull, constant ache with periodic spikes of incident
pain. The common sites of bone metastases are the spine, pelvis, ribs, skull,
and proximal femur. Bone is the third most common location for metastasis,
after the lung and liver. The microenvironment of the marrow tends to favor
particular types of cancer, including prostate, breast, and lung cancers. In
prostate cancer, bone tends to be the only site of metastasis.
Question 732
• Question 732: A 45-year-old female presents with a history of serosanguinous
discoloration on her left inner bra for three weeks. There is no weight loss, loss of
appetite, swelling, discoloration, or tenderness in her left breast. She has no similar
history in the past but was diagnosed with fibroadenoma of left breast at 16 years of age,
which was later removed surgically because of its large size. She has no family history of
breast cancer. Her vitals include heart rate: 90/minute, blood pressure: 110/85 mm of Hg,
temperature: 98.5 degrees Fahrenheit. A mammogram is normal. What is the next best
step in the management of this patient?
• Choices:
• 1. Follow-up mammography in one year
• 2. Follow-up mammography in six months
• 3. Ultrasound of the left breast
• 4. Cytology of the discharge
Answer: 4
• Answer: 4 - Cytology of the discharge
• Explanations: In this clinical context, the two major concerns are
papilloma and cancer. In the face of a negative mammogram, cytology
of discharge and pictogram is needed. Cytology of the fluid will reveal
pleomorphic cells in case of malignancy. If a filling defect is found on
ductography, correlative directed ultrasound is often helpful. This
patient is 45 years old which puts her at a higher risk for malignancy.
Follow up studies will be detrimental to the management of this
patient.
Question 733
• Question 733: What is the most common malignancy seen in HIV
infected individuals?
• Choices:
• 1. Colorectal cancer
• 2. Prostate cancer
• 3. Kaposi sarcoma
• 4. Malignant melanoma
Answer: 3
• Answer: 3 - Kaposi sarcoma
• Explanations: Kaposi sarcoma (KS) is the most common malignancy
seen in HIV infected individuals. It serves as an AIDS-defining illness.
KS is linked to human herpesvirus 8. The availability of HAART (highly
active antiretroviral therapy) has significantly decreased the incidence
and severity of KS.
Question 734
• Question 734: A 32-year-old African American female has a breast
mass. What is the standard treatment option for a premenopausal
patient with breast cancer and negative for ER, PR, and HER2?
Choices:
• 1. Estrogen
• 2. Chemotherapy
• 3. Tamoxifen
• 4. Radiation
Answer: 2
• Answer: 2 – Chemotherapy
• Explanations: ER or PR negative breast cancer is treated with chemotherapy. Hormone receptor-
positive breast cancer cells have either estrogen (ER) or progesterone (PR) receptors. These can be
treated with hormone therapy drugs that lower estrogen levels or block estrogen receptors. This
includes cancers that are ER-negative but PR-positive. Hormone receptor-positive cancers grow more
slowly than those that are receptor-negative. Women with hormone receptor-positive cancers have a
better outlook in the short-term, but these cancers come back many years after treatment. Hormone-
receptor-negative breast cancers have neither estrogen nor progesterone receptors. Treatment with
hormone therapy is not helpful. These cancers grow faster than hormone receptor-positive cancers. If
they come back after treatment, it is usually within the first few years. Hormone receptor-negative
cancers are common in women who have not yet gone through menopause. Triple-negative breast
cancer cells do not have estrogen or progesterone receptors and also do not make the protein HER2.
These cancers tend to bemore common in young women and in women who are African-American or
Hispanic/Latina. Triple-negative cancers grow and spread faster. Because the cancer cells do not have
hormone receptors, hormone therapy is not helpful. Drugs that target HER2 are not helpful either.
Chemotherapy can be useful.
Question 735
• Question 735: Which of the following is the correct definition of
glottic carcinoma?
• Choices:
1. Tumor that occurs on the laryngeal surface of the epiglottis
• 2. Tumor that occurs on the true vocal cords
• 3. Tumor that occurs in the piriform sinus
• 4. Tumor that occurs in the laryngeal ventricle
Answer: 2
• Answer: 2 - Tumor that occurs on the true vocal cords
• Explanations: Glottic cancer is a malignancy of the larynx. Although
cancers may occur in several subsites of the larynx, glottic tumors, by
definition, occur on the true vocal cords and may involve the
posterior and anterior commissures. Because of the anatomical
location, glottic cancer significantly affects basic vital functions like
swallowing, breathing, and voice. Mortality is very high with this
malignancy. The two major risk factors include smoking and alcohol. A
diet that lacks fruits and vegetables is also a risk factor for this
malignancy. .
Question 736
• Question 736: At what level of serum calcium would one suspect a
malignant cause?
• Choices:
• 1. 7 mg/dL
• 2. 9 mg/dL
• 3. 11 mg/dL
• 4. 13 mg/dL
Answer: 4
• Answer: 4 - 13 mg/dL
• Explanations: A calcium level over 12 mg/dL is most likely caused by
cancer. Symptomatic, severe hypercalcemia associated with malignancy
portends a poor prognosis. Symptoms of hypercalcemia are nausea,
polyuria, confusion, kidney stones, and weakness. Symptomatic
hypercalcemia requires emergent treatment. Cancer-associated
hypercalcemia occurs in about one-third of patients. In most cases,
these patients have advanced cancer and a poor prognosis. Common
malignancies associated with hypercalcemia include lung or breast
cancer and multiple myeloma. Hematologic malignancies like
lymphomas and leukemias also may be associated with hypercalcemia.
Question 737
• Question 737: Which statement regarding T cells is correct?
• Choices:
• 1. They mature in the bone marrow
• 2. They are found in the germinal centers of lymph nodes
• 3. They are involved in humoral and cell-mediated immunity
• 4. They are progenitors of plasma cells
Answer: 3
• Answer: 3 - They are involved in humoral and cell-mediated immunity
Explanations: T cells are found in the paracortex of lymph nodes once
they leave the thymus and go into circulation. T cells are the effectors
of cell-mediated immunity and aid in humoral immunity by activating
B cells to respond to protein-based antigens. T cells mature in the
thymus. Plasma cells are mature cells involved with humoral
immunity, producing antibodies. B cells, not T cells, are the
progenitors of plasma cells.
Question 738
• Question 738: A postmenopausal 68-year-old female, gravida 2, para 2 is diagnosed with endometrial
cancer. She initially presented to the emergency department with the passage of clots, heavy vaginal
bleeding, and feeling pre-syncopal. Her blood pressure was 87/55 mmHg, her heart rate was 120
beats per minute, and her hemoglobin was noted to be 7.6 g/dl. She received 1 unit of packed red
blood cells with symptomatic improvement. Further history reveals that she underwent menarche at
the age of 11 and menopause at the age of 60. She is obese, but otherwise, her past medical, past
surgical, family and social history are noncontributory. Pelvic examination was concerning for blood
coming from the cervical os without any other masses or abnormalities. Pelvic ultrasound
demonstrated a 7 mm heterogenous endometrial stripe. What is the most sensitive method for
diagnosing endometrial cancer?
• Choices:
• 1. Pelvic ultrasound
• 2. Fractionated endometrial curettage
• 3. Endometrial biopsy
• 4. Pap smear
Answer: 2
• Answer: 2 - Fractionated endometrial curettage
• Explanations: Endometrial carcinoma comes from columnar cells of
the endometrial lining. The definitive diagnosis of endometrial cancer
requires an endometrial tissue sample via curettage of the
endometrium. Risk factors for endometrial carcinoma include
infertility, delayed menopause, nulliparity, unopposed estrogen use,
polycystic ovary syndrome, diabetes mellitus, chronic tamoxifen use,
hypertension, obesity, and adenomatous hyperplasia. Previous breast,
colon, or ovarian cancer also increases the risk of developing
endometrial cancer. Pap smears only detect endometrial cancer 38%
of the time.
Question 739
• Question 739: What is the most common symptom of laryngeal
cancer?
• Choices:
• 1. Hoarseness
• 2. Cough
• 3. Odynophagia
• 4. Weight loss
Answer: 1
• Answer: 1 - Hoarseness
• Explanations: Hoarseness, or change in voice, would be the most
common symptom of laryngeal cancer. Other symptoms may include
ear pain, halitosis, odynophagia, cough, sore throat, and a palpable
lump in the neck. The diagnosis is usually made by an
otolaryngologist. The predominant type of laryngeal cancer is
squamous cell carcinoma.
Question 740
• Question 740: A 66-year-old male has multiple osteolytic punched-out
lesions on plain radiography. What is the most likely diagnosis?
• Choices:
• 1. Prostate cancer
• 2. Multiple myeloma
• 3. Tuberculosis
• 4. Hyperparathyroidism
Answer: 2
• Answer: 2 - Multiple myeloma
• Explanations: Multiple myeloma should be a top consideration in any
lytic lesion in a patient over the age of 50 years. The most common
locations for these lesions include the skull, spine, pelvis, and ribs.
Multiple punched-out, well-defined lesions are classic for multiple
myeloma. Multiple myeloma is less evident on bone scintigraphy than
on plain radiography. It is distinct from metastatic disease because
myelomas have less of an osteoblastic reparative response than
metastatic disease.
Question 741
• Question 741: A 65-year-old male complains of itching, jaundice, anorexia,
and weight loss over 4 weeks. His urine is dark, and her stools are light.
Abdominal examination shows hepatomegaly and gallbladder distention.
Liver enzymes are elevated. Ultrasound and CT shows dilated bile ducts and
intrahepatic biliary dilation but no definitive cause. What is the next step in
evaluation?
• Choices:
• 1. Percutaneous transhepatic cholangiography (PTC)
• 2. Laparoscopy
• 3. Magnetic resonance cholangiopancreatography (MRCP)
• 4. Ultrasound-guided liver biopsy
Answer: 3
• Answer: 3 - Magnetic resonance cholangiopancreatography (MRCP)
Explanations: The most likely diagnosis is cholangiocarcinoma. MRCP
or ERCP would be the next step in evaluation. Liver biopsy is not
indicated. PTC is reserved for when ERCP is not successful.
Question 742
• Question 742: A 17-year-old male presents with a slowly enlarging, painless
mass on his forearm near the wrist. A biopsy confirms the diagnosis of
epithelioid sarcoma. Which of the following is true regarding this tumor?
• Choices:
• 1. Epithelioid sarcoma is considered malignant only if it metastasizes to
distant sites
• 2. Epitheloid sarcoma commonly metastasizes to regional lymph nodes
• 3. Epithelioid sarcoma is an indolent, benign tumor
• 4. Epithelioid sarcoma is a misnomer; it is a variant of squamous cell
carcinoma arising from the epidermis
Answer: 2
• Answer: 2 - Epitheloid sarcoma commonly metastasizes to regional
lymph nodes
• Explanations: The most common site of distant metastasis of
epithelioid sarcoma is the lung. Lymph node examination should be
performed, as epithelioid sarcoma commonly spreads via lymphatics.
Epithelioid sarcoma, though an indolent tumor, is locally aggressive
with metastatic potential. Although there is no known cell of origin,
epithelioid sarcoma is thought to be a true sarcoma of mesenchymal
origin with epithelial differentiation.
Question 743
• Question 743: Which of the following viruses is associated with the
development of Kaposi sarcoma?
• Choices:
• 1. Human herpesvirus 8
• 2. Epstein-Barr virus
• 3. Parvovirus B19
• 4. Human herpesvirus 6
Answer: 1
• Answer: 1 - Human herpesvirus 8
• Explanations: The etiology of Kaposi sarcoma is human herpesvirus 8
infection. Though many people are infected with human herpesvirus
8, a co-infection with HIV increases the risk of Kaposi sarcoma. Co-
infection with HIV leads to reprogramming of the host's blood
endothelial cells to resemble lymphatic endothelium, which leads to
the development of Kaposi sarcoma. Epstein-Barr virus is associated
with infectious mononucleosis. Parvovirus B19 is associated with
erythema infectiosum or fifth disease. Human herpesvirus 6 is
associated with exanthem subitum or roseola.
Question 744
• Question 744: A 65-year-old male patient with a past medical history of testicular
cancer diagnosed at the age of 35, for which he received chemoradiation, now
presents to the hospital with shortness of breath. On examination, his vitals are
stable except for oxygen saturation of 90% on room air. His lung exam reveals
scattered fine crepitations. A CT chest is done, which shows pulmonary fibrosis.
Which of the following agents he may have received 20 years ago, to develop this
pulmonary disorder?
• Choices:
• 1. Doxorubicin
• 2. Dexamethasone
• 3. Bleomycin
• 4. Vincristine
Answer: 3
• Answer: 3 – Bleomycin
• Explanations: Bleomycin pulmonary toxicity (BPT) is seen in 10-40% of all treated
with bleomycin, usually 4-10 weeks after initiation of treatment, but it can occur
later as well. Pulmonary toxicity is age and dose-related, occurring mostly in
patients in the geriatric age, or in those who receive a total dose greater than 400
units. However, it has been reported in doses as low as 20-60 units. Fatal pulmonary
fibrosis has been seen in geriatric patients on dosage as low as 20 units. It may be
reversible or fatal, however, evidence suggests that in patients who have survived,
pulmonary parameters return to baseline in approximately 2 years. Identified
clinically by the presence of pulmonary symptoms with bilateral interstitial
infiltrates and no evidence of an infectious etiology. In patients receiving other anti-
neoplastic drugs or radiation, pulmonary toxicity can occur at even low bleomycin
doses with mortality as high as 10%. The sequential measurement of pulmonary
Question 745
• Question 745: Using the American Joint Committee on Cancer
classification for head and neck lymph node location, what is the level
of a submental node?
• Choices:
• 1. I
• 2. II
• 3. III
• 4. IV
:Answer
• Answer: 1 - I
• Explanations: The AJCC lymph node classification system has ten
levels. Levels II, III, IV, and V are bilaterally symmetric. Levels I, VI, and
VII are midline. Level I is submental and divided into A and B, A being
medial to the anterior belly of the digastric muscle, formerly termed
"submental lymph nodes"
Question 746
• Question 746: A patient is prescribed dolasetron mesylate. There is a
particular concern about precipitating the most dangerous side effect of
this medication when it is prescribed for long durations. What test
would be ordered to risk stratify this patient for the likelihood of
experiencing this specific side effect?
• Choices:
• 1. Liver function tests
• 2. Complete blood count
• 3. EKG
• 4. Pulmonary function test
Answer: 3
• Answer: 3 - EKG
• Explanations: Dolasetron is prescribed to decrease nausea and
vomiting during chemotherapy. Dolasetron functions as a 5-HT3
receptor antagonist. QT prolongation is the most feared side effect of
antiemetics. In order to assess for risk of developing QT prolongation
from chronic dolasetron usage, one should assess a baseline EKG to
evaluate the patient's baseline QTc.
Question 747
• Question 747: Which of the following is true about salivary gland
tumors?
• Choices:
• 1. Parotid masses usually present in the tail of the gland
• 2. Submandibular neoplasms cause diffuse enlargement of the entire
gland
• 3. Sublingual tumors produce a fullness in the floor of the mouth
• 4. Fine needle biopsy revealing predominance of lymphocytes
requires antibiotic therapy
Answer: 4
• Answer: 4 - Fine needle biopsy revealing predominance of lymphocytes
requires antibiotic therapy
• Explanations: Most salivary glands present with a slowly enlarging, painless
mass. A discrete mass is usually found on an accidental exam. The use of a
fine needle biopsy in the work-up of a salivary gland mass is still being
debated. Evevn if the needle biopsy is negative, this does not mean there is
no tumor. When a fine needle biopsy reveals the predominance of
lymphocytes, one should always consider a lymphoma. These tumors often
present in the submandibular gland. Fine needle biopsy for warthin tumors
can lead to false positives and good clinical judgment is required before
proceeding to aggressive treatment. In addition, fine needle biopsy can lead
to infarction and subsequent parotitis.
Question 748
• Question 748: Lung cancers most commonly spread to which organ?
• Choices:
• 1. Brain
• 2. Adrenal
• 3. Liver
• 4. Spleen
Answer: 3
• Answer: 3 - Liver
• Explanations: The most common site of metastatic spread of lung
cancers is the liver. The bone is also a common site for the spread of
lung cancer. These patients often complain of bone pain and tender
spots. Spread to the pericardium may present as tamponade or a
large pleural effusion. Once the lung cancer spreads to the liver, the
prognosis is very poor.
Question 749
• Question 749: In a 50-year-old female with a hard 2 x 2 cm malignant
breast mass, which of the following features will worsen her
prognosis?
• Choices:
• 1. Family history
• 2. Positive estrogen receptor test
• 3. Tethering of skin
• 4. Presence of 4 or more lymph nodes in axilla
Answer: 4
• Answer: 4 - Presence of 4 or more lymph nodes in axilla
• Explanations: The presence of lymph nodes indicates that the tumor
has spread. Invasion of lymph nodes is consistent with an invasion of
the lymphatic system and is considered a poor prognostic sign. Once
the spread of lymph nodes has occurred, treatment may include
hormone blocking therapy, chemotherapy, and monoclonal
antibodies. When three or more lymph nodes are positive with a 5 cm
mass, it is considered stage 2.
Question 750
• Question 750: Methotrexate is a drug that can be used to treat some
forms of cancer and some autoimmune disorders. It functions by
inhibiting an enzyme involved in the production of tetrahydrofolate.
Which of the following statements regarding tetrahydrofolate is false?
• Choices:
• 1. Functions as an enzyme derived from folic acid
• 2. Functions as a single-carbon group donor
• 3. Involved in the synthesis of DNA
• 4. Involved in the synthesis of amino acids
Answer: 1
• Answer: 1 - Functions as an enzyme derived from folic acid
Explanations: Tetrahydrofolate is a coenzyme, not an enzyme, that is
derived from folic acid. It functions as a single-carbon group donor
and is involved in the biosynthesis of nucleic acids and amino acids.
Tetrahydrofolate is produced by the enzyme dihydrofolate reductase
(DHFR) acting on dihydrofolic acid. It is this reaction that is inhibited
by methotrexate. As a result, methotrexate's action leads to inhibited
synthesis of DNA, RNA, and proteins, from which it derives its
antineoplastic activity. Methotrexate's activity in the treatment of
rheumatoid arthritis appears to involve other mechanisms in addition
to the inhibition of DHFR.
Question 751
• Question 751: A 52-year-old male patient presented with a history of an
allogeneic bone marrow transplant (matched sibling donor) performed in
a reference hospital due to acute myeloid leukemia one year before. The
patient started having oral and skin lesions. Which of the following is the
investigation of choice to rule out graft versus host disease in this patient?
• Choices:
• 1. Complement levels
• 2. HLA crossmatch
• 3. Mixed lymphocyte culture
• 4. Biopsy
Answer: 4
• Answer: 4 – Biopsy
• Explanations: No blood test can identify a graft versus host reaction.
A biopsy is a primary tool for the evaluation of suspected graft versus
host disease. The diagnosis can be complicated from other viral
exanthems and drug eruptions and requires clinical correlation. The
most common cause of death in acute graft versus host reaction is
overwhelming sepsis.
Question 752
• Question 752: Which of the following cancers has the worst
prognosis?
• Choices:
• 1. Hodgkin lymphoma
• 2. Choriocarcinoma
• 3. Basal cell carcinoma
• 4. Actinic keratosis
Answer: 2
• Answer: 2 – Choriocarcinoma
• Explanations: Choriocarcinoma is a malignant type of gestational
trophoblastic cancer. Choriocarcinoma is much more likely than other
types of gestational trophoblastic cancers and tends to spread quickly.
Choriocarcinoma can develop from both a complete hydatidiform
mole and a partial mole. Choriocarcinoma can also develop in man
and not be related to pregnancy. Although choriocarcinoma most
often develops from a complete hydatidiform mole, it can also occur
after a partial mole, a normal pregnancy, or a pregnancy in which the
fetus is lost early.
Question 753
• Question 753: What is the most likely diagnosis in a 64-year-old
female with bloody nipple discharge from the right nipple?
• Choices:
• 1. Paget disease of nipple
• 2. Intraductal papilloma
• 3. Phyllodes tumor
• 4. Lobular carcinoma
Question 754
• Question 754: A 40-year-old male with a history of renal cell carcinoma presents with a feeling of
heaviness in his scrotum. He noticed it two months ago but did not want to see physicians yet.
He denied any weight loss, cough, hemoptysis, abdominal pain. He was a former smoker but quit
five years ago. Vitals show blood pressure of 120/70 mmHg, a pulse of 60 beats per min,
respiratory rate 15 breaths/minute with oxygen saturation of 92% on room air. On physical exam,
the left testicle is distended, and there is no tenderness on palpation. No skin lesions or lumps
are present. Laboratory workup reveals a hemoglobin of 20 g/dl. What is the next step in the
evaluation of this patient after the ultrasound confirmation of the testicular pathology?
• Choices:
• 1. MRI abdomen pelvis
• 2. CT abdomen pelvis
• 3. Urinalysis
• 4. Erythropoietin levels
Answer: 2
• Answer: 2 - CT abdomen pelvis
• Explanations: CT abdomen and pelvis with contrast should be performed in
patients with a history of renal cancer to rule out the extension of cancers
into renal veins. The clot or actual tumor invading renal veins have a filling
defect appearance on CT scan abdomen with contrast. New studies
recommend that a CT scan of the abdomen and pelvis should be ordered in
a high-risk patient such as those with renal cell cancer, co-presence of risk
factors (e.g. old age) if the patient has the only history of left-sided
varicocele. Tumor involvement of the renal vein changed renal cell
carcinoma staging from stage 2 to 3, altering the course of the disease.
Erythropoietin level is high in renal cell carcinoma but cannot help identify
the cause of varicocele.
Question 755
• Question 755: Which precaution regarding the preparation and
administration of chemotherapy is most appropriate?
• Choices:
• 1. A pair of surgical gloves should always be worn when handling
chemotherapeutic agents
• 2. Infusions of chemotherapeutic agents can be prepared the same way as
infusions of other drugs
• 3. Spills of chemotherapeutic agents should be cleaned immediately with 40
percent isopropyl alcohol
• 4. All supplies used in preparing chemotherapeutic agents should be
discarded in designated chemotherapy waste containers
Answer: 4
• Answer: 4 - All supplies used in preparing chemotherapeutic agents
should be discarded in designated chemotherapy waste containers
Explanations: Chemotherapeutic agents may cause cancer, birth
defects, and infertility. Precautions should be taken when handling
these drugs. Gloves should always be worn. Chemotherapy gloves are
preferred, but two pairs of surgical gloves can be substituted.
Infusions of chemotherapeutic agents should be prepared in areas
specifically designed for this purpose, for example, biosafety cabinets.
Spills of chemotherapeutic agents should be cleaned immediately
with approved solutions, such as 70 percent isopropyl alcohol.
Question 756
• Question 756: What is the most important factor associated with the
development of penile cancer?
• Choices:
• 1. Balanitis
• 2. Smoking
• 3. Herpes virus infections
• 4. Intact foreskin
Answer: 4
• Answer: 4 - Intact foreskin
• Explanations: There is evidence that an intact foreskin is associated
with the development of penile cancer. Jewish and Muslim males
rarely develop penile cancer. It appears that circumcision protects
against penile cancer. Infection with some types of human
papillomavirus appears to be an important risk factor for penile
cancer.
Question 757
• Question 757: What is the most common cancer to affect bones?
• Choices:
• 1. Osteosarcoma
• 2. Osteochondroma
• 3. Multiple myeloma
• 4. Chondrosarcoma
Answer: 3
• Answer: 3 - Multiple myeloma
• Explanations: Although not a primary cancer of bone tissue, multiple
myeloma is the most frequent neoplasm found in the bones. Bone
lesions in multiple myeloma are classically osteolytic. Multiple
myeloma is mostly a disease of the elderly. The lesions are usually
painful.
Question 758
• Question 758: Which of the following is used for the treatment of
post-lung transplant malignancy?
• Choices:
• 1. Additional chemotherapy
• 2. Radiation therapy
• 3. Increased dose of present chemotherapy
• 4. Reduced dose of present chemotherapy
Answer: 4
• Answer: 4 - Reduced dose of present chemotherapy
• Explanations: Post-transplant malignancies are associated with
Epstein-Barr infections. The malignancies are treated by decreasing
the dose of chemotherapy and giving acyclovir. In the last decade,
rapamycin inhibitors have been advocated in transplant patients, both
to suppress the immune system and decrease the risk of cancer.
Retrospective studies have shown a decreased incidence of de novo
cancers in patients treated with rapamycin inhibitors.
Question 759
• Question 759: Thalidomide is approved by the FDA to be used for the
treatment of which of the following?
• Choices:
• 1. Multiple myeloma
• 2. Ectopic pregnancy
• 3. Tuberculosis
• 4. Acne
Answer: 1
• Answer: 1 - Multiple myeloma
• Explanations: Thalidomide has been approved by the FDA for use in
the treatment of multiple myeloma. Its anticarcinogenic properties
may be secondary to the inhibition of angiogenesis. The drug also is
used to treat some subtypes of leprosy, graft versus host disease, and
wasting syndrome in patients with HIV. However, to prevent its
misuse, the drug is used under strict control.
Question 760
• Question 760: Whis is true of soft-tissue radionecrosis?
• Choices:
• 1. Delayed effects of radiation therapy which results in damage to capillary
beds and arterials, obliterative endarteritis, leading to fibrosis of the tissue
involved and poor wound healing after trauma
• 2. Acute damage done to tissue after radiation therapy that leads to
inflammation in the tissue from the damage done to DNA at the cellular level
• 3. Hyperbaric oxygen therapy has a very limited adjunctive role in the
treatment of soft-tissue radionecrosis
• 4. Soft-tissue radionecrosis is oftentimes a mild disease needing only
supportive therapies such as topical steroids and skin moisturizers
Answer: 1
• Answer: 1 - Delayed effects of radiation therapy which results in damage to capillary beds
and arterials, obliterative endarteritis, leading to fibrosis of the tissue involved and poor
wound healing after trauma
• Explanations: The delayed effects of radiation therapy can occur months to several years
after the initial radiation damage. The damage done to the smaller arterioles and capillary
beds leads to an obliterative endarteritis which then decreases tissue perfusion resulting in a
relatively hypoxic environment. There are fibro atrophic changes that develop in the tissue
and these changes are associated with several cytokines that increase after radiation injury.
TGF-beta has been the most frequently studied cytokine associated with radiation injury. The
pathogenesis of soft-tissue radionecrosis is complex and is not fully understood at this point.
Hyperbaric oxygen therapy has been shown to be beneficial for the treatment of soft-tissue
radionecrosis in numerous studies and has an important adjunctive role. Some cases of soft-
tissue radionecrosis may be mild resulting in fibrotic changes of the skin. Skin moisturizers
may be suitable for treating the mild symptoms. More often the damage to the tissue is
much deeper than
• the skin and results in wounds that do not heal even after mild unintentional
trauma or from surgical trauma. Approximately 5 to its 10 percent of the patients
who have had radiation therapy to the head and neck for cancers located in this
region will suffer from soft-tissue radionecrosis or osteoradionecrosis especially
of the mandible. Soft-tissue radionecrosis may involve multiple organ systems
depending upon the dose of the radiation therapy and the anatomic location
relative to the radiation field. Common locations of soft-tissue radionecrosis
include: radiation cystitis, proctitis, enteritis, and damage to skin, muscle, and
bone of the chest wall. Soft-tissue radionecrosis of the extremities is less
common because cancers located in those regions of the body are less common.
The central nervous system can also have a delayed response to radiation,
termed radiation myelitis. Fortunately, this difficult to treat condition is rare. In
this condition there is a temporary demyelination that can occur. When it
involves the spinal cord it is termed Lhermitte syndrome. Patients experience
electric like shocks down the legs with spinal flexion.
Question 761
• Question 761: A 72-year-old female smoker with type 2 diabetes and alcoholic
cirrhosis dies of complications from her chronic illnesses. Autopsy shows a mass
in the pancreas that is hard and infiltrative. The rest of the pancreas shows
fibrosis with acinar loss, dilation of ducts, moderate infiltration of lymphocytes,
but normal islets. Select the most likely findings for the pancreatic mass.
• Choices:
• 1. Dense, fibrotic stroma with moderately differentiated cells that secrete mucus
• 2. Fibrosis with acinar loss, dilation of ducts, moderate infiltration of
lymphocytes
• 3. Necrosis with neutrophilic infiltrate
• 4. Atrophy of exocrine glands, fibrosis, and plugging of ducts
Answer: 1
• Answer: 1 - Dense, fibrotic stroma with moderately differentiated cells
that secrete mucus
• Explanations: Risk factors for pancreatic cancer include smoking
tobacco, chronic pancreatitis, and diabetes mellitus. The findings for
most of the pancreas were consistent with chronic pancreatitis. The
description in the first option was that of ductal pancreatic
adenocarcinoma.
Question 762
• Question 762: A patient complains of headaches and blurry vision
that has gotten worse over the past year. MRI shows a well-defined
mass attached to the meninges with a dural tail. Select the most likely
diagnosis.
• Choices:
• 1. Oligodendroglioma
• 2. Low-grade astrocytoma
• 3. Meningioma
• 4. Glioblastoma
Answer: 3
• Answer: 3 – Meningioma
• Explanations: This is most likely a meningioma that is usually benign
and attached to the dura. Meningiomas rarely invade the brain and
are more common in women than in men. Surgical resection is usually
curative. Extra-axial tumors, such as schwannomas and meningiomas,
have sharp margins
Question 763
• Question 763: Increased levels of parathyroid hormone secondary to
a tumor will increase the activity of which of the following?
• Choices:
• 1. Osteoclasts
• 2. Osteoblasts
• 3. C-cells (parafollicular cells)
• 4. Osteoclasts and C-cells
Answer: 4
• Answer: 4 - Osteoclasts and C-cells
• Explanations: Parathyroid hormone (PTH) increases bone resorption
by osteoclasts. Increased blood calcium stimulates the secretion of
calcitonin by the C-cells of the thyroid. In metastasis, bones can be
invaded by cancer stimulating local osteolytic activity increasing
serum calcium levels. In certain cancers like breast and prostate,
tumors may release 1-alpha-hydroxylase increasing the synthesis of
calcitriol allowing for excessive reabsorption of calcium. In bladder,
kidney, ovarian, uterine, head and neck, lung, and breast carcinomas
the primary tumor may secrete parathyroid hormone-related protein
(PTHrP).
Question 764
• Question 764: Chemotherapeutic agents have adverse effects that
pass through three stages: onset, maximum depression, and recovery.
Which of the following indicates maximum depression?
• Choices:
• 1. Start time
• 2. Retention time
• 3. Suppression time
• 4. Nadir time
Answer: 4
• Answer: 4 - Nadir time
• Explanations: Nadir is the time of deepest depression. In
chemotherapy, the term refers to the length of time before maximum
bone marrow depression occurs. Many chemotherapeutic agents
cause low leukocyte counts. When the white cell count drops, the
patient is prone to many types of bacterial and viral infections. By
anticipating
Question 765
• Question 765: During an outpatient laparoscopic cholecystectomy on a 69-
year-old female for acute or chronic cholelithiasis, a 1 cm nodule is found at
the dome of the gallbladder during the visual examination. There is
puckering of the serosa but no other findings at the gallbladder. What should
be done?
• Choices:
• 1. Convert to open cholecystectomy
• 2. Continue the open cholecystectomy procedure without a frozen section
• 3. Laparoscopic biopsy of the nodule and wait for frozen section report
• 4. Aspirate the bile and send for cytology. If positive, use a retrieval bag to
prevent spillage
Answer: 1
• Answer: 1 - Convert to open cholecystectomy
• Explanations: Surgeons performing laparoscopic cholecystectomy should have a high
index of suspicion for gallbladder cancer in patients with risk factors such as chronic
cholelithiasis or others. Conversion to open procedure allows staging of the tumor
and adequate resection. A positive frozen for malignancy will require extended
cholecystectomy, for example, to include partial regional lymphadenectomy and
hepatic resection. For any suspected malignant lesion encountered intraoperatively
during laparoscopic cholecystectomy, it is recommended not to sample the lesion
laparoscopically to reduce the hazard of seeding. A frozen section has a higher
diagnostic yield when compared to aspiration cytology, which is not indicated or
routinely performed. Surgeons on an outpatient center may not perform a more
complex hepatobiliary surgery, closing the patient with or without a simple
cholecystectomy and referring to a high-volume center is appropriate.
Question 766
• Question 766: A 53-year-old male patient was admitted due to a space-occupying lesion of the right liver observed by
physical examination 20 days previously. A computed tomography scan revealed disproportion of the liver lobes and a 4-
cm low-density focus in the right posterior liver lobe. Preoperative cytomegalovirus (CMV), Epstein-Barr virus (EBV),
Coxsackievirus, Herpes simplex virus, and autoimmune antibody series were negative. The preoperative human leukocyte
antigen type was also negative. The patient tested positive for hepatitis B surface antigen, hepatitis B e-antibody, and
hepatitis B c-antibody and had an alpha-fetoprotein level of 288,810 µg/L and B-type blood. Liver function tests,
biochemistry, and blood routine tests were within the normal range. The patient received classic orthotropic liver
transplantation. Postoperatively, a scattered red rash was observed behind the left ear and on the neck. Laboratory
analysis at 33rd postoperative day revealed that the white blood cell (WBC) count was 3,740 cells/microliter, the red blood
cell count was 2.65 million cells/microliter, the platelet count was 83,000/microliter, the concentration of blood urea
nitrogen (BUN) was 11.8 mmol/L, the creatine level was 96 µmol/, and liver function was near the normal value. The
patient had yellow watery diarrhea following surgery and continued to have a fever of unknown cause, a rash, symptoms
of the digestive tract, leukocytopenia, and pancytopenia. Which of the following is the most likely mediator cell
responsible for the patient's condition?
• Choices:
• 1. B cell
• 2. T cell
• 3. Neutrophil
• 4. Macrophage
Answer: 2
• Answer: 2 - T cell
• Explanations: Graft versus host disease is mediated by T-cells
attacking the host cells. It is characterized by fever, rash, high alkaline
phosphatase, and elevated levels of bilirubin. Acute graft versus host
reaction has a distinct presentation of hepatitis, dermatitis, and
enteritis, which usually develops within the first three months of a
transplant. Chronic graft versus host is more diverse and may occur de
novo. It can present with multiple organ involvement. Acute graft
versus host reaction is treated with prophylactic therapy with the use
of cyclosporine.
Question 767
• Question 767: What is suggested by dimpling of the overlying skin of
the breast?
• Choices:
• 1. Fibrocystic breast changes
• 2. Breast cancer
• 3. Bacterial mastitis
• 4. Fibroadenoma
Answer: 2
• Answer: 2 - Breast cancer
• Explanations: Breast cancer usually starts in the lactiferous (milk)
ducts. Dimpling of the skin is also known as peau d'orange. It signifies
involvement of the skin. Fibrocystic breast changes are characterized
by fibrous tissue and benign cysts. Fibroadenomas are benign tumors
of the breast tissue. Mastitis is an infection of the breast ducts and
can involve the skin but does not cause skin dimpling.
Question 768
• Question 768: A postmenopausal 68-year-old gravida 2 para 2 female is diagnosed with stage 1
endometrial cancer. She initially presented to the emergency department with the passage of clots,
heavy vaginal bleeding, and feeling pre-syncopal. She was hypotensive, tachycardic, and her
hemoglobin was noted to be 7.6 g/dL. Her symptoms improved after the transfusion of 1 unit of
packed red blood cells. Pelvic examination was concerning for blood coming from the cervical os
without any other masses or abnormalities. Pelvic ultrasound demonstrated 7mm heterogenous
endometrial stripe. She underwent menarche at the age of 11 and menopause at the age of 60.
She is obese, but otherwise, her past medical, past surgical, family and social history are
noncontributory. What is the overall 5-year survival for stage 1 endometrial cancer?
Choices:
• 1. 93%
• 2. 76%
• 3. 62%
• 4. 38%
Answer: 2
• Answer: 2 - 76%
• Explanations: Stage 1 endometrial cancer has an overall 5-year
survival of 76%. Due to endometrial cancer often presenting with
abnormal uterine bleeding, it commonly is diagnosed at an early
stage, when cure is more likely, and the prognosis is excellent. White
race, nulliparity, obesity, tamoxifen use, and history of colorectal,
breast, or ovarian cancer are risk factors. Generally, a higher stage is
associated with a less favorable prognosis.
Question 769
• Question 769: Exposure to beta-naphthylamine is known to cause
which of the following?
• Choices:
• 1. Bladder cancer
• 2. Hepatitis B
• 3. Crohn disease
• 4. Acne
Answer: 1
• Answer: 1 - Bladder cancer
• Explanations: Naphthylamine is found in tobacco smoke and is
associated with many illnesses. Prolonged exposure to naphthylamine
is linked to bladder cancer. Naphthylamine is rapidly inactivated in the
blood, but active products can form in the bladder. Bladder cancer is
the fourth most common cancer in men and is typically transitional
cell or urothelial cell type.
Question 770
• Question 770: Expression of which of the following in breast cancer
would indicate probable response to trastuzumab?
• Choices:
• 1. Estrogen receptors
• 2. Progesterone receptors
• 3. Her2/neu receptors
• 4. Androgen receptor
Answer: 3
• Answer: 3 - Her2/neu receptors
• Explanations: The three receptors currently important for treatment
of breast cancer are estrogen, progesterone, and Her2/neu.
Trastuzumab is a monoclonal antibody directed toward Her2/neu.
Aromatase inhibitors and selective estrogen receptor modulators are
used for estrogen positive tumors. Progesterone receptors present in
breast cancer indicate probable response to hormonal manipulation
but no medications are currently being used that are directed at this
receptor.
Question 771
• Question 771: A 65-year-old farmer comes to the physician's office due to a
lesion on the tip of his nose. He first noticed the spot a couple of years ago,
but it has since enlarged. He has years of unprotected sun exposure. He does
not visit the doctor often and has not been sick lately. Biopsy of the lesion
confirms malignant melanoma. Based on the diagnosis and location of the
lesion, what is the most likely location to find the first evidence of metastasis?
• Choices:
• 1. Upper superficial cervical lymph nodes
• 2. Submandibular lymph nodes
• 3. Retropharyngeal lymph nodes
• 4. Upper deep cervical lymph nodes
Answer: 2
• Answer: 2 - Submandibular lymph nodes
• Explanations: The anterior nasal cavity and external nose drain
anteriorly to the face that then makes their way to the submandibular
lymph nodes. Submandibular lymph nodes are located in level 1B. The
specific levels and location of lymph nodes help determine if cancer
has spread. The upper cervical lymph nodes and retropharyngeal
lymph nodes drain the posterior nasal cavity and paranasal sinuses.
Question 772
• Question 772: Widespread cancer is diagnosed in a 34-year-old
Vietnamese immigrant. Serologic studies reveal a high level of alpha-
fetoprotein. This laboratory finding favors the diagnosis of a primary
carcinoma of which organ?
• Choices:
• 1. Breast
• 2. Colon
• 3. Lung
• 4. Liver
Answer: 4
• Answer: 4 – Liver
• Explanations: Hepatocellular carcinoma is associated with elevated
blood alpha-fetoprotein in about 70% of patients. Alpha-fetoprotein
also is elevated by cirrhosis, but a sudden increase indicates possible
cancer. Thus levels are monitored. Germ cell tumors also can have
elevated alpha-fetoprotein. The global geographic distribution of
hepatocellular carcinoma incidence also favors this diagnosis.
Hepatocellular carcinoma is very common in sub-Saharan Africa,
China, and other parts of Asia.
Question 773
• Question 773: Which of the following is true regarding cervical cancer?
• Choices:
• 1. The death rate from cervical cancer continues to increase despite
cervical cancer screening
• 2. Immunosuppression has not been identified as a risk factor for
cervical cancer
• 3. Human papillomavirus (HPV) types 1 and 3 are most frequently
associated with cervical cancer
• 4. Smoking has been linked to the development of cervical cancer
Answer: 4
• Answer: 4 - Smoking has been linked to the development of cervical
cancer
• Explanations: The death rate from cervical cancer in developed
countries is decreasing due to cervical cancer screening. Women who
are immunosuppressed have a higher risk of developing cervical
cancer. Human papillomavirus (HPV) types 16 and 18 are most
frequently associated with cervical cancer. Smoking nearly doubles
the risk of developing cervical cancer.
Question 774
• Question 774: In which of the following does gastric adenocarcinoma
usually arise?
• Choices:
• 1. Mucosa
• 2. Smooth muscle
• 3. Serosa
• 4. Submucosa
Answer: 1
• Answer: 1 – Mucosa
• Explanations: Gastric cancer is not very common and often presents
with vague symptoms. The cancer usually arises from the superficial
mucosa that lines the surface of the stomach. The chief risk factor for
stomach cancer is H. pylori. At least 90% of gastric cancers are related
to H. pylori. Eradication of H. pylori has not led to complete reduction
in gastric cancer.
Question 775
• Question 775: Which layer of bowel must colon cancer cross in order
to spread?
• Choices:
• 1. Mucosa
• 2. Muscularis mucosa
• 3. Adventitia
• 4. Endothelial layer
Answer: 2
• Answer: 2 - Muscularis mucosa
• Explanations: Colon cancer starts in the mucosa and must cross the
muscularis mucosa before it can spread. A colon cancer which invade
the submucosa is a T1 lesion. When the colon cancer invade
muscularis propria, it is a T2 cancer. When the cancer has invaded the
subserosa or beyond it is a T3 cancer.
Question 776
• Question 776: Which cancer is associated with the use of tamoxifen?
• Choices:
• 1. Breast
• 2. Skin
• 3. Uterine
• 4. Ovarian
Answer: 3
• Answer: 3 - Uterine
• Explanations: Tamoxifen is an estrogen receptor partial agonist used
to treat estrogen receptor-positive breast cancer. Tamoxifen may
activate the estrogen receptors in the uterus causing the uterine
lining to grow, which can lead to endometrial cancer. Adverse effects
of tamoxifen include hot flushes, vaginal bleeding, hypercalcemia, and
peripheral edema. Patients should receive regular gynecologic follow-
up while taking tamoxifen.
Question 777
• Question 777: Kaposi sarcoma has been linked to which infection?
Choices:
• 1. HLTV-1
• 2. Cytomegalovirus
• 3. Human herpesvirus
• 4. Epstein Barr virus
Answer: 3
• Answer: 3 - Human herpesvirus
• Explanations: Human herpes virus 8 is responsible for Kaposi
sarcoma. The virus is usually acquired through saliva. It often presents
in patients with HIV. Typical Kaposi sarcoma occurs on the soles and
toes as a reddish or blackish macule or a patch The endemic type of
Kaposi sarcoma is usually seen in young individuals in the subsahara.
This variant is very aggressive and extensively invades the deeper
layers of skin.
Question 778
• Question 778: Which of the following primary cancer with metastases
to the liver has the best prognosis after liver resection?
• Choices:
• 1. Colon
• 2. Breast
• 3. Melanoma
• 4. Ovary
Answer: 1
• Answer: 1 - Colon
• Explanations: If liver metastases are resectable, the 5 year survival is best
for colon cancer among those listed. The most common cause of death
from colorectal cancer is presence of liver metastases. At least 30-70
percent of individuals who die of cancer have liver metastases at autopsy.
Isolated liver metastases from colon cancer have the best results if both
the colon and liver lesion can be completely excised. Treatment of
colorectal liver metastases with systemic chemotherapy is futile and less
than 10 percent of patients are alive at 3 years. Other therapies for
colorectal metastases include embolization, octreotide, tumor
chemoembolization, and intra-arterial administration of 5-fluorouracil.
Question 779
• Question 779: In which scenario is mastectomy not a component of
initial therapy?
• Choices:
• 1. Inflammatory cancer
• 2. Intraductal cancer
• 3. Phyllodes tumor
• 4. Paget disease
Answer: 1
• Answer: 1 - Inflammatory cancer
• Explanations: Inflammatory breast cancer is very aggressive and
usually presents with a red, swollen, inflamed breast. It rarely
presents with a mass and is not detected on a mammogram.
Inflammatory breast cancer may present with pain, skin changes,
itching, nipple retraction, nodes, tethering of skin, peau d'orange,
warm breast, or swelling of the arm. Diagnosis is made by biopsy.
Treatment of
Question 780
• Question 780: What is the most common testicular malignancy?
• Choices:
• 1. Leydig cell tumor
• 2. Sertoli cell tumor
• 3. Nonseminomatous germ cell tumor
• 4. Seminomatous germ cell tumor
Answer: 3
• Answer: 3 - Nonseminomatous germ cell tumor
• Explanations: Nonseminomatous germ cell tumors are the most
common malignant testis tumors, usually with multiple tumor types
in the same mass. Second most common are the seminomatous
tumors. Leydig and Sertoli cell tumors are both gonadal stromal
tumors. They are uncommon and usually benign. In children, the two
most common testicular tumors are teratomas and yolk sack tumors.
Question 781
• Question 781: What kind of drug is paclitaxel?
• Choices:
• 1. Alkylating agent
• 2. Antimetabolite
• 3. Plant alkaloid
• 4. Taxane
Answer: 4
• Answer: 4 - Taxane
• Explanations: Paclitaxel is obtained from the bark of Taxus brevifolia,
commonly known as the Pacific yew tree. It is a taxane. It is a
cyclodecane that stabilizes polymerized microtubules causing cell
death. It is used in intracoronary stents to prevent restenosis. It is
used for lung, breast, ovarian, and head and
Question 782
• Question 782: What is the most common histology of penile cancer?
• Choices:
• 1. Squamous cell
• 2. Basal cell
• 3. Actinic keratosis
• 4. Melanoma
Answer: 1
• Answer: 1 - Squamous cell
• Explanations: Penile cancer is very rare but can be devastating. The
majority of penile cancers are squamous cell carcinomas. Penile
cancer is slow-growing and rarely interferes with erection or voiding,
hence its diagnosis is often delayed. Further, because of
embarrassment, many delay seeking help. The majority of penile
cancers occur on the glans and prepuce. Go to the next page if you
knew the correct answer, or click the link image(
Question 783
• Question 783: Which of the following is a causative factor for the
development of thyroid carcinoma?
• Choices:
• 1. Use of synthetic thyroid medications
• 2. Radiation exposure
• 3. Excessive iodine intake
• 4. Treated hypothyroidism
Answer: 2
• Answer: 2 - Radiation exposure
• Explanations: Radiation exposure is known to cause thyroid
carcinoma. In the early twentieth-century external beam radiation
was used to treat benign conditions such as tonsillar hypertrophy,
acne, tinea capitis, and thymic enlargement before it was found to be
carcinogenic. In 1986, the Chernobyl disaster caused as much as a
100-fold increase in pediatric thyroid cancer in those exposed. When
radiation exposure causes thyroid cancer it is almost always papillary.
Alos, hypothyroidism, if left untreated, can increase the risk of
malignancy and the growth of thyroid cancer cells secondary to the
trophic effect of increased levels of thyroid-stimulating hormone.
Question 784
• Question 784: A 63-year-old man with a past medical history of essential
hypertension and type 2 diabetes mellitus presents to the clinic for follow up. He
has been receiving aggressive immunosuppressive therapy for the past two months
to treat Hodgkin Lymphoma. Per patient history, he has developed orthopnea,
fatigue, and lower extremity edema. His most recent diagnostic studies indicate he
has a reduced ejection fraction. Which of the following derivative does the
chemotherapeutic agent most likely used in this patient belong to?
• Choices:
• 1. Chromomycins
• 2. Ribonucleotide reductase inhibitor:
• 3. Plant alkaloids
• 4. Anthracyclines
Answer: 4
• Answer: 4 – Anthracyclines
• Explanations: Anthracyclines, including idarubicin, doxorubicin,
epirubicin, daunorubicin, anthracenedione, and mitoxantrone, cause
cardiotoxicity. Patients need to be evaluated before treatment and
monitored, usually with echocardiography. Doxorubicin is used to
treat cancer of the breast, ovary, prostate, stomach, and thyroid. It is
also used to treat Hodgkin disease, lymphomas, ALL, AML, small cell
cancer of lung, liver, squamous cell cancer of head and neck, and
multiple myeloma. Patients with prior mediastinal irradiation may
show delayed cardiotoxicity.
Question 785
• Question 785: Which of the following is a tumor marker for a
nonseminoma testicular cancer?
• Choices:
• 1. Alkaline phosphatase
• 2. CA-125
• 3. Carcinoembryonic antigen
• 4. Alpha-fetoprotein
Answer: 4
• Answer: 4 - Alpha-fetoprotein
• Explanations: Tumors that appear to have a seminoma histology, but
have elevated serum levels of alpha-fetoprotein (AFP), should be
treated as nonseminomas. Elevation of the beta subunit of human
chorionic gonadotropin (HCG) alone is found in approximately 10% of
patients with pure seminoma. Levels of AFP and HCG are followed
after treatment to monitor for recurrence. Treatment for non-
seminomatous germ cell tumors may include surveillance, but
chemotherapy is more commonly used, especially in Europe.
Retroperitoneal lymph node dissections may also be done.
Question 786
• Question 786: What is the major histological subtype of uterine
cancer?
• Choices:
• 1. Melanoma
• 2. Adenocarcinoma
• 3. Squamous cell cancer
• 4. Rhabdomyosarcoma
Answer: 2
• Answer: 2 – Adenocarcinoma
• Explanations: The majority of uterine cancers are adenocarcinomas.
Other histological subtypes include sarcomas, leiomyosarcomas,
rhabdomyosarcomas, and chondrosarcomas. Currently, there are no
screening tests for uterine cancer in asymptomatic women. No
evidence suggests that routine endometrial sampling can detect the
cancer early. A major risk factor for uterine cancer is exposure to
estrogens. Thus, women who take tamoxifen or who have polycystic
ovary syndrome are at a high risk for this malignancy.
Question 787
• Question 787: A 30-year-old lady presents with a severe backache
following the lifting of 10 kg weight at the gym. The pain is radiating to
the right lower limb. She is being prescribed ketorolac intravenously for
pain relief. What is the best method to assess the intensity of this pain
and its response to analgesic?
• Choices:
• 1. Description
• 2. Alphabet
• 3. Numeric
• 4. Gesturing
Answer: 3
• Answer: 3 – Numeric
• Explanations: The adequacy of pain medication is based on patient
self-assessment. The most efficient self-assessment is with a numeric
rating system. Nonverbal patients may be able to use a visual number
line. Face expression rating tools can be used.
Question 788
• Question 788: An individual with lung cancer experiences weakness,
confusion, ataxia, sensory loss, and nystagmus. What is the most likely
diagnosis?
• Choices:
• 1. Transient ischemic attack
• 2. Hypercalcemia
• 3. Lupus
• 4. Paraneoplastic syndrome
Answer: 4
• Answer: 4 - Paraneoplastic syndrome
• Explanations: Paraneoplastic syndromes can involve the nervous
system causing these signs and symptoms. Lung cancers are the most
common causes of paraneoplastic syndrome. Other paraneoplastic
syndromes can involve the rheumatologic, renal, endocrine,
gastrointestinal, or hematologic systems.
Question 789
• Question 789: A 28-year-old female, gravida 0, presents to a reproductive endocrinologist's office
desiring fertility. She was recently diagnosed with polycystic ovarian syndrome (PCOS) based on
chronic anovulation, signs of hirsutism (facial hair), and an ultrasound that was significant for
multiple follicles (>10 mm) in each ovary. At her initial visit, her body mass index (BMI) was noted
to be 38 kg/m2, and her heart rate was 120 beats per minute. All of her other vital signs were
within normal limits. Her past medical history is significant for type II diabetes that is controlled
with metformin 500 mg twice daily. She denies any past surgical history. Her family history is
significant for hypertension, dyslipidemia, and obesity in her mother and father and PCOS in her
older sister. Upon questioning, it is revealed that menarche was at age 14, and she received all
childhood vaccinations, including the human papillomavirus (HPV) series as a teenager. A cycle
was induced using the progesterone withdrawal method; however, she continued to have heavy
bleeding with the passage of clots for four weeks. A pelvic ultrasound was performed and
demonstrated a diffusely thickened endometrium without any other anomalies. An endometrial
biopsy was collected and the result was concerning for endometrial adenocarcinoma. Which of
the following is associated with her increased risk of endometrial cancer?
• Choices:
• 1. Diabetes
• 2. Late menarche
• 3. Age less than 35
• 4. Induction of cycle using progesterone
Answer: 1
• Answer: 1 - Diabetes
• Explanations: Diabetes is associated with endometrial cancer,
primarily due to the link to obesity. The risk of endometrial cancer
increases after the age of 35 years. Today obesity is considered to be a
common risk factor for this cancer. in addition, nulliparity also
increases the risk of cancer Unopposed estrogen or hormone
replacement therapy can increase the risk of endometrial cancer.
Some studies show that tamoxifen is also associated with a high risk
of endometrial cancer; however, it is not as strong of a correlation as
diabetes.
Question 790
• Question 790: Which factor is most important for prognosis for
colorectal cancer?
• Choices:
• 1. Lymph node involvement
• 2. Local invasion
• 3. Tumor size
• 4. Histological type
Answer: 1
• Answer: 1 –Lymph node involvement
• Explanations: when colon cancer has spread to lymph node the cure
is seldom possible
• Patients have developed distant metastasis associated with colon
cacer has a life expectancy of 7 months
• When there are a localized rigonal node involved , chemotherapy and
surgery may cure few patients , however most patients has
reoccurrence of cancer.
• When colon cancer has spread to the liver tha chance of survival is
poor
Question 791
• Question 791: In a 65-year-old with ductal carcinoma in situ (DCIS) of
the left breast, what other treatment is administered besides a
lumpectomy?
• Choices:
• 1. Chemotherapy
• 2. Radiation
• 3. Axillary node dissection
• 4. Bilateral mastectomy
Answer: 2
• Answer: 2 – Radiation
• Explanations: When a lumpectomy is performed for DCIS, radiation is added to
decrease the risk of local recurrence from 30% to 40% with lumpectomy alone to
15% to 16% for lumpectomy with radiation. For DCIS, the recommended margin is
greater than or equal to 2 mm after surgery. If the margins are positive, reexcision
is recommended. Otherwise, a radiation boost can be added if re-excision is not
feasible. Chemotherapy is not used in people with DCIS, but hormone therapy
may be recommended if they are estrogen or progesterone receptor positive. The
addition of hormone therapy after lumpectomy and radiation can reduce the risk
of local recurrence further. There are certain indications for omission of radiation
after breast-conserving surgery. These include patients with all of the following
factors: age greater than 70 years, T1N0, negative margins, ER+, and receiving
hormone therapy.
Question 792
• Question 792: A 52-year-old man presents to the clinic with a painless mass in
his left breast for the past 2 months. He has a BMI of 34. Physical exam shows a
2 cm firm mass in the left breast with irregular edges. There is nipple retraction
with minimal nipple discharge. The patient has no other complaints. He denies
any family history of cancer. He has worked in a steel mill for the past 20 years.
What is the best initial test to confirm the diagnosis in this patient?
• Choices:
• 1. MRI breast
• 2. Mammogram now
• 3. Ultrasound-guided fine-needle aspiration
• 4. Ultrasound-guided core biopsy
Answer: 4
• Answer: 4 - Ultrasound-guided core biopsy
• Explanations: While this patient does not have a family history of
breast cancer, his other risk factors include, age, previous occupation
and exam findings raise the suspicion of breast cancer. Ultrasound-
guided core biopsy of the left breast is favored over fine-needle
aspiration for definitive diagnosis. Mammography can be effective in
some patients as it has high sensitivity and specificity, however, in this
obese patient, ultrasound-guided core biopsy would provide a more
definitive diagnosis. Further invasive testing should
Question 793
• Question 793: A patient with squamous cell lung cancer presents with
confusion, constipation, anorexia, and bloating. Further review
reveals two fractured hips and ongoing renal failure. What is the most
likely cause of these findings?
• Choices:
• 1. Metastatic spread
• 2. Osteoporosis
• 3. Hypercalcemia
• 4. Hyperkalemia
Answer: 3
• Answer: 3 – Hypercalcemia
• Explanations: This is a paraneoplastic syndrome causing
hypercalcemia. The most common cause of hypercalcemia is
hyperparathyroidism followed by hypercalcemia of malignancy. The
ectopic parathyroid hormone secretion also is associated with
hypophosphatemia. Hypophosphatemia needs to be corrected while
treating hypercalcemia. Despite the hypercalcemia, the tumor is
resectable, and the levels will normalize following excision. Two
percent of all cancers are associated with hypercalcemia. If severe,
hypercalcemia can be life-threatening and is associated with poor
prognosis.
Question 794
• Question 794: Cancer cachexia is the status of extreme loss of muscle
and adipose tissue. This is frequently seen in patients with advanced
cancer of the respiratory tracts and gastrointestinal tract. Which of
the following is correct?
• Choices:
• 1. This status is similar to simple starvation or hunger
• 2. This status can be reversed by nutritional supplements
• 3. This status can be reversed by chemotherapy and/or radiotherapy
• 4. This status is secondary to altered metabolism
Answer: 4
• Answer: 4 - This status is secondary to altered metabolism
• Explanations: In simple starvation or hunger, fat replaces glucose as the preferred
fuel to spare lean body mass and is not associated with systemic increases in
proinflammatory factors. Cancer cachexia cannot be reversed by nutritional
supplements. Patients are less tolerant of systemic chemotherapy and
radiotherapy. Several metabolic alterations have been recognized and have been
regarded as the cause of cancer cachexia, which includes ATP production mainly
from glycolysis rather than from TCA cycle, futile metabolic cycles, and increased
free fatty acids production in plasma. Cancer cells are unable to produce ATP via
TCA cycle because of activation of HIF (hypoxia-inducible factor) by tumor
suppressors and oncogenes, as well as mutations in metabolic enzymes,
especially isocitrate dehydrogenase 1 (IDH1) and IDH2 and other TCA cycle
enzymes.
Question 795
• Question 795: During your clinical rotation in a cancer clinic, you will
take special precautions when handling and mixing which of the
following medications?
• Choices:
• 1. Lorazepam
• 2. Ranitidine
• 3. Promethazine hydrochloride
• 4. Vinblastine sulfate
Answer: 4
• Answer: 4 - Vinblastine sulfate
• Explanations: Special precautions should be taken when handling
vinblastine sulfate. Vinblastine sulfate is an antineoplastic medication
whose arrest of cell growth is dependent on blocking cell division.
Because of its potent effect on mitosis, it has been associated with
cancer risk for those exposed to the drug, including those who
prepare the medication prior to its administration. Lorazepam,
ranitidine, and promethazine hydrochloride have no documented
risks for persons who handle them.
Question 796
• Question 796: Which of the following neoplastic drug is used in the
treatment of seminomas and works by inhibiting topoisomerase II?
• Choices:
• 1. Cisplatin
• 2. Ciprofloxacin
• 3. Etoposide
• 4. Cyclophosphamide
Answer: 3
• Answer: 3 – Etoposide
• Explanations: Etoposide is an immunosuppressant drug used to treat
a variety of cancers. The drug is also known as VP 16 and is often
given in combination with other cancer drugs to reduce toxicity. VP 16
works by inhibiting topoisomerase II. After VP 16, one must drink lots
of fluid to prevent damage to the bladder and kidneys. Side effects
include hypotension, hair loss, metallic taste, and bone marrow
suppression.
Question 797
• Question 797: A patient develops shortness of breath and a dry cough
after starting a chemotherapeutic agent. Which of the following is the
most likely cause?
• Choices:
• 1. Bleomycin
• 2. Dacarbazine
• 3. Doxorubicin
• 4. Cyclophosphamide
Answer: 1
• Answer: 1 - Bleomycin
• Explanations: The most serious side effect of bleomycin is pulmonary
fibrosis and impaired lung function. Bleomycin-induced free radical
damage is caused by sensitivity to oxygen. Patients with poor lung
function should not be given bleomycin as this can induce respiratory
failure. Other side effects of bleomycin include Raynaud
phenomenon, hyperpigmentation, and alopecia.
Question 798
• Question 798: A 4-year-old boy with a past medical history of Li-Fraumeni syndrome is
brought in by his mother after she noticed a bulge in his neck. The mother says she is
unsure of how long he has had it, though notes that he has not complained of any
pain or trouble swallowing. She denies that he has been around any sick contacts. The
exam is notable for a mass on the left side of his neck that is not painful to palpation.
Subsequent ultrasound and MRI concern for a mass. A biopsy is taken and evaluated
with immunohistochemistry. What will most likely be positive?
• Choices:
• 1. Demin
• 2. Keratin
• 3. CK
• 4. CD45
Answer: 1
• Answer: 1 - Demin
• Explanations: A rhabdomyosarcoma may arise in any location but 28%
arise in the head and neck. 24% arise in the extremities, 18% the
genitourinary tract, and 11% the trunk. To confirm the diagnosis of
rhabdomyosarcoma immunohistochemical, molecular genetic and/or
ultra-structural techniques may be needed. Rhabdomyosarcoma
immunohistochemistry is usually positive for myogenin, demin,
sarcomeric actin, myogenin, and myoglobin. It is usually negative for
NKX2.2, CD99, CD45, CK, S100, and NSE.
Question 799
• Question 799: What tests are performed to assess a patient suspected
of having a colon cancer?
• Choices:
• 1. Total body CT scan
• 2. Colonoscopy
• 3. CBC, CEA, liver function, and electrolyte levels
• 4. Barium swallow
Answer: 2
• Answer: 2 - Colonoscopy
• Explanations: Colonoscopy is usually done to diagnose colon cancer.
Once a patient is diagnosed with colon cancer, CBC, electrolytes, and
liver function tests should be done. Once a colon cancer has been
diagnosed, staging should be done with a CT scan. PET scan is usually
done for recurrent disease.
Question 800
• Question 800: A 6-year-old boy is brought to his pediatrician for an annual visit. The patient has a history
of advanced osteosarcoma and finished treatment last week. He has a history of asthma and is treated
with an albuterol rescue inhaler as needed. He is recovering from acute otitis media one month prior. He
has no pertinent family history and lives at home with his mother, father, and sisters in a carpeted home
with a pet dog. His mother mentions during the visit that she often has to say his name multiple times to
get his attention and that sometimes “he seems oblivious to what is happening around him” and she
wonders if he is haven’t trouble with attention like his sister who was diagnosed with ADHD. You have to
speak loudly to get his attention, and he continuously tries to play with his toys instead of participating in
the examination. The physical exam is unremarkable. He is afebrile and reports no ear pain when on the
manipulation of the pinna and denies discharge from the ear. What is the next best step in the
assessment?
• Choices:
• 1. Repeat course of antibiotics
• 2. Multimodality assessement for ADHD
• 3. Audiometry testing
• 4. MRI of the brain
Answer: 3
• Answer: 3 - Audiometry testing
• Explanations: This patient has a history of advanced osteosarcoma and
has just finished treatment. Advanced osteosarcoma is sometimes treated
with a regimen containing Cisplatin, which is known to be ototoxic.
Ototoxicity may be manifested by tinnitus, loss of high-frequency hearing,
and occasionally deafness. Index of suspicion should be high in children
demonstrating signs of ototoxicity such as hearing loss. Pediatric patients
should receive audiometry testing to indicate a baseline, and testing
should be repeated before each subsequent round of chemotherapy. Loss
of hearing acuity can be detrimental to the development of language in
pediatric populations and should be assessed with audiometry.

You might also like